Você está na página 1de 312

MicroMash

Bar Review

MBE In Brief
Condensed Outlines

The MBE REFERENCE Volumes consist of: Introductory Volume: Introduction The MicroMash Way to the MBE Reference Volume I: Constitutional Law Criminal Law & Procedure Evidence Reference Volume II: Contracts Real Property Torts MBE In Brief: Condensed Outlines Bar Exam Alerts At-A-Glance

Acknowledgements: The contents of the MicroMash MBE Review were initially developed and written by SMH. Walter McLaughlin, Jr. and his partners, Frederick M. Hart and James W. Smith, founded the SMH Bar Review in Massachusetts in 1964. The SMH Bar Review expanded to 22 additional states and the District of Columbia. In 1996, MicroMash purchased the SMH Bar Review and converted its MBE product into the current computer-based Review. Walter McLaughlin continues as the primary contributing author of the MicroMash MBE Review. We recognize the accomplishments and dedication of our team of employees whose visions have made such an outstanding product. They contributed countless hours to deliver this package and are each fully dedicated to helping candidates pass the Bar Exam. We particularly thank the following members of the MicroMash MBE Review team: Software designers, developers, and testers: H. Bart Rogers, Ph.D. Jeff Owen James Otis Olga Zaturenskaya Nelson Adams Susan Wines Kirk Langman Carlos Seegmiller Legal Editorial Department: Amy 0. Poggioli, Esq. Lesley A. Yosses, Esq. Sheryl Botnick, Esq. Brian Page, J.D.

Marketing: Mary Howard Megan Robinette

This book, the accompanying software, and printed supplements contain questions and answers from the Multistate Bar Examination copyrighted 1992, Sample MBE copyrighted 1995, Sample MBE II copyrighted 1997, and Sample MBE III copyrighted 2002 by the National Conference of Bar Examiners ("NCBE"), all rights reserved, and are reprinted with permission. MBE Subject Matter Outlines are copyrighted 1997 by the NCBE and are reprinted with permission. "MBE" and "Multistate Bar Exam" are trademarks of the NCBE. Certain publicly disclosed questions and answers from past MBEs have been included herein with the permission of NCBE, the copyright owner. These questions and answers are the only actual MBE questions and answers included in MicroMash's materials. Permission to use the NCBE's questions does not constitute an endorsement by NCBE or otherwise signify that NCBE has reviewed or approved any aspect of these materials or the company or individuals who distribute these materials. All other trademarks are the property of their respective companies. This book is written to provide accurate and authoritative information concerning the covered topics. It is not meant to take the place of professional advice. Software copyright CD1984-2003 by M-Mash, Inc. Copyright 2003 by M-Mash, Inc. All rights reserved. No part of this work may be reproduced or transmitted in any form or by any means, electronic or mechanical, including photocopying and recording, or by any information storage or retrieval system, except as may be expressly permitted by the 1976 Copyright Act or in writing by the Publisher. Requests for permission should be addressed in writing to: Legal Editorial Department, MicroMash, 6402 South Troy Circle, Englewood, Colorado 80111-6424.

ISBN: 0314147446

MicroMash MBE In Brief: Condensed Outlines Bar Exam Alerts At-A-Glance

MicroMash
MBE IN BRIEF CONSTITUTIONAL LAW

MicroMash BAR REVIEW


MBE IN BRIEF CONSTITUTIONAL LAW
Table of Contents
I. THE NATURE OF JUDICIAL REVIEW 1

A. ORGANIZATION AND RELATIONSHIP OF STATE AND 1 FEDERAL COURTS IN THE FEDERAL SYSTEM
1. 2. Federal Court Jurisdiction State Court Jurisdiction Original Jurisdiction Appellate Jurisdiction To Review Federal Court Decisions 1 2 2 2

B.

CONSTITUTIONAL BASIS OF SUPREME COURT JURISDICTION2


1. 2.

C.

CONGRESSIONAL POWER TO DEFINE AND LIMIT FEDERAL 2 COURT JURISDICTION


1. 2. 3. The Supreme Court Lower Federal Courts Legislative (Article I) Courts General Principles Of Constitutional Adjudication The Case-or-Controversy Requirement Standing Timing Of Litigation Justiciability Political Questions Appellate Jurisdiction To Review State Court Decisions Burden Of Proof In Constitutional Litigation 2 2 3

D.

JUDICIAL REVIEW IN OPERATION


1. 2. 3. 4. 5. 6. 7.

3
3 3 3 4 4 4 5

II.

THE SEPARATION OF POWERS


A. THE POWERS OF CONGRESS
1. 2. Enumerated And Implied Powers The Federal Commerce Power

5 5
5 5

3. 4. 5. 6. 7. 8. 9.

The Taxing Power The Spending Power

6 6

Power Over Territories 6 The Federal Property Power 6 War And Defense Powers 6 Congressional Power To Enforce The Thirteenth, Fourteenth, And Fifteenth Amendments 6 The Investigatory Power 7

B.

POWERS OF THE PRESIDENT


1. 2. 3. 4.
As Chief Executive The Powers Of The President As Commander-In-Chief Powers Of The President Over International Affairs The Appointment And Removal Powers
-

7
7 8 8 8

C.

FEDERAL INTER BRANCH RELATIONSHIPS


1. 2. 3. 4. Congressional Limits Upon The Executive

8 The Presentment Requirement And The President's Power To Veto Or To Withhold Action 9 Delegation Doctrine 9 Executive, Legislative, And Judicial Immunities 10

III. THE RELATION OF THE NATION AND THE STATES IN THE FEDERAL SYSTEM 11
A. INTERGOVERNMENTAL IMMUNITY
1. 2. 1. 2. 3. 1. 2. 3. 4. 5. Immunity Of The Federal Government Immunity Of State Government Negative Implications Of The Commerce Power Limitations On State Power In Taxation Specific Constitutional Limitations On State Power Direct Conflict State Act Enhances Federal Policy State Act Contravenes Federal Policy Preemption

11
11 11

B. AUTHORITY RESERVED TO THE STATES

11
11 12 13

C. NATIONAL POWER TO OVERRIDE STATE AUTHORITY

13
13 13 13

14 Congressional Authorization For Otherwise Impermissible State Activity14

ii

D. RELATIONS AMONG STATES


I. 2. Interstate Compacts Full Faith And Credit

14
14 14

IV. INDIVIDUAL RIGHTS


A. STATE ACTION AND THE ROLE OF THE COURTS
1. 2. 3. 1. 2. 3. 4. 1. 2. 3. 4. 1. 2. State Agency Public Function State Involvement Substantive Due Process Takings Procedural Due Process What Process Conforms To The Requirement? Fundamental Interests Nonfundamental Interests Suspect Classifications Nonsuspect Categories Under The Fourteenth Amendment Under Article IV, 2 The Comity Clause

15
15
15 15 15

B. DUE PROCESS

16
16 17 18 18

C. EQUAL PROTECTION OF THE LAWS

19
19 20 20 21

D. PRIVILEGES AND IMMUNITIES CLAUSES

22
22 22

E. THE OBLIGATION OF CONTRACTS, BILLS OF ATTAINDER, 22 AND EX POST FACTO LAWS


1. 2. 3. 1. 2. Obligation Of Contracts Bills Of Attainder Ex Post Facto Laws Freedom Of Religion And Separation Of Church And State Freedom Of Expression And Association 22 22 22

F. FIRST AMENDMENT FREEDOMS

23
23 24

iv

CONSTITUTIONAL LAW

I. THE NATURE OF JUDICIAL REVIEW A. ORGANIZATION AND RELATIONSHIP OF STATE AND FEDERAL COURTS IN THE FEDERAL SYSTEM
1. Federal Court Jurisdiction The federal court system consists of a constitutionally created Supreme Court and inferior courts created by Congress. Their jurisdiction is constitutionally limited to cases or controversies arising under the Constitution, laws of the United States, and treaties of the United States; cases affecting ambassadors, ministers and consuls, and admiralty and maritime jurisdiction; cases in which the United States is a party; and cases based upon diversity of citizenship. a. Limitation of the Eleventh Amendment The Eleventh Amendment prohibits a citizen from suing a state in the federal court. b. Exceptions to the application of the Eleventh Amendment Under the ex parte Young exception, a state official can be sued in federal court to prevent his enforcement of an unconstitutional statute. The claim may not be based on state law. A state may expressly waive its Eleventh Amendment protection. Personal damage suits against state officers are not prohibited by the Eleventh Amendment. An injunction may require an official to make prospective disbursements of state funds, but damages as to past violations may not be assessed against the state. Pursuant to the Fourteenth Amendment, Congress can authorize a suit against a state to enforce rights created by that amendment. c. Abstention There are two types of abstention: 1) Discretionary abstention A federal court can, as a matter of discretion, refuse to adjudicate an issue involving an unresolved point of state law until the state court has ruled on the matter. The federal court retains jurisdiction over the case, and completes adjudication if necessary after the state court opinion is rendered. 2) Abstention in cases to involving state criminal statutes There is also abstention based upon considerations of federalism, which prevents the federal district court from enjoining enforcement of a state statute or declaring it unconstitutional if there is a pending criminal prosecution under that statute. Federal review of such a statute is limited to certiorari, appeal to the Supreme

MicroMash MBE In Brief: Constitutional Law

Court, or habeas corpus in the district court. The federal district court can hear a declaratory judgment action challenging the constitutionality of a state statute when brought by a proper plaintiff if there is no pending state court prosecution.

2. State Court Jurisdiction


State courts have concurrent jurisdiction over matters within the jurisdiction of the federal courts, except where Congress or the Constitution vests exclusive jurisdiction in the federal courts. State courts are required to adjudicate federal claims, unless the states have valid, nondiscriminatory excuses.

B. CONSTITUTIONAL BASIS OF SUPREME COURT JURISDICTION 1. Original Jurisdiction


The Supreme Court has original and exclusive jurisdiction in all cases between two or more states. It has original and concurrent jurisdiction in suits between a state and the United States, suits by a state against the citizens of another state or against aliens, and suits to which ambassadors or other public ministers of foreign states are parties. To sue in the Supreme Court, a state must either place in issue its own rights, or, under the parens patriae doctrine, the rights of a large class of its citizens.

2. Appellate Jurisdiction To Review Federal Court Decisions


The United States Supreme Court can review lower federal court decisions by appeal when there is an appeal from a three-judge district court convened because the relief sought is a restraining order against enforcement of a state statute. All other cases in the federal courts of appeal, including an appeal when the court of appeals holds a state statute invalid or when the federal court has held an act of Congress unconstitutional, can be reviewed by certiorari.

C. CONGRESSIONAL POWER TO DEFINE AND LIMIT FEDERAL COURT JURISDICTION 1. The Supreme Court
Congress can neither add to nor subtract from the constitutional grant of original jurisdiction to the Supreme Court, but can vest concurrent jurisdiction in other courts. Congress can control the appellate jurisdiction of the Supreme Court, so long as the regulation of appellate jurisdiction does not destroy the role of the Court in the constitutional scheme. 2. Lower Federal Courts Congress has the power to create and abolish lower federal courts and, within the grant of the judicial power, to determine their jurisdiction. Once it has created those courts and authorized the judgeships, it cannot limit the judges' terms or reduce their compensation. Congress cannot interfere with the inherent judicial functions of courts it has created.

MicroMash MBE In Brief: Constitutional Law

3. Legislative (Article I) Courts


As an incident to its Article I powers, Congress can create courts with nonjudicial functions, such as rendering advisory opinions.

D. JUDICIAL REVIEW IN OPERATION 1. General Principles Of Constitutional Adjudication


The Court will not adjudicate a suit unless the parties are genuine adversaries. It will not anticipate a constitutional issue in advance of the need to decide it, nor formulate a rule of constitutional law broader than that required by the facts to which it is applied. The Court will not reach a constitutional issue if it is possible by construction to decide the case on nonconstitutional grounds.

2. The Case-or-Controversy Requirement


The judicial power of the United States extends only to cases or controversies that is, to actual disputes between adversaries over concrete facts. This is a constitutional limitation. The courts may not adjudicate abstract questions or render advisory opinions.

3. Standing
The standing requirement focuses on the relationship of the plaintiff to the claim he is making. Before a court will decide a case, it must be satisfied that the parties have standing that is, a direct and substantial interest in the outcome. There are two levels of standing: the constitutional requirement derived from the case-or-controversy requirement, and the self-imposed prudential rules.

a. The constitutional standard


To satisfy the constitutional standard, the plaintiff must show that she has been injured by the action she is complaining about, but the injury need not be different from the injury suffered by the public at large. Plaintiff must also show that the relief she is requesting will redress the harm that is alleged. A person has standing to challenge an administrative decision, even if a harm to a legally protected interest has not occurred, if she has been injured in fact and is within the zone to be protected by the statute in question. Damage to environmental interests will give standing so long as there is an allegation of personal injury.

b. Self-imposed requirements
Even though the constitutional standard is met, the Court is likely to deny standing if the plaintiff is not harmed in a manner different from the public at large, if he is asserting rights of third parties, or if Congress or the Constitution did not intend to confer standing.

c. Specialized problems in standing 1) Taxpayer standing


A federal taxpayer does not have standing to challenge federal expenditures unless there is a logical nexus between the status asserted and the claim sought to be

MicroMash MBE In Brief: Constitutional Law

adjudicated. The claim must seek to enjoin a spending (not a regulatory) statute, and must allege a positive limitation on the spending power. The only such limitation that has been recognized is the First Amendment Establishment Clause. 2) Standing as a citizen It is unlikely that a plaintiff can use her status as a citizen to obtain standing. 3) Standing to raise rights of third parties A party ordinarily may raise only constitutional deprivations that affect him personally. However, he may raise the constitutional rights of third parties if there is a special relationship with that third party, and if that party would not otherwise have his rights adjudicated. 4. Timing Of Litigation a. Mootness When the passage of time resolves the issues between the parties so that the plaintiff no longer has a stake in the outcome, the case will be dismissed as moot. The mootness doctrine will not apply: (1) when a party has a continuing, residuary interest in the outcome; (2) when the injury is capable of repetition, yet evades review because the passage of time makes each such case moot before it can be litigated; (3) when the case is a class action and other members of the class have a continuing interest in the outcome, and (4) when an adjudication is necessary to prevent recalcitrant officials from returning to their old practices once a case has been dismissed for mootness. b. Ripeness A suit will be dismissed because it is not ripe when the action that is the subject of the claim is indefinite, i.e., the plaintiff has not yet violated the statute. Ripeness can also bar adjudication when the defendant's alleged actions affect the plaintiff in an imprecise manner, i.e., it is not clear that the defendant would enforce the statute against the plaintiff. 5. Justiciability Political Questions The Supreme Court will decline to review a case if the final determination of the issue has been committed by the Constitution to another branch of the government, or when the Court is inherently incapable of managing the controversy. 6. Appellate Jurisdiction To Review State Court Decisions The United States Supreme Court has the right to review decisions of the highest state court by certiorari (a discretionary form of review available when four justices of the Supreme Court agree to hear the case) when the validity of a federal statute is upheld; when the validity of a state statute is successfully challenged on the ground that it is repugnant to federal law, a treaty, or the United States Constitution; or when the state court upholds the validity of a state statute, or decides against the validity of a

MicroMash MBE In Brief: Constitutional Law

federal statute. Review by appeal (a nondiscretionary form of review) has been all but
eliminated. Before a case is ripe for Supreme Court review, all state court appeals must be exhausted and the judgment must be final. Appeals on interlocutory matters are not permitted unless the injury would be irreparable and the issue is separable. Supreme Court review is not available even if federal issues are involved if the state court decision is based upon an adequate and independent state ground. State decisions on matters of substantive state law are adequate except when state law purports to incorporate a federal standard under a statute creating a federal cause of action. If the basis of decision is ambiguous, the Court will review it as decided on federal law, unless the state court has made a "plain statement" in the decision that federal cases are being used only for guidance. State procedural grounds are adequate if the procedure is uniformly applied and furthers a legitimate state interest.

7. Burden Of Proof In Constitutional Litigation a. The usual standard


The usual rule is that the plaintiff has the burden of showing that a challenged statute is unconstitutional. The ordinary standard in a due process or equal protection case is whether the statute serves a rational purpose.

b. The exceptional standard


The exceptional standard applied when the state is classifying by means of a suspect classification or affecting a fundamental interest, or is infringing upon the free exercise of religion or upon speech fully protected by the First Amendment, is that the state has the burden of justifying the statute by showing that it is necessary to serve a compelling state need.

II. THE SEPARATION OF POWERS


A. THE POWERS OF CONGRESS
1. Enumerated And Implied Powers
The Constitution vests specific legislative powers in Congress. Incidental to the power to pass legislation is the power to conduct investigations and hearings. While there is no general delegation of legislative power to Congress, the Necessary and Proper Clause gives Congress the implied power to pass legislation appropriate and reasonably related to the specific delegation of legislative powers under Article I, 8.

2. The Federal Commerce Power


Where Congress is regulating either the instrumentalities of interstate commerce or the channels of interstate commerce, its power pursuant to the Commerce Clause is plenary, limited only by positive limitations in the Constitution such as the Establishment Clause. However, when Congress is regulating intrastate activities because of their relation or effect on interstate commerce, the court must find a substantial effect on interstate

MicroMash MBE In Brief: Constitutional Law

commerce for the act to be constitutional. The legislation can make findings concerning the perceived effect which is likely to aid the court in finding the act constitutional. 3. The Taxing Power The taxing power of Congress is limited by the requirements that it be exercised in a uniform manner throughout the United States, that a direct tax (head taxes or ad valorem taxes on real property) be apportioned among the states, and that no tax be levied on exports. A tax statute, even though it regulates the subjects it taxes, will be sustained under the taxing power if one of its purposes is to raise revenue, even though the amount raised is minimal. If legislation cannot be sustained under the taxing power, it will still be valid if the regulation imposed by the statute would be valid under the commerce power. Incidental to the taxing power, Congress may enact administrative provisions that have a substantial regulatory effect, unless such regulations violate basic individual constitutional guarantees such as the right not to incriminate oneself. 4. The Spending Power Although Congress does not have the right to legislate for the general welfare, it does have the right to spend money for the general welfare, a power not subject to judicial review unless the money is spent for a purpose that is specifically prohibited by the Constitution. Congress can accomplish ends through conditions attached to the expenditure of funds that it could not accomplish by direct regulation. 5. Power Over Territories In the District of Columbia and other federal enclaves, Congress has the power that is normally possessed by a state legislature. 6. The Federal Property Power Article IV, 3 of the Constitution gives Congress the right to dispose of federal property, and to enter into competition with private businesses. Congress may also protect property owned by the federal government. 7. War And Defense Powers Pursuant to its power to declare war, raise and support armies, provide and maintain a navy, and to make rules for the government of armed forces, Congress can control the economy during wartime and for a reasonable period thereafter, authorize the destruction of private property, operate the draft, and accomplish other acts incidental to its delegated powers. It may set up military courts to try military personnel for crimes committed on military posts or in service connected activities. Except for military spies or enemy combatants, military courts may not try civilians, including military dependents. 8. Congressional Power To Enforce The Thirteenth, Fourteenth, And Fifteenth Amendments While early cases construed the Civil War Amendments in a restrictive manner, the Court has in recent years construed legislation passed under the Fourteenth Amendment to

MicroMash MBE In Brief: Constitutional Law

reach not only state action, but also actions of officers acting under color of state law. Dicta indicate that Congress has the power to legislate directly against individual conduct that restricts Fourteenth Amendment freedoms. Under the Thirteenth Amendment, Congress not only can legislate against slavery and involuntary servitude, but also can pass remedial legislation designed to remove the incidents or badges of slavery. Statutes preventing discrimination against black people in housing and in contracting for private education have been upheld under the Thirteenth Amendment, which is not limited by the state action concept. The Court has upheld the power of Congress to regulate local elections under the Fifteenth Amendment. Katzenbach v. Morgan implied, but the Court has never held, that Congress has the power to define the content of the equal protection and due process guarantees of the Fourteenth Amendment.

9. The Investigatory Power


As an incident to its power to legislate, Congress may conduct investigations on matters on which it might take action even if there is no specific legislation pending. However, an investigating committee is limited to the scope of the legislative resolution authorizing it. Congress may penalize a reluctant witness by trying her for contempt of Congress and imprisoning her for the remainder of the congressional term, or by trying her in a judicial proceeding. In such a judicial proceeding, a defendant may raise as a defense that the investigation is beyond the scope of the legislative power, that she was not given a fair opportunity to determine if she was within her rights in refusing to answer, that she properly raised her right against self-incrimination, or that her First Amendment rights were infringed.

B. POWERS OF THE PRESIDENT 1. As Chief Executive a. Power and obligation to execute the laws
As chief executive, the president is under the obligation to execute the laws and to spend funds that Congress directs the president to spend.

b. Inherent power in domestic affairs


In domestic affairs, the president has an undefined quantum of inherent authority to act unless Congress has specifically forbidden the proposed action.

c. The legislative power


The legislative power of the president is limited to proposing legislation and vetoing bills that have passed Congress. The veto power empowers the president to veto a bill in its entirety. The Court held the line item veto unconstitutional in Clinton v. City of New York. The president's veto can be overridden by a two-thirds vote of each house of Congress, except when Congress adjourns within 10 days of the time a bill is delivered to the president. In that case, the legislation is not enacted unless the president signs it.

MicroMash MBE In Brief: Constitutional Law

d. The pardon power The president has plenary power to pardon for federal crimes, and to impose conditions when granting a pardon. e. Power to refuse to disclose information executive privilege The president has an absolute privilege to refuse to disclose confidential communications dealing with military or diplomatic secrets, and a qualified privilege with respect to all other confidential communications. The qualified privilege is suspended when there is a substantial governmental interest outweighing the president's interest in preventing disclosure. 2. The Powers Of The President As Commander-In-Chief As commander-in-chief, the president has the power to deploy military forces, to establish governments in occupied territories, and, during wartime, to seize private property. The president does not have the power to declare war or to raise and support armed forces, both of which are given to Congress. 3. Powers Of The President Over International Affairs In international affairs, the president's inherent authority is much more extensive. With the approval of two-thirds of the Senate, the president has the power to conclude treaties, which, if self-executing, have a status equal to that of laws passed by Congress. Nonself-executing treaties, such as the United Nations Charter, have no effect as law. In addition, the president has the power to make executive agreements with foreign nations; such agreements have a status superior to state law, but not to congressional legislation. 4. The Appointment And Removal Powers As chief executive, the president has the power to appoint officers of the United States. As an incident to the power to make appointments, the president has the power to remove appointees without cause except those appointed to fixed terms on independent regulatory commissions.

C. FEDERAL INTER BRANCH RELATIONSHIPS


-

Although it was the intent of the framers of the Constitution to prevent the abuse of power by separating the executive, legislative, and judicial functions of government, none of the three branches of government has unfettered power in its own sphere because of restraints imposed by other branches. 1. Congressional Limits Upon The Executive a. Impeachment power The ultimate control that can be exercised by the legislature over the executive is impeachment. Article II, 4 of the Constitution provides that the president, vice president, and all civil officers of the United States can be removed from office by impeachment and conviction. The grounds for impeachment are treason, bribery, or other high crimes and misdemeanors. The Nixon impeachment process raised the question whether noncriminal activity that constituted a breach of trust or a violation

MicroMash MBE In Brief: Constitutional Law

of the oath of office was an impeachable offense. Historically, criminal conduct has not been required. The House of Representatives is given the sole power to impeach, an action akin to indictment in the criminal law. The sole power to try impeachments is in the Senate. A two-thirds vote is required for conviction. The Chief Justice of the Supreme Court presides over the impeachment trial of a president. A conviction after an impeachment removes the official from office, and disqualifies him from holding office again. b. Investigatory power The power of Congress to carry on investigations is only incident to its legislative power. In the process of investigating the executive branch for the ostensible purpose of finding the facts necessary to pass legislation, however, Congress may bring to light inadequacies in the administration of the executive branch of government. The public exposure of these inadequacies often serves to correct them. c. Appropriation power Through its ability to refuse appropriations for activities, Congress can control the policies of the executive. If Congress desires that a project go forward, it can mandate that appropriated funds be spent, and the president is required to spend them, because the president has a constitutional obligation to execute the laws. 2. The Presentment Requirement And The President's Power To Veto Or To Withhold Action a. Power and obligation to execute the laws The essence of the executive power is to execute the laws. In some cases that power is discretionary. Where Congress has not mandated that the president spend specific money that Congress has appropriated, the president has the discretion not to spend it. However, Congress can mandate that funds be spent or that laws be carried out. b. Presidential veto power Once Congress has approved legislation, the president has the power to veto it within 10 days of the time it is sent to him or her. Congress has the power to override a veto by a two-thirds vote of each house, in which case the proposed legislation will become law despite the president's veto. If Congress adjourns within 10 days of the time the legislation is delivered to the president, however, it cannot become law without the presidential signature. Therefore, the president can "pocket veto" a bill passed at the end of a congressional session by not signing it, and Congress has no authority to override such a veto. 3. Delegation Doctrine Congress frequently vests substantial discretion in the president and presidential subordinates regarding the manner in which laws are executed, and in many instances the members of the executive branch make legislative policy determinations. Such

10

MicroMash MBE In Brief: Constitutional Law

delegation of power is constitutionally permissible unless there are absolutely no Legislative veto provisions have been employed by Congress to delegate authority, while reserving to Congress the power to block specific actions or regulations. However, the veto is legislation and therefore must follow the procedures prescribed in Article I bicameral approval of legislation, presentment of legislation to the president, the president's veto, and Congress' power to override a veto. 4. Executive, Legislative, And Judicial Immunities a. Executive immunity The president is entitled to absolute immunity from civil liability for damages predicated on official acts. While the separation of powers doctrine does not bar every exercise of jurisdiction over the president, a court, before exercising jurisdiction, must balance the constitutional weight of the interest to be served against the dangers of intrusion on the authority and functions of the executive branch; private suits for damages based on the president's official acts do not warrant such an intrusion. Executive officials in general are usually entitled to only qualified or good faith immunity. This means that officials performing discretionary functions are generally shielded from liability for civil damages insofar as their conduct does not violate clearly established statutory or constitutional rights of which a reasonable person would have known. Executive officials serving as presidential aides may sometimes be entitled to absolute immunity, if the presidential aide can show that the responsibilities of her office embraced a function so sensitive as to require a total shield from liability, and that she was discharging that function when performing the act for which liability is asserted. b. Legislative immunity, the speech or debate clause Members of Congress cannot be held accountable in any forum except Congress for acts that are an integral part of the deliberative and political process, such as speaking or voting on the floor of Congress or conducting legislative hearings. Aides of members of Congress assisting them in performing these functions are also protected. The Speech or Debate Clause does not protect a member of Congress from criminal prosecution for taking a bribe to influence his vote. c. Judicial immunity Judges enjoy absolute immunity from any decisions made in their official capacity. In defamation an absolute privilege applies to statements which are part of a judicial proceeding, including statements by the judge, the attorneys, and the witnesses, provided that the statements are pertinent to the cause at issue. The test is not whether the statement is legally relevant, but whether it has reference and relation to the subject matter of the action.

MicroMash MBE In Brief: Constitutional Law

11

III. THE RELATION OF THE NATION AND THE STATES IN THE FEDERAL SYSTEM
A. INTERGOVERNMENTAL IMMUNITY
1. Immunity Of The Federal Government a. Suits against the federal government The United States can sue a state without its consent, but a state cannot sue the United States without permission. b. Regulation of the federal government The states cannot regulate the federal government in a manner that prevents it from carrying out its responsibilities, such as requiring a state license for a contractor to work on a federal project. The state may regulate the federal government if the state does not interfere with federal policies. c. Taxation of the federal government The federal government itself is exempt from direct state taxation. Congress has the power to exempt one of its instrumentalities from state taxation. Absent express congressional intention, a state tax levied directly on a nongovernmental entity, the economic burden of which is on the federal government, is constitutional. Federal employees are liable for state income taxes. 2. Immunity Of State Government a. Immunity from suit A state may be sued by the federal government or by a sister state without its consent, but may not be sued by a private citizen. b. Immunity from regulation The federal government has the right to regulate the states, and the fundamental limitation on the federal commerce power is the political process, not a judicial distinguishing of "traditional government functions." c. Taxation The federal government has the power to tax a state unless the function being taxed is one that can only be performed by a sovereign.

B. AUTHORITY RESERVED TO THE STATES 1. Negative Implications Of The Commerce Power


If Congress fails to exercise the full scope of the commerce power, states may regulate commerce, provided that the state does not discriminate against out of state commerce and does not impose an excessive burden on interstate commerce. Health and safety regulation is a traditional police power of the states and can justify nondiscriminatory,

12

MicroMash MBE In Brief: Constitutional Law

incidental burdens on interstate commerce so long as there is no less burdensome means the state could use to protect its interests. a. Exceptions A state can, in the absence of congressional legislation to the contrary, discriminate between out of state and resident buyers when it acts as a market participant. The state has plenary control over the consumption of alcoholic beverages in that state under the Twenty-First Amendment. 2. Limitations On State Power In Taxation a. Foreign commerce The states do not have the power to tax imports, but may impose nondiscriminatory, ad valorem taxes upon goods once they are physically within the state, even though they are in their original packages. States cannot impose ad valorem property taxes on instrumentalities used in foreign commerce or otherwise use taxes to discriminate against foreign commerce. b. State power to tax interstate commerce 1) General principles There are four conditions which must be met for a state to tax interstate commerce constitutionally: (1) there must be a substantial nexus between the taxing state and the subject of the tax a due process requirement; (2) the tax must be fairly apportioned to prevent interstate commerce from paying more than its share of the tax burden; (3) the tax must not discriminate in favor of local commerce and against interstate commerce; and (4) the tax must be fairly related to the services provided by the state. 2) Stream of commerce Ad valorem taxes on property cannot be constitutionally levied upon goods in interstate commerce. Once movement from one state to another has begun, goods are in interstate commerce until they reach their destination, even though there are delays in transit. If the goods come to rest and can easily be diverted into local commerce, they are no longer immune from local taxation. 3) Instrumentalities of interstate commerce Instrumentalities of interstate commerce are subject to property taxes, provided they are in a state for a sufficient period to obtain a taxable situs and the tax is fairly apportioned. The state of domicile need not apportion property taxes unless it can be shown that the property was habitually employed in another state. 4) Privilege and license taxes Privilege and license taxes are constitutional even on the privilege of conducting an interstate business within the state, provided that they meet the basic test described above. Flat fee license taxes for itinerant merchants (drummer taxes)

MicroMash MBE In Brief: Constitutional Law

13

are likely to be found unconstitutional because they place a higher tax burden on the itinerant merchant than on the local merchant.

c. Due process requirements


A state may impose a sales tax only on a transaction that is consummated in the state, but may impose a use tax on sales made outside the state if the goods are thereafter brought into the state. If the seller has sufficient contact with the state imposing the use tax, that state may require the seller to collect the use tax and remit the proceeds to the taxing state.

d. Equal protection or privileges and immunities


Net income taxes on out of state residents are constitutional if they are apportioned to the income earned within the state, and are assessed on the same basis as resident income taxes. Since foreign corporations are not citizens for purposes of the Privileges and Immunities Clause, a state can exact a discriminatory fee for the right to do business in the state. Once a foreign corporation is qualified to do business in the state, however, the state may not tax it in a discriminatory manner. As expressed in the Tenth Amendment, the federal government is one of specific delegated powers, leaving the residuum of sovereignty in the states.

3. Specific Constitutional Limitations On State Power


The states are specifically forbidden from making treaties with other countries, and impliedly forbidden from interfering with the conduct of foreign relations by the federal government. States may not establish a monetary system, pass bills of attainder, or pass ex post facto laws. Without congressional authority, they may not wage war, maintain a peacetime army, enter into a compact with another state or foreign country, or tax imports and exports. States lack the power to increase the qualifications for Congress set forth in the Constitution and therefore could not impose term limits on senators and representatives elected as part of the state's congressional delegation.

C. NATIONAL POWER TO OVERRIDE STATE AUTHORITY 1. Direct Conflict


A state statute that conflicts with an act of Congress is invalid under the Supremacy Clause.

2. State Act Enhances Federal Policy


If a state act furthers a congressional policy, it is unlikely that it will be invalid unless Congress has preempted the field.

3. State Act Contravenes Federal Policy


On the other hand, a state act that contravenes federal policy will be invalid, even if Congress did not preempt the field.

14

MicroMash MBE In Brief: Constitutional Law

Even if there is no conflict, a state statute will not be vatic it it attempts to legislate in a field in which Congress has intended to exercise exclusive legislative power. Since preemption is a matter of congressional intention, it can either be explicit or implied in congressional legislation. The factors that will help a court find preemption are that the legislation is in a field not traditionally controlled by the states, the federal legislation is very detailed, the federal government has a substantial interest in the field regulated, and there is a need for a uniform system of national regulation. 5. Congressional Authorization For Otherwise Impermissible State Activity Congress can specifically permit the states to regulate interstate commerce, and can nullify a court decision that prohibits states from acting because of the negative implications of the commerce power. D. RELATIONS AMONG STATES 1. Interstate Compacts Interstate compacts are agreements between states concerning cooperative efforts at various governmental functions. Article 1, 10, Clause 3 provides that a state is forbidden to enter into such a compact without approval of Congress. Congressional consent is not necessary for every agreement between states, on those situations where the interstate compact "tends to increase the political power of the states that may encroach upon the supremacy of the United States." For example, an agreement that vested a tri state governmental body with regulatory powers that ordinarily would be exercised by the federal government would require congressional approval. 2. Full Faith And Credit Article IV, Section 1 of the United States Constitution (the Full Faith and Credit Clause) requires that "Full Faith and Credit shall be given in each State to the public acts, records and judicial proceedings of every other State." "Public acts" include both statutes and case law. The Full Faith and Credit Clause has often been interpreted narrowly, allowing the forum state to apply its own statutes as long as the forum has a legitimate interest in applying its law and has sufficient contacts with the parties or the subject of the litigation. The Full Faith and Credit Clause does not require a state to apply another state's laws in violation of its own legitimate public policy. a. Enforcement of foreign judgments Under the Full Faith and Credit Clause, a judgment rendered by a court in State X must be given the same effect in State Y that it would have in State X. At common law, a second suit was necessary in State Y in order to establish the original judgment as enforceable there. Many states, however, have adopted the Uniform Enforcement of Foreign Judgments Act, which provides in numerous instances for a registration of the foreign judgment in the second state and thus eliminates the necessity of a second suit to enforce the judgment.

MicroMash MBE In Brief: Constitutional Law

15

A "foreign judgment" generally means any judgment, decree, or order of a court of the United States or of any other court which is entitled to full faith and credit in a sister state. A judgment that is not on the merits will be recognized in other states only as to the issues actually decided.

b. Defenses to recognition or enforcement


Fundamentally, the law of another state or another country has no extraterritorial effect. Such foreign law will generally be recognized and given effect extraterritorially, however, subject to certain limitations. A state may, under certain circumstances, refuse to recognize or enforce foreign laws or rights and liabilities based thereon on the ground that the law in question is a penal or revenue statute, that it violates the public policy or positive law of the forum state, or that to give effect to the foreign law would prejudice the state's own rights or the rights of its citizens. Furthermore, a foreign judgment will not be recognized or enforced where it was rendered without proper judicial jurisdiction or without requisite due process.

IV. INDIVIDUAL RIGHTS


A. STATE ACTION AND THE ROLE OF THE COURTS
The Fourteenth Amendment applies only to the states. Individuals do not have to comply with due process or equal protection standards. The problem for the Court is to determine when state participation with individuals, or the quality of the individual's activity, makes it appropriate to subject an individual to Fourteenth Amendment requirements.

1. State Agency
The Fourteenth Amendment applies to the state, its officers and agents, and all of its political subdivisions.

2. Public Function
The Fourteenth Amendment is applicable to actions of private individuals who act without state involvement when the private activity takes over what is essentially a public function, such as running a company town or conducting an election.

3. State Involvement
When the state acts in concert with individuals, the Court strikes a balance in determining whether there is state action, and is more likely to find state action in cases of racial discrimination than in violations of the First Amendment or of procedural due process.

a. Judicial involvement
Shelley v. Kraemer found state involvement in the enforcement by judicial decree of a racially restrictive covenant, but the implications of this decision have not been developed. State action is not found in the mere enactment of a statute giving a lienholder a right to "self help" upon default by the debtor.

16

MicroMash MBE In Brief: Constitutional Law

b. Business involvement A partnership-like arrangement between the state and an individual is sufficient involvement for state action. c. State services vs. subsidies The furnishing of services on a non-exclusive basis to an individual or entity is not state action, but a state subsidy (i.e., special, exclusive aid) to a racially restricted school, or permitting such a school to use state facilities on an exclusive basis, is state action. d. State licensing and regulation The fact that a state licenses or regulates an industry does not make the business of that industry state action. e. State encouragement The repeal of a fair housing law by referendum and the prohibition by constitutional amendment of a new fair housing law is state encouragement of racial discrimination, and constitutes state action.

B. DUE PROCESS
The First, Fourth, Fifth, and portions of the Sixth and Eighth Amendments to the Constitution have been incorporated into the Due Process Clause of the Fourteenth Amendment by a standard which makes guarantees that are fundamental to the American scheme of justice applicable to the states. Citizens, aliens, and corporations are "persons" within the meaning of the Due Process Clause.
1. Substantive Due Process a. Economic interests

The substantive Due Process Clause is no longer a limitation on the power of the legislature to enact economic legislation, unless the statute is arbitrary and unreasonable and has no legitimate purpose. b. Family and privacy interests When legislation affects personal and family rights such as the right to marry, the right in limited instances to privacy, or the right to an abortion, the legislature must justify the legislation by a compelling state interest or it will violate substantive due process. c. Retroactivity Statutes that retroactively deprive individuals of vested economic rights violate the Due Process Clause.

MicroMash MBE In Brief: Constitutional Law

17

d. Vagueness in criminal statutes Due process also requires that criminal statutes be specific enough so that an individual knows what conduct is prohibited before she takes action. Court opinions defining vague statutes can cure vagueness for future cases, but not in the case in which they are announced. 2. Takings Both the federal government and state governments and instrumentalities of both have the power to take private property by eminent domain. However, the Fifth Amendment to the Constitution prohibits the United States from taking private property for public use without just compensation. That amendment is applicable to the states through the Fourteenth Amendment. a. Requirement of a public purpose A purported taking is invalid if it is not for a public purpose. However, the court has construed a "public purpose" broadly, as coterminous with the scope of the sovereign's police power. If the taking is held invalid because it did not serve a public purpose, the landowner is entitled to damages for the temporary taking during the period when he was unable to use his property because of the invalid taking. b. What constitutes a taking? Not all actions by the government that regulate the use of land and in many instances substantially diminish its value are takings for which the government must pay compensation. Zoning ordinances, environmental protections laws, and landmark preservation are usually found to be valid regulations under the police power instead of compensable takings. There are a number of situations, however, where the regulations constitute a taking. Any physical intrusion on private property by the government or the establishment of a non-possessory property interest such as an easement constitutes a taking. A land use regulation, even if enacted for valid police power purposes will constitute a taking if it deprives the owner of all economically viable use of the land. The only basis upon which the state could justify a regulation depriving the land of all economic value under the police power is to prove that the building on the land would constitute a common law nuisance. To be valid regulation instead of a taking, the regulation must also substantially advance the governmental objective being pursued and there must be a tight fit between the regulation and the governmental interest. While the government can condition a permit upon the landowner providing some kind of public benefit, the benefit which the landowner is to provide must be roughly proportional to the burden which she is placing upon the public by obtaining the permit.

18

MicroMash MBE In Brief: Constitutional Law

3. Procedural Due Process The state must afford some form of procedural due process when it interferes with a property right or the liberty of a person, in order to ensure the accuracy of the determination by the government official and to avoid arbitrary government action. a. When must the government conform to due process requirements? 1) Loss of liberty Due process must be followed when an individual is tried under the criminal justice system, when it revokes parole or probation, or cancels the "good time" credits of a convicted criminal. In determining whether a sentence is to be commuted or parole is to be granted, due process safeguards must be maintained when an expectancy, rather than possibility, of commutation or parole is created. Civil commitment proceedings and infliction of physical punishment are deprivations of liberty interests necessitating procedural due process. 2) Fundamental constitutional rights The government must observe procedural due process when it regulates fundamental constitutional rights, such as the control of speech through obscenity laws, or the right of association in the family unit. 3) Property interests The right to drive an automobile is a protected property right, not a privilege. The government may not terminate an individual's participation in a welfare program without a hearing but may legislatively modify such benefits. A person holding government employment under an express or implied tenure possesses a protected property interest, but there is no property right to have the government renew a fixed term contract. State law will ordinarily determine if there is a protected property right. A government employee whose contract is not renewed because of alleged exercise of First Amendment rights is entitled to a hearing in which the state must show that it would have terminated his employment even if he had not attempted to exercise those rights. The temporary deprivation of property through prejudgment attachment is interference with a property right that requires notice and hearing. Damage to reputation alone is not a protected property right, but a hearing is required if damage to reputation is coupled with damage to another interest. 4. What Process Conforms To The Requirement? The three factors determining the amount of process required are: (1) the private interest affected by the official action; (2) the risk of erroneous action through the procedures used, and (3) the governmental burden in providing process. At a minimum, procedural due process requires that the affected person be given notice of the official action to be taken against her and an opportunity to be heard. The hearing may promptly follow instead of precede the official action if there is sufficient reason for this procedure. If the deprivation is more substantial, such as the revocation of parole,

MicroMash MBE In Brief: Constitutional Law 19

of charges, the opportunity to confront the evidence against one, the opportunity to present evidence, and the right to a neutral factfinder who makes written findings. Arbitrary presumptions that prevent persons deprived of liberty or property rights from presenting their claims are likewise unconstitutional. C. EQUAL PROTECTION OF THE LAWS
The purpose of the Equal Protection Clause is to ensure that the government will treat similar persons in a similar manner. Legislation rarely achieves this goal, however, because a statute places individuals in a class who should not be there, or fails to include persons who should be. The crucial test for equal protection is the degree of tolerance that the Court will afford to legislation failing to classify persons appropriately to achieve a governmental purpose. The Equal Protection Clause applies to the states because it is part of the Fourteenth Amendment, and to the federal government through the Due Process Clause of the Fifth Amendment. Citizens, aliens, and corporations, provided that they are qualified to do business within a state, are protected by the Equal Protection Clause. A statute many times will classify explicitly, but those who administer a statute may make classifications not present in the statute itself. A statute, even if neutral on its face and in its application, may have a discriminatory purpose and effect.

there are additional procedural requirements. These requirements include written notice

1. Fundamental Interests
If a fundamental interest is affected by the classification, the strict scrutiny test is applied. The state, not the plaintiff, must demonstrate that the classification is necessary to satisfy a compelling state need. Fundamental interests include the right to vote, the right to fair representation, the right to be a candidate, the right to travel, rights as a criminal defendant, and the right to marry.

a. Regulation of voting and legislative representation 1) The right to the franchise


While the state has the right to limit the franchise to residents of the community, it may not impose a durational requirement of more than fifty days. It may not impose property or interest qualifications, nor may it condition the right to vote upon the payment of a poll tax. While the constitutionality of literacy tests has been upheld, their operation has frequently been suspended by congressional action.

2) The right to be a candidate


The right to be a candidate and have one's name on the ballot is subject to more substantial governmental restrictions than the right to vote. The state may impose durational residency requirements, filing fees, and petitions signed by substantial numbers of voters. However, the Court will scrutinize the burdens and hold them unconstitutional if they unreasonably restrict ballot access.

20

MicroMash MBE In Brief: Constitutional Law

3) The right to fair representation A voter has a fundamental right to have his vote count as much as that of any other voter in the election of a legislative or governmental body. Legislative districts in both the House of Representatives and in the state legislature must therefore be uniform in size. A higher degree of mathematical precision is required in congressional districting, however, than for state elections. The "one person, one vote" principle applies to municipal elections, but not to the election of executives, to special purpose districts, or to purely administrative bodies. Flagrant racial or political party gerrymandering will be found unconstitutional. Multi-member districts are constitutional so long as they are not designed to reduce minority representation. States lack the power to increase the qualifications for Congress set forth in the Constitution and therefore could not impose term limits on senators and representatives elected as part of the state's congressional delegation. b. Other fundamental interests Because the right to travel is fundamental, discrimination in the payment of welfare benefits on the basis of length of residency is unconstitutional. In criminal cases, the state must afford the indigent defendant counsel for at least one appeal, and a free transcript for that appeal. It may not imprison a defendant for failure to pay a fine. The right to marry is also a fundamental interest. 2. Nonfundamental Interests Under ordinary or lower-tier equal protection standards, the Court will not substitute its judgment for that of the legislature. The legislature may fail to include in a class those who should logically be in, and include those who should logically be left out. Mathematical certainty is not required, and some unfairness will be tolerated so long as the classification is reasonable. a. Economic regulation Economic regulation is judged by the rational purpose test, and is rarely held unconstitutional under the Equal Protection Clause. b. Social welfare legislation When no fundamental interest is affected, the Court has applied the rational basis test to governmental benefits. 3. Suspect Classifications a. Race A classification by race, even if it treats all races equally, is suspect and will not be sustained in the absence of a compelling state interest. Schools classified by race under state law must be desegregated immediately. The Court will devise a desegregation plan using general equitable powers if the school board fails to do so. Once a racially neutral plan is in place, the plan need not be altered to meet changing

MicroMash MBE In Brief: Constitutional Law

21

housing patterns. School districts that have segregated schools because of segregated housing patterns are not in violation of the Constitution unless a discriminatory purpose can be found. If part of a school district is deliberately segregated, there is a presumption of unlawful purpose in the rest of the school district. A desegregation remedy cannot include districts beyond those engaged in unlawful segregation. When race is used as a criterion for governmental action, it must meet the strict scrutiny standard and will not be upheld unless the remedy is closely tailored to remedy the effects of past discriminations.

b. Alienage
Discrimination against aliens is permitted by the federal government under its power to further foreign policy, and by state governments to prevent aliens from entering the governmental process and those non elective government jobs which formulate or execute public policy. However, a state may not discriminate against aliens with respect to education, public welfare, or the right to earn a livelihood or engage in the "learned professions." Discrimination against illegal aliens in these matters is subject to an intermediate level of review.

c. Quasi-suspect categories
An intermediate standard of review is applied when the legislature uses sex or legitimacy as a classification. Under this standard the governmental objective must further important governmental purposes, there must be a close relationship between the classification and its purposes, the legislature must specify the valid objectives which it is trying to further, and the legislature must tailor the classification to achieve the objective.

1) Illegitimacy
Illegitimacy is not a truly suspect classification, but the state interest in promoting morality or the unity of the traditional family is not sufficient to justify discrimination against illegitimate children. Statutes denying illegitimate offspring rights to Workers' Compensation benefits, wrongful death benefits, and intestacy benefits when the parent child relationship is readily proven have been held unconstitutional. However, a state may distinguish among different classes of illegitimate offspring based on whether paternity was acknowledged or adjudicated to promote the orderly disposition of an estate.

2) Gender
Discrimination on the basis of sex is valid if it serves important governmental purposes, and is substantially related to those purposes. Classifications based upon the notion that a female is to stay in the home and raise children are invalid, but those classifications designed to remedy past discrimination have been upheld.

4. Nonsuspect Categories
Poverty, age, and mental retardation are not suspect categories.

22

MicroMash MBE In Brief: Constitutional Law

D. PRIVILEGES AND IMMUNITIES CLAUSES


1. Under The Fourteenth Amendment The Fourteenth Amendment Privileges and Immunities Clause only protects matters which are incidents of national citizenship, such as the right to travel from state to state, to vote in national elections, and to petition Congress. 2. Under Article IV, 2 The Comity Clause The Comity Clause protects a citizen of one state from discrimination on that basis in another state in fundamental matters. To justify discrimination on a fundamental issue, the state must show a substantial relationship between the discrimination against the nonresidents and the problems caused by them.

E. THE OBLIGATION OF CONTRACTS, BILLS OF ATTAINDER, AND EX POST FACTO LAWS


1. Obligation Of Contracts While the Constitution flatly prohibits states from impairing the obligation of contract, the Court has permitted states to construe contracts very narrowly in favor of the government, and has permitted legislation to affect the remedy for breach of contract so long as there is some means of vindicating the right. States may impair the obligations of contracts if there is a valid police power reason. However, a more stringent standard is applied when a state attempts to avoid the obligation of a contract to which it is a party. Such an impairment will be upheld only if less drastic means were not available, and the reason for the impairment was not foreseen at the time of the contract. A state statute imposing substantial burdens on one contracting party that is enacted for the benefit of a narrow group will also violate the Contract Clause. 2. Bills Of Attainder States and the federal government are forbidden to pass a bill of attainder, Article 1, 9 and 10. A bill of attainder covers legislative acts which applies either to named individuals or to easily ascertainable members of a group in such a way as to inflict punishment on them without a judicial trial. Legislation prohibiting specific government employees from being paid constitutes an unconstitutional bill of attainder. Legislative punishment also includes legislation whose purpose is deterrent or preventative. General regulations that do not target specific individuals, however, are not within the constitutional ban. 3. Ex Post Facto Laws Article 1, 9 and 10 of the Constitution prevent both the federal government and the states from passing an ex post facto law that is, a law which has a retroactive punitive effect. A law is an ex post facto law if it (1) creates a new crime or alters the definition of an existing crime so that an act which was not criminal at the time the statute was enacted is made criminal, 2) redefines a crime so that an act which was criminal at the time the act was passed is made a more serious crime, 3) increases the punishment for an act which was a crime at the time the act was passed, or 4) changes the rules of evidence

MicroMash MBE In Brief: Constitutional Law

23

which permits a conviction for an existing crime on lesser evidence than was required at the time the act was committed. The ex post facto prohibition applies to criminal conduct only. Any punishment in the form of imprisonment or a criminal fine would constitute punishment for criminal conduct. However, a statute that retroactively provides that a felony conviction disqualifies an individual from a professional license or subjects her to deportation does not constitute an ex post facto law. Statutes that increase the penalties for individuals who have had previous convictions and who commit a crime after the law was enacted do not constitute ex post facto laws even thought the crimes which serve as the basis to increase the penalty were committed before the law increasing the punishment was enacted.

F. FIRST AMENDMENT FREEDOMS


1. Freedom Of Religion And Separation Of Church And State There are two different prohibitions on the power of Congress with respect to religion contained in the First Amendment and carried over to the states through the Fourteenth Amendment. The Establishment Clause focuses upon government aid to religion and whether such aid promotes particular religious beliefs. The Free Exercise Clause prohibits governmental encroachment on an individual's religious freedom. a. The Establishment Clause The present tests to determine if governmental activity is constitutional under the Establishment Clause, all of which must be satisfied, are: (1) Does the activity reflect a secular legislative purpose? (2) Does it have a primary effect that neither advances nor inhibits religion? (3) Does it avoid excessive entanglement between church and state? Under the above tests, the loan of textbooks to parochial school pupils, college dormitory construction, and Sunday closing laws have been upheld, but prayers in public schools, payment of the teachers' salaries, and tax credits only available to parents of parochial school children have been held unconstitutional. b. Free exercise of religion 1) Freedom of thought or belief The state does not have the power to regulate or prescribe manifestations of thought or belief. 2) Regulation of activity based upon belief The state must justify regulation of activity based upon religious belief by a compelling state need. However, if religious belief engenders activity harmful to society, regulation of such conduct will be permitted. The time, place, and

24

MicroMash MBE In Brief: Constitutional Law

manner of religious activity can be regulated by narrowly drawn statutes that do not discriminate on the basis of religious belief. While the state has an interest in compulsory education for children, it cannot require a public school education, nor require education beyond the eighth grade if, for example, the agrarian lifestyle of a religious community would render additional education detrimental in the eyes of that religious community. Doorto-door solicitation to publicize religious beliefs and the incidental sale of religious literature are also constitutionally protected activities. While Congress need not exempt religious objectors from the draft, the state cannot inquire into the reasonableness of those beliefs if they are made a ground for exemption; only the sincerity of the belief may be challenged. 3) Judicial resolution of church disputes Courts may not decide ecclesiastical questions to resolve secular disputes concerning internal church management or church property. 2. Freedom Of Expression And Association The First Amendment provides in part that "Congress shall make no law abridging the freedom of speech or of the press; or the right of the people peaceably to assemble and to petition the government for a redress of grievances." The guarantees of the First Amendment are protected from abridgment by the states by the Fourteenth Amendment. Speech is protected by the Court because uninhibited speech is essential to the political process, because truth is more likely to emerge if all ideas may be freely expressed, and because free speech is an end in itself in a civilized society. a. Regulation of the communicative aspects of expression 1) What constitutes speech? In many cases, speech is exercised in conjunction with action. If that action is primarily a form of communication (a substitute for words, such as wearing a black armband), it will be considered symbolic speech. If the action is coercive, such as picketing to prevent ingress and egress, the state power to regulate the speech coupled with action is significantly greater than it is to regulate pure speech. Even though the action is intended to be symbolic speech, it can be regulated to protect valid governmental policy divorced from the content of the symbolic expression itself. 2) What constitutes regulation of content? If the government classifies speech according to its content, requires individuals to promulgate speech, or places a dollar limitation on the amount to be spent to proliferate speech in a political campaign, it is regulating speech by content.

MicroMash MBE In Brief: Constitutional Law

25

3) Content regulation of protected speech must be justified by compelling state interest


Regulation of speech by content is generally invalid unless: (1) the state can show a compelling need closely related to the regulation, or (2) the speech can be shown to be beyond the protection of the First Amendment or entitled only to marginal protection.

(a) Clear and present danger


Speech in the political sphere can be regulated only when the speech creates a "clear and present danger" of the evils that the state has a duty to prevent. Congress cannot regulate the content of speech when the speaker is advocating abstract doctrine, but can regulate speech that advocates illegal action. In its most recent formulations, the Court has held that before the state can proscribe the content of speech, it must be directed toward inciting imminent lawless action and must be likely to incite that action. Regulation of speech merely because it is likely to provoke a hostile reaction from an unreceptive audience is rarely justified. The government has an affirmative obligation to protect a speaker before a hostile audience, but can require him to stop speaking if there is a genuine likelihood of immediate violence.

(b) The administration of justice as a compelling state need


The clear and present danger test has been applied to state laws prohibiting the criticism of judges. An order by a judge to prevent the press from publishing statements made by the accused is unconstitutional unless the gravity of the evil, discounted by its improbability, justifies the invasion of free speech. Although the right of access to criminal trials is not absolute, denial of the right must be necessitated by a compelling governmental interest and must be narrowly tailored to serve that interest. The First Amendment does not create a privilege in news reporters to refuse to disclose the source of their information in the course of a legitimate, good faith grand jury investigation. The state may not prohibit a witness from divulging her own grand jury testimony after the grand jury proceedings are concluded when there is no longer a compelling need to preserve secrecy.

(c) The election process as a compelling state need


A limitation on the total amount a candidate can spend on his own election, or on the amount that an individual might spend on political activities or on contributions to support a ballot measure, is unconstitutional; however, a limitation on campaign contributions, a restriction on campaigning near polling places, and required contribution disclosures serve the important state purpose of preserving the integrity of the electoral process.

26

MicroMash MBE In Brief: Constitutional Law

b. Regulation of the time, place, and manner of speech The government has substantially greater latitude in regulating the time, place, and manner of speech than in regulation of its content. The interests to be balanced are the state's requirement of public peace and the orderly flow of transportation and the method used to regulate speech, versus the means and location used by the individual to promulgate the speech. 1) The nature of the forum (a) Public forum If a speaker desires to communicate in a public forum, the state cannot completely prevent such speech on the ground that an alternative forum is available. Public parks and streets are areas traditionally used to express views, and only a narrow type of restriction on speech is permitted. (b) Semipublic or private forum The use of property open to the public, but which is not traditionally used for speech related activities, such as schools, libraries, and courthouses, can be regulated to prevent the speech from interfering with their governmental functions. Jails, private government offices, and military bases are closed areas where the government need not grant access or free speech rights. Private property that is not within the state action concept can be regulated by the owners without regard for the rights of others to speak. 2) The interests of privacy and tranquillity as limitations on the time, place, and manner of speech The government has the right to limit the use of loudspeakers in residential areas under narrowly drawn statutes, but cannot completely ban door-to-door solicitation. A statute requiring a mailer to remove an addressee's name from a mailing list is constitutional. However, passengers in a public bus have no constitutional right to prevent the broadcast of unwanted commercials. 3) Licensing as a control of the time, place, and manner of speech A licensing statute controlling speech that confers excessive discretion on a public official is overbroad and invalid on its face. Licensing statutes cannot discriminate on the basis of the content of speech. If the statute is impermissibly overbroad or vague, failure to apply for a license does not prevent a challenge to the constitutionality of the statute; however, one must apply for a license under a narrowly drawn statute before a defense that the statute is unconstitutional as applied will be sustained. One cannot successfully challenge the constitutionality of a statute in a prosecution for contempt for violation of an injunction brought pursuant to that statute. 4) Regulation of the media While the state may not require that a newspaper give individuals a right to publish in that paper, it may require the electronic media to give persons whom it

MicroMash MBE In Brief: Constitutional Law

27

attacks a right to reply. The difference in result is because there is a finite supply of radio and television frequencies. However, there is no constitutional right to purchase air-time to promulgate a message. Because of the ability of radio broadcasts to intrude into the home and be heard by the young, the government can prohibit material that is offensive but not obscene from being broadcast during daytime or early evening hours. c. Regulation of unprotected speech Traditional First Amendment analysis classifies certain categories of speech as unprotected by the First Amendment and subject to any rational regulation. This concept has eroded so that almost all content-related regulation of speech must be related to an important governmental purpose. 1) Obscenity If speech is found obscene, it is not entitled to First Amendment protection. To be obscene, the challenged material: a. must appeal to the prurient interest of an average person applying contemporary community standards; b. must depict or describe sexual conduct in a patently offensive way; and c. must, taken as a whole, lack serious literary, artistic, political, or scientific value. A jury will decide each of these matters as a question of fact, but will not have unbridled discretion. Immorality, sexual expletives, and nudity by itself are not necessarily obscene. An individual can be convicted for advertising in a manner designed to appeal to the prurient interest, even though the material itself is not obscene. The right to privately possess material depicting or describing adults engaged in obscene acts is constitutionally protected, but the right to show it to consenting adults or to sell it is not. Private possession of child pornography may be prohibited. The state also has greater rights to regulate the content of sexual material exhibited to minors than it does to consenting adults. Before the state can seize allegedly obscene material, there must be authorization for seizure by a judicial officer that specifically identifies the material to be seized, so that the officer has little or no discretion to determine what material shall be seized. The Court has permitted content regulation of adult motion pictures and nude dancing which were not obscene, in a manner similar to time, place, and manner regulations. Child pornography is unprotected by the First Amendment. States have greater leeway in regulating child pornography because of its harm to the children

28

MicroMash MBE In Brief: Constitutional Law

involved and because its value is de minimis. Such pornography need not be obscene under the Miller standard to be regulated. 2) Other unprotected speech, fighting words, and offensive speech Words which by their very utterance tend to incite an immediate breach of the peace are not protected by the First Amendment. However, a speaker cannot be punished for words that are offensive but do not tend to invoke an immediate breach of the peace. d. Regulation of commercial speech Corporations and other business associations are entitled to the protection of the First Amendment when speaking on issues of political or public interest. However, "commercial speech" does not enjoy the same amount of First Amendment protection of content as that enjoyed by other forms of speech. It is subject to reasonable regulation for the protection of consumers and other legitimate governmental interests, but the outright prohibition of advertising in the professions and price advertising on drugs has been held unconstitutional. e. Regulation of or imposition upon public employment, licenses, or benefits based upon exercise of expressive or associational right 1) Public employment The government may not condition public employment upon the waiver of First Amendment rights. It may deny employment based upon membership in an association only if an individual is an active member of a subversive association, knows the illegal ends of the organization, and has a specific intent to further those ends. If an oath is a requirement for public employment, it must be clear, concise, and narrow in scope, or must merely pledge allegiance to constitutional processes. A prospective employee cannot be required to take an oath concerning past conduct in joining an organization, unless the oath specifies that it was knowing membership with specific intent to further illegal purposes. The employee may not be dismissed for a claim of Fifth Amendment rights. However, if she is given immunity and still persists in claiming the privilege against selfincrimination, she may be discharged. Information given pursuant to a grant of immunity may be used to terminate employment. 2) Admission to the bar The state has a right to inquire into the character of candidates for admission to the bar, but may not deny admission for political association unless the candidate knowingly belonged to a subversive organization with a specific intent to further its ends. The state may ask a bar candidate questions concerning his knowing membership in subversive organizations, even though he did not have specific intent to further the purposes of those associations.

MicroMash MBE In Brief: Constitutional Law

29

f. Freedom of association
The First Amendment protects the right to join with others in the exercise of First Amendment rights. Mere membership in an organization cannot be made criminal. When disclosure of membership would destroy the right of association, the Court has limited the right of the government to obtain membership lists. The freedom of association includes a right not to associate with certain types of persons. An organization may exclude selected groups if the organization is relatively small, the purpose of the organization is exclusive, and nonmembers cannot participate in critical activities of the association.

g. Regulation of defamation and invasion of privacy 1) Public officials and public figures
The First Amendment only applies to defamation in the form of libel against media defendants. State law governs libel and slander actions against private individuals. New York Times v. Sullivan holds that for a "public figure" to prevail against a media defendant, he must show that the falsehood was published either with knowledge that it was false or with reckless disregard for whether it was false. Public figures are either public officials or persons who have achieved substantial fame or notoriety (public figures in all contexts), or persons who inject themselves into a public controversy (public figure only with respect to that controversy). Once a public figure satisfies the New York Times test by proving "malice," he may collect any damages allowed under state law, including punitive damages.

2) Private figures
Libel suits by private figures on matters of public concern are governed by Gertz v. Robert Welch. The plaintiff may not recover on strict liability, and in most cases must prove negligence. She may recover only actual damages (special or out-of-pocket losses, plus loss of reputation, humiliation, and mental suffering). She may not recover punitive damages without proof of malice. If the defamation does not involve matters of public concern, however, a private figure can recover presumed or punitive damages without proof of actual malice.

h. Procedural problems peculiar to the first amendment 1) Prior restraint


If the government restrains speech prior to the time it is published, the government regulation will undergo more searching scrutiny than if the speech is punished after it has taken place. Any system of prior restraint of expression comes before the Court with a heavy presumption against its constitutional validity.

2) "Overbreadth"
A statute will violate the First Amendment if, in its attempt to regulate speech in a permissible manner, it also regulates protected speech. This overbreadth concept

30

MicroMash MBE In Brief: Constitutional Law

will permit an attack upon a statute on its face rather than as it applies to the particular litigant, because a statute restricting speech that is overbroad has a chilling effect upon the attempt to exercise First Amendment freedoms. The Burger Court restricted the scope of the overbreadth doctrine by holding that an overbreadth attack on a statute regulating conduct as well as speech would only prevail when the overbreadth was substantial, and when the statute could not be narrowed by construction. The reason a defendant is able to raise the defense of another party to a statute is that the person whose First Amendment rights are affected by an overly broad statute is unlikely to litigate the matter himself. The overbreadth doctrine does not apply to commercial speech. 3) "Vagueness" A statute can be challenged for vagueness if it does not specify in precise terms the activity that is forbidden. Vagueness differs from overbreadth as follows: a statute is vague when it does not precisely define an activity that is prohibited, whereas an overly broad statute prohibits activity which is protected by the First Amendment. An individual does not have the right to raise the argument that a statement is vague with respect to a third party but not to herself. A hard core violator to whom the statute is not vague cannot raise a vagueness defense. A court can construe a statute so that its scope is precise, and subsequent litigants will not be successful in a challenge on the ground of vagueness.

MicroMash BAR REVIEW BAR EXAM ALERTS AT-A-GLANCE CONSTITUTIONAL LAW

I. THE NATURE OF JUDICIAL REVIEW


A. ORGANIZATION AND RELATIONSHIP OF STATE AND FEDERAL COURTS IN THE FEDERAL SYSTEM 1. FEDERAL COURT JURISDICTION
A federal court has discretion to abstain from deciding an issue of state law if a decision by a state court on the state issue might obviate the need for a decision on a federal constitutional issue. A federal court will abstain from a case asking for an injunction against the enforcement of a state criminal statute if a prosecution under that statute has commenced.

2. STATE COURT JURISDICTION


Congress can require state courts to hear causes of action based upon federal statutes. State governments or agencies are not citizens of a state for the purpose of federal diversity jurisdiction.

A private citizen cannot sue a state in a federal court.


A private citizen can challenge the constitutionality of a state statute in a federal court by suing a state officer to enjoin the enforcement of the statute on the ground that it is unconstitutional. Political subdivisions of a state can be sued by citizens in federal court because they do not enjoy the protection afforded a state under the Eleventh Amendment.

B. SUPREME COURT JURISDICTION


A state has the right to sue another state in the United States Supreme Court on behalf of its citizens on claims affecting a multiplicity of citizens (the parens patriae doctrine).

MicroMash MBE In Brief: Constitutional Law Bar Exam Alerts

There is no direct right of appeal to the Supreme Court from a federal district court decision holding an act of Congress unconstitutional.

C. CONGRESSIONAL POWER TO DEFINE AND LIMIT JURISDICTION


Congress has control of the jurisdiction of the federal courts and can establish or abolish lower federal courts. Congress cannot alter the jurisdiction of the Supreme Court in such a way as to interfere with the Court's essential function of preserving constitutional order. Congress cannot interfere with inherent judicial functions in courts it has created. Congress can set up courts pursuant to its powers under Article I of the Constitution. Judges of such courts are not constitutionally entitled to life tenure. Due process requires that there must ultimately be a right of appeal to an Article III court from the decision of an Article I court or an administrative body.

D. JUDICIAL REVIEW 1. STANDING A person has standing by virtue of being a taxpayer only to challenge
legislation authorizing expenditures on the basis that those expenditures contravene specific constitutional limitations on the spending power. The only specific constitutional limitation on the spending power ever found has been through the Establishment Clause of the First Amendment.

A mere philosophical, ethical, or intellectual interest in the outcome of a case is not sufficient to qualify for standing. Standing exists in a party that has a close relationship to the party actually injured if the injured party is unlikely to successfully assert its rights.

2. MOOTNESS AND RIPENESS A case will not be dismissed for mootness if the issue is capable of repetition
and will consistently evade review.

A case will be dismissed as not ripe if events that will raise material issues in
the case have not yet occurred.

3. POLITICAL QUESTIONS AND JUSTICIABILITY


Only the president has the right to appoint officers of the United States, and an attempted appointment by Congress or by members thereof is unconstitutional.

The Supreme Court has the right under the Constitution to decide which branch of government is vested with final authority to decide a particular matter.

MicroMash MBE In Brief: Constitutional Law Bar Exam Alerts

While the Senate has the right to advise and consent on presidential appointments, it does not have the right to advise and consent when the president removes officers of the executive branch. Under the political question doctrine, the United States Supreme Court will not review an issue on the merits if it determines that the Constitution places final authority to resolve the issue in another branch of government.

4. SUPREME COURT REVIEW OF STATE COURT DECISIONS


If a case has been decided by a state court on an independent state ground, there is no jurisdiction for Supreme Court review, even if the state court decides a federal issue in the case that is not essential to the decision. If, on the other hand, the state court decides a state issue on the basis of federal decisions on the same point, then the state ground is not independent and there is a basis for Supreme Court review. There is no right to appeal a state court advisory opinion to the United States Supreme Court, even if it involves federal constitutional issues, because there is no case or controversy as required by the United States Constitution for federal court jurisdiction.

5. CONSTITUTIONAL LITIGATION
If constitutional litigation involves the strict-scrutiny tier of equal protection, the denial of substantive due process rights that are highly protected, or the deprivation of the right of free speech or of freedom of religion, the state must show: (1) a compelling state need and (2) that no less burdensome method would achieve that objective. If constitutional litigation involves sexual discrimination, the state must show that the classification has an important governmental objective and is substantially related to achieving those objectives. If constitutional litigation involves matters other than those described above, the plaintiff must prove that the legislation lacked a rational basis. The state never has the burden when only lack of rational basis must be shown, and the plaintiff never has the burden when highly protected rights are involved.

MicroMash MBE In Brief: Constitutional Law Bar Exam Alerts

II. SEPARATION OF POWERS


A. POWERS OF CONGRESS The Supremacy Clause itself is not a source of congressional power.

1. COMMERCE POWER
Congress may exercise the commerce power to regulate purely local activities that have a substantial effect on interstate commerce. Congress may exercise the commerce power to regulate the conduct of private individuals with respect to racial discrimination (even though such private action could not be regulated by legislation under the Fourteenth Amendment) so long as the individual's conduct affects interstate commerce. Congress may delegate rulemaking power to an administrative agency, but cannot reserve to itself the right to change such rules by anything short of legislation adopted by the full constitutional process. 2. TAXING AND SPENDING POWERS Through Congress' power to condition expenditures on compliance with its standards, Congress can persuade the states and individuals to adopt measures that it could not directly require through legislation. While Congress can tax and spend for the general welfare, the General Welfare Clause is not a source of congressional regulatory power. Congress can achieve a regulatory effect through a taxing statute as long as the statute has a revenue-raising purpose. 3. POWER OVER FEDERAL PROPERTY The property power, not the commerce power, is the best source of congressional authority to regulate or dispose of property owned by the United States. Congress holds all of the regulatory power of territories that would be possessed by the state legislature if the territory were a state. Judges appointed to serve in the territories are not entitled to lifetime tenure.

4. POWER TO ENFORCE THE THIRTEENTH, FOURTEENTH, AND FIFTEENTH AMENDMENTS


Congress has power under the Thirteenth Amendment to affect individual conduct, but only to eradicate slavery or the effects of slavery. Pursuant to that authority, it has the power to regulate the manner in which black people are treated.

MicroMash MBE In Brief: Constitutional Law Bar Exam Alerts

Congress has power under the Fourteenth Amendment only to reach state action (or action accomplished under the color of state law) that abrogates the rights guaranteed by that amendment. Congress has power under the Fifteenth Amendment to directly regulate voting procedures in the states for the purpose of eradicating procedures that affect the rights of minorities to vote or to have their vote counted.

5. INVESTIGATORY POWER
Congress has the power to investigate and subpoena witnesses for the purpose of obtaining information with respect to potential legislation that it might pass. An individual can successfully defend against a contempt-of-Congress charge for failing to answer a question from a congressional committee only if the witness can show that the subject matter of the questioning was beyond the power of Congress to pass potential legislation or beyond the scope of the power delegated by Congress to that committee.

B. POWERS OF THE PRESIDENT


The president is obligated to carry out legislation mandating that the president act in a specific manner. The pardon power only extends to federal crimes. Executive privilege is absolute with respect to defense and foreign policy matters. Confidential communications between the president and advisors in all other areas are presumptively privileged; disclosure can be required only when a specific communication is subpoenaed and a substantial governmental interest outweighs the president's interest in nondisclosure.

III. THE RELATION OF THE NATION AND THE STATES

A. INTERGOVERNMENTAL IMMUNITIES
Absent congressional intention to the contrary, states can tax buildings leased by the federal government and contractors doing business with the federal government, as long as such tax is not discriminatory. The federal government has the right to tax and regulate the instrumentalities and employees of state government.

B. AUTHORITY RESERVED TO THE STATES


A state has the right to regulate interstate commerce as long as it does not contravene an express federal policy, does not discriminate against interstate commerce, and does not unduly burden interstate commerce.

MicroMash MBE In Brief: Constitutional Law Bar Exam Alerts

In determining the validity of a state action that burdens interstate commerce, the court will consider whether the state used the least restrictive means to achieve a legitimate state objective. A state regulatory statute that discriminates in favor of local commerce and against out-of-state commerce is unconstitutional because of the negative implications of the Commerce Clause. The negative implications of the Commerce Clause prevent a state from requiring that a resource of the state be sold to in-state customers only. The negative implications of the Commerce Clause prevent a state from excluding trash from a sister state if its landfills accept in-state trash. A state acting in a proprietary rather than a regulatory capacity may discriminate in favor of local business and against interstate commerce. The police power is a source of state power, not a source of congressional power.

C. NATIONAL POWER TO OVERRIDE STATE AUTHORITY


Congress has the right to expressly authorize a state to burden commerce or discriminate in favor of local commerce, even if the Supreme Court has held such burden or discrimination unconstitutional under the negative implications of the Commerce Clause.

IV. INDIVIDUAL RIGHTS A. STATE ACTION AND THE ROLE OF THE COURTS
The Supremacy Clause is the source of constitutional power for a court to hold state statutes and decisions unconstitutional because they conflict with the Constitution, laws, or treaties of the United States or acts done in furtherance of them. If Congress has provided a comprehensive scheme of regulation in an area, Congress may be said to have occupied the field and any state regulation (even if complementary to the federal legislation) will be preempted, unless Congress' intent was to allow state regulation. Congress has the ability to permit states to operate in areas where it has legislated. States may not enact any legislation that affects foreign policy, because foreign policy is the exclusive province of the federal government. State legislation or decisions that are contrary to a federal policy expressed in an executive agreement are invalid.

MicroMash MBE In Brief: Constitutional Law Bar Exam Alerts

A state or municipal law in conflict with a federal regulation dealing with standards applicable to federal offices is invalid because of the Supremacy Clause. The activity of a state in regulating or taxing an activity does not render the activity itself "state action" subject to Fourteenth Amendment scrutiny. The action of any political subdivision of a state constitutes state action. The activities of an entity in which the state has a partnership interest constitute state action. B. SUBSTANTIVE DUE PROCESS When called upon in a multiple-choice question to consider all arguments attacking the constitutionality of a statute, the argument that it violates substantive due process should be considered, especially if it can be shown that the statute operates in an arbitrary and unreasonable manner, and that it is difficult or impossible to find a legitimate reason for the legislature to pass such a statute. An economic regulation violates the substantive strand of the Due Process Clause if there is no rational basis for it. Substantive due process prohibits states from limiting fundamental privacy interests, absent a showing of a compelling state need. The right to use contraceptives and the right of an extended family to live together are examples of such interests. A state activity is no more likely to withstand constitutional challenge because it is part of the state constitution or enacted by referendum. C. PROCEDURAL DUE PROCESS Procedural due process is required only if the action of the decision-maker constitutes state action. An individual has a property interest in continued employment if the individual has an employment contract or tenure. The factors in determining what process is "due" are the type of interest infringed, the likelihood of an erroneous decision, and the burden on the government in providing process. The minimum necessary to satisfy due process is notice and an opportunity to be heard. Criminal statutes violate due process if they are so vague that they do not inform a citizen of the conduct deemed criminal. A judicial construction of the statute can cure the vagueness with respect to future violators, but not with respect to any person charged before the decision was rendered.

MicroMash MBE In Brief: Constitutional Law Bar Exam Alerts

D. EQUAL PROTECTION
The Equal Protection Clause is contained in the Fourteenth Amendment and does not apply to the federal government. However, the principles of equal protection are applied to the federal government through the Due Process Clause of the Fifth Amendment. The "one man, one vote" rule applies to municipal legislative bodies. The state may impose limited residency requirements (e.g., two months) on the right to vote to assure that voters are bona fide residents. The state may impose reasonable requirements regarding filing fees, residency, and petition signatures to achieve ballot access. 1. OTHER (NON-FUNDAMENTAL) RIGHTS Economic regulation need only satisfy the rational-basis standard. The right to be free from poverty is not a fundamental right. 2. SUSPECT CLASSIFICATIONS Neither the state nor the federal government can discriminate on the basis of race except to further a compelling state need. A regulation or decision that classifies on the basis of race in order to remedy specific past racial discrimination is valid. A classification based upon a racially neutral principle such as residence, which also indirectly discriminates by race, is not unconstitutional unless there is an intention to discriminate by race. The federal government has broad discretion to discriminate on the basis of alienage in the furtherance of foreign policy. A state cannot discriminate on the basis of alienage except in elective governmental positions and nonelective governmental jobs that formulate or execute public policy. 3. QUASI-SUSPECT CLASSIFICATIONS Discrimination on the basis of gender is valid only if it serves an important governmental purpose and is substantially related to achieving that purpose. The state cannot deny workers' compensation benefits, wrongful death benefits, or intestacy benefits based upon illegitimacy where the parent-child relationship has been adjudicated or acknowledged, but can make distinctions where proof of the relationship is difficult.

E. PRIVILEGES AND IMMUNITIES CLAUSES The Privileges and Immunities Clause of the Fourteenth Amendment applies only to the privileges of national citizenship and is rarely if ever a valid reason for holding a statute unconstitutional.

MicroMash MBE In Brief: Constitutional Law Bar Exam Alerts

The Privileges and Immunities Clause of Article IV of the Constitution is an alternative analysis where the state discriminates on a matter of fundamental interest in favor of its own citizens and against out-of-staters.

F. FIRST AMENDMENT FREEDOMS


1. FREEDOM OF RELIGION AND SEPARATION OF CHURCH AND STATE
a. THE ESTABLISHMENT CLAUSE
State aid to religions is constitutional only if the activity reflects a secular purpose, it has a primary effect that neither advances nor inhibits religion, and there is no excessive entanglement between church and state. State activity that aids all religions equally can still violate the Establishment Clause. State laws requiring that religious theory be taught in public schools violate the Establishment Clause.

b. FREE EXERCISE OF RELIGION


A state has the right to regulate action based upon religious activity if there is a compelling state need. When religious belief is the basis for resisting government rules (e.g., conscientious-objector status) the courts have a right to examine the sincerity of the belief, but not the belief itself. Courts cannot decide ecclesiastical questions to settle disputes concerning church management or property.

2. FREEDOM OF EXPRESSION AND ASSOCIATION


a. REGULATION OF CONTENT OF EXPRESSION
Action that is a substitute for words can be protected symbolic speech. Even if action is intended as symbolic speech, however, it can be regulated to protect a legitimate governmental interest divorced from the content of the symbolic speech itself (e.g., burning draft cards). All speech is protected speech for purposes of content regulation except fighting words, defamatory speech, obscene speech, and to some degree, commercial speech. Neither the state nor the federal government can regulate the content of protected speech unless it can show a compelling state need. A compelling state need is present and the state can proscribe the content of protected speech that is directed toward inciting immediate lawless action and is likely to incite that action.

10

MicroMash MBE In Brief: Constitutional Law Bar Exam Alerts

Requiring an individual to display a message prescribed by the state is the equivalent of regulating the content of speech. The state has an affirmative obligation to protect a speaker before an audience, but the speaker can be required to stop speaking if there is a genuine likelihood of immediate violence that the state cannot prevent. b. REGULATION OF TIME, PLACE, AND MANNER OF EXPRESSION The state cannot completely prohibit the exercise of free speech rights in a public forum such as streets or parks, but can regulate such speech pursuant to narrowly drawn statutes conferring limited discretion on officials to ban speech at particular times and places and in particular ways, as long as the prohibition of speech does not turn on its content. The state has the right to forbid speech near semipublic forums such as schools, libraries, and courthouses to prevent interference with governmental functions. The state has the right to prohibit the exercise of free speech rights in places closed to the public such as jails, military bases, and private governmental offices. Unless the regulation of speech on private property becomes state action (as in the operation of a company town), the owner of private property can regulate and prohibit the exercise of speech on that property. c. OBSCENITY Speech is obscene and subject to complete prohibition if it appeals to the prurient interest of an average person applying contemporary community standards, depicts or describes sexual activity in a patently offensive way, and taken as a whole, lacks serious literary, artistic, political, or scientific value. Child pornography is totally unprotected speech. Communications portraying nudity or sexual activity can be regulated concerning the time, place, and manner of their exhibition even if the communication is not pornographic and the regulation is content-based. d. COMMERCIAL SPEECH Commercial speech can be subject to reasonable governmental regulation for the protection of consumers and other legitimate governmental interests, but outright prohibition of commercial speech is unconstitutional.
e. REGULATION OF PUBLIC EMPLOYEES' SPEECH

A public employee's freedom of speech with respect to matters of public concern cannot be infringed unless the employer's interest in operating

MicroMash MBE In Brief: Constitutional Law Bar Exam Alerts

11

the public service outweighs the employee's interest in expressing the employee's political views. A public employee who has joined a subversive organization cannot be dismissed from public employment unless the employer can prove that the employee would have been dismissed even if the employee had not exercised the right of freedom of association by joining the organization.

3. PROCEDURAL PROBLEMS PARTICULAR TO THE FIRST AMENDMENT


If a court has issued an injunction banning the exercise of free speech rights, the constitutional issues raised by the issuance of the injunction cannot be litigated in a contempt prosecution for violation of the injunction. A statute that is overly broad (i.e., prohibits protected speech as well as properly regulated speech) or vague (i.e., a person of ordinary intelligence cannot distinguish permitted from prohibited activities) is unconstitutional on its face and can be successfully challenged even by those who could be regulated if the statute were clear and narrowly drawn. An individual is entitled to notice and a hearing before an injunction is granted limiting the time, place, and manner of her expression, unless there is a genuine emergency justifying an ex parte application.

12

MicroMash MBE In Brief: Constitutional Law Bar Exam Alerts

FUNDAMENTAL CONSTITUTIONAL LAW ISSUES


Many multiple-choice questions in Constitutional Law will list various clauses of the Constitution as the best answer to resolve the question presented. An attempt is made here to review the pertinent characteristics of such clauses to assist in choosing the best one. The Fourteenth Amendment can include all of the amendments to the Constitution that are incorporated into it. Therefore, it can be the best answer in a case where the First Amendment guarantees of freedom of speech or freedom of religion would be more precise. The Privileges and Immunities Clause of the Fourteenth Amendment was made a dead letter by the Slaughter-House cases, and is almost never the right answer. On the other hand, the Privileges and Immunities Clause of Article IV prevents states from discriminating against nonresidents in matters of fundamental interest unless the regulation is specifically targeted at a problem caused by such nonresidents. The Commerce Clause is the most important source of congressional power to regulate, but is not a source of power to cajole states or individuals into acting. The commerce power has been the source of congressional power to regulate civil rights. The dormant-commerce power is also the most significant curb on state economic regulation. The taxing power can be used for regulatory purposes as long as there is some revenue-raising purpose. It is likely to be the best answer if the commerce power is not one of the choices presented. The general-welfare power (or spending power) is only a power to spend money, not to regulate activity. It is the best answer when a result can be achieved only if an individual or state voluntarily cooperates with the federal government. The Due Process Clause is the best argument that a statute is unconstitutional only if there is (a) state action and (b) a failure to grant a hearing or a violation of a personal or privacy right. Substantive due process on economic matters is a dead letter. The Equal Protection Clause will be the best answer where state action is present, and the state is interfering with a fundamental interest, such as the right to travel, vote, or marry, or the classification is made by race, alienage, illegitimacy, or sex. The Thirteenth Amendment and the powers of Congress under it is likely to be the right answer when the discrimination is by private individuals where activity cannot qualify as state action. The Contract Clause has been revitalized in cases where the state is abrogating a contract to which it is a party. The police power is an answer that will justify only state as opposed to federal governmental action. The Supremacy Clause is not the best answer unless there is a congressional statute or a constitutional provision that is in conflict with a state activity. The Supremacy Clause is not a power of Congress.
--.

MicroMash BAR REVIEW MBE IN BRIEF CONTRACTS


Table of Contents
I. FORMATION OF CONTRACTS
A. MUTUAL ASSENT
1. 2. 3. 4. 1. 2. 3. 4. 5. 1. 2. 3. 1. 2. Offer And Acceptance Excuse Problems Of Communication And Battle Of The Forms Indefiniteness Or Absence Of Terms Infancy Mental Illness Intoxication Guardianship Corporate Incapacity Illegality Unconscionability Public Policy
-

1
1
1 4 7 9

B. CAPACITY TO CONTRACT

9
9 10 10 10 10 10 11 12

C. ILLEGALITY, UNCONSCIONABILITY, AND PUBLIC POLICY 10

D. IMPLIED IN FACT CONTRACTS AND QUASI CONTRACTS


Implied-In-Fact Contracts Quasi Contracts

12
12 12

E. F.

PRE-CONTRACT OBLIGATIONS BASED UPON DETRIMENTAL 13 RELIANCE EXPRESS AND IMPLIED WARRANTIES IN SALE-OF-GOODS 14 CONTRACTS
1. 2. Express Warranty Warranty Of Merchantability 14 14

3.

Warranty Of Fitness For A Particular Purpose

14

II.

CONSIDERATION AND ITS SUBSTITUTES


A. B. BASIC CONCEPT LEGAL DETRIMENT
1. 2. 3. 4. 5. Adequacy Of Consideration Pre-Existing Duty Rule Modification Of Contract Compromise Of Claims Illusory Promises

14
14 14
15 15 15 15 16

C.

BARGAIN ASPECT OF CONSIDERATION AND PROMISSORY ESTOPPEL 16


1. 2. 3. 4. 5. "Gift" vs. "Bargain" vs. "Reliance" Charitable Subscriptions Past Or Moral Consideration Debts Barred By The Statute Of Limitations Quasi-Contractual Recovery 16 16 17 17 17

III.

THIRD-PARTY BENEFICIARY CONTRACTS


A.

17
17 17 18 18

IN GENERAL DEFINITIONS VESTING OF BENEFICIARY'S RIGHTS DEFENSES

B. C. D.

IV.

ASSIGNMENT AND DELEGATION


A. B. IN GENERAL ASSIGNMENT OF RIGHTS
1. 2. 3. When Allowed Requirements Rights Of The Assignee

18
18 18
18 19 19

C. V.

DELEGATION

19

STATUTE OF FRAUDS
A.

19
19 19

GENERALLY MEMORANDUM

B.

C.

TYPES OF CONTRACTS WITHIN THE STATUTE OF FRAUDS 20


1. 2. 3. 4. 5. 6. Land Contracts Sale Of Goods One-Year Provision Suretyship Provision Executor-Administrator Provision Marriage Provision 20 20 21 21 21 21

VI.

PAROL EVIDENCE
A. B. INTENT OF THE PARTIES OPERATION OF THE RULE

22
22 22 23 23
23 23 23 24

VII. CONDITIONS
A. EXPRESS CONDITIONS
1. 2. 3. 4. Creation Of Condition Burden Of Proof On Performance Satisfaction Architect's Certificate Constructive Conditions Of Exchange Partial Performance Strict Performance By Seller Under The U.C.C. Buyer's Obligations Divisible Or Installment Contracts

B.

CONSTRUCTIVE
1. 2. 3. 4. 5.

24
24 25 26 26

C. D.

IMPLIED DUTIES OF GOOD FAITH AND FAIR DEALING

27

SUSPENSION OR EXCUSE OF CONDITIONS BY WAIVER, 27 ELECTION, OR ESTOPPEL


1. 2. 3. Waiver Election Estoppel 27 27 28

E. PROSPECTIVE INABILITY TO PERFORM, EFFECT ON OTHER 28 PARTY


1. 2. Seller's Remedies Under The U.C.C. Stoppage In Transit 28 28

iii

VIII. REMEDIES A. TOTAL AND PARTIAL BREACH OF CONTRACT


1. 2.

29 29

Buyer's Remedies Under The U.C.C. For Breach Of Contract By The 29 Seller 30 Seller's Remedies Under The U.C.C. For Breach by Buyer

B.

ANTICIPATORY BREACH 1.
2. At Common Law Anticipatory Repudiation Under The Uniform Commercial Code Multiple Recoveries Not Permitted When Does Election Occur Election Of Remedies Under The Uniform Commercial Code

32
32 32

C.

ELECTION OF SUBSTANTIVE RIGHTS AND REMEDIES 1.


2. 3.

33
33 33 34

D.

SPECIFIC PERFORMANCE, INJUNCTION AGAINST BREACH, 34 DECLARATORY JUDGMENT 1.


2. 3. 4. 5. 6. In General Factors Considered Real Property Limitations On Court's Powers To Order Specific Performance Specific Performance Under The U.C.0 Declaratory Judgment Rescission Reformation 34 34 34 34 35 35

E.

RESCISSION AND REFORMATION 1.


2.

35
35 36

F.

MEASURE OF DAMAGES IN MAJOR TYPES OF CONTRACT 36 AND BREACH


1. 2. Expectancy Damages Nominal Damages 36 37

G. CONSEQUENTIAL DAMAGES, CAUSATION, CERTAINTY, AND 37 FORESEEABILITY


1. 2. 3. 4. Consequential Damages Causation Certainty Foreseeability And The Rule Of Hadley v. Baxendale 37 37 38 38

H. LIQUIDATED DAMAGES AND PENALTIES

39

iv

I.

RESTITUTION AND RELIANCE RECOVERIES


1. 2. Restitutionary Damages Reliance Damages

39
39 39

J. K. L.

REMEDIAL RIGHTS OF DEFAULTING PARTIES AVOIDABLE CONSEQUENCES UNIFORM COMMERCIAL CODE ISSUES 1.
2. 3. Risk Of Loss Insurable Interest Title And Good-Faith Purchasers

39 39 40
40 40 41

IX.

IMPOSSIBILITY OF PERFORMANCE A. B. IMPOSSIBILITY IMPRACTICABILITY OF PERFORMANCE


1. 2. Total Impracticability Partial Impracticability

41 41 41
41 42

C. X.

FRUSTRATION OF PURPOSE

42 42

DISCHARGE

vi

CONTRACTS

I. FORMATION OF CONTRACTS
A. MUTUAL ASSENT
1. Offer And Acceptance a. Offer An offer is a communication that gives to the recipient of the communication the power to conclude a contract by accepting. A statement is an offer only if the person to whom it is communicated could reasonably interpret it as an offer. The primary test of whether a communication is an offer is whether an individual receiving the communication would believe that he or she could enter into an enforceable deal by satisfying the condition. Another way of looking at an offer is as a "conditional promise," meaning that a person promises something if the other person either returns a promise or does an act. If a return promise is requested, then the contract is called a "bilateral contract." If an act is requested, then the contract is a "unilateral contract." b. What is not an offer Offers must be distinguished from statements of opinion. For example, a statement by a physician that a person will be out of the hospital in two or three days is probably not an offer. Offers must be distinguished from statements of intention. For example, the statement, "I plan to sell my house for $25,000," is probably not an offer. Offers must be distinguished from invitations to deal or preliminary negotiations. When a person says something like, "What is your lowest price?" this is not an offer but merely an inquiry. A response to that question such as, "We can quote you $5 per gross for immediate acceptance," generally would be an offer. Advertisements normally are not offers unless they offer a reward. Statements made in jest or anger are not offers. Thus, if someone is disgusted with his or her automobile and says, "I'll sell it to you for $5," this is not an offer. c. Termination of offers An offer must be accepted while it is still outstanding. Offers can be terminated in the following ways: 1) Lapse of time stated in the offer If the offer states a date upon which it is terminated, then the offer terminates at midnight of that day. If the offer is terminated after a certain number of days

MicroMash MBE In Brief: Contracts

("This offer terminates in three days"), then the time starts to run from the time the offer is received, unless the offeree knows of the offer and that its transmission was delayed. 2) Lapse of a reasonable period of time What is a reasonable period of time depends upon the circumstances of the case. When the parties are dealing face to face or over the telephone, and there is no acceptance at the time they part, then the offer terminates. 3) Incapacity By majority rule, the offer terminates upon the death or mental incapacity of the offeror, even though the offeree does not know of the death or mental incapacity. 4) Revocation The basic rule is that an offer can be terminated by the offeror at any time. This is true even though the offeror specifically states that it will be held open for a specified period of time. There are, however, some exceptions to this rule: Firm or Irrevocable Offer. If consideration is paid for the promise to keep the offer open (an option contract), then the offer cannot be revoked. Promissory Estoppel. In some cases, it has been held that the doctrine of promissory estoppel prevents revocation of the offer. This occurs when the offeree relies to his detriment upon the offeror's promise to keep the offer open. Part Performance. When the offer is for a unilateral contract, once the offeree has started performance of the act requested, the offeror cannot revoke. U.C.C. Firm Offer Rule. When the offer is to buy or sell goods, the offer is irrevocable if (1) the offeror is a merchant, (2) there are assurances that it will be held open, and (3) the assurance is contained in a signed writing.

A revocation is not effective until communicated (it must be received by the offeree). Even though the offeror does not directly inform the offeree of the intent to revoke, if the offeree acquires reliable information that the offeror has taken definite action inconsistent with the offer, then the offer is automatically revoked. A classic example of this occurs when the offer is to sell real property, and the offeree discovers that the property has been sold to someone else. A general offer must be revoked by publishing the revocation in the same way in which the offer was publicized. However, remember that actual notice of the intent to revoke a general offer is effective as to the person who receives that notice, even though equal publicity is not given to the revocation. 5) Rejection or counteroffer Rejection of the offer by the offeree or the making of a counteroffer by the offeree operates to terminate the offer.

MicroMash MBE In Brief: Contracts

6) Impossibility or illegality

If the subject matter of the offer is destroyed, or if the contract becomes illegal, the offer is terminated.
d. Acceptance 1) Generally

An acceptance is an exercise of the power to conclude a contract given to an offeree by the offeror. The acceptance forms a contract between the parties. Only a person to whom an offer is made may accept. There can be no assignments of offers. There can, however, be assignments of options. The offeree must know of the offer in order to accept. Thus, when offers cross in the mail, there is no contract. However, the rule is that the offeree must know only at the time that she completes the act. Hence, if the act is started without knowledge of the offer, but completed after the offeree has learned of the offer, a contract is formed.
2) Method of acceptance

The offeror is the master of the offer, and can specify how acceptance is to occur. In determining whether there is a valid acceptance, the following points must be considered.
(a) Bilateral vs. unilateral offer

If the offer requests an act, then the act must be performed in order to accept. If the offer requests a return promise, then a return promise must be given. However, completion of the act will operate as an acceptance, even though a promise is requested by the offeror.
(b) Means of acceptance

The traditional view is that if the offer is sent by letter, it must be accepted by letter; if sent by telegram, it must be accepted by telegram, etc. The trend, however, is to find any reasonable method of acceptance proper. When the same means of acceptance is adopted as is used to communicate the offer, e.g., a letter offer is accepted by a letter, then all risks of mistake or loss are upon the original offeror. Thus, if the acceptance letter is lost or delayed in transit, a contract is formed. However, if the letter is misaddressed, this rule does not apply and its delay or loss prevents the formation of a contract.
3) Silence as acceptance

Normally, silence does not operate as an acceptance of an offer. Thus, even when the offer says, "If I don't hear from you within 10 days, I will assume you have accepted," no contract is formed by the offeree's remaining silent. However, a contract is formed if:

MicroMash MBE In Brief: Contracts

the offeror has given the offeree reason to believe that the offer can be accepted by silence, the offeree has remained silent, and the offeree intended to accept by silence, or because of previous dealings or otherwise, it is reasonable that the offeree should notify the offeror if she does not intend to accept, e.g., if the offeree has always accepted offers from this offeror in the past.

4) Notice of acceptance If the offer is for a bilateral contract, the offeree must give notice of acceptance, i.e., notice must be given of the return promise. However, an acceptance becomes valid when posted. Hence, a letter sent by the offeree operates as an acceptance as soon as it is placed in the mailbox, even though the offeror has not as yet received notice. This is not true if the letter is misaddressed. If the offer is for a unilateral contract, notice of acceptance is required only when the offeror is not likely to become aware that the act is being performed. e. Counteroffers The basic rule is that the acceptance must be on the same terms as the offer. Any changes, additions, or subtractions from the terms of the offer operate to make the attempted acceptance a "counteroffer" which, in legal terms, is a rejection of the offer and the making of a new offer by the offeree. However, suggestions or inquiries in a response by the offeree do not amount to a counteroffer. Also, if the acceptance simply spells out the details of the transaction, this does not make it a counteroffer. For example, if the offer does not note the kind of title to be given in a sale of real property, and the offeree requires "marketable title," the acceptance is valid provided that marketable title would be required by law. When the contract is for the sale of goods, Article 2 of the U.C.C. applies a different rule with regard to counteroffers. Under a literal reading of the code (2-207), a response by the offeree that purports to be an acceptance operates as an acceptance even though it changes, adds to, or subtracts from the terms of the offer. Cases, however, have held that a material change in the terms will prevent the formation of a contract under the code provision. 2. Excuse a. Mistake 1) In general The law divides mistake into "unilateral mistake" and "mutual mistake." In the unilateral mistake cases generally, no relief is granted, whereas in the mutual mistake cases, a party is excused from performance. 2) Unilateral mistake Unilateral mistake means that only one of the parties is mistaken. Here, either party can enforce the contract on its terms unless the party knew or had reason

MicroMash MBE In Brief: Contracts

to know that the other party was making a mistake, or the party had a duty to disclose the fact as to which the other party was mistaken. A similar situation arises when there is a mistake in a telegraph transmission. The sender may want to sell the goods at a specific price, but the telegraph company transmits the telegram so that it contains a lower price term. Here, if the buyer accepts, a contact is made at the lower price and it can be enforced in spite of the mistake. 3) Mutual mistake Mutual mistake means that both parties were mistaken as to an essential element of the contract. There must be a substantial difference between the deal as contemplated and the actual deal, with no intent by the parties to take a risk on this element of the transaction. For example, in the leading case, there was a contract to sell a cow, with both parties believing that it was a barren animal, fit only for meat. Actually, the cow was "with calf" As a meat animal, it was worth $80, whereas it was valued at $750 as a breeding animal. Rescission of the contract was granted. 4) Reformation When reformation of the contract is available to cure a mistake, neither party can avoid the contract. For example, assume that A agrees to sell Redacre to B, with B agreeing to pay $50,000 and to "assume a mortgage in the amount of $100,000." If the parties fail to include a provision regarding the assumption of the $100,000 mortgage, A can obtain reformation of the agreement to reflect B's promise. A has no right to avoid the contract because reformation adequately remedies the mistake in drafting the written agreement. b. Misunderstanding If the prospective parties to a contract manifest think they are agreeing to the same terms but in fact assent to different terms, and neither knows or should know that there is a misunderstanding that causes them to assent to different terms, there is no contract. c. Misrepresentation, nondisclosure, and fraud 1) Relationship to tort law In contract law, a misrepresentation made in connection with an agreement between parties may prevent the formation of a contract or make a contract voidable. 2) Elements of fraudulent misrepresentation In general, a misrepresentation is an assertion that is not in accord with the facts. If the misrepresentation is made knowingly (with scienter), it is clearly fraudulent. Under the Restatement (Second) of Contracts 162, it is also fraudulent if the

MicroMash MBE In Brief: Contracts

person either (a) does not have confidence in the truth of her assertion, or (b) knows that she does not have a basis for her assertion.

3) Nondisclosure
A person's nondisclosure of a fact known to him is equivalent to an assertion that the fact does not exist only if: (a) he knows that disclosure is necessary to prevent some previous assertion from being fraudulent; (b) he knows that disclosure would correct a mistake of the other party as to a basic assumption, and nondisclosure would amount to lack of good faith and fair dealing; (c) he knows that disclosure would correct a mistake of the other party as to the contents or effect of a writing evidencing or embodying their agreement; or (d) the other person is entitled to know the fact because of a confidential or fiduciary relationship between them.

4) Effect of fraudulent misrepresentation (a) Fraud in the factum


Fraud in the factum, or as it is also called fraud in the execution, occurs when the fraudulent misrepresentation prevents a party from knowing the character or essential terms of the transaction. In such cases, no contract is formed, i.e., the apparent contract is void. Where, however, the person could have discovered with the exercise of reasonable diligence the character or essential terms of the transaction, he is estopped from asserting that the contract is void. In most cases, whether the contract is void or voidable is immaterial because the person has a defense in both cases. The distinction is more important when the rights of third parties are involved. A voidable contract can be ratified after the fraud is discovered, but a void one may not be.

(b) Fraud in the inducement


When the misrepresentation is used to induce someone to enter into a contract, it is called "fraud in the inducement," and the contract is voidable. The rationale of the modern theory is that a contract is voidable whenever the misrepresentation is material, regardless of fraudulent intent.

d. Undue influence and breach of a confidential relationship


A plaintiff need not meet the same standards to avoid a contract that must be met in the case of fraud where there is a trusting relationship between the contracting party and a person in whom she places trust and confidence.

1) Undue influence
Undue influence occurs in a relationship between two parties where one is dominant, usually because he is sophisticated in business relationships and the other is dependent, either because of lack of education and experience in the financial world or because the dependent person has diminished mental capacity

many times caused by advanced age. In addition there is a relationship of trust


between the dominant and dependent person. Where that type of relationship

MicroMash MBE In Brief: Contracts

exists, the person in the dominant position is held to a higher standard of disclosure and fairness than in the ordinary world of arms-length contracts. The dominant person may have the burden to show that a contract was fair to the dependent person before he can withstand an action to avoid the contract. His affirmative duty to disclose facts is much higher when there is a possibility of undue influence. While most lawsuits in this area deal with situations where there is a contractual relationship between the dominant and the dependent person, or where the dominant person has assisted the dependent person in making a testamentary disposition, a person who has been the subject of undue influence can also avoid a contract with a third person, unless that person would be harmed because he is a bona fide purchaser. 2) Breach of a confidential relationship There are certain relationships between individuals, usually described as fiduciary relationships, where the higher standards of conduct in contractual dealings (described above in dealing with undue influence) arise out of the existence of the relationship itself These are generally described as fiduciary relationships and automatically impose of the party who has the fiduciary obligation of fair dealing and full disclosure. The most common examples of such fiduciary relationships occur between trustee and beneficiary, lawyer and client, doctor and patient, financial advisor and client, and in some cases parent and child. The burden of proving that the contract is fair is usually placed upon the fiduciary. e. Duress Like fraud, duress can result in either a void or voidable contract. Generally, any wrongful act or threat that deprives a party of meaningful choice constitutes duress. When a party's agreement is the result of physical duress, e.g., a strong person taking the other's hand and compelling her to sign a contract, the contract is void. When the duress is in the nature of a threat, then the contract is voidable. However, not all threats are improper. For example, a "threat" that one "will never talk to the person again" if she refuses to enter into a contract is not duress. 3. Problems Of Communication And Battle Of The Forms Section 2-207 of the U.C.C. significantly changes the counteroffer rule in regard to contracts for the sale of goods. Section 2-207 (1) provides: A definite and seasonable expression of acceptance or a written confirmation which is sent within a reasonable time operates as an acceptance, even though it states terms additional to or different from those offered or agreed upon, unless acceptance is expressly made conditional on assent to the additional or different terms.

MicroMash MBE In Brief: Contracts

a. When a contract is formed despite varying terms in the acceptance From a reading of 2-207(1), it appears that a contract is formed whenever there is a "definite and seasonable expression of acceptance" or a "written confirmation," unless acceptance is expressly made conditional on assent to the additional or different terms. The extent to which the terms of the acceptance vary or add to the offer does not seem to be relevant, except that the changes may bear upon whether the writing can be construed to be a "definite" acceptance or confirmation. It is only where the acceptance is expressly conditioned upon the offeror's assent to the new or different terms that a contract is not formed. However, where the "expression of acceptance" deviates substantially from the offer, courts have indicated a reluctance to find that a contract is formed. b. When additional or different terms become part of the contract Assuming that a contract has been formed even though the acceptance contains terms additional to or different from the offer, the question arises whether these additional or different terms become a part of the contract formed. If both parties are merchants, additional or different terms do become a part of the contract, unless they: (1) materially alter the offer, or (2) are objected to by the offeror in advance of the acceptance, or (3) are objected to by the offeror within a reasonable time after the offeror obtains notice of them. Note that the U.C.C., by its terms, treats different and additional terms differently. Under the plain wording of the U.C.C., different terms (those that vary or contradict a term of the offer) never become part of the contract. However, courts have either ignored or avoided this rule, presumably viewing it as too harsh and impractical. Thus, the rule applied on the Multistate Bar Exam is that different terms are incorporated into a contract if they meet the criteria stated above. c. Acceptance by conduct Subsection (3) of 2-207 provides for the formation of a contract by virtue of the conduct of the parties. If the conduct includes some written expression of the parties' agreement, then the terms of the contract include whatever terms the parties agreed to in writing. As to other terms, those implied by the code in the absence of agreement apply. For example, if the writings of the parties agreed only to the description and quantity of the goods, the code would supply the price term, the place of delivery, warranties, and so on. Under 2-206 of the U.C.C., an offer to buy or sell goods may be accepted in any reasonable manner; thus, the code has rejected the bilateral-unilateral distinction. The section specifically says that an order for goods may be accepted either by shipping the goods, or by promising to ship them. Under the section, the shipment of defective, nonconforming goods is an acceptance, and a contract is formed unless the

MicroMash MBE In Brief: Contracts

seller notifies the buyer that they are not meant to fill the order. The buyer may reject them and sue for any damages resulting from the seller's failure to deliver conforming goods, or if the buyer accepts the goods, he may have a remedy for any damages resulting from the nonconformity.
4. Indefiniteness Or Absence Of Terms

At common law the courts were reluctant to supply terms to a contract if the parties had not agreed upon them and thus often held that an agreement which missed essential terms was not a contract because it was indefinite. The U.C.C. has taken a different approach with respect to contracts within its scope. Article 2 makes contracts somewhat easier to form. Under 2-204(1), a contract for the sale of goods may be made in any manner sufficient to show agreement, including conduct by both parties which recognizes the existence of a contract. For example, if the parties agree to the sale of a specific quantity of goods but do not agree on a price, there is a contract for a reasonable price. There is no need to determine the exact time when a contract is made. This provision obviates the necessity of denominating a particular communication an "offer" and another an "acceptance." The general approach of Article 2 is to determine whether from the totality of the communication it is clear that the parties intended to enter into a binding agreement; if so, a contract exists. Under the Code, an offer to buy or sell goods normally can be accepted either by a return promise or by a performance of the act requested. Note, however, that if the offeror specifically requires a return promise or a completion of the act, this must be done to effectuate an acceptance. Under the Code's general rule, however, unless otherwise unambiguously indicated, an order from a buyer for current shipment of goods may be accepted by the seller either by a return promise or by actual shipping of the goods.
B. CAPACITY TO CONTRACT

Another requirement of a contract is that the parties be competent. Incompetency arises because of infancy, mental illness or defect, intoxication, guardianship, and corporate incapacity.
1. Infancy a. Disaffirmance

When a contract is made by an infant, it is voidable. This means that the infant may disaffirm the contract and avoid any liability under it. The disaffirmance can be effectuated for a reasonable time after the infant reaches majority. The infant must restore any benefits received under the contract if possible, but if not possible, the infant still can disaffirm. Even if the infant misrepresented her age, disaffirmance is still allowed.

10

MicroMash MBE In Brief: Contracts

b. Liability for necessities When necessities are furnished to the infant, the infant must pay for them, but the recovery by the person furnishing the necessities is on a quasi-contract theory. Thus, the supplier can recover only the reasonable value of the services or goods, and not the agreed upon price. Food, shelter, and clothing are clearly necessities, and, depending upon the status of the minor, an automobile, an education, etc., may also be necessities. 2. Mental Illness If a party is adjudicated mentally incompetent and is under guardianship, the contracts made by the individual are void. On the other hand, if there has been no adjudication or guardianship, the contracts are voidable and must be disaffirmed. If a contract is made during a lucid period, the contract is fully enforceable, unless the person has been adjudicated an incompetent. 3. Intoxication Technically, intoxication resulting from alcohol or drugs renders a contract voidable if the person entering into the contract was unable to understand the nature of the transaction. 4. Guardianship A person has no capacity to incur contractual duties if his property is under guardianship by reason of an adjudication of mental illness or defect, habitual intoxication, narcotics addiction, or because the person is a spendthrift, aged, or a convict. The policy of appointing a guardian is to preserve the property from squandering or improvident use. The powers of a guardian are usually defined by statute. The guardianship proceedings are treated as giving public notice of the incapacity of the ward. Property under guardianship may be reached to satisfy the torts or quasi-contractual obligations of the ward. 5. Corporate Incapacity When a corporation acts ultra vires (outside its powers), the contract is voidable, but today most states take the position that if one party has performed, there may be recovery in quantum meruit.

C. ILLEGALITY, UNCONSCIONABILITY, AND PUBLIC POLICY 1. Illegality


If the performance that is to occur under a contract is illegal, the contract itself is illegal and is unenforceable. a. Types of illegal contracts Clearly, in most states, wagering contracts, usurious bargains, etc., are illegal. By public policy, contracts in restraint of marriage are illegal. Contracts in restraint of trade are illegal, but covenants not to compete contained in a sale of a business or an employment contract will be enforced if they are reasonable in time and geographical

MicroMash MBE In Brief: Contracts

11

area. Contracts to bribe an official and bribes themselves are illegal, as are contracts to commit a tort or a crime. b. Effect of illegality The law will not enforce or even recognize transactions that are illegal. The contract itself cannot be enforced; if one party performs under the contract, she cannot receive any compensation for the performance, either in an action on the contract or an action in quasi contract. The law also will not grant restitution if someone has paid another under an illegal contract. c. Exceptions 1) Ultimate purpose illegal If a contract has an ultimate purpose that is illegal, but its performance is legal in itself, e.g., a contract to sell gambling equipment, a person can recover under the contract, provided he does not participate in the illegal purpose of the deal. For example, the seller of gambling equipment would be able to recover the price, provided he did not become involved in the gambling. This is true even if the seller knows of the illegal purpose. 2) Divisible contracts In some instances, contracts are separable in that part of the consideration is legal and the other part is illegal. For example, B painted A's house in return for A's promise to pay $500 and to smuggle goods into the country. In such cases, the courts may allow recovery of the legal part of the promise, i.e., the $500, and deny recovery of any damages for A's failure to smuggle the goods into the country. 3) Rini Delia (equally at fault) Where the parties are not in pari delicto (equally at fault), recovery may be allowed. For example, a contract whereby an employer promises to pay an employee for overtime work is enforceable by the employee, even though it is illegal to work overtime in the particular occupation. 4) Locus poenitentiae repudiation If one of the parties repents of the illegal bargain prior to the time of the illegal performance, she is normally allowed to recover whatever was given for the promised performance. 2. Unconscionability Article 2 of the code provides that a court may refuse to enforce a contract or part of a contract on the grounds that it is "unconscionable." Generally, unconscionability goes to unfair dealings by one of the parties. Normally, it arises where the parties have unequal bargaining power, and the more powerful party has attempted to limit severely the rights of the other party, or has taken unfair advantage. The question whether a contract is unconscionable is for the court to decide, and the issue does not go to the jury.

12

MicroMash MBE In Brief: Contracts

3. Public Policy
Even if a contract is neither illegal nor unconscionable it may be unenforceable if it violates a significant public policy. Many kinds of adhesion contracts are unenforceable because they are against public policy. A contract requiring an individual who needs an essential public service, such as medical care, to waive any claim for negligence on the part of the provider is likely to be found to violate public policy and therefore be unenforceable.

D. IMPLIED IN FACT CONTRACTS AND QUASI CONTRACTS


-

1. Implied-In-Fact Contracts
The agreement or mutual assent necessary for the formation of a contract most frequently results from words expressed by the parties. When this is the case, the contract is called an "express contract." However, conduct may also indicate assent or agreement. Thus, if one takes a pack of cigarettes from the counter of a drug store, this conduct indicates her intent to purchase the cigarettes. When the agreement is formed by virtue of conduct rather than expressed words, this gives rise to what is called an "implied contract." Of course, in certain cases the terms of a contract are determined both by the expressed words of the parties and by conduct on their part. Whether the contract is express or implied does not affect the legal relationship between the parties or the rules of law that apply to this relationship. A primary characteristic of a contract, whether express or implied in fact, is that the law will "enforce the promise." Usually this is done by granting "benefit of the bargain" damages that will attempt to put the plaintiff in the position that he or she would have been in had the contract been performed. In some cases the court will specifically enforce the contract, i.e., will order the defendant to perform his or her promise.

2. Quasi Contracts "Quasi contracts," or, "contracts implied in law," are not true contracts at all. They do
not depend upon assent between the parties, nor is recovery based upon a promise. In a quasi contract, the law imposes an obligation because it appears just. Such an obligation is very close to the type of obligation imposed by the law of torts, and it has become associated with contract law largely because of the forms of action which were prevalent in early English law. Since the defendant has not made a promise in cases seeking quasi-contractual recovery, the law cannot "enforce the promise" as it does in contract actions. Rather, the law implies a promise (establishes a duty) that the defendant must make restitution to the plaintiff of any benefit that the plaintiff has conferred upon the defendant. This is accomplished by awarding the plaintiff money damages in the amount of the value of the benefit. Thus, the theory of the action is restitutionary in nature; the law will restore the plaintiff to the position he or she was in prior to the transaction or event. Although the theoretical distinction between contracts implied in fact and quasi contracts is clear, the differences sometimes fade and even appear entirely nonexistent in certain

MicroMash MBE In Brief: Contracts

13

fact situations. For example: In a contract for services where no price term is agreed upon, the recipient of the services is obligated to pay the reasonable market value of the services. In such a case, it is difficult to determine whether the law finds this obligation because it assumes that the parties implicitly agreed on the fair market value as the price for the services, or because it believes that it is just to impose the obligation as a matter of law.

E. PRE-CONTRACT OBLIGATIONS BASED UPON DETRIMENTAL RELIANCE


The general rule except for the case of firm offers under the U.C.C. is that an offer is revocable at any time, even though the offeror has agreed to keep the offer open for a specified time. An offer can be made irrevocable if the offeror promises to keep the offer open for a specific period of time and that promise is supported by consideration. An option contract is then formed and the offeree usually pays an agreed amount to the offeror in order to make the offer irrevocable. There is one set of circumstances where the promissory estoppel substitute for consideration is the basis for keeping the offer open. This occurs in the construction industry where the owner puts a contract out to bid. The construction of most modern buildings is supervised by a general contractor who bids a specific price to the owner for the entire job. In order to complete the job, the general contractor usually hires subcontractors to perform portions of the construction such as electrical and plumbing work. Prior to submitting a bid to the owner for the entire job, the general contractor solicits bids for portions of the work from subcontractors, which the subcontractors agree to leave outstanding for a reasonable time after the contract for the entire job is awarded by the owner. The agreement not to revoke the sub-bid offer is enforceable because it is supported by the substitute for consideration known as promissory estoppel. When making the bid, the subcontractor knows that the contractor is relying on it by using it in the calculation of his costs for the job in preparation for making his bid as general contractor. It would be unjust to permit the subcontractor to revoke that bid after inducing justifiable reliance. There, upon receiving the contract, the general contractor has a right to accept the sub-bid, turning it into a contract even though the sub-bidder has attempted to withdraw the sub-bid. The contractor must accept the sub-bid within a reasonable time to turn it into a contract. Since the sub-bid is only an outstanding offer, the general contract is not bound to accept it upon becoming the successful bidder for the general contract. He can enter into a subcontract with someone else for a lower price. The sub-bidder has no right to require that the general contractor accept his bid if the general contractor is the successful bidder on the project. If the general contractor attempts to negotiate a lower price with the sub-bidder, then he has made a counteroffer and can no longer accept the original bid.

14

MicroMash MBE In Brief: Contracts

F. EXPRESS AND IMPLIED WARRANTIES IN SALE-OF-GOODS CONTRACTS


There are three basic warranties of quality: (a) express, (b) merchantability, and (c) fitness for a particular purpose. The trend is to allow a warranty action to lie even though there is no privity between the parties. 1. Express Warranty All statements and promises made by the seller that form a part of the basis of the bargain are express warranties unless merely the seller's opinion or commendation of the value of the goods. Express warranties include almost any positive affirmation by word or conduct, including descriptions of goods, and disclaimer clauses are ignored when they conflict with these representations. 2. Warranty Of Merchantability An implied warranty of merchantability is given whenever the seller is a merchant. Goods must be fit for their ordinary purpose and pass without objection in the trade under the contract description. The disclaimer may be oral, but must use the term merchantability," and must be conspicuous if in writing. The warranty can be disclaimed in some instances by use of "as is" or similar language.
"

3. Warranty Of Fitness For A Particular Purpose This implied warranty is given whenever the seller has reason to know that (1) the buyer has a particular use for the goods, and (2) the buyer is relying upon the seller's skill to select the goods. This warranty can be disclaimed by general language, but the disclaimer must be in writing.

II. CONSIDERATION AND ITS SUBSTITUTES A. BASIC CONCEPT


There are two basic elements of consideration: (1) legal detriment, and (2) bargained-for exchange. You must look at the promise that the plaintiff is trying to enforce, and determine whether or not that promise is supported by consideration. Generally, this means you must ask whether or not something has been received for the promise, and whether this has been bargained for. Consideration can take the form of: a return promise to do something; a return promise to refrain from doing something; the actual doing of some act; or refraining from doing some act.

B. LEGAL DETRIMENT
The basic concept of legal detriment is that there must be something of substance, either an act or a promise, that is given in exchange for the promise which is to be enforced.

MicroMash MBE In Brief: Contracts

15

1. Adequacy Of Consideration Normally, adequacy of consideration is not an issue except when specific performance is being requested. Even though the thing bargained for may be worthless, e.g., a promissory note which cannot be enforced, so long as the promisor wanted that thing, the giving of it will constitute adequate consideration. Section 2-302 of the Uniform Commercial Code provides that a court may refuse to enforce a contract, or a part of a contract, on the grounds of unconscionability. Cases have held that when a consumer agrees to pay far in excess of the value of goods to be sold and there is some other indication of unfair dealing, a contract is unconscionable under this provision. 2. Pre-Existing Duty Rule The pre-existing duty rule arises when the "legal detriment" is something that the promisee is already obligated to do. For example, if A says to B, "I will give you $100 in thirty days if you will refrain from killing my cat," there is no consideration for A's promise to pay $100, because B is already under an obligation not to kill the cat. 3. Modification Of Contract The traditional rule is that a modification of an existing contract must be supported by consideration. The courts, however, allow enforcement of agreements to modify a contract under the following three rules: where there has been a rescission of the existing contract by tearing it up or by some other outward sign, and then the entering into a new contract, whereby one of the parties must perform more than he or she was to perform under the original contract; where there are unforeseen difficulties, and one of the parties agrees to compensate the other when the difficulties are discovered; where there are new obligations on both 'sides.

Article 2 modifies the doctrine of consideration in two ways: A promise to keep an offer open made by a merchant need not be supported by consideration if it is in writing and signed. Under 2-209 of the Uniform Commercial Code, no consideration is necessary to modify a contract for the sale of goods, but there is a requirement of good faith.

4. Compromise Of Claims If P claims that D owes her a thousand dollars, and P agrees to accept $500 in full satisfaction, the payment of the $500 operates to discharge the entire $1,000 debt only where there is some dispute either as to the validity of the debt or the amount of the debt. However, the debt will be discharged if the $500 payment is earlier than it had to be, or if something in-addition to the $500 is given to P. If P has an invalid claim against D, and D promises to pay money in return for P's dismissing the action or agreeing not to bring it, D's promise to pay the money is

16

MicroMash MBE In Brief: Contracts

enforceable, provided P had a reasonable belief that she had a legitimate claim even though the claim turns out to be groundless. 5. Illusory Promises Some promises, although appearing to be real, are in fact illusory. Thus, if A says, "I will sell you my car if I want to," this is an illusory promise and does not constitute legal detriment. Where a contract has been entered into but one party or the other may cancel, the general rule is that a contract is enforceable so long as some notice must be given of the cancellation. Also, output and requirements contracts are enforceable.

C. BARGAIN ASPECT OF CONSIDERATION AND PROMISSORY ESTOPPEL


In order for the legal detriment to constitute good consideration, it must be bargained for in exchange for the promise. Promissory estoppel is very similar to consideration, the only difference being that the legal detriment must have been suffered in reliance upon the promise rather than having been bargained for. 1. "Gift" vs. "Bargain" vs. "Reliance" If A says, "I will give you $1,000 if you attain the age of 21," this promise is not enforceable. The act of attaining the age of 21 is not bargained for; hence, it cannot operate as consideration. Likewise, there is no reliance on the promise, so promissory estoppel does not apply. If A says, "I will give you $1,000 if you graduate from high school," this may or may not be enforceable. It is possible that A wanted the promisee to graduate from high school and was bargaining for this. If that is the case, the promise to pay the $1,000 is supported by good consideration. It is also possible that the promisee may have relied upon the promise in completing the act, and if this is the case, the promise is supported not by consideration, but by the doctrine of promissory estoppel. If A says, "I will pay you $1,000 if you give up smoking for a year," this is an enforceable promise. The law assumes that A was bargaining for the act. The test in these cases is generally said to be whether the offeree could have reasonably believed that the intent of the offeror was to induce the action. If that is the case, the promise is enforceable. 2. Charitable Subscriptions The doctrine of promissory estoppel is used frequently to enforce promises to charitable institutions. For example, if A promises to give a university $1,000,000 in 1990, and the university purchases land in reliance upon the promise, then the promise is enforceable under the doctrine of promissory estoppel. In charitable subscription cases, there is also the possibility of finding consideration from the exchange of promises among a number of contributors. The concept is that the promise of each contributor operates as the consideration for the promise of each other contributor. The donee is treated as a third-party beneficiary of the promises.

MicroMash MBE In Brief: Contracts

17

3. Past Or Moral Consideration Past or "moral" consideration is generally not recognized in the United States. Thus, if P saves D's life and D then promises to pay $1,000 because P has saved his life, D's promise is unenforceable. P's act of saving D's life could not have been bargained for, nor could it have been done in reliance upon the promise. However, there is a modern trend toward enforcing such promises when necessary to "prevent injustice." 4. Debts Barred By The Statute Of Limitations If D owes P $5,000, but the statute of limitations has run on the claim, a new promise by D to pay the $5,000 made after the running of the statute is enforceable without any new consideration. 5. Quasi-Contractual Recovery Recovery may be available where the plaintiffs performance was neither bargained for nor in reliance on an offer, but it would be unjust to treat it as a gift. For example: P sees D's horse running free and knows that D is out of town. P feeds and houses the horse for two weeks awaiting D's return. When D returns, D thanks P and promises to pay P $50 at the end of the month. This promise would not be enforceable in many courts because it is for "past consideration." There was neither any bargain for the act, nor could P have relied upon D's promise in feeding the horse. However, P may be able to recover in quasi contract. In order to recover, P would have to show that she reasonably expected to be compensated, and that there was a benefit rendered to D by virtue of taking care of the horse. The recovery in this case might be $50 if this were the reasonable value of the services and food provided, but it would not be based upon D's promise. Rather, it would be based upon the theory that P rendered a benefit to D and, in justice, D ought to compensate P for that benefit.

III. THIRD PARTY BENEFICIARY CONTRACTS


-

A. IN GENERAL
A third-party beneficiary contract results when two parties enter into a contract with the understanding and intent that the performance to be rendered by one will go to a third person.

B. DEFINITIONS
An intended beneficiary is one to whom the promisee wishes to make a gift of the promised performance or to satisfy an obligation to pay money owed by the promisee to the beneficiary. For example, A agrees to paint B's house in return for B's promise to pay $500 to C. This is a third-party beneficiary contract in which C is the beneficiary. Because C is an intended beneficiary, C has the right to bring an action against B for the $500. An "incidental beneficiary" is one who just happens to be benefited by the contract, and there is no contractual intent to benefit that person. For example, if A and B entered into a contract to build a theater and C owned a restaurant across the street, C would be benefited by the theater but would not gain any rights against A or B under the contract. C would be an "incidental beneficiary" because there was no intent to benefit C.

18

MicroMash MBE In Brief: Contracts

C. VESTING OF BENEFICIARY'S RIGHTS


Only an intended beneficiary has a right to sue on the contract. A beneficiary of a "gift promise" may sue only the promisor, but a beneficiary to whom the promisee owed money may sue either the promisor or the promisee on the underlying obligation. The rights of an intended beneficiary vest when he manifests assent to the contract, changes position in reliance on the contract, or brings suit on the contract. Once the rights of a beneficiary "vest," the original parties to the contract cannot modify it in any way, nor can the promisor be discharged by the promisee to the detriment of the thirdparty beneficiary. Thus, in our first hypothetical, if C's rights had vested, A could not discharge B from B's obligation to pay $500 to C, nor could A agree to a lesser payment in satisfaction of B's obligation.

D. DEFENSES
The promisor can raise any defense against the third-party beneficiary that she had against the original promisee. For example, in our first hypothetical, if A fails to paint the house, B can raise this as a defense in an action brought by C. The promisor, however, cannot raise rights of the promisee against the third-party beneficiary. Thus, if C were a creditor beneficiary in our hypothetical, but the debt that A owed to C was barred by the statute of limitations, a discharge in bankruptcy, or by virtue of the fact that A had a defense against C, these defenses could not be raised by B in a suit brought by C. This would be a so-called jus tertii (rights of a third party) defense.

IV. ASSIGNMENT AND DELEGATION A. IN GENERAL


Assignment refers to the transfer of rights under a contract, and delegation involves the obtaining of someone else to perform a party's obligations under a contract. Although the two are clearly distinct in concept, the term "assignment" is often used to refer to both assignment of rights and delegation of duties.

B. ASSIGNMENT OF RIGHTS
Almost all contract rights can be assigned. Partial assignments are permissible, as is the assignment of future or unearned rights. 1. When Allowed There can be no assignment which: (a) materially increases the duty or risk of the obligor; or (b) materially reduces the obligor's chance of obtaining performance. Generally, prohibitions against assignment in the contract are strictly construed, and the assignment is permissible. Even if validly prohibited by the contract, the parties retain the power to assign, and the only consequence is that an assignment operates as a breach of the contract.

MicroMash MBE In Brief: Contracts

19

2. Requirements No formalities are needed for an assignment, but there must be a present intent to transfer the right immediately. Thus, a statement, "When the money comes in I will give you 10%," is not an assignment. There is no consideration needed. 3. Rights Of The Assignee
An assignee takes all of the rights of the assignor, but takes subject to any defenses that could be raised against the assignor. The rights of the assignee are subject to set-offs if the transaction giving rise to the set-off occurred prior to the time the obligor was given notice of the assignment. The assignee is also subject to any modification of the contract made prior to the time the obligor obtained notice of the assignment. Thus, payment by the obligor to the assignor can be raised as a defense, provided the payment was made before the obligor had notice of the assignment.

C. DELEGATION
The general rule is that obligations under a contract can be delegated. However, there can be no delegation when (a) the other party has a substantial interest in having the individual perform (for example, in a personal services contract involving taste or a special skill); or (b) when delegation is prohibited in the contract. When obligations are delegated, the delegator is not released from liability, and recovery can be had against the delegator if the delegatee does not perform. Also, the other party to the contract becomes a third-party beneficiary to the contract of assignment and has a right to sue the delegatee immediately.

V. STATUTE OF FRAUDS A. GENERALLY


The Statute of Frauds requires that there be a written memorandum of the contract in certain cases. The following contracts require a memorandum in most states: land contracts; sale of goods, if the price is $500 or more; contracts that cannot be performed within one year; suretyship contracts; promises by executors and administrators; contracts in consideration of marriage.

In addition, many states have writing requirements for contracts to make a will and real estate brokers' contracts.

B. MEMORANDUM
The memorandum that is required must be in writing, signed by the party to be charged, and it must contain the essential elements of the deal. The memorandum need not be formal:

20

MicroMash MBE In Brief: Contracts

receipts, telegrams, exchange of correspondence, etc., can serve as memoranda. The essential elements may be in more than one writing, but if so, one of the writings must contain something referring to the others. The memorandum need not be delivered. If it is lost or destroyed, it still operates to satisfy the Statute of Frauds, and its prior existence can be proved by oral evidence.

C. TYPES OF CONTRACTS WITHIN THE STATUTE OF FRAUDS


1. Land Contracts a. Types
Practically all contracts and conveyances involving real property are included and must be in writing, e.g., contracts to buy or sell, conveyances, mortgages, grants of easements, leases, and interests created by restrictive covenants. Excluded are licenses, leases for less than one year in most states, and assignments of mortgages.

b. Part performance
Once a vendor conveys, the contract is enforceable, and the vendee must pay the price even if there is no memorandum. However, payment by the vendee alone is not sufficient to take the transaction out of the Statute of Frauds. There must be some other act, such as taking of possession, showing the existence of the contract.

2. Sale Of Goods a. Sufficiency of the writing


When the price of goods is $500 or more, the U.C.C. requires a memorandum of the sale. Here, the memorandum need only: (1) indicate that a contract has been made; (2) identify the parties; (3) contain a quantity term, and (4) be signed by the person to be charged. Specifically, a mistake in the memorandum or the omission of other terms does not destroy its validity. An omitted term, such as the price term, can be proved by parol evidence. There can be no enforcement, however, beyond the quantity term actually stated in the memorandum. Thus, if the memorandum calls for the delivery of 10,000 widgets, whereas the parties actually agreed to 15,000, the contract can be enforced only to the extent of 10,000.

b. Exceptions
There are several exceptions to the sale of goods Statute of Frauds.

Specially Manufactured Goods. When (a) the goods are specially


manufactured for the buyer; (b) a substantial beginning has been made on their production, or a commitment has been made to purchase them elsewhere by the seller; and (c) the goods are not salable in the seller's ordinary course of business, there is no requirement of a writing.

Part Payment. When part of the purchase price has been paid, the contract is taken out of the statute to the extent of payment. Hence, if the contract calls for 200 widgets, and 10% of the price has been paid, the contract can be enforced as to 20 widgets.

MicroMash MBE In Brief: Contracts

21

that goods are received and accepted. Again, if only part of the goods are received and accepted, a contract can be enforced only as to those goods.

Receipt and Acceptance. The contract is taken out of the statute to the extent

Admission in Pleadings. If the party to be charged admits in the pleadings or

otherwise in court the existence of the contract, no writing is required as to the amount admitted.

Failure to Respond to Memorandum. If a memorandum sufficient against

one party is sent to the other party, who does not object within ten days, the contract is enforceable against the other party even though he has not signed it.
3. One-Year Provision

Contracts that cannot be performed within one year of their making must be in writing. The year starts the day after the contract is made. It is the time that the contract is made that is important, not the length of performance. For example, a contract made in October, with performance to begin in December and to continue through November of the following year, is clearly covered by the statute. If there is a possibility of performance within a year, no writing is required. Thus, a contract "for life" is not covered by this provision of the Statute of Frauds because the employee might die within the year. However, a contract whereby a person is to work for "two years" is covered. Even though the person may die within the two years, his or her death would not constitute full performance. Part performance of the contract does not take it out of the Statute of Frauds; the only remedy is in restitution (quasi contract). Full performance by one of the parties, however, does take it out of the statute, and recovery can be had on the contract.
4. Suretyship Provision

When a person promises to answer for the debt of another, the person's promise must be in writing in order to be enforceable. This provision does not include "indemnity contracts," i.e., those in which there is a promise to reimburse someone if she loses money on a deal. The "main purpose rule" provides that if the main purpose of the contract is to benefit the promisor (surety), rather than the principal debtor, the contract may be enforced even though there is no writing. Neither partial nor full performance takes the contract out of this section of the Statute of Frauds.
5. Executor-Administrator Provision

This is simply a special application of the suretyship provision. Remember that the debt must be a debt of the estate in order for this section of the Statute of Frauds to apply.
6. Marriage Provision

This is practically obsolete. It applies to any agreement in consideration of marriage except mutual promises by the two parties to marry each other.

22

MicroMash MBE In Brief: Contracts

VI. PAROL EVIDENCE


The parol evidence rule provides that when parties have adopted a writing as their agreement, no evidence can be admitted to vary, contradict, add to, or subtract from the obligations as they are stated in the writing.
A. INTENT OF THE PARTIES

In order to invoke the parol evidence rule, it must be shown that the parties intended to adopt the writing as their agreement. It is possible that there is:
total integration: the writing is the final and complete expression of the agreement between the parties, and no evidence can be introduced as to any additional promises or representations made prior to the time of the writing. partial integration: the writing is a final expression of the parties' agreement to the matters covered in the writing, and no extrinsic evidence can be introduced that varies or contradicts the parties' agreement on those matters.

parties.

no integration: the writing in no way was intended to represent the agreement of the

The intent of the parties determines whether there is total, partial, or no integration. This is a question of fact but is determined by the court. Generally, the court looks at the document to determine how complete and how formal it is. Normally, the court will then consider the proffered evidence to see whether it is covered by the agreement. If there is a "merger clause" ("this writing contains the entire agreement of the parties and no evidence of other agreements is admissible"), the merger clause is evidence of the intent to integrate.
B. OPERATION OF THE RULE

If the proffered evidence contradicts the writing, it is not admissible unless the court finds that there was no intent whatsoever to integrate the agreement of the parties into the writing. If the evidence merely supplements (adds to the agreement as written), it is normally admitted if it is consistent with the writing, unless the court finds that there is total integration or that the matter is covered by the agreement and the additional matters would naturally have been reduced to writing if the parties intended it to be a part of their contract. When the parol evidence rule is applicable, evidence may still be admitted under a number of theories: the court may find that there is only partial integration, that this matter is not covered by the writing, that the evidence is consistent with the writing, and that it simply adds to the agreement and does not contradict it. even where there is full integration, the court may find that the evidence offered represents a separate deal. For example, if the writing covers the sale of a house and the oral evidence goes to show a sale of personal property in connection with the sale of the house, the court may find that the oral evidence proves a distinct and separate contract.

MicroMash MBE In Brief: Contracts

23

if there is an ambiguity in the contract, the evidence may be admitted for the purpose of interpreting the agreement. the evidence may be admitted to prove a defense such as fraud in the inducement, mistake, failure of consideration, failure of a condition, or that the contract is void or voidable.

The parol evidence rule does not apply in any way to evidence of agreements between the parties subsequent to the time the writing was signed.

VII. CONDITIONS
A. EXPRESS CONDITIONS 1. Creation Of Condition
Performance by one or both of the parties may be made expressly conditional in the contract. The effect of such condition is that some event must happen before performance is due. For example, a vendor and purchaser of real estate may provide that the contract is "conditioned upon the purchaser's ability to obtain a mortgage." Express conditions can be identified by the language used. Normally, such words as, "if," "provided that," "on the condition that," indicate an express condition. Also, such language as "when I get my Christmas bonus . . . " indicates a condition.

2. Burden Of Proof On Performance


Express conditions can be "precedent" or "subsequent." A condition precedent means the event must occur before any obligation to perform arises. A condition subsequent, on the other hand, means the obligation exists under the contract, but it will be discharged by the happening of an event. The only distinction between conditions precedent and subsequent is procedural. If the condition is precedent, the plaintiff has the burden of proving that the condition occurred in order to recover; whereas, if the condition is subsequent, the defendant must prove the happening of the condition to avoid liability. The Second Restatement does away with this distinction in terminology, but the language used in making the condition or event of discharge and the policies involved in allocating the burden of proof will be examined.

3. Satisfaction
If a contract provides that performance will be "to the full satisfaction" of the other party, this amounts to a condition. When the aesthetic taste of an individual is involved, e.g., when a contract is for the enlargement of a photograph, the person is not liable if he or she in good faith determines that the work is not satisfactory. If, however, the contract requires performance not involving aesthetics but rather mechanical fitness, utility, or marketability, the question is whether a reasonable person would be satisfied with the performance.

24

MicroMash MBE In Brief: Contracts

4. Architect's Certificate When a construction contract calls for an architect's certificate of performance, this is treated as an express condition, and the builder will not be able to recover unless the certificate is procured and presented to the landowner. There is no excuse for this condition, and no recovery will be granted without it unless there is evidence of fraud or collusion between the owner and the architect. Even if the architect makes a mistake or fails to exercise reasonable judgment in refusing to give the certificate, recovery will be denied on the contract. In such cases, however, recovery can be had by the builder in quasi contract for the reasonable value of the work and the materials.

B. CONSTRUCTIVE
1. Constructive Conditions Of Exchange When parties enter into a bilateral contract, the doctrine of constructive conditions of exchange provides that a party to the contract cannot recover unless he or she has either performed or tendered performance to the other party. Thus, if a builder agrees to construct a house on the owner's land, the builder will not be able to recover from the owner unless he completes the building. 2. Partial Performance a. Meaning To alleviate the harshness of the above rule when a party has almost but not fully completed performance, the courts have developed the doctrine of substantial performance. This doctrine provides that if a party substantially performs, he or she can recover on the contract even though full performance has not been tendered. What constitutes "substantial performance" varies from case to case, but it means something like "almost fully performing." b. Applicability When the breach is a "willful breach," no recovery is allowed under the doctrine of substantial performance. A willful breach, however, requires something more than knowledge, and implies some attempt to cheat the other party or to provide less than was called for by the contract. The doctrine of substantial performance does not apply either to express conditions, or to contracts for the sale of goods. c. Measure of damages When a builder recovers under the doctrine of substantial performance, the primary measure of damages is the contract price minus the cost of replacing or repairing any defect in the builder's performance. This gives to the owner sufficient money to get what she bargained for. Where, however, constitutes "economic waste," the measure of damages applied is the contract price minus any diminution in value resulting from the defective performance.

MicroMash MBE In Brief: Contracts

25

d. Waiver
The owner may "waive the breach" by accepting the performance of the builder knowing that the condition was not fulfilled. Waiver is defined as "a voluntary relinquishment of a known right." If there is a waiver by the owner, he can still recover damages for any defect in the performance. Thus, the end result is similar to a situation where the substantial performance doctrine is applied.

3. Strict Performance By Seller Under The U.C.C.


The basic obligations of a seller are: (1) to transfer ownership of the goods to the buyer; and (2) to tender goods conforming to the warranty obligations.

a. Transfer of ownership
Unless the contract provides otherwise, the seller must convey clear title to the goods; otherwise, there is a breach of the warranty of title.

b. Tender of the goods


The seller must tender the goods in accordance with the contract if there are specific provisions on tender; otherwise, she must tender them in accordance with the code provisions. There are basically four methods of tender stated by the code, the correct one in a particular transaction depending upon the place of tender: Seller's place of business: the seller must place the goods at the disposition of the buyer and give the buyer notice. Shipment contract: the seller has a duty to ship the goods, but no duty to deliver them to a particular place. The seller must deliver to the carrier, make a proper contract for their shipment, and give the buyer notice that the goods have been shipped. Destination contract: the seller must deliver the goods to a particular place, and tender them there by holding the goods at the buyer's disposition and giving the buyer notice. Where goods in hands of bailee: the seller must negotiate a negotiable document of title or obtain acknowledgment from the bailee of the buyer's rights in the goods.

Mercantile terms F.O.B., etc. can affect the seller's obligations. F.O.B. is the most common term. If it is F.O.B. seller's place of business or city, it is a shipment contract, and the seller is obligated only to put the goods in the possession of the carrier. If it is F.O.B. buyer's place of business or some point other than seller's place of business, it is a destination contract, and the seller at her own expense and risk must transport the goods to the specified place.

26

MicroMash MBE In Brief: Contracts

4. Buyer's Obligations
When a conforming tender is made, the buyer is obligated to accept. If he accepts, this amounts to performance of one of the buyer's obligations. If the buyer rejects, he has breached the contract.

5. Divisible Or Installment Contracts a. At common law


A divisible or installment contract is one in which the various units of performance are divisible into separate parts. In other words, part of the consideration by one person is set against part of the consideration promised by the other. Typical divisible contracts are employment contracts for a set period of time with payment to be made periodically. Thus, a contract for employment during a six-month period, with a provision that the employee is to be paid at the end of each week, is a divisible contract and the separate units are the individual weeks. A contract for the construction of a building where the owner is to make progress payments when certain parts of the building are completed, is not a divisible contract unless expressly made so. The theory is that the payments are simply to allow the builder to continue construction, and that the owner wants a completed building, not parts of a building. If a contract is divisible, the doctrine of constructive conditions of exchange is applied to each part of the contract separately. Hence, a person can recover the amount promised for a segment of the contract, even though she does not perform the other segments, e.g., an employee can recover the week's salary even though she quits at the end of the week. Damages are recoverable by the other party for breach of the other segments.

b. Installment contracts under the U.C.C. 1) Defined


An installment contract is defined as one in which the goods are to be delivered in a number of shipments, and each shipment is to be separately accepted by the buyer. Payment by the buyer is due upon each delivery, unless the price cannot be apportioned.

2) Effect of breach as to that installment


Where there is an installment contract and the seller makes a nonconforming tender or tenders nonconforming goods under one segment of the contract, the buyer may reject only if the nonconformity substantially impairs the value of that shipment to the buyer, and if the nonconformity cannot be cured. Thus, in an installment contract, the strict performance rule does not apply; rather, there is something like the substantial performance rule. Of course, even if the buyer is forced to accept because the nonconformity does not substantially impair the value of the shipment, the buyer may recover damages for any loss resulting from the nonconformity.

MicroMash MBE In Brief: Contracts

27

3) Effect of breach on rest of contract


Where there is a nonconforming tender or a tender of nonconforming goods under one segment of an installment contract, the buyer may call off the rest of the contract, i.e., cancel the contract, only if the nonconformity substantially impairs the value of the entire contract to the buyer.

C. IMPLIED DUTIES OF GOOD FAITH AND FAIR DEALING


A duty of good faith and fair dealing is implied into all contracts. A subspecies of this duty is a duty not to hinder the other party's performance and a duty to cooperate, where necessary.

D. SUSPENSION OR EXCUSE OF CONDITIONS BY WAIVER, ELECTION, OR ESTOPPEL


Even though a party to a contract would be unable to sue successfully on the contract because he has not complied with an express or implied condition, he can postpone the time in which he must comply with the condition if it is suspended. He can possibly sue successfully despite the existence of the condition if he can show that it was waived, that the other party elected his rights in a way that terminated the condition, or that the other party is estopped from raising the condition as a defense to suit. If a condition is suspended, the party having the benefit of the condition has only postponed the time in which he can interpose the noncompliance with the condition as a defense to the contract. When the period of suspension has expired, the condition is restored to its original effect. On the other hand if the condition is excused, then the party having the benefit of the condition can never raise it as a defense.

1. Waiver
A waiver occurs when a party, having the benefit of a an express or implied condition which the other party must comply with in order to perform her obligations under a contract voluntarily and knowingly relinquishes that right. The relinquishment can occur explicitly by language or implicitly by action. If a waiver occurs, an issue can arise concerning its scope. If the waiver is of a condition that is effective at one time only, then the waiver is irrevocable and cannot be unilaterally reinstated even if no consideration is given for the waiver. In some cases the waiver is not a total relinquishment of the condition but only a postponement of it. After the period of postponement has elapsed, the condition is reinstated. If there is an installment or divisible contract, a waiver with respect to conditions on one installment is not a waiver for all future installments unless the language of the waiver clearly indicates that the waiver applies to future installments.

2. Election
In some contracts, one party has the choice of alternative methods of performance from

the other party. For example, a contract could require a seller to deliver either red

28

MicroMash MBE In Brief: Contracts

widgets or blue widgets at the buyer's election that is to be communicated at a specific time. If the buyer makes an election for red widgets, then the seller has performed his contractual obligation by delivering red widgets even though the seller later changes his mind and asks for blue widgets.

3. Estoppel
A party is estopped to insist upon satisfaction of a condition as a defense to performance by the other party, if she by her actions creates an impression that she will not insist on a condition, and the other party reasonably relies on that impression to her detriment. She has taken a position indicating that the condition will not be enforced and if estoppel applies, will not be able to take a position inconsistent with her previous position.

Reasonable reliance is an essential element of estoppel. If the buyer realized that the seller was under the impression that she could deliver at a later date, and called the seller to correct that impression before the seller reasonably relied on that impression, the seller would not have the defense of estoppel.
If the contract were divisible, and one party created an estoppel so that she could not enforce the condition with respect to one party of the contract, she could insist on the condition in later portions of the contract if she insisted on the condition before the other party reasonably relied with respect to later portions of the contract.

E. PROSPECTIVE INABILITY TO PERFORM, EFFECT ON OTHER PARTY


Between the time a contract is made and the time for performance, each party should enjoy a reasonable expectation that performance will be forthcoming when due. A party's expectations of performance may be diminished by the occurrence of some event after the contract was made. A common example of this is a credit sale when the buyer becomes insolvent prior to the time the goods are delivered. Such cases have generally been classified as examples of prospective inability to perform. The law is less settled in the area of prospective inability to perform than it is in cases of anticipatory repudiation, and in general, there is a reluctance to excuse a party from performing solely on the ground that he does not expect counter-performance to be given.

1. Seller's Remedies Under The U.C.C.


The seller can cancel credit terms or stop goods in transit if the buyer becomes insolvent before delivery of the goods. Either party can demand assurance of performance if she has reasonable grounds for insecurity about the other party's ability or willingness to perform. Assurances must be in writing, and until they are provided, the other party may suspend performance.

2. Stoppage In Transit
A seller who has shipped goods to a buyer may stop the goods in transit if the buyer either becomes insolvent or breaches the contract. If the ground is insolvency, then stoppage is available even if the goods were sent in less than carload lots; otherwise, only if shipped in carload lots.

MicroMash MBE In Brief: Contracts

29

The right to stop in transit terminates where: buyer has received the goods; or carrier or warehouseman has acknowledged buyer's rights; or goods have been reshipped by carrier; or a negotiable document of title has been given or negotiated to the buyer.

VIII. REMEDIES A. TOTAL AND PARTIAL BREACH OF CONTRACT 1. Buyer's Remedies Under The U.C.C. For Breach Of Contract By The Seller
When the seller's time for performance arises, the seller may do one of three things: nothing (breach by seller); make a nonconforming tender (breach by seller); make a conforming tender (performance by seller). The seller must strictly perform all obligations under the contract. There is no doctrine of substantial performance, except in installment contracts.

a. If seller makes no tender


If the seller does not tender the goods when the time for performance arises, there is a breach by the seller. The buyer has three basic remedies:

(1) Damages: (market price minus contract price); (2) Cover: Purchase similar goods elsewhere and sue for cover price minus contract price; (3) Specific Performance: Unique goods; or (4) Replevin: Identified goods where similar goods are unavailable in the marketplace.
The buyer also can obtain incidental and consequential damages where damages or the cover remedy is sought. The buyer can obtain the goods from the seller if partial payment has been made and the seller becomes insolvent within ten days of payment. The buyer must tender the rest of the price.

b. If seller makes nonconforming tender


If either the tender is nonconforming or the goods are nonconforming, the buyer has the right to reject all, accept all, or reject some and accept some. The buyer has the right to inspect the goods before deciding whether to accept or reject. Inspection can be at or in any reasonable time, place, and manner.

30

MicroMash MBE In Brief: Contracts

1) Rejection Rejection requires the buyer to give the seller notice that he is rejecting. If the buyer has possession of the goods at the time of rejection, he must hold them for a reasonable time to allow the seller to remove them. If the seller has no local agent at the place of rejection (usually the city in which the buyer is located), the buyer's obligations depend upon whether the goods are perishable. In the absence of other instructions from the seller, (1) the buyer must sell perishable goods for the account of the seller; and (2) the buyer may store non-perishable goods at the seller's expense, reship them to the seller, or sell them for the seller's account. If the buyer rejects, the buyer has the same basic remedies as if no tender was made by the seller: (1) damages, (2) cover and cover damages, and (3) specific performance or replevin. Incidental and consequential damages can also be recovered. 2) Acceptance Acceptance occurs when: (1) the buyer says she is accepting, (2) the buyer uses the goods, or (3) the buyer fails to reject. If the buyer accepts a nonconforming tender, the buyer can still recover for any damages resulting from the defect, provided notice of the defect is given within a reasonable time. If the nonconformity is in the goods (breach of warranty), then the measure of damages is the value the goods would have had if as warranted, minus the actual value with the defect. If the breach was in the tender obligation (e.g., delivered late), then the buyer may recover all losses resulting in the ordinary course of events. The buyer can revoke acceptance (thus putting herself in the same position as if she had rejected), if (1) she accepted with a reasonable expectation that the seller would cure and the seller did not, or (2) the defect was hidden. 3) Right to cure The seller has a right to cure a defective tender if: there is still time to perform under the contract; or the seller had reasonable grounds to believe the buyer would accept despite the nonconformity.

If the seller cures, it is just as though he had made proper a tender in the first place. 2. Seller's Remedies Under The U.C.C. For Breach by Buyer a. Seller's right to price upon acceptance If the buyer has accepted the goods, the seller has a right to the price. The seller's remedy when the buyer refuses to pay the price is to sue for the price.

MicroMash MBE In Brief: Contracts

31

The amount is generally set by contract, but if not, the buyer must pay a reasonable price. The parties can postpone agreement about the price when they enter the contract. Payment by check is permitted unless the seller demands cash. If the seller demands cash, the buyer is entitled to a reasonable time to obtain it. The price is due after the goods are physically delivered to the buyer and the buyer has an opportunity to inspect, unless the contract provides otherwise. The buyer has no right to inspect before paying in the following cases: C.O.D. contract; C.I.F. or C.& F. contract; payment is against documents.

Of course, payment does not constitute "acceptance" if there is no right of inspection before payment. b. Seller's right to reclaim goods If the buyer was insolvent when the goods were delivered and the price is not paid, the seller can recover the goods if demand is made in ten (10) days, provided no good-faith subpurchaser has bought the goods from the buyer. In a C.O.D. sale, if the buyer has given a worthless check, the seller can reclaim the goods. c. Seller's remedies upon wrongful rejection If the buyer wrongfully rejects, the seller has three basic remedies: damages, resale, or recovery of the price. The seller may also recover incidental damages along with contract damages or resale. d. Damages for wrongful rejection The measure of damages is generally contract price minus market price. However, if Contract Price minus Market Price does not fully compensate the seller, she can sue for lost profits as an alternate measure of damages. In either case the seller is entitled to incidental damages and the buyer must be credited with any partial payments and with expenses saved by the seller (e.g., by the seller's resale of the goods). e. Resale If the seller elects to resell and sue for contract price minus resale price, the resale must be: only of goods identified to the contract, and commercially reasonable.

f. Price

The seller can recover the price after rejection only if the goods are not salable in the
seller's ordinary course of business.

32

MicroMash MBE In Brief: Contracts

The seller can retain deposits paid by the buyer up to the amount stated in a liquidated-damages clause, or, in the absence of such a provision, (2) 20% of the value of performance, or $500, whichever is less.

B. ANTICIPATORY BREACH
1. At Common Law a. Repudiation of promise The doctrine of anticipatory breach is applicable when a promisor, before the time for performance arises, repudiates his promise. A repudiation must be "clear and unequivocal." It may be by acts instead of words. The bankruptcy of the promisor, prior to performance, has been held a repudiation. However, a good-faith denial of an obligation under the contract is not a repudiation. b. Promisee's options When a promisor repudiates, the promisee has the option of ignoring the repudiation and demanding that the promisor perform at the agreed upon time, or treating the repudiation as a breach of the contract. If treated as a breach, the promisee may cancel the contract, sue for damages or, if appropriate, seek specific performance. If the promisee ignores the repudiation, he cannot continue to perform under the contract if this will increase the damages of the promisor. The repudiation relieves the promisee of any obligations under the contract. However, he must be able to prove that he would have been "ready, willing, and able" to perform but for the repudiation. c. Retraction of repudiation A repudiation may be retracted until the promisee: (a) acts in reliance on the repudiation; (b) signifies her acceptance of the repudiation; or (c) commences an action for breach of contract. Notice of the retraction sufficient to allow the promisee to perform her contractual obligations must be given. d. Unilateral contracts The doctrine of anticipatory repudiation does not apply to unilateral contracts and may not be used if the promisee has completed performance prior to the repudiation. 2. Anticipatory Repudiation Under The Uniform Commercial Code Anticipatory repudiation occurs when there has been an unequivocal refusal of the buyer or seller to perform, or when a party creating reasonable grounds for insecurity fails to provide adequate assurances within 30 days of demand for assurances. The repudiation can be retracted if the other party has not canceled the contract or materially changed position. Repudiation allows the nonrepudiating party to resort to any remedy given by the contract or code.

MicroMash MBE In Brief: Contracts

33

C. ELECTION OF SUBSTANTIVE RIGHTS AND REMEDIES When the defendant induces the plaintiff to enter into a contract by fraud, or when the defendant materially breaches the contract, the plaintiff has three types of remedies available to him. In limited instances he can require that the defendant perform the contract by obtaining a decree for specific performance.

He can seek damages for breach of contract. He can rescind the contract and recover what he put into performing the contract. The issues are when he can pursue more than one of these remedies, and when the election to pursue one remedy bars him from changing his mind and pursuing another, and what action is considered an election.
1. Multiple Recoveries Not Permitted

One principle that is clear is that the plaintiff cannot pursue multiple remedies in order to obtain a double recovery. For example the plaintiff could not obtain a decree for specific performance of a land contract and at the same time obtain expectancy damages for breach of the contract to convey. On the other hand, she would be able to obtain specific performance some incidental damages, related to her equitable remedy. In a lawsuit for breach of contract, a plaintiff under the Federal Rules of Civil Procedure and similar state procedural rules can bring a lawsuit asking for inconsistent remedies such as specific performance and damages, but at the successful conclusion of the lawsuit, she will only obtain a judgment for one remedy. If her preferred remedy is specific performance, but the court in its discretion refuses to order the contract performed, it can order the damage remedy in the alternative.
2. When Does Election Occur

The most common situation involving election involves the choice between damages, where the plaintiff affirms the existence of the contract and seeks monetary compensation for the difference between the performance he bargained for and that which he received, and rescission where the plaintiff seeks to restore the status quo which existed before the contract was formed. It is clear that if one of these alternative remedies is pursued to judgment, the plaintiff cannot change his mind and pursue the other. The procedural rules of res judicata would bar such action. Likewise, if the plaintiff communicates to the defendant that he is seeking one of these alternative remedies and the defendant relies on that representation, the principals of estoppel will prevent the plaintiff from abandoning one remedy and seeking the other. Some courts have found an election at a much earlier stage, and prohibited the plaintiff from changing his mind and seeking a different remedy once he has communicated his choice of remedy to the defendant, even if the defendant has not relied upon that decision.

34

MicroMash MBE In Brief: Contracts

A plaintiff may also be denied the remedy of rescission if he has knowledge of the basis for seeking rescission and fails to pursue that remedy for a period of time during which the defendant changes his position. The plaintiff in this case would still be entitled to the damages remedy, as long as the statute of limitations has not expired. 3. Election Of Remedies Under The Uniform Commercial Code Comment to U.C.C. Section 2-703 dealing with seller's remedies provides that the article rejects any doctrine of election of remedy as a fundamental policy and thus the remedies are essentially cumulative in nature and include all of the available remedies for breach. Whether the pursuit of one remedy bars another depends entirely on the facts of the individual cases. In dealing with buyer's remedies 2-711 of the code provides that the buyer may cancel and whether or not she has done so may in addition to recover so much of the price as has been paid, obtain damages based upon cover or otherwise.

D. SPECIFIC PERFORMANCE, INJUNCTION AGAINST BREACH, DECLARATORY JUDGMENT


1. In General When damages are an inadequate remedy, equity will order specific performance of the contract. If the equity decree is not observed, the breaching party is in contempt and may be fined or imprisoned. 2. Factors Considered In determining whether damages are adequate, the following factors should be taken into consideration: the difficulty of proving damages with reasonable certainty (e.g., the difficulty may be posed by sentimental associations and aesthetic interests, not measurable in money, which would be affected by breach); the difficulty of procuring a suitable substitute performance by means of money awarded as damages; and the likelihood that an award of damages could not be collected.

3. Real Property Because every parcel of real property is considered unique, contracts involving the transfer of an interest in real property may be enforced by an order of specific performance. 4. Limitations On Court's Powers To Order Specific Performance Even if the remedy of damages is inadequate, specific performance will not be granted where the court cannot supervise enforcement. Thus, courts do not grant specific enforcement of contracts for personal services, although they may restrain the breaching party from working for another. For example, if a performer agrees to sing in a particular theater, the court will not grant a decree ordering the person to sing at that theater, but it

MicroMash MBE In Brief: Contracts

35

will grant a decree restraining the performer from appearing at competing theaters. Lumley v. Wagner, 42 Eng. Rep. 687 (1852). Also, courts will usually refuse to grant specific performance in an action where the act or forbearance will occur outside the jurisdiction of the court. Thus, if A agrees to sell land to B, and both A and the land are located outside the jurisdiction of the court, the court will usually not grant a decree of specific performance upon A's refusal to deed the land. Since a decree of specific performance is an equitable remedy, the usual rules for seeking equitable relief apply, e.g., the clean hands doctrine. Whether the court will grant the relief requested is always within the discretion of the court.

5. Specific Performance Under The U.C.C.


In sale of goods cases, 2-716 of the Uniform Commercial Code provides that specific performance may be granted where the goods are unique or in other proper circumstances, such as for breach of a requirements contract where there is not another convenient supplier.

6. Declaratory Judgment
If the rights and obligations of the parties under a contract are unclear, and an actual dispute exists between the parties concerning those rights and obligations, either party may bring a declaratory judgment action to obtain an adjudication of those rights and duties. It is not available, however, to resolve moot issues or theoretical problems which have not risen to an actual dispute. For example, this form of litigation can be brought even before a breach of contract, to adjudicate whether a specific proposed action by one party would constitute a breach. If the court found that a specific action constituted a breach and a party pursued that course of action, the principles of collateral estoppel would constitute an adjudication of breach which could not be relitigated at a subsequent action for damages. Thus a declaratory judgment action is often an effective, indirect way of enforcing a contract prior to a breach.

E. RESCISSION AND REFORMATION 1. Rescission


The non-defaulting party to a contract can choose the rescission remedy, which cancels the contract, and requires a return of any deposits or other benefit conferred on the other party. The rescission remedy is advantageous to the nonbreaching party when he has made a disadvantageous contract, and will be better off if he can completely undo the contract. For example, if homeowner hires contractor to paint his house for two thousand dollars, and gives contractor a five hundred dollar contract, and contractor defaults on the contract by not starting to paint the house in a timely way, homeowner can rescind the contract and get his five hundred dollars back, and then make a more advantageous agreement with another painter to paint his house for fifteen hundred dollars. Rescission can also occur by the mutual agreement of the parties.

36

MicroMash MBE In Brief: Contracts

2. Reformation When reformation of the contract is available to cure a mistake, neither party can avoid the contract. For example, assume that A agrees to sell Redacre to B, with B agreeing to pay $50,000 and to "assume a mortgage in the amount of $100,000." If the parties fail to include a provision regarding the assumption of the $100,000 mortgage, A can obtain reformation of the agreement to reflect B's promise. A has no right to avoid the contract because reformation adequately remedies the mistake in drafting the written agreement. F. MEASURE OF DAMAGES IN MAJOR TYPES OF CONTRACT AND BREACH The primary objective of contract damages is to put the nonbreaching party in the same position that she would have been in had the contract been performed. Thus, it is sometimes said that the plaintiff in a contract action is entitled to the "benefit of her bargain" or her "expectancy damages." Although "expectancy damages" are the normal means of determining damages in a contract case, two alternative types of damages are recognized: "restitution damages" and "reliance damages." In both of these the plaintiff is not to be put in the position that she would have been in had the contract been performed, but rather to be put in the position she was in at the time the contract was made. In addition, a plaintiff may have suffered damages that go beyond loss of the benefit of her bargain. These are classified as "consequential damages." 1. Expectancy Damages a. In general Expectancy damages are normally measured by a formula that looks at the value of the performance of the breaching party and the consideration promised for that performance. The general formula is the market value of the promised performance less the consideration promised by the nonbreaching party. For example, assume a contract were for the painting of Homeowner's house which was breached by Painter. The damages would be Homeowner's cost to hire a new painter less the original contract price. Thus, if the original contract price was $2500 and Homeowner, after Painter's breach, hired Brush to paint the house for $3000, Homeowner's damages would be $500. This $500 damages from Painter puts Homeowner in the position he would have been in had the contract been performed he has his home painted for a net cost of $2500. Conversely, assume that that Homeowner had breached in the example above, and Painter's cost of painting the house was $2000, Painter would be able to collect $500 in damages. Even if no second contract is entered into by which to measure damages, the same basic formula exists: In contracts for services; compare the difference between the value of the services to be performed under the contract with the price that was promised for those services.

MicroMash MBE In Brief: Contracts

37

b. Partial performance
If one of the parties has partially performed at the time of the breach of the other party, the performing party can recover for work done at the contract rate plus "expectancy damages" for the work not yet performed. Thus, assume a contractor agreed to paint O's house for $10,000, with $9,000 representing the cost of performance and $1,000 representing the contractor's profit. If 0 breaches when Contractor is half finished, having thus far incurred $4,500 in costs, Contractor could recover the $4,500 and the $1,000 in anticipated profit, for a total of $5,500. She could not recover the remaining $4,500, which represents the costs not incurred by not yet finishing the contract.

2. Nominal Damages
Damages are not an essential element in a cause of action for breach of contract. If no damages are alleged or none are proved, the plaintiff is still entitled to a judgment for "nominal" damages: usually six cents or $1.

G. CONSEQUENTIAL DAMAGES, CAUSATION, CERTAINTY, AND FORESEEABILITY 1. Consequential Damages


A breach of contract may result in damages to the nonbreaching party that go beyond the difference between the value of the nonbreaching party's performance and what the nonbreaching party would have received had there been no breach. For example, if a contract calls for the construction of a motel and the builder fails to perform, the basic measure of damages would be the difference between the contract price and the amount that it cost the owner to have someone else construct the building. However, the failure to construct the building may cause additional damages to the owner in the way of lost profits because of delay, or by causing him to breach contracts he may have made with third parties for the use of the building. If such damages can be proven by the plaintiff and were foreseeable by the breaching party at the time of the contract, consequential damages are recoverable.

2. Causation
Contract law has a doctrine similar to the requirement of "but for" causation in tort law. If the plaintiff is seeking damages, the defendant can defend on the ground that the losses that the defendant seeks to recover would have occurred whether or not the defendant breached her contract. This issue can be simply illustrated where the plaintiff seeks to recover damages from a components supplier who delivered late, because that supplier knew that the plaintiff would be penalized if she delivered the final product beyond a specified date. If the defendant delivered the parts late, but at the time they should have been delivered the plaintiffs employees were on strike so that the final product could not have been delivered on time even if the parts were delivered on time, the plaintiff could not recover for delay from the defendant, because the defendant's breach was not a "but for" cause of the plaintiffs damage.

38

MicroMash MBE In Brief: Contracts

3. Certainty In order to recover damages, a plaintiff must prove the dollar amount of the damages with reasonable certainty. To put this in the negative: the damages must not be too speculative. The Restatement (Second) of Contracts 352 requires that the evidence of the amount of damage must afford "a sufficient basis for estimating their amount with reasonable certainty." This does not mean that the plaintiff can recover nothing if part of the damages he attempts to prove fails to meet the requirement of certainty; those damages that were proved with sufficient certainty can be recovered, although others may not be allowed. The Restatement gives the following examples, among others, of the application of this rule: A and B contract to form a partnership and to continue it for a specified period. B dissolves the partnership prior to the time specified. A could recover damages for lost profits by proving what the profits were during the life of the partnership. A contracts to allow B to operate his established coal mine and to pay B $25 per ton for coal produced. A breaches the contract by refusing B entry to the land. If the mine has been operating for an extended period of time and the veins are well established, B can recover damages by showing the cost of producing the coal, the amount that could have been produced during the contract period, and the market price at which it could have been sold.

Courts are hesitant to award damages for lost profits because they are, in the eyes of most courts, too speculative. This is particularly true in a new or relatively young enterprise. For example, if a plaintiff is suing to recover lost profits because of the defendant's failure to deliver a boiler necessary for plaintiff's new factory, a court may award the plaintiff the rental value of his factory during the time it was inoperative, but will not grant an award on the presumption that the new enterprise would have made a profit. 4. Foreseeability And The Rule Of Hadley v. Baxendale The leading case on consequential damages is Hadley v. Baxendale, 9 Exch. 341 (1854), which involved an action by the owner of a factory against a carrier. The owner had contracted to have a shaft needed to run machinery in the factory transported to a third party, who was to use it as a model for the manufacture of a new one. The shipment was delayed by the defendant-carrier, and the plaintiff asked damages to compensate for the loss resulting from the stoppage of work in the factory during the delay. The court held that the damages recoverable in a contract action are those which "may fairly and reasonably be considered either arising naturally, i.e., according to the usual course of things, from such breach of contract itself, or such as may reasonably be supposed to have been in the contemplation of both parties at the time the contract was made, as the probable result of the breach of it." Thus, the rule under Hadley v. Baxendale is that damages are recoverable if they were the "natural and probable consequences," or if they were "in the contemplation of the

MicroMash MBE In Brief: Contracts

39

parties at the time the contract was made." In other words, the damages must be "foreseeable." H. LIQUIDATED DAMAGES AND PENALTIES Parties to a contract may fix the amount of damages that will be recoverable in the event of breach; however, a party may not be "penalized" for her breach of contract. Therefore, penalty clauses in a contract are unenforceable. A provision for liquidated damages will be enforced, and not construed as a penalty, if the amount of damages stipulated in the contract is reasonable in relation to either the actual damages suffered, or the damages that might be anticipated at the time the contract was made. I. RESTITUTION AND RELIANCE RECOVERIES 1. Restitutionary Damages Restitutionary damages restore to the plaintiff whatever benefit he has conferred upon the defendant prior to breach. Restitutionary damages may be recovered even though the plaintiff would have suffered a loss had the defendant not breached. They may be combined with expectancy damages. 2. Reliance Damages Reliance damages may be recovered where the nonbreaching party has incurred expenses in reasonable reliance upon the defendant's performing her contractual obligations. Unlike restitutionary damages, there is no requirement that the defendant have benefited from the plaintiffs expenditures. J. REMEDIAL RIGHTS OF DEFAULTING PARTIES A party who commits a material breach of his contract obligations cannot sue for contract damages. For example, assume Painter enters into a contract with Homeowner to paint Homeowner's house for $5,000 by July 1 payment to be made upon completion of the job. If Painter abandons the job after the home is 60% painted, he may not collect $3,000, 60% of the contract price. However, if Homeowner hires Brush who completes the work for $4,000, Painter has conferred a benefit upon Homeowner for the fair value of his work in painting the house and can sue in quantum meruit for the fair value of the benefit conferred. In this case the court would probably find that the benefit conferred was $1,000 because Homeowner placed a value of $5,000 in having his house painted, and only expended $4,000 to have Brush complete the job. K. AVOIDABLE CONSEQUENCES A party to a contract has the obligation of avoiding or mitigating damages to the extent possible by taking such steps as do not involve undue risk, expense, or inconvenience. The nonbreaching party is held to a standard of reasonable conduct in preventing loss. Thus, the plaintiff in a contract action cannot recover damages which were foreseeable by her, and which she could have avoided by the expenditure of reasonable effort. For example: A contracts to manage B's farm for a year. Several weeks before the planting season, A quits. If B could have found another manager by the exercise of reasonable effort, B cannot recover for damages resulting from the fact that no crops were planted.

40

MicroMash MBE In Brief: Contracts

L. UNIFORM COMMERCIAL CODE ISSUES


1. Risk Of Loss a. General rules
Where the goods are lost or destroyed through no fault of the seller or the buyer, a determination must be made as to who must initially suffer this loss. The basic rule is that the risk of loss is on the seller until he or she completes his or her delivery obligations under the contract. Thus, if there is a "shipment contract," the seller completes the delivery obligations by tendering the goods to the carrier, and making a proper contract for their shipment. At that time, there is a shifting of the risk of loss from the seller to the buyer, and if the goods are destroyed or lost while they are in transit, the risk of loss is on the buyer and the buyer must pay the contract price.

b. Special rules
There are three "special" rules regarding risk of loss, all of which deal with situations

where one of the parties is in breach:


Where the seller delivers nonconforming goods, the risk of loss remains on the seller until the buyer accepts or there is a cure. Where the buyer rightfully revokes his or her acceptance, the risk of loss shifts back to the seller to the extent of any lack of insurance coverage by the buyer. Where the buyer repudiates or breaches after the goods have been identified to the contract but before the risk of loss has shifted to him, the risk of loss is immediately shifted to the buyer to the extent of any lack of insurance coverage on the part of the seller.

2. Insurable Interest a. Seller's insurable interest


In order for a person to insure goods, he or she must have an "insurable interest" in the goods. Under Article 2, the seller retains an insurable interest as long as he or she has title to the goods or has a security interest in them. Absent a contrary provision in the contract, the title passes from the seller to the buyer when the seller completes his or her delivery obligations. Thus, at that time, the seller's insurable interest ceases unless the seller retains a security interest in the goods. Remember that a seller does not automatically get a security interest, even though the buyer is to pay sometime in the future. There must be a voluntary granting of a security interest by the buyer to the seller in accordance with the provisions of Article 9 of the code.

b. Buyer's insurable interest


The code provides that the buyer obtains an insurable interest in the goods as soon as the goods are "identified to the contract."

MicroMash MBE In Brief: Contracts

41

c. Overlapping insurable interests It is possible for both the seller and the buyer to have insurable interests at the same time under the provisions of Article 2 of the code. 3. Title And Good-Faith Purchasers a. Entrusting provisions The code provides that delivery of goods by the owner to one who sells goods of that kind gives to the transferee the power to convey good title to a buyer in ordinary course. Thus, if I give my watch to a jeweler to have it repaired and the jeweler also sells used watches, if the jeweler sells the watch to a buyer in ordinary course, the buyer would get good title even against me. A buyer in ordinary course is one who in good faith and without knowledge of a third party's ownership rights or security interest, buys from someone selling goods of that kind. b. Voidable title Where the true owner of goods sells them to another, but the sale is voidable because of fraud or because it was a cash sale and the buyer has not paid or has paid with a dishonored check, the code takes the position that the buyer has the power to transfer good title to a good-faith purchaser. This is true even though the fraud may be punishable as larceny under the local law.

IX. IMPOSSIBILITY OF PERFORMANCE


A. IMPOSSIBILITY Impossibility of performance arises when the performance of one of the parties becomes impossible. It is essential that the event giving rise to the impossibility be unforeseeable at the time the contract was made. The typical situations are: where the performance becomes illegal after the contract is made; where there is a lease or sale of specific goods and they are destroyed; where there is a personal services contract, and the person who is to provide the services dies or becomes incapacitated.

When impossibility of performance arises, a party is excused from performing the contract. However, a person cannot require that the other party accept substitute performance. If the parties desire, one or the other can assume a greater risk in their contract, e.g., become liable to pay damages even if performance becomes impossible. Also, a liquidated damages clause can be made to apply to impossibility situations by specific agreement of the parties.

B. IMPRACTICABILITY OF PERFORMANCE 1. Total Impracticability


Impracticability of performance provides an excuse (defense) for nonperformance of the contract similar to impossibility in contract law. This excuse exists when: goods identified at the time of contracting are destroyed;

42

MicroMash MBE In Brief: Contracts

performance becomes illegal; or performance has been made "impracticable."

The excuse applies only when the event was not foreseeable and when nonoccurrence of the event was a basic assumption of the contract. 2. Partial Impracticability When impracticability affects only part of the seller's ability to perform, the goods actually produced must be apportioned among all the buyers with whom the seller has contracted. The buyer, however, may refuse to accept and may cancel the contract. When the agreed method of transportation or payment becomes impracticable: the performing party must use a commercially reasonable substitute if available, and the substitute performance must be accepted.

C. FRUSTRATION OF PURPOSE The doctrine of frustration of purpose applies when the value of the contract is totally or almost totally destroyed for one of the parties. Again, the event destroying the value of a contract must have been unforeseeable. If the doctrine applies, the party is excused from performing the contract. However, American courts very rarely find the doctrine applicable.

X. DISCHARGE
A party's duties under the contract may be discharged by full performance or without full performance by operation of law or by act of the parties. Discharge by operation of law occurs when enforcement of the obligor's duty is discharged in bankruptcy or by the running of the statute of limitations. A duty may be discharged by impossibility or frustration of purpose. Discharge may occur pursuant to the terms of the contract by nonoccurrence of a condition precedent to performance or by occurrence of an event of discharge. A duty may be discharged by merger into an award in arbitration, into a judgment, or into a substituted contract or novation. The parties may also modify their duties by means of a mutual agreement of rescission, an accord and satisfaction, or an account stated. Unilateral forms of discharge, such as a renunciation of rights or a release, generally must be supported by consideration. The most common exception today is that a renunciation or release contained in a signed and delivered writing will be effective under the Uniform Commercial Code without consideration.

MicroMash BAR REVIEW BAR EXAM ALERTS AT A GLANCE


-

CONTRACTS

I. FORMATION OF CONTRACTS
A. MUTUAL ASSENT 1. OFFERS
A communication is an offer for a bilateral contract if it sets forth a proposed exchange of promises in such a manner that the person to whom it is directed reasonably believes that she can enter into a binding contract by accepting those terms. The person selling goods at auction is not bound by the highest bid unless she advertises the auction as "without reserve," in which case placing the goods at auction is making an offer to the highest bidder. 2. ACCEPTANCE At common law, an offer can only be accepted by the offeree agreeing, before the offer is revoked, to all of its terms in the time and manner specified by the offeror (or in a reasonable time and in a reasonable manner, if the offeror did not specify the manner of acceptance). Unless the offeror specifically states that her offer may be accepted by silence or the course of dealings between the parties indicates that the offer will be accepted if the offeree does nothing, silence will not operate as an acceptance. 3. REJECTION If the offeree rejects an offer or makes a counteroffer, the original offer is terminated and cannot thereafter be accepted, even if the time for expiration of the offer has not yet occurred. If the offeror has made an offer that she permits to be accepted in part, acceptance of part can be considered a rejection of the remainder. An inquiry in response to an offer ("Would you consider a lesser price?") is not a rejection.

MicroMash MBE In Brief: Contracts Bar Exam Alerts

4. REVOCATION OF OFFERS
At common law, an offer is generally revocable even if the offer says it will remain open for a specified time. At common law, an offer is irrevocable for the time specified only if an option contract is formed, i.e., if the offeree has given consideration to the offeror in exchange for the offeror's agreement to keep the offer open. Under the U.C.C., a "firm offer" cannot be revoked before the expiration date. Such a "firm offer" can only be made by a merchant, must state in writing that the offer is irrevocable until a date certain, and cannot remain irrevocable for more than three months. An offer for a unilateral contract is irrevocable by the offeror if the offeree has, with the knowledge of the offeror, started substantial performance. An offer is revoked if the notice of revocation is communicated to the offeree in any manner before the offer is accepted. The notice of revocation can be any communication that fairly indicates to the offeree that the offeror has withdrawn the offer. In a real estate brokerage transaction where the owner makes an offer for a unilateral contract that the broker accepts by producing a buyer ready, willing, and able to buy at the listing price, the offer is automatically revoked by the seller's acceptance of an offer to purchase the property from a buyer not produced by the broker. An offer for a contract that would fall within the Statute of Frauds can be revoked orally. If the offeror dies before the offer is accepted, it is revoked. However, if the offer is accepted, then death does not terminate the obligations of the contract. 5. MISTAKE, FRAUD, AND DURESS A contract can be avoided on the grounds of unilateral mistake if the mistake was so obvious that the offeree should have known of the mistake at the time he accepted the offer. A party has the right to rescind a contract if it was entered into in reliance on an untrue material fact. If each of the parties innocently has a different understanding of the meaning of the words of the agreement, then there is no contract. 6. INDEFINITENESS AND ABSENCE OF TERMS If the price term is missing in a U.C.C. transaction, there is a contract at a reasonable price.

MicroMash MBE In Brief: Contracts Bar Exam Alerts

7. BATTLE OF THE FORMS Under the U.C.C., a valid contract is formed if the offeree accepts the offer, even if she proposes different or additional terms. Between merchants, the different or additional terms become part of the contract if they do not materially alter the offer and the offeror does not object. Under the U.C.C., a seller can accept an offer either by a promise to sell the goods requested by the buyer or by shipping conforming goods in accordance with the offer.

B. CAPACITY TO CONTRACT
A minor can disaffirm a contract, even one that has been completed, within a reasonable time of reaching the age of majority.

C. IMPLIED IN FACT CONTRACTS AND QUASI CONTRACTS


-

If a person accepts services from someone in the business of providing those services, there is an implied-in-fact contract to pay for the reasonable value of those services. If necessary services are rendered to a person at a time when she lacks the mental capacity to request such services, (e.g., she is unconscious at the time medical services are rendered), there is an implied-in-law contract to pay for them. A quasi contract exists when there is no enforceable contractual relationship between the parties, but one party has conferred a benefit on the other not intended to be gratuitous. The party conferring the benefit is entitled to collect the fair value of the services rendered. A party does not have the right to sue in quantum meruit for a benefit conferred if there is an enforceable right to sue under a contract.

II. CONSIDERATION

A. BARGAIN AND EXCHANGE


The concept of bargain is the essence of consideration. If a party asks for something that she wants, even though it does not directly benefit her, and promises something in return, there is valid bargained-for consideration. A gift contingent on a minor condition that is not bargained for by the donor, such as "I will give you a birthday present if you come down and pick it up," does not amount to a contract supported by consideration.

MicroMash MBE In Brief: Contracts Bar Exam Alerts

B. ADEQUACY OF CONSIDERATION
The value of what a party promises or requests is irrelevant for purposes of determining if there is valid consideration. If a party to a contract performs the promise he makes, he is entitled to enforce the contract according to its terms, even if he is getting far more than he has given. An illusory promise, one that gives the party the unilateral right to do anything they want, is not valid consideration.

C. MORAL OBLIGATIONS AND DETRIMENTAL RELIANCE


One party's promise to make a gift is enforceable under the doctrine of promissory estoppel if (1) the donor-promisor knows that the promise will induce substantial reliance on the part of the promisee, and (2) failure to enforce the promise will cause substantial hardship. A service that has already been gratuitously rendered is not valid consideration for a later promise to pay for that service. A promise in writing to pay a debt that is barred by the statute of limitations is enforceable according to its terms without new consideration.

D. MODIFICATION OF CONTRACTS; PRE EXISTING DUTY RULE


-

In a non-U.C.C. contract, consideration is required to support a modification. In a U.C.C. contract, consideration is not required to support a modification. If each side to an existing contract modifies its rights and obligations in exchange for modification of the rights of the party(ies) on the other side, there is consideration. If the parties agree to rescind an executory (i.e., uncompleted) contract, consideration is found in the mutual agreements to give up rights under the contract. The agreement to perform an act that a person is already legally obligated to perform is not valid consideration.

III. THIRD-PARTY BENEFICIARY CONTRACTS A. INTENDED BENEFICIARIES


Intended beneficiaries are those persons who have a right to sue on a third-party beneficiary contract because the original contracting parties either explicitly or implicitly intended to benefit them.

MicroMash MBE In Brief: Contracts Bar Exam Alerts

If the third-party beneficiary contract is designed to satisfy an obligation of the promisee to the third party, the third-party beneficiary does not give up his rights against the promisee until such time as the promisor renders performance to the third-party beneficiary.

The third party need not provide consideration to be able to sue on a third-party beneficiary contract. The promisor of a third-party beneficiary contract has a valid defense in a suit by the intended beneficiary if the promisee fails to perform his obligations to the promisor.

B. INCIDENTAL BENEFICIARIES
An incidental beneficiary, a person that the original contracting parties did not intend to benefit, has no right to enforce a third-party beneficiary contract.

C. MODIFICATION OF THIRD PARTY BENFICIARY'S RIGHTS


-

The promisor and promisee of a third-party beneficiary contract can modify their contract to the detriment of the intended beneficiary only until the beneficiary either assents to the contract at a party's request, sues on the contract, or changes his position in reliance on it.

IV. ASSIGNMENT AND DELEGATION A. ASSIGNMENT OF RIGHTS


The benefits of a U.C.C. contract are assignable even if the contract prohibits assignment. The benefits of a contract can be assigned without the assignee becoming bound to perform the obligations of the contract. An assignee takes rights under the contract subject to any defenses that the contracting party has against the assignor. If a party to a contract is notified of the assignment of rights under that contract, the contracting party cannot raise against the assignee rights against the assignor which accrue after notice of the assignment.

B. DELEGATION OF DUTIES
A contract is not delegable if the party wishing to delegate possesses unique characteristics such that the performance rendered by a delegatee would vary materially from that bargained for. A contract is also not delegable if the contract specifically prohibits it.

MicroMash MBE In Brief: Contracts Bar Exam Alerts

The party delegating duties (the delegator) remains liable on the contract as a surety for the performance of the delegatee, the party now principally liable on the contract.

V. STATUTE OF FRAUDS
A. SURETYSHIP An oral promise to pay the debt of another is usually unenforceable because of the Statute of Frauds. However, if a person agrees to pay the debt of another for the primary purpose of furthering his own goals, rather than those of the debtor, the Statute of Frauds will not prevent enforcement of the promise.

B. LAND INTEREST
In a contract for the sale of land, the memorandum required to satisfy the Statute of Frauds must contain the price. A real estate brokerage contract is not within the Statute of Frauds.

C. CONTRACTS THAT CANNOT BE PERFORMED WITHIN ONE YEAR


In determining if a contract can be performed within one year, measure from the time of the making of the contract to the time prescribed for the end of performance, not just the time when performance will take place. A personal services contract for more than a year is within the Statute of Frauds despite the fact that the contract would be prematurely terminated if the personal service supplier died within the year. A contract for life is not within the Statute of Frauds because death could occur within a year, which would be the natural termination of the contract. D. SALE OF GOODS Contracts for the sale of goods for $500 or more must satisfy the Statute of Frauds, unless they are specially manufactured goods and not suitable for sale to others in the ordinary course of business. The Statute of Frauds is satisfied to the extent that there is part performance. The Statute of Frauds with respect to the sale of goods does not apply where the goods have been received and accepted. The Statute of Frauds with respect to the sale of goods is satisfied in a contract between merchants where one merchant sends a written, signed memorandum of the transaction sufficient to bind him to the contract and the receiving merchant does not object within 10 days.

MicroMash MBE In Brief: Contracts Bar Exam Alerts

In a U.C.C. contract, a memorandum satisfies the Statute of Frauds if it indicates there is a contract, it contains a description of the goods and the quantity, and is signed. It need not contain the price. A modification of a U.C.C. contract, if it involves a sale of goods for more than $500, requires compliance with the Statute of Frauds.

VI. PAROL EVIDENCE RULE


Parol evidence is admissible to show that there is a condition precedent to a contract's coming into existence. Parol evidence is admissible to explain an ambiguity.

Parol evidence is admissible to show that the parties used words in a nontraditional manner or spoke in code. Parol evidence is admissible to prove a mistake in reducing the terms of an oral agreement to writing.

In a U.C.C. contract, a provision that requires subsequent modifications be in writing In a non-U.C.C. contract, a provision requiring that subsequent is valid. modifications be in writing is invalid. Except in a U.C.C. contract requiring subsequent amendments to be in writing, evidence of an oral modification subsequent to the making of a written contract is admissible.

VII. CONDITIONS A. EXPRESS CONDITIONS


A party seeking to sue on a contract must either show compliance with an express condition, or that the other party was in bad faith with respect to the condition, thereby excusing its performance. If a contract contains a condition that performance must be satisfactory to the purchaser, that satisfaction will be judged by an objective standard, unless the contract involves personal taste, in which case the performance must be subjectively satisfactory to the purchaser (limited only by the purchaser's obligation to exercise good faith).

B. CONSTRUCTIVE CONDITIONS OF EXCHANGE


Unless otherwise specified, each party must perform its obligations under the contract to be able to demand performance from the other side. Such mutual conditions precedent are constructive conditions of exchange.

MicroMash MBE In Brief: Contracts Bar Exam Alerts

1. SUBSTANTIAL PERFORMANCE AT COMMON LAW


Under the common law, the plaintiff can sue for breach of contract if she has substantially performed the contract, even if there is an immaterial (nonwillful) breach.

2. DIVISIBLE CONTRACTS
If a contract is divisible, then performance of one divisible portion permits the plaintiff to demand performance from the defendant for that divisible portion, even if the plaintiff is in breach with respect to another divisible portion. The fact that a construction contract requires periodic payments does not make it a divisible contract. Under the U.C.C., except for an installment contract, the seller must tender the correct number of conforming goods at the time specified in the contract or the buyer can reject the goods without liability. However, the seller has a limited right to "cure" after a nonconforming tender. Under the U.C.C., if a contract is determined to be an installment contract, the buyer can reject a nonconforming shipment only if it substantially impairs the value of the installment and cannot be cured. Under a U.C.C. installment contract, a breach with respect to one installment is a breach of the total contract only if the nonconformity substantially impairs the value of the entire contract.

C. IMPLIED DUTIES OF GOOD FAITH AND FAIR DEALING


Each party to a contract has an implied duty to cooperate with the other party in achieving the objects of the contract. Failure of the plaintiff to discharge that implied duty is a defense in a suit on the contract.

D. PROSPECTIVE INABILITY TO PERFORM


Under the U.C.C., when a party has reasonable grounds for insecurity, she may demand adequate assurances from the other party and suspend her performance until she receives them. If a party repudiates a contract before the time for performance, the other party may treat the repudiation as a total breach, seek performance elsewhere, and sue for breach. If the nonrepudiating party has not canceled the contract or materially changed position, the repudiating party may retract the repudiation, providing she gives adequate assurances. The nonrepudiating party then has no right to sue for breach.

MicroMash MBE In Brief: Contracts Bar Exam Alerts

VIII. REMEDIES
A. BUYER'S REMEDIES UNDER THE U.C.C. The buyer may seek specific performance and replevin the goods where they are unique (and in other special circumstances). The buyer may seek damages the difference between the market price and the contract price. The buyer may cover, that is, purchase the goods elsewhere and collect the difference between the cover price and the contract price. B. SELLER'S REMEDIES UNDER THE U.C.C. The seller may sell the goods in a commercially reasonable manner and collect the difference between the contract price and the sales price, plus incidental damages. If the difference between the sales price and the contract price does not reasonably reflect the seller's damage because he has an unlimited supply of the goods, then the measure of damage is the seller's profit, the difference between his production cost for the goods and the contract price. The seller may sue for the contract price if the goods cannot be sold in the seller's ordinary course of business. C. SPECIFIC PERFORMANCE Both the buyer and the seller are entitled to sue for specific performance of enforceable land contracts. D. DAMAGES Consequential damages are limited to those damages that were reasonably foreseeable. Liquidated damages are only collectible if the liquidated amount is reasonable either in respect to the amount of damages which the parties anticipated at the time of making the contract, or in respect to the actual damages incurred. If a party is prevented from suing on the contract because the contract is unenforceable or because she has breached the contract, she is entitled to collect restitution damages in quantum meruit, measured by the fair value of the benefit conferred on the other party. E. RISK OF LOSS UNDER THE U.C.C. The seller shifts the risk of loss to the buyer when she completes her delivery obligation for conforming goods. If the contract is FOB seller's place of business, the delivery obligation is completed by placing conforming goods on a common

10

MicroMash MBE In Brief: Contracts Bar Exam Alerts

carrier with arrangements that they be shipped to the buyer. If the contract is FOB buyer's place of business, the delivery obligation is completed when the goods are delivered to buyer's place of business; the seller retains the risk of loss during transit. If the seller ships nonconforming goods on a shipment or a destination contract, she retains the risk of loss until the goods are accepted. If the buyer rightfully revokes acceptance of the goods, the risk of loss is on the seller to the extent that the goods are not covered by buyer's insurance. If the buyer breaches or repudiates the contract before the risk of loss passes to her, the risk of loss is on the buyer for a commercially reasonable time to the extent that the loss is not covered by seller's insurance.

F. IMPOSSIBILITY AND FRUSTRATION


At common law, the excuse of impossibility applies when the subject matter of the contract is destroyed or a party to a personal service contract dies. Under the U.C.C. doctrine of impracticability, performance is excused when (1) goods identified to the contract are destroyed, (2) performance becomes illegal, or (3) performance is prevented by a nonforeseeable event the nonoccurrence of which was a basic assumption of the contract.

MicroMash

MBE IN BRIEF CRIMINAL LAW & PROCEDURE

MicroMash BAR REVIEW MBE IN BRIEF


CRIMINAL LAW AND PROCEDURE Table of Contents
1 1 1
1 1 1 2 2

I.

HOMICIDE A. B. DEFINITION CRIMINAL HOMICIDE 1.


2. 3. 4. 5. Murder Degrees Of Murder Voluntary Manslaughter Involuntary Manslaughter A Killing While Resisting Arrest Justifiable Homicide Excusable Homicide Self-Defense Defense Of Others Defense Of Property Public Authority Arrest Prevention Of Felony

C.

INNOCENT HOMICIDE 1.
2.

2
2 2

D.

DEFENSES TO HOMICIDE 1.
2. 3. 4. 5. 6.

3
3 3 3 3 3 3

E. II.

ANALYSIS OF HOMICIDE CRIMES

3 4
4 4 4 4

PROPERTY CRIMES A. THEFT CRIMES


1. 2. 3. Introduction Specific Intent Required Common Law Larceny

4. 5.

Embezzlement Obtaining Property By False Pretenses

5 5

B. C. D.

RECEIVING STOLEN GOODS ROBBERY BURGLARY


1. 2. 3. 4. 5. Breaking Entering Dwelling House Nighttime Specific Intent

6 6 6
6 6 6 6 6

E. F. G. H. III.

ASSAULT AND BATTERY RAPE KIDNAPPING ARSON

6 7 7 7 7 7
7 8 8

INCHOATE CRIMES; PARTIES A. INCHOATE OFFENSES


1. 2. 3. Attempt Solicitation Conspiracy

B.

PARTIES TO CRIME 1. Principal First Degree


2. 3. 4. 5. 6. Principal Second Degree Accessory before the Fact Accessory After The Fact Misprision Compounding

9
9 9 9 9 9 9

IV.

GENERAL PRINCIPLES A. B.
ACTUS REUS MENS REA 1.
2. 3. 4. General Intent Specific Intent Constructive Intent Statutory Intent

9 9
10
10 10 10 10

ii

5. 6. 7.

Strict Liability Crimes Mistake Of Fact As A Defense Mistake Of Law As A Defense Insanity Intoxication Duress Necessity Public Authority Domestic Authority

11 11 11

C. RESPONSIBILITY
1. 2. 1. 2. 3. 4.

11
11 11

D. JUSTIFICATION

11
12 12 12 12

V. CONSTITUTIONAL PROTECTION OF ACCUSED PERSONS


A. ARREST, SEARCH, AND SEIZURE
1. 2. 1. 2. 3. Arrest Search and Seizure Voluntariness Standard Miranda Standard Fourth Amendment Standard

12
12
12 12

B. CONFESSIONS

14
14 14 15

C. D.

RIGHT TO COUNSEL LINEUPS AND OTHER FORMS OF IDENTIFICATION


1. 2. 1. 2. 3. 1. 2. 3. 4. 5. Right To Counsel Due-Process Standard Pretrial Stage Trial Stage Post-Trial Stage What Is The "Same Offense"? When Jeopardy Attaches When Retrial Is Permitted Appeals By The Prosecution Collateral Estoppel

15 15
15 15

E. FAIR TRIAL AND GUILTY PLEAS

15
15 16 17

F. DOUBLE JEOPARDY

17
17 17 18 18 18

iii

iv

CRIMINAL LAW AND PROCEDURE

I. HOMICIDE A. DEFINITION
Homicide is the death of a human being caused by another. In the extremes, the victim must be born alive (infanticide) or be not yet dead (euthanasia). "Cause" refers to legal or proximate cause.

B. CRIMINAL HOMICIDE
1. Murder Murder is a criminal homicide committed with malice aforethought. "Criminal" means that the defenses of justification or excuse are not available. "Malice aforethought" means: intent to kill, in the sense of desiring the homicide or taking such action that its occurrence is substantially certain (intent is implied from the use of a dangerous weapon); intent to inflict serious bodily harm; a wanton and willful disregard of an unreasonable human risk (depraved heart); a homicide occurring in the commission or attempted commission of a dangerous felony.

2. Degrees Of Murder First-degree murder is murder committed with deliberately premeditated malice aforethought, with extreme atrocity and cruelty, in the commission of a dangerous felony, by poison or by lying in wait. Second-degree murder is murder committed with malice but without deliberate premeditation, and any murder that is not first-degree murder. 3. Voluntary Manslaughter Voluntary manslaughter is an intentional homicide without malice aforethought. Malice aforethought is not present if the homicide is committed in the heat of passion caused by adequate provocation. a. Adequate provocation A serious battery, observing a spouse in the act of adultery, and words conveying inflammatory information (if they are such as to inflame the passions of a reasonable person), constitute adequate provocation; abusive language does not.

MicroMash MBE In Brief: Criminal Law & Procedure

b. Actual provocation The defendant must be actually provoked (a subjective test), and the homicide must occur before the defendant has cooled off. c. Causation The adequate provocation must cause the heat of passion. d. Imperfect self-defense Voluntary manslaughter also occurs when there is a right of self-defense or defense of another, but it is imperfectly exercised because the defendant unreasonably perceived a risk of death or serious bodily harm or used excessive force. e. Where necessity or coercion present Voluntary manslaughter also occurs when the defendant would have the defense of coercion or necessity if it were a nonhomicide crime. 4. Involuntary Manslaughter Involuntary manslaughter is an unintentional homicide committed with criminal negligence or in the commission of a misdemeanor malum in se. a. Criminal negligence Criminal negligence occurs when there is a substantial departure from the standard of reasonable care, resulting in a high degree of risk of death or serious bodily injury (reckless conduct). b. In the course of an act malum in se When a homicide occurs in the course of a misdemeanor that is bad in itself, a breach of public order, or contrary to public decency or good morals, then there is involuntary manslaughter under the theory of misdemeanor manslaughter. 5. A Killing While Resisting Arrest This is not a separate category of homicide. It is murder if intentional while resisting a lawful arrest. It can be manslaughter if the arrest is unlawful and passion is aroused by the excessive use of force.

C. INNOCENT HOMICIDE
Innocent homicide occurs when the death caused by another is either justified or excused. 1. Justifiable Homicide Homicide is justified if it is commanded by law or is committed in self-defense, defense of another, or in preventing a dangerous felony. 2. Excusable Homicide Homicide is excused if the defense of mistake of fact or insanity is available, or if it was an unintentional homicide committed without criminal negligence, and not while in the commission of an unlawful act.

MicroMash MBE In Brief: Criminal Law & Procedure

D. DEFENSES TO HOMICIDE
1. Self-Defense a. Nondeadly force
Nondeadly force, i.e., force neither intended nor likely to cause death, which in fact does cause death, is justified even though the defendant is not threatened with death or serious bodily injury as long as there is a threat of some harm.

b. Deadly force
Deadly force is justified where the defendant reasonably believes she is in immediate danger of death or serious bodily harm.

c. Retreat when required


Retreat is not required under the majority view, and the minority view does not require retreat unless the defendant is outside of his home and can retreat in safety.

d. Right of aggressor to use self-defense


An aggressor has the right to use deadly force if she originally used nondeadly force and was met with deadly force, or if she withdrew from aggression, communicated that fact, and otherwise had the right to use self-defense.

2. Defense Of Others
A person has the right to defend others under the same circumstances where he can defend himself. Reasonable mistake does not vitiate the defense. It is not limited to defending family members.

3. Defense Of Property
Except when protecting one's home against a person intent on committing a serious felony, there is no right to use deadly force in defending property.

4. Public Authority
Killings commanded by public authority are justified.

5. Arrest
A police officer or a civilian acting under police direction can use deadly force if necessary to arrest for a felony, and nondeadly force to arrest for a misdemeanor.

6. Prevention Of Felony
Anyone can use deadly force to prevent the commission of a serious felony.

E. ANALYSIS OF HOMICIDE CRIMES


If intentional homicide, then it is murder unless it is manslaughter (when the killing is in the heat of passion caused by adequate provocation), or innocent homicide if it is justified or excused.

MicroMash MBE In Brief: Criminal Law & Procedure

If not intentional, then it is murder if there is an intention to inflict serious bodily harm,
commit a willful act in serious disregard of a severe danger, or if the homicide is in the course of a dangerous felony. It is involuntary manslaughter if committed with criminal negligence or in the course of a misdemeanor malum in se; otherwise, it is innocent homicide.

II. PROPERTY CRIMES


A. THEFT CRIMES 1. Introduction
Larceny, embezzlement, and obtaining property by false pretenses are mutually exclusive common law crimes.

2. Specific Intent Required a. Intent to deprive another permanently


The specific intent necessary for theft crimes is the intent to steal or to permanently deprive the owner of property. It is present when property is taken with intent to claim a reward, intent to abandon, intent to sell the property back to the owner, or intent to pledge or pawn if there is not a substantial capacity to redeem.

b. No intent to steal
The specific intent is not present where the intent is only to borrow and an ability to return exists; where there is the intent and capacity to pay for property that is for sale; and where there is the intent to take money to repay a debt. The specific intent is not present if the taking is caused by a mistake of fact or mistake of law.

c. Coincidence of specific intent and prohibited conduct


The specific intent must coincide with the prohibited conduct for a particular crime to occur.

3. Common Law Larceny


Common law larceny is the trespassory taking and carrying away of the personal property of another with the intent to steal.

a. Possession
The distinguishing characteristic of larceny is that it is a taking from one in possession. Possession in its ordinary sense means the legal right to control an object for a reasonably long period of time.

b. Constructive possession
"Constructive possession" means legal possession where factual possession does not exist.

Employers have constructive possession of property in the hands of lower-level


employees.

MicroMash MBE In Brief: Criminal Law & Procedure

Owners have constructive possession of a package's contents when a bailee breaks bulk and steals part of a package. Owners of mislaid property or lost property with a clue to ownership have constructive possession of it. Owners have constructive possession of property when actual possession but not title is taken from them by fraud. The crime is called "larceny by trick." Owners have constructive possession of property delivered by mistake. A joint owner has possession of all parts of jointly owned property.
c. Taking

The taking requirement is satisfied by severing the article from the owner's possession, but not by destroying property while it is in the owner's possession. An innocent agent can accomplish the taking.
d. Trespass

The trespassory requirement means that the property must be taken without the consent of the person in possession. If the original taking was without consent, yet was not unlawful because there was no intent to steal at the time of the taking, a crime is committed when the intent to steal is formed, because the original trespass is deemed to be "continuing" until that time.
e. Carrying away

The carrying away requirement is satisfied by a very slight movement.


4. Embezzlement Embezzlement is the fraudulent conversion of the property of another by one who is already in lawful possession. Conversion is a serious act interfering with the owner's rights.

The property embezzled must belong to another. Inability to fulfill a contractual obligation is not embezzlement. The converter must be in lawful possession. There must be the intent to defraud.
5. Obtaining Property By False Pretenses "False pretenses" is a false representation of a material fact relied upon by the victim which induces the victim to pass title to property.

The representation must actually be false and be of a material past or present fact. A prediction of a future event or a false promise is not sufficient. The victim must rely upon the false representation and pass title. The defendant must know that the representation is false and intend to defraud.

MicroMash MBE In Brief: Criminal Law & Procedure

B. RECEIVING STOLEN GOODS


This crime requires the receiving of stolen property, the knowledge that it is stolen, and the intent to deprive the true owner of the property. The test for knowledge is subjective. The knowledge must coincide with the act of receiving the property.

C. ROBBERY Robbery is larceny from the person by force or intimidation. All of the elements of larceny
are necessary for robbery. The term "from the person" means goods within the control of the victim. The victim must be placed in fear by either force or intimidation.

D. BURGLARY
Common law burglary is the breaking and entering of the dwelling house of another with the intent to commit a felony.

1. Breaking
Breaking is the opening of any enclosure, even though it is unlocked. If entry is obtained by fraud, there is a breaking. If entry is gained without a breaking, a breaking can occur if the defendant enters another part of the real estate by opening a door or window.

2. Entering
Entering occurs when any portion of the defendant's body, or an instrument used by him to accomplish the crime, crosses into the dwelling through the opening created by the breaking.

3. Dwelling House
A dwelling house of another is a structure regularly occupied for habitation. It need not be occupied at the time of the breaking, but must not be abandoned.

4. Nighttime
Nighttime occurs during the period of darkness between sunset and sunrise.

5. Specific Intent
At the time of the breaking and entering, the defendant must have the intention to commit a felony that is causally connected to the breaking and entering.

E. ASSAULT AND BATTERY


A battery is the unlawful application of force to the person of another. The force need not be great or applied directly, but must be applied either intentionally, or with criminal negligence, or in the commission of an act malum in se. Consent, self-defense, and the right to apply force because of a position of authority are defenses to a battery. An assault is either an attempt to commit a battery, or the intentional placing of the victim in apprehension of receiving an immediate battery.

MicroMash MBE In Brief: Criminal Law & Procedure

F. RAPE
Rape is unlawful sexual intercourse with a female without her consent. Intercourse requires penetration. Intercourse is without consent if procured by force, threat of harm, or when the female is unable to consent because of drunkenness, a drug-induced stupor, or unconsciousness. Consent is not a defense if it is produced by fraud with respect to the nature of the act itself fraud in the factum. If the fraud relates to a collateral matter, fraud in the inducement, the consent is valid. Consent is not valid if the female is under the age of consent, usually 16. Reasonable mistake of fact concerning age is not a defense.

G. KIDNAPPING
Kidnapping is the unlawful forcible confinement and asportation of a person against his or her will. Some statutes define kidnapping to include secret confinement, and dispense with the asportation requirement. Kidnapping for ransom is an aggravated form of kidnapping.

H. ARSON
Arson is the malicious burning of the dwelling of another. There must be the intent to burn. The burning must ignite the real estate. The test for the "dwelling of another" requirement is the right to possession or occupancy, and not title.

III. INCHOATE CRIMES; PARTIES


A. INCHOATE OFFENSES
1. Attempt An attempt is a step in the direction of committing a crime, coupled with the intention to commit that crime. If the crime is successfully completed, the attempt is merged.

a. Close to actual commission


The activity must reach the perpetration stage. The closer to the time and place of the scheduled execution of the criminal act, the more likely that the perpetration has been reached. If the defendant has completed the last acts she is required to perform, she is guilty of attempt.

b. Specific intent
The defendant must intend to commit the crime attempted.

c. Defense of impossibility
There is no impossibility defense if the crime attempted, because of circumstances unknown to the actor, is factually impossible to commit. However, if the attempt is inherently impossible or the act intended is not a crime, there is no attempt. If the act attempted is not legally a crime because of facts unknown to the actor, the common law crime of attempt is not committed. d. Abandonment If the attempt is abandoned because of a change of heart, there is no defense at common law, although there is a defense under some statutes.

MicroMash MBE In Brief: Criminal Law & Procedure

2. Solicitation Solicitation occurs when the defendant, with the intention that another commit a crime, entices, advises, incites, orders, or otherwise encourages the other to commit that crime. If the crime is completed, solicitation is merged into the solicitor's liability as an accomplice. If the person solicited agrees to commit the crime, then a conspiracy has been formed and the solicitation merges into the conspiracy. 3. Conspiracy a. Does not merge Conspiracy is a combination for an unlawful purpose. It does not merge into the target offense. b. Agreement can be inferred The agreement necessary for a combination need not be formal, and can be inferred from a concerted action. c. Specific intent The parties to a conspiracy must intend both to agree and to accomplish the objective of the conspiracy. A supplier who knows the criminal purpose will be a conspirator if he has a stake in the outcome, if the goods are highly regulated, or if the crime contemplated is very serious. d. Overt-act requirement An overt act was not required at common law, but is required by many statutes. If an overt act is required, the conspiracy crime is not complete until the overt act is completed. The overt act can be committed by any conspirator and need only be in preparation to commit the crime. e. Number of persons required A person cannot commit a conspiracy alone. If one party feigns agreement, or one of the persons is a member of a legislatively protected class, or the two persons are husband and wife, or only one person is acting for two corporations, there is no conspiracy under the common law. There is also no conspiracy if only the parties essential to the commission of the offense are participants. If all conspirators but one are acquitted, then the other must likewise be acquitted. f. Unlawful conduct

Unlawful conduct encompasses more than criminal conduct. g. Impossibility Impossibility, whether legal or factual, is not a defense to conspiracy. h. Scope of a conspiracy A conspiracy begins at the time of agreement, or in overt-act jurisdictions, at the time of the overt act. It ends with the attainment of its objective or with abandonment. If

MicroMash MBE In Brief: Criminal Law & Procedure

the agreement covers several crimes, there is only one conspiracy. If parties know of necessary acts of others to complete the criminal activity, they are all parts of one conspiracy. i. Withdrawal To withdraw from a conspiracy, it is necessary to notify all conspirators of the withdrawal. j. Effects of existence of a conspiracy

Once a person is a conspirator, she is guilty of all crimes committed by the other conspirators in furtherance of the conspiracy which are within the scope of the conspiracy, and all statements made by one conspirator in furtherance of the conspiracy are admissible against the others in evidence.

B. PARTIES TO CRIME 1. Principal First Degree


The person who actually perpetrates a crime is a principal in the first degree. 2. Principal Second Degree The person who is present but does not actually perpetrate the felony is a principal in the second degree. 3. Accessory Before The Fact The person who is not present at the scene but aids or encourages the commission of a felony is an accessory before the fact.

4. Accessory After The Fact


The person who aids or assists a felon to avoid apprehension is an accessory after the fact. The person must know of the commission of the felony. Most jurisdictions exempt assistance to close relatives. 5. Misprision Misprision of a felony, a common law misdemeanor, is the failure to report or prosecute a known felon. 6. Compounding Compounding a felony is receiving consideration for failure to report or prosecute a known felon.

IV. GENERAL PRINCIPLES A. ACTUS REUS


Before there can be a crime, there must be a criminal act. It may consist of words or actions. The act must be voluntary and be committed while conscious. Failure to act is criminal when

10

MicroMash MBE In Brief: Criminal Law & Procedure

a duty to act is imposed by law as a result of a contract, a previous act, or a relationship. Failure to act is not criminal, however, if the act would be futile if performed.

B. MENS REA
The required mental state must occur simultaneously with the actus reus. 1. General Intent Except for strict liability crimes, all crimes require at least general intent, that is, the intent to accomplish the required act. Intent means either desiring the result or taking such steps that the result is substantially certain to occur. 2. Specific Intent Specific intent is the intent to cause the harmful consequence of said act. The following crimes require a specific intent in addition to a general intent: murder: malice; theft crimes, robbery, and receiving stolen property: intent to deprive the owner of property; burglary: intent to commit a felony within the dwelling; solicitation: intent to have the crime committed by the solicited; attempt and conspiracy: intent to commit the target crime.

3. Constructive Intent Intent can be inferred from the accomplishment of an act. In some instances, either negligence or recklessness constitutes a constructive intent, even though an actual intent does not exist. 4. Statutory Intent Specific intent is also required in statutory crimes where the act must be done willfully, knowingly, wantonly, or with malice. a. Malice Malice only requires the doing of a criminal act without excuse, justification, or mitigation. b. Knowledge Knowledge includes the situations where one perceives a fact, believes it to be true, or deliberately avoids discovering it. c. Willfully "Willfully" means intentionally or purposely and in some instances with evil intent. d. Wantonness "Wantonness" means an act done without regard to the consequences, so as to show a wicked or mischievous intent.

MicroMash MBE In Brief: Criminal Law & Procedure

11

5. Strict Liability Crimes


Strict liability crimes do not require a general intent; the mere accomplishment of the act invokes criminal liability.

6. Mistake Of Fact As A Defense


Mistake of fact is a defense to a crime when, in the case of general intent crimes, the defendant reasonably believes facts, which, if true, would make his actions noncriminal. In specific intent crimes, an honestly held, though unreasonable, belief is sufficient. Reasonable mistake of fact is not a defense to strict liability crimes.

7. Mistake Of Law As A Defense


Mistake of law is a defense only where there is reliance on the decision of a court or high administrative official, or where an honestly held mistake of law prevents or negates the specific intent required.

C. RESPONSIBILITY 1. Insanity
There are four tests for insanity:

a. The M'Naghten Test


D is not guilty where, because of a defect of reason or disease, she did not know the nature and quality of her act, or if she did know, then she did not know it was wrong.

b. Irresistible Impulse Test


D is not guilty if mental disease prevented him from controlling his conduct.

c. Durham Rule
D is not guilty if her unlawful act was the product of a mental disease or defect.

d. Model Penal Code Test


D is not guilty if at the time of the conduct, as a result of a mental disease or defect, D lacked substantial capacity to appreciate the criminality of the conduct, or to conform his conduct to the requirements of the law. The Model Penal Code also requires that the degree of certain offenses be reduced because of diminished mental capacity.

2. Intoxication
Voluntary intoxication is not a defense to a general intent crime, but can be a defense to a specific intent crime where the intoxication prevents the formation of the required intent. Intoxication can also prevent the formation of the premeditation required for first-degree murder. Involuntary intoxication is a defense where the defendant does not know the nature and quality of her act, or that it was wrong.

D. JUSTIFICATION
An action is justified and, therefore, not criminal when authorized or commanded by law.

12

MicroMash MBE In Brief: Criminal Law & Procedure

1. Duress An act short of homicide is justified if the defendant was coerced to commit the act by the use of, or the threat of use of, unlawful force against his person or the person of another, which a person of reasonable firmness in the situation would be unable to resist. 2. Necessity An individual who commits a criminal act other than homicide in response to a nonhuman force has the defense of necessity if she has not voluntarily put herself in that position, and she commits the act in order to avoid a greater harm. 3. Public Authority A public officer pursuant to public duty may perform acts that would be criminal if not done pursuant to that duty. 4. Domestic Authority A parent or schoolteacher may use reasonable physical force to discipline a child.

V. CONSTITUTIONAL PROTECTION OF ACCUSED PERSONS


A. ARREST, SEARCH, AND SEIZURE
1. Arrest An arrest occurs when an individual is deprived of freedom of movement for the purpose of commencing a criminal action. A misdemeanor amounting to a breach of the peace, or a reasonable belief that a person has committed a felony, gives grounds to arrest without a warrant. Otherwise, a warrant is required. Incriminating statements that are the products of an illegal arrest are inadmissible for substantive purposes. 2. Search And Seizure Evidence obtained as a result of an illegal search or seizure is generally inadmissible. a. Search A search occurs when the governmental activities violate the right to privacy upon which the defendant justifiably relies. This protection covers the area of the home and its surroundings, as well as affording protection from electronic eavesdropping. An item in plain view is not the subject of a search except where the viewer is illegally on the premises. Statements made to informers are not the subject of a search. b. Warrantless searches A search without a warrant is valid: if it is made incident to a valid arrest; if it is a limited threshold inquiry ("stop and frisk") when an officer has reason to believe his physical safety may be threatened;

MicroMash MBE In Brief: Criminal Law & Procedure

13

if the police enter a private building in hot pursuit; if it is on premises subject to regulatory licenses, and at a reasonable hour; if it is incident to protecting the borders of the country; if the object searched is in official custody; (1) if voluntary consent, not under threat or compulsion, is given by the defendant; or (2) if a third party who has joint control over the property with the defendant gives consent; if the search is of an automobile which is capable of being moved by its occupants and the police have probable cause to believe that it is carrying contraband, or where other exigent circumstances require an immediate search; if necessary for national security in dealing with foreign affairs; if there is some other emergency which makes it reasonable.

c. Administrative searches Administrative searches are permitted either without warrants, or with blanket warrants which do not have to be obtained in accordance with strict procedures, and which may cover large geographic areas. d. Search warrants A valid search warrant requires that the officer applying personally contact an independent official and submit an affidavit which sets forth reliable facts which establish grounds for the issuance of the warrant. The application for the warrant must specifically state the material or person to be seized and the place to be searched. The affidavit may be based upon hearsay evidence as long as the neutral magistrate can determine the reliability of the informant. The defendant can attack the warrant on the ground that the affiant deliberately lied, and that the warrant would not be sufficient without those lies. Search warrants can be issued to search for the fruits, instrumentalities, or evidence of a crime held by third parties. e. Fruits of an illegal search If there is an illegal search, all evidence and verbal statements that are obtained are fruits of that search, and are inadmissible for substantive purposes unless they are found by means sufficiently distinguishable to be purged of the taint of the illegal search. Illegally seized evidence may be used to impeach credibility. f. Standing

A defendant has standing to raise a search-and-seizure issue only when she can show both a possessory interest in the items seized and a legitimate expectation of privacy in the premises searched.

14

MicroMash MBE In Brief: Criminal Law & Procedure

B. CONFESSIONS
1. Voluntariness Standard
An involuntary confession, i.e., one obtained by the threat of or application of physical force or psychological coercion, will be excluded from evidence. The age, sex, health, and education of the defendant are relevant in determining psychological coercion. The voluntariness of a confession is a preliminary question of fact to be determined by the trial judge. 2. Miranda Standard Statements made as a result of custodial interrogation are inadmissible unless Miranda warnings, or their equivalent, are given, and Miranda rights are waived.

a. Custody
Custody occurs when the defendant is arrested or deprived of freedom in a significant way.

b. Interrogation
"Interrogation" refers not only to express questioning, but any words or actions on the part of the police that they know or should know are likely to elicit an incriminating response from a suspect. c. Miranda rights The Miranda rights are: (1) the right to remain silent; (2) the right to an attorney; (3) the right to a court-appointed attorney if indigent; and (4) the right to be told that anything said by a defendant can be used against him.

d. Waiver
The warning must be given before interrogation begins. If the right to counsel is exercised, it cannot thereafter be waived until counsel is furnished. If the right to remain silent is exercised, it can be waived later. A waiver of Miranda rights must be intelligently and knowingly made after the warnings are given. If a confession is made soon after an earlier tainted confession, it will probably be excluded as the fruit of the tainted confession.

e. Impeachment
Statements that are excluded for substantive purposes by Miranda can be used to impeach.

f. Adoptive admissions
Miranda excludes the operation of the adoptive admission-by-silence rule when the defendant is in custody.

g. Private persons
Miranda does not apply to custodial interrogation by private persons.

MicroMash MBE In Brief: Criminal Law & Procedure

15

h. Corroboration
A confession must be corroborated before a conviction can be obtained.

3. Fourth Amendment Standard


A statement is inadmissible if it is obtained as a result of detention following an illegal arrest.

C. RIGHT TO COUNSEL
An indigent criminal defendant has a right to court-appointed counsel in all felony cases, in misdemeanor cases where the defendant is actually incarcerated, in juvenile proceedings where incarceration is a possibility, and in civil-commitment hearings. The right to counsel attaches during custodial interrogation and at arraignment. If the state provides an appeal procedure as of right for criminal convictions, the defendant has a right to counsel, and a free transcript for one appeal. Convictions obtained when counsel was not provided cannot be used to impeach. Incompetence of counsel is not a denial of the right to counsel, unless it is a result of conflict of interest or is so pronounced that counsel has not given effective assistance, and actual prejudice is shown.

D. LINEUPS AND OTHER FORMS OF IDENTIFICATION 1. Right To Counsel


A criminal defendant has the right to have counsel present at all post-indictment lineups. Failure to comply with this obligation renders identification made at the lineup per se inadmissible, and a later in-court identification inadmissible unless the prosecution can show by clear and convincing evidence that the identification made at trial was made from sources independent of the identification at the lineup.

2. Due-Process Standard
Where an identification is made before indictment, either in a lineup or showup, the results of that identification, and a subsequent in-court identification, are inadmissible only where the pretrial identification was unnecessarily suggestive and conducive to irreparable mistaken identification. The totality of the circumstances surrounding the identification is relevant in applying this due-process standard.

E. FAIR TRIAL AND GUILTY PLEAS 1. Pretrial Stage a. Pretrial release


The purpose of bail is only to assure that the defendant will be present at trial. Bail may be denied to those arrestees who may be dangerous if released.

b. Discovery
Pretrial discovery is not constitutionally required, but the defendant is entitled to the same discovery rights as the prosecution.

16

MicroMash MBE In Brief: Criminal Law & Procedure

c. Publicity A defendant is entitled to be tried by a jury which is unaffected by media coverage of the events and the subject matter of the trial. However, media coverage, including incourt television coverage, is not prejudicial unless the jury is unable to adjudicate fairly or the defendant is unable to present effectively her defense due to such coverage. d. Speedy trial The factors that determine if the defendant's constitutional right to a speedy trial has been violated are: (1) the length of the delay, (2) the reason for the delay, (3) the defendant's assertion of his right, and (4) the prejudice to the defendant. The right to a speedy trial does not commence until the defendant is formally accused of the crime. The right can be waived, but a violation of this right requires complete dismissal of the charges. 2. Trial Stage a. Competence The defendant must be able to understand the charges against her, to assist in her own defense, and to consult with her lawyer; otherwise, she is incompetent to stand trial. b. Impartial judge The trial judge must be impartial and have no interest in the outcome of the case. c. Public trial The defendant has a right to a public trial, but neither the press nor the public has a right to be present at a pretrial hearing, nor do they have an absolute right to be present at trial. A defendant does not have a right to compel a private trial. d. Prosecutor's misconduct The prosecutor may not use false or perjured testimony, and has the obligation to furnish the defense with favorable evidence. e. Jury trial Less than unanimous and fewer than 12-person juries are constitutionally permissible, but a jury may not consist of fewer than six members. Where six-person juries are used, a unanimous verdict is required. A defendant has the right to a jury trial in any criminal proceeding (including a contempt proceeding) where there is a possibility of a jail sentence in excess of six months. The defendant is entitled to a jury in which persons of any racial or ethnic background or sex have not been systematically excluded, and, generally, in which the jurors have been examined for bias or prejudice. The prosecution in a capital case can exclude persons whose opposition to the death penalty is so strong as to substantially impair the performance of their sentencing duties.

MicroMash MBE In Brief: Criminal Law & Procedure

17

f. Guilty pleas
Before a defendant can plead guilty, he must be informed of his right to trial, the elements of the charge against him, and the maximum sentence he can receive. The plea must be voluntarily given.

g. Confrontation
The defendant's constitutional right to confront witnesses against her is not violated when she has had the opportunity to cross-examine the witness at an earlier occasion and the witness is genuinely unavailable at trial. Out-of-court statements of witnesses present at the trial offered for their truth likewise do not violate the right of confrontation.

h. Severance
The defendant has a constitutional right to a severance when the out-of-court statement of a co-defendant implicating both defendants will be admitted at the trial but the co-defendant will not testify, unless the statements implicating the nonconfessing defendant can be excised.

i. Burden of proof
Due process requires that the prosecution prove all of the elements of its case beyond a reasonable doubt, but not matters of affirmative defense, such as insanity.

3. Post-Trial Stage
The defendant is entitled to be represented by counsel at sentencing, and may not be sentenced to jail solely because he is indigent and lacks the ability to pay a fine as an alternative. The Eighth Amendment prohibits imposition of punishments that are physically barbarous, or involve the unnecessary and wanton infliction of pain.

F. DOUBLE JEOPARDY
A defendant may not be put in jeopardy for the same offense twice.

I. What Is The "Same Offense"?


Two offenses are considered the same offense for purposes of double jeopardy unless each offense contains an element not contained in the other. If one offense is a lesser included offense of the other, the trial of either offense will bar a later trial of the other, unless there is a waiver, or one of the elements of the greater offense had not occurred at the time of the earlier trial. If one activity is a crime under two different jurisdictions, double jeopardy will not bar either prosecution.

2. When Jeopardy Attaches


The defendant is in jeopardy in a jury trial when the jury is impaneled, and in a nonjury trial when the introduction of the evidence begins.

18

MicroMash MBE In Brief: Criminal Law & Procedure

3. When Retrial Is Permitted Retrial is permitted after jeopardy attaches if the defendant successfully appeals her conviction; if there is a failure in the mechanics of trial that is not the fault of the prosecutor; or if the defendant waives the defense of double jeopardy. 4. Appeals By The Prosecution The prosecution can constitutionally appeal a criminal case when the decision by the appellate court will not subject the defendant to a new trial. 5. Collateral Estoppel The prosecution may not relitigate issues that were determined in the defendant's favor in a previous criminal trial. A judge after a retrial may not sentence a defendant to a greater term unless his reasons for doing so affirmatively appear on the record.

MicroMashv BAR REVIEW

BAR EXAM ALERTS AT-A-GLANCE CRIMINAL LAW & PROCEDURE


I. HOMICIDE
A. MURDER 1. INTENT TO KILL
A mercy killing is murder because it is an intentional killing, even if the victim asks the person to kill her. A person who sets up a mechanical device that kills a person is guilty of the crime that would have been committed if the person had personally set off the device intentionally. If a person takes steps to make substantially certain that an event will occur, she has intended the act even if she subjectively does not desire that the result occur.

2. INTENT TO DO GREAT BODILY HARM


If a person commits an act that would not ordinarily inflict fatal injury, but would likely cause great bodily harm, and the victim dies, that person is guilty of murder, even if the victim died because of a peculiar medical condition. Intent to do great bodily harm can be inferred from the use of a weapon to inflict bodily injury.

3. DEPRAVED HEART
Firing bullets in a confined space or through a wall, or playing Russian roulette, is abandoned-heart murder if a death results. Deliberately and unjustifiably driving a car onto a crowded sidewalk would constitute abandoned-heart murder if a death results.

4. FELONY MURDER
A defendant is not guilty of felony murder unless he is guilty of the commission or attempted commission of the underlying felony. A defendant is not guilty of felony murder if the commission of the felony has not yet begun or is completed at the time the death occurs.

MicroMash MBE In Brief: Criminal Law Bar Exam Alerts

A co-conspirator of the felon who actually commits the killing is not guilty of felony murder if the killing was beyond the scope of the conspiracy. The felonies of manslaughter or assault and battery cannot be the underlying felony for felony murder. If a third party kills a co-felon in the course of a felony, the surviving felon is not guilty of felony murder because the killing is justifiable homicide.

5. DEGREES OF MURDER A homicide accompanied by malice in the form of a deliberate intentional killing is first-degree murder. A death in the course of the serious common law felonies of Mayhem, Rape, Sodomy, Burglary, Arson, Kidnapping, Escape, and Robbery ("Mrs. Baker" felonies) is first-degree murder ("felony murder"). A murder accompanied by malice in the form of intent to do great bodily harm is usually second-degree murder. A murder accompanied by malice in the form of a depraved heart is seconddegree murder.

B. MANSLAUGHTER
1. VOLUNTARY MANSLAUGHTER
To reduce a murder crime to voluntary manslaughter, there must be adequate provocation to inflame a reasonable person into the heat of passion and the defendant must have actually been in such a state. A violent battery, spousal adultery, mutual affray, and an illegal arrest are adequate provocation. The killing must also take place in a time frame where the passions of a reasonable person would not have cooled and the passion of the defendant must not in fact have cooled. A murder crime can be reduced to manslaughter if the defendant had a defense (e.g., a right to defend herself or another), but used that defense imperfectly (e.g., by employing excessive force). If the defendant would only have been guilty of voluntary manslaughter if she had killed A, she is only guilty of voluntary manslaughter if she shoots at A, misses and kills V.

2. INVOLUNTARY MANSLAUGHTER A death occurring in the course of willful, wanton conduct is involuntary manslaughter. A death occurring in the course of a misdemeanor ma/um in se is involuntary manslaughter.

MicroMash MBE In Brief: Criminal Law Bar Exam Alerts

A person under a duty to aid another person because of a contractual or family relationship is guilty of involuntary manslaughter if a death occurs because of unreasonable failure to give that aid. A person under no duty, however, can unreasonably refuse to give aid without any criminal liability. C. INNOCENT HOMICIDE

1. SELF-DEFENSE
An aggressor does not have the right of self-defense, unless she attacked with nondeadly force and is met with deadly force, or unless she completely ends her aggression and makes that known to the person attacked. A person committing a felony does not have the right of self-defense. An individual has the right to use deadly force to apprehend a felon committing a dangerous felony or to prevent a dangerous felony from being committed. Only nondeadly force can be used if the crime is a misdemeanor. Force is classified as deadly or nondeadly by whether it is likely to cause death, not whether death in fact occurred.

A belief that the person defended has the right of self-defense is a defense in a criminal prosecution, even if the person defended does not in fact have the right of self-defense (e.g., because she was the aggressor). (To avoid liability in tort, though, the person defended must have actually had a right to self-defense.) A person does not have the right to use self-defense to avoid being arrested by a police officer. If an individual has a perfect right of self-defense but, in the exercise of that right, kills the wrong person, the homicide is still excused. 2. DEFENSE OF PROPERTY Defense of property is not sufficient to justify the use of deadly force. A killing commanded by the law, such as an execution or killing on the battlefield in time of war, is not murder because it is a justifiable homicide. 3. EXCUSABLE HOMICIDE Duress relates to coercion by a human force, whereas necessity relates to coercion by nonhuman elements. Duress and necessity cannot be defenses to a homicide crime, but can be defenses to an underlying felony, which would then be a defense to felony murder.

MicroMash MBE In Brief: Criminal Law Bar Exam Alerts

II. PROPERTY CRIMES


A. THEFT CRIMES

1. LARCENY
The specific intent necessary for larceny is not present if the defendant intends to return the property at the time he committed the trespassory taking. If the defendant intended to return the property, the fact that it was not returned because it was unintentionally destroyed does not transform the intent into the intent to steal. However, the intent to destroy is equivalent to the intent to steal. If possession is obtained by fraud, the crime is larceny by trick, not embezzlement. The trespassory act necessary for larceny can be committed by an innocent agent of the defendant. A person with title to property can be guilty of larceny if he wrongfully takes that property from a person rightfully in possession. A lower-level employee in possession of the goods of an employer, or a bailee who breaks the bulk of the goods bailed, does not have a sufficient possessory interest to have the taking of those goods constitute embezzlement. 2. EMBEZZLEMENT Embezzlement only occurs when a person rightfully gains possession of another's property and then converts it to his own use. 3. OBTAINING PROPERTY BY FALSE PRETENSES To be guilty of obtaining property by false pretenses, the victim must give up title to property in reliance on a false representation of material fact by the defendant. The defendant's honest belief that the representation is true prevents the defendant from having the specific intent necessary for the crime of obtaining property by false pretenses, even if the belief is unreasonable.

4. RECEIVING STOLEN GOODS A belief that the goods were not stolen is a defense to the crime of receiving
stolen goods. If the goods are not in fact stolen goods, the defendant cannot be convicted of receiving stolen goods even if she believes that the goods are stolen.

MicroMash MBE In Brief: Criminal Law Bar Exam Alerts

5. ROBBERY Larceny is an essential element of (and merges into the more serious crime of) robbery when all of the elements of robbery are found. The battery that constitutes the force employed in a robbery merges into the more serious crime of robbery. The use of force or intimidation to retain possession of property already stolen is not robbery. The threat to use force in the future is the threat necessary for extortion, not robbery. 6. BURGLARY In order to be guilty of burglary, the defendant must have the specific intent to commit a felony on the premises at the moment of the entering. Burglary is committed if the defendant breaks and enters a part of a dwelling house, even if she does not break and enter when she first enters the dwelling. The breaking and entering need not occur simultaneously. The breaking and the entering necessary for burglary are present if entry is obtained by fraud. A person cannot be guilty of burglary for breaking and entering into her own home. A defendant is guilty of burglary even if not successful in completing the intended felony.

B. OTHER CRIMES
1. ASSAULT AND BATTERY The defendant never has the obligation to retreat if she is using nondeadly force as a defense to an assault or battery. 2. RAPE; STATUTORY RAPE Consent to intercourse is not a defense if the victim's assent to the act performed is procured through fraud that obscures the fact that intercourse is taking place. An underage female who engages in intercourse cannot be held guilty of conspiracy to commit statutory rape or as an accessory to statutory rape. 3. KIDNAPPING Demand for a ransom is not an element of simple kidnapping.

MicroMash MBE In Brief: Criminal Law Bar Exam Alerts

4. ARSON - A person cannot be guilty of the common law crime of arson for burning her own house. A minimal burning of part of the dwelling house is all that is required for arson, but the burning of the contents alone is not sufficient.

III. INCHOATE CRIMES


A. OFFENSES 1. ATTEMPT
To be guilty of the crime of attempt, the defendant must intend to commit the crime that he is attempting. If the act that the defendant intended to accomplish is not a crime, the defendant is not guilty of an attempt even if he thinks he has committed a crime. If the defendant is successful in his attempt and commits the substantive crime, there is no separate crime of attempt; the attempt and the crime merge. 2. CONSPIRACY A co-conspirator is guilty of the substantive crimes committed by any other co-conspirator during the course of the conspiracy and within the scope of the conspiracy. If a co-conspirator withdraws from the conspiracy and informs her coconspirators of the withdrawal, she is not guilty of the substantive crimes committed by the conspirators after the withdrawal, but is guilty of the conspiracy crime. An overt act is not necessary to complete the conspiracy crime at common law, but is required today for federal conspiracy crimes and is required in some states. The impossibility of accomplishing the object of the conspiracy is not a defense to conspiracy, but there is no conspiracy if the parties mistakenly believe that the lawful object of the conspiracy is a crime. A person is not a conspirator unless she combines with another human being to commit an unlawful act or a lawful act by unlawful means. Persons who do not have the requisite intent to qualify as a conspirator do not count as that other person.

MicroMash MBE In Brief: Criminal Law Bar Exam Alerts

A person is not a conspirator if she does not intend to combine to commit a crime. For example, a person is not a conspirator if she intends to combine only to do something which she believes is legal. A person is not guilty of conspiracy if she combines with another person who is essential to the commission of the substantive crime (e.g., adultery). Conspiracy does not merge into the substantive offense.

3. SOLICITATION
If the party solicited agrees to commit the crime, there is a conspiracy and the crime of solicitation is merged into it.

B. PARTIES TO CRIME
To be guilty as an accomplice, the person must know that the principal is committing a crime and must intend to help the principal. If the act being committed by the principal is not in fact a crime, the accessory is not guilty despite his intent to help with an illegal act. Presence at the scene of the crime plus encouragement of the principal to commit the crime is sufficient for accomplice liability.

IV. GENERAL PRINCIPLES


A. STATE OF MIND 1. GENERAL INTENT
To be guilty of a general intent crime, the intent to accomplish the act must coincide with the doing of the act. If a person desires a result and that result occurs, even through an unexpected means, the person has intended the act for purposes of the criminal law.

2. SPECIFIC INTENT
To be guilty of a specific intent crime, the defendant must have the required specific intent at the time he is accomplishing the specific act. 3. STRICT LIABILITY The doing of the actus reus is all that is required for the defendant to be guilty of a strict liability crime. Specifically forbidding an agent to perform an illegal act is not a defense for a principal if performing that act constitutes a strict liability offense.

MicroMash MBE In Brief: Criminal Law Bar Exam Alerts

To be guilty of an attempt to commit a strict liability offense, the defendant must have the specific intent to commit the offense. 4. MISTAKE OF LAW OR FACT A reasonable mistake of fact is a defense to a general intent crime. A reasonable or unreasonable mistake of fact that prevents the specific intent from being formed is a valid defense to a specific intent crime. A mistake of law is not a defense to a general intent crime, but a mistake of law that prevents the specific intent from being formed is a defense to a specific intent crime.

B. RESPONSIBILITY 1. MENTAL DISORDER


A mental illness that causes delusions will not create the defense of insanity under the MNaghten test if the individual knows what she is doing and knows that it is a crime. The irresistible-impulse test is not part of the MNaghten test of insanity.

2. INTOXICATION
In a specific intent crime, intoxication is a defense if the intoxication prevents the defendant from forming the required specific intent. Intoxication can prevent the formation of the malice necessary to constitute first-degree murder and reduce the crime to second-degree murder, but not to manslaughter.

C. CAUSATION
If a person mortally wounds a victim, but death occurs from a totally independent cause, that person is not guilty of murder. Improper medical treatment resulting in death is within the scope of the risk when an individual causes bodily harm. Therefore, lack of causation is not a defense to a homicide crime if there was the required intent or misconduct.

D. JUSTIFICATION
A police officer is justified in using deadly force to apprehend a person who it reasonably appears is either committing or escaping from a dangerous felony. The use of deadly force to arrest a person for a nondangerous felony or any misdemeanor is not justified. A person assisting a police officer is justified in using the same force that a police officer would be justified in using.

MicroMash MBE In Brief: Criminal Law Bar Exam Alerts

V. CONSTITUTIONAL PROTECTION OF ACCUSED PERSONS


A. ARREST; SEARCH AND SEIZURE 1. WARRANTS A search warrant can only be issued by a neutral and detached magistrate on
the basis of probable cause. Probable cause can be based on the totality of the circumstances and does not specifically require evidence on both the basis for the search and the reliability of the informant. The warrant must state with particularity the place to be searched and the objects of the search. If a magistrate grants a search warrant, and the police execute it believing in good faith that it is valid, the property seized pursuant to the search is admissible. 2. SEARCH INCIDENT TO LAWFUL ARREST If the arrest is invalid, or the search is made before there is a valid ground to arrest, then the evidence obtained by the search is inadmissible. A search incident to an arrest must be essentially contemporaneous with the arrest and only of the area within the arrestee's immediate control.

3. AUTOMOBILE SEARCHES
The random stopping of automobiles without any probable cause constitutes an invalid search. However, stops of all vehicles at a fixed checkpoint are permissible. Once an automobile is stopped with probable cause, the entire automobile (including the trunk and containers in the automobile) may be searched. The search need not take place immediately. The police may stop a car and search containers within the car if they have probable cause as to the contents of the containers.

4. CONSENT SEARCHES
The consent given by an individual to search her property must be voluntary, but the suspect need not be warned that she need not give consent. A third party can give valid consent to search areas over which she has joint access. A hotel manager cannot validly consent to the search of rooms in the hotel that are rented to guests.

10

MicroMash MBE In Brief: Criminal Law Bar Exam Alerts

5. OTHER SEARCHES A search at a border (or the functional equivalent thereof) does not require probable cause or a warrant. A regulatory search does not require probable cause. A "stop and frisk" is permitted only if there is a suspicion of criminality and may extend only to a "pat down" search. If the "pat down" uncovers an object that may be a weapon, an intrusive search may be made for weapons. B. CONFESSIONS If a confession is coerced, even by a private individual, then it is inadmissible for any purpose. If improperly admitted, however, such admission is subject to the harmless-error rule. 1. INTERROGATIONS
Miranda only applies when the suspect is interrogated while he is in custody.

Interrogation can take the form of behavior by the police likely to induce the defendant to make a statement. If the defendant exercises his Miranda rights by demanding a lawyer, no further questioning can take place until a lawyer is present and the defendant agrees to interrogation after consultation with his lawyer. If a defendant agrees to submit to interrogation, he can be questioned about more subjects than the crime that is the primary object of the police interrogation. If evidence is inadmissible substantively because of a Miranda violation, it is nevertheless admissible to impeach. C. LINEUPS There is a right to counsel at a lineup only after the criminal process has commenced. If the likelihood of a proper identification is so remote or the lineup is so prejudicial that it offends due process standards, then both testimony about the lineup identification and a subsequent in-court identification are inadmissible.

D. RIGHT TO COUNSEL
A defendant has the right to counsel in all felonies and all misdemeanors for which she is actually incarcerated. An individual has the right to act as her own counsel and, if she does, she cannot later complain that she was denied her right to counsel. A defendant has the right to counsel for one appeal.

MicroMash MBE In Brief: Criminal Law Bar Exam Alerts

11

The defendant has been deprived of effective assistance of counsel if her attorney has a conflict of interest because she is also representing a codefendant.

E. FAIR TRIAL 1. JURY TRIAL


The defendant is entitled to be tried by a jury if the period of incarceration can exceed six months. The defendant is entitled to be tried by a jury chosen from a venire in which there is no systematic racial exclusion. Neither the defendant nor the prosecutor may systematically exclude individuals of one race from the jury by peremptory challenges. The jury need consist of only six persons. If the jury consists of only six persons, however, the verdict must be unanimous. If the jury consists of 12 persons, a nine-person verdict is constitutional. 2. SPEEDY TRIAL The right to speedy trial does not commence to run until the defendant is charged with the crime. The defendant is not denied the right to a speedy trial solely by the passage of time. There must also be prejudice to the defendant. 3. PUBLIC TRIAL Even if both the prosecutor and defendant want a private trial, the public has a right to a public trial. A trial must be public unless there is either a substantial likelihood of prejudice to the defendant or a need to limit access to ensure an orderly proceeding.

4. FAIR CONDUCT BY PROSECUTOR


A prosecutor has the obligation to disclose to the defendant all exculpatory material known to or in the possession of the prosecutor's office.

5. RIGHT TO CONFRONTATION The confrontation clause is satisfied if the defendant had the right to cross If the statement of one defendant is admissible against the confessor, but also implicates a co-defendant and is inadmissible against the co-defendant, the court must either excise the offending portions of the statement or grant a severance. examine the witness at a pretrial hearing and there is a valid excuse for the witness' absence from the trial.

12

MicroMash MBE In Brief: Criminal Law Bar Exam Alerts

6. AFFIRMATIVE DEFENSES The prosecution must prove all elements of the offense beyond a reasonable doubt. However, the state can place upon the defendant the obligation to plead affirmative defenses and prove them by a preponderance of the evidence. 7. SENTENCING A defendant's criminal record is admissible after the verdict, for purposes of deciding the appropriate sentence. 8. RETRIAL The appellate court may constitutionally order a new trial if the verdict is against the weight of the evidence. 9. COLLATERAL ESTOPPEL If issues are litigated in one criminal case between the prosecution and the defendant, they cannot be relitigated in a separate criminal case between the same parties.

F. DOUBLE JEOPARDY
Jeopardy attaches in a criminal jury trial when the jury is sworn, and in a jurywaived trial when the first witness begins to testify. Double jeopardy does not apply when the judge declares a mistrial to benefit the defendant or the appellate court orders a new trial as the result of the defendant's appeal.

The prosecution can only appeal a judgment if a victory for the prosecution will not result in a retrial.

MicroMash BAR REVIEW MBE IN BRIEF


EVIDENCE Table of Contents
I. PRESENTATION OF EVIDENCE
A. INTRODUCTION OF EVIDENCE
1. 2. 3. 4. 5. 6. 7. 8. 1. 2. 3. 4. 5. 1. 2. 3. 1. 2. 3. 4. 5. Requirement Of Personal Knowledge Refreshing Recollection Objections And Offers Of Proof Lay Opinions Competency Of Witnesses Judicial Notice Roles Of Judge And Jury Limited Admissibility Conclusive Presumptions Presumptions Of Fact Presumption Of Innocence True Or Rebuttable Presumptions Legislatively Created Presumptions Control By The Court Form Of Questions, Leading Questions, And Narrative Testimony Exclusion Of Witnesses Prior Inconsistent Statements Bias Conviction Of A Crime Specific Instances Of Conduct Character For Truthfulness

1
1
1 1 1 2 2 2 3 3

B. PRESUMPTIONS

4
4 4 4 4 4

C. MODE AND ORDER

4
4 5 5

D. IMPEACHMENT

5
6 6 6 7 7

6. 7. 8. 9.

Inability To Observe, Remember, Or Relate Accurately Contradiction Impeachment Of The Hearsay Declarant Rehabilitation

7 7 7 7

E. PROCEEDINGS TO WHICH EVIDENCE RULES APPLY

II.

RELEVANCY AND REASONS FOR EXCLUDING RELEVANT EVIDENCE 8


A. CONCEPT OF PROBATIVE VALUE
1. 2. Relevancy Counterweights To Relevancy Method Of Authentication Voice Identification Self-Authentication Character In Issue Character Not Directly In Issue Methods Of Proving Character Other Crimes Habit, Custom, And Routine Practice Similar Happenings And Transactions Expert Witnesses Experimental And Scientific Evidence Real Evidence Demeanor And Views Sound And Picture Recordings

8
8 8

B.

AUTHENTICATION
1. 2. 3.

8
8 9 9

C.

CHARACTER AND RELATED CONCEPTS 1.


2. 3. 4. 5. 6.

9
9 9 10 10 10 10

D.

EXPERT WITNESSES AND SCIENTIFIC EVIDENCE 1.


2.

11
11 11

E.

DEMONSTRATIVE EVIDENCE 1.
2. 3.

11
11 12 12

III.

PRIVILEGES AND OTHER POLICY EXCLUSIONS


A. INTERSPOUSAL PRIVILEGE
1. 2. Right Of Spouse To Refuse To Testify In Criminal Cases Right To Keep Confidential Communications Out Of Evidence
-

12
12
12 12

B.

ATTORNEY CLIENT AND WORK PRODUCT


1. Statement Of Rule

13
13

2. 3. 4. 5. 6. 7. 8. 1. 2.

Communications That Are Protected Communications Between Lawyer And Subordinate Eavesdroppers When A Communication Is Confidential Client Is Holder Of The Privilege Exceptions To, And Termination Of, The Privilege Work Product
-

13 13 13 13 13 13 13

C. PHYSICIAN/PSYCHOTHERAPIST PATIENT PRIVILEGE


Physician-Patient Privilege Psychotherapist-Patient Privilege
-

14
14 14

D. SELF INCRIMINATING TESTIMONY


1. 2. 1. 2. 3. 4. 5. 6. Witness Privilege Privilege Of Accused Priest-Penitent Required Reports Vote Accountant-Client Newsperson Sources Government Secrets

15
15 15

E. OTHER PRIVILEGES

16
16 16 16 16 16 16

F. INSURANCE COVERAGE G. H. SUBSEQUENT SAFETY MEASURES

16 16

COMPROMISE, PAYMENT OF MEDICAL EXPENSES, PLEA 16 NEGOTIATIONS


1. 2. 3. 4. 5. Statement Of Rule Exceptions Medical Payments Settlements With Third Parties Criminal Plea Bargaining 16 17 17 17 17

I. PAST SEXUAL CONDUCT

17

IV. WRITINGS, RECORDINGS, AND PHOTOGRAPHS


A. REQUIREMENT OF AN ORIGINAL
1. 2. Rule Applies When Contents Must Be Proven Original Document

18
18
18 18

iii

3.

Degrees Of Secondary Evidence

18

B. C.

SUMMARIES COMPLETENESS RULE

18 19

V. HEARSAY AND CIRCUMSTANCES OF ITS ADMISSIBILITY A. DEFINITION OF HEARSAY


1. 2. 3. 4. Statement Out-Of-Court Statements That Are Not Hearsay Admissions Multiple Hearsay

19
19
19 19 20 21

B. EXCITED UTTERANCES AND STATEMENTS OF PRESENT 21 SENSE IMPRESSIONS


C. D. E. F. G. H. STATEMENTS OF MENTAL, EMOTIONAL, OR PHYSICAL 21 CONDITION STATEMENTS FOR THE PURPOSE OF DIAGNOSIS OR 21 TREATMENT PAST RECOLLECTION RECORDED BUSINESS RECORDS PUBLIC RECORDS LEARNED TREATISES DEPOSITIONS
Unavailability As A Requirement 1. 2. Former Testimony Depositions 3.

21 22 22 22 22
22 22 22

I. THE PRIOR RECORDED TESTIMONY EXCEPTION,

J. STATEMENTS AGAINST INTEREST EXCEPTION K. OTHER EXCEPTIONS


1. 2. 3. 4. 5. Dying Declaration Vital Statistics And Family History Ancient Documents Market Tabulations Residual Exception

23 23
23 23 23 23 23

iv

EVIDENCE

I. PRESENTATION OF EVIDENCE A. INTRODUCTION OF EVIDENCE


1. Requirement Of Personal Knowledge
A witness may not testify to a matter unless evidence is introduced sufficient to support a finding that she has personal knowledge of the matter. The testimony of the witness herself may be used to prove that personal knowledge. The personal knowledge requirement does not apply to expert testimony. Hearsay comes within this rule only insofar as the witness must have personal knowledge of the statement.

2. Refreshing Recollection
When the memory of a witness on a subject is exhausted, the examiner can attempt to refresh it by calling particular attention to some fact or event. A witness may also refresh his memory by reference to a writing. Any document so used while on the stand, or, in the discretion of the judge, any document so used before taking the stand, may be examined by opposing counsel for purposes of cross-examination, and relevant portions thereof may be admitted into evidence.

3. Objections And Offers Of Proof a. Objections 1) Timeliness


An objection must be made as soon as opposing counsel knows she has grounds to object.

2) Specificity
The objection must delineate the specific ground for objection unless the specific ground is obvious from the context.

3) Waiver
A party can waive grounds for objecting by introducing similar inadmissible evidence himself.

4) Probative worth
Evidence admitted without objection, even though inadmissible under an exclusionary rule, is entitled to its full probative worth.

b. Offer of proof
If evidence offered by a party is excluded by a trial judge, the party must make an offer of proof so that the appellate court can effectively review the trial judge's

MicroMash MBE In Brief: Evidence

actions. An offer of proof is merely an explanation read into the record, out of the hearing of the jury, of the expected content of the excluded evidence. The offer of proof serves the additional purpose of informing the trial judge about the testimony, allowing her an additional chance to pass on its admissibility. Federal Rule of Evidence 103(a)(2) requires such an offer of proof unless the substance of the offer was apparent from the context of the questions asked. An offer of proof is required only when the examiner can be expected to know what the answer to her question will be. Thus, an offer of proof is required when counsel is examining her own witness on direct examination, and when she is interrogating a friendly witness on crossexamination. It is not ordinarily required when a question asked on crossexamination is excluded. When required, the offer must be specific and give the court sufficient facts to determine the question of admissibility.

4. Lay Opinions
A layperson may testify to inferences or give opinions if they are rationally based upon the perception of the witness and helpful to a clear understanding of his testimony or the determination of the facts in issue. Lay opinions are not permissible on subjects reserved for expert opinions and on legal conclusions.

5. Competency Of Witnesses
Every person is competent to be a witness except: a witness who would be incompetent under state law if state law controls, as it does in diversity cases; a witness who lacks personal knowledge, except for an expert witness; a witness who cannot understand that she must tell the truth; the trial judge; a juror.

6. Judicial Notice a. Adjudicative facts


A court must upon request, or may of its own motion, take judicial notice of any fact which is either generally known within the territorial jurisdiction or is capable of ready and accurate determination by sources whose accuracy cannot be questioned. The judge's personal knowledge of the fact is irrelevant to this determination.

b. Judicial notice of law


A court will judicially notice the law of a state in which it is sitting and federal law. The law of sister states and foreign countries will be noticed if there is statutory authorization.

MicroMash MBE In Brief: Evidence

c. Procedure Counsel is entitled to a hearing on the propriety of taking judicial notice. Once judicial notice is taken, evidence is not admissible on the issue and, except in criminal cases, the jury must accept the fact judicially noticed. 7. Roles Of Judge And Jury a. Preliminary questions of fact The jury decides preliminary questions of fact when the relevancy of other evidence is dependent upon such a finding, as long as there is sufficient evidence to find that preliminary question. The trial judge determines those preliminary questions of fact on which the applicability of exclusionary rules depends. He is not bound by any rules of evidence (except privilege) in making preliminary findings. Hearings on preliminary matters must be out of the jury's hearing in all cases on the admissibility of confessions, and in other cases where justice requires. The accused may testify at a preliminary hearing without subjecting himself to cross-examination on all issues. b. Function of appeals court on evidentiary matters In addition to the limitations discussed above with respect to the proper preservation of an issue for appeal, an appellate court will uphold a judgment if the trial judge acted within his discretion, or if the evidentiary error did not affect the substantial rights of the parties and was therefore harmless. 8. Limited Admissibility In some instances evidence will be admissible for one purpose and inadmissible for another purpose. If the trial judge rules adversely to either a request to admit such evidence or an objection to its admissibility, it is the obligation of the lawyer who lost the ruling to specify the limited basis for its admission. The general principle behind this rule covering the offering of evidence and the objections thereto is that the trial judge is entitled to the assistance of counsel in making her rulings. If counsel are not specific on either the admission or exclusion of evidence, the trial judge's ruling will be upheld if it is right for any reason. a. Admitting evidence A party desiring to offer physical or documentary evidence does so by having it authenticated by a witness, showing it to the trial judge and opposing counsel, and then asking the judge to admit it. Oral evidence is offered by asking questions of a witness on the witness stand. Once the offering party has complied with these formalities, the obligation is upon the opponent to object to the admissibility of the evidence. b. Specifying reason for admissibility If the trial judge sustains a general objection, and the evidence is objectionable for any reason, the trial judge will be upheld upon appeal unless counsel specifies the

MicroMash MBE In Brief: Evidence

limited ground for which he is offering the evidence, and the trial judge improperly excludes the evidence offered for that limited purpose.

B. PRESUMPTIONS 1. Conclusive Presumptions


Irrebuttable or conclusive presumptions are merely a different way of stating rules of substantive law which provide that proof of the first set of facts conclusively proves the second proposition.

2. Presumptions Of Fact
Presumptions of fact are rules holding that inferences which can be drawn from certain facts have enough probative value on a second issue that a verdict cannot be directed on that second issue if those facts are proven.

3. Presumption Of Innocence
The presumption of innocence is another way of stating that in a criminal case the prosecution has the burden of proving all the facts necessary for a conviction beyond a reasonable doubt.

4. True Or Rebuttable Presumptions


A true or rebuttable presumption operates to shift the burden of production when the person having the benefit of the presumption introduces evidence of the basic facts necessary to bring the presumption into operation. If the basic facts are proven and the person against whom the presumption operates offers no evidence on the presumed fact, then the presumed fact has likewise been proven. If she does offer evidence on the presumed fact, the artificial procedural effect of the presumption disappears, but any inferences that can be drawn about the presumed fact from the existence of the basic fact may still be drawn.

5. Legislatively Created Presumptions


Legislatively created presumptions in criminal cases are unconstitutional unless the presumed fact is more likely than not to flow from the proven fact on which it is made to depend.

C. MODE AND ORDER 1. Control By The Court


While there is a traditional order in which a case is presented (in which the plaintiff presents his affirmative case first, followed by the defendant's case, followed by rebuttal), the order of presentation of the case and the order of the presentation of witnesses are controlled by the sound discretion of the trial court.

a. Direct examination
The scope of a witness' testimony on direct examination is limited to the matters which are relevant to the issues presented in the case, and if the witness is called for

MicroMash MBE In Brief: Evidence

the purpose of impeaching another witness, a witness' testimony is limited to proper matters of impeachment discussed below. b. Cross-examination 1) Definition Cross-examination is defined as the examination commencing after the examination by the party calling the witness. Because cross-examination is a fundamental right, testimony given on direct examination will be stricken if that right has been denied. 2) Improper form Even though leading questions are permissible on cross-examination, questions will be inadmissible if they are argumentative, misleading, or contain more than one question. 3) Scope of cross-examination Examination on cross-examination is limited to the subject matter of direct examination and matters affecting the credibility of the witness. In its discretion, the court may permit inquiry into additional matters as if on direct examination. 4) Redirect and recross-examination On redirect examination, counsel may inquire into matters raised in crossexamination and matters rehabilitating the credibility of the witness. Recrossexamination is limited to matters raised on redirect examination. 2. Form Of Questions, Leading Questions, And Narrative Testimony Leading questions are those in which the examiner suggests in the question the answer which she desires. Interrogation by leading questions is permissible on crossexamination, except where the witness is the cross-examiner's own client or is closely allied with that client. Leading questions are not allowed on direct examination except: where the witness is the party opponent or is hostile; to cover preliminary matters; where the witness' memory is exhausted; where the witness has a weak memory or is of tender age.

Narrative testimony is permitted in the discretion of the trial judge. 3. Exclusion Of Witnesses A judge must at the request of a party, and may on his own motion, exclude all prospective witnesses from the trial, except for a party, an officer of a party, or a person shown to be essential to a party.

D. IMPEACHMENT
Any party, including the party calling her, can impeach a witness in one or more of the following ways.

MicroMash MBE In Brief: Evidence

1. Prior Inconsistent Statements a. No hearsay problem if used only for impeachment Prior inconsistent statements are relevant for impeachment because the very fact that they were made casts doubt upon the veracity of the statement made on the stand; therefore, they are not hearsay when used only for impeachment. b. When admissible substantively Such statements are admissible substantively when they come within an exception to the hearsay rule, or if they were made under oath. c. Inconsistency required To be admissible, such statements must be inconsistent with a statement made on the stand. d. Foundation required for use of extrinsic evidence No foundation is required to inquire about an inconsistent statement on crossexamination, but if extrinsic proof of the inconsistent statement is to be made, then the witness must be afforded an opportunity to explain or deny it. e. Extrinsic evidence on collateral matters inadmissible Extrinsic evidence of a prior inconsistent statement about a collateral matter is not admissible. 2. Bias Evidence of bias is admissible to impeach credibility because it shows reasons why a witness might lie. Hatred, love, kinship, or interests in the outcome of the litigation are common examples of bias. A witness must be asked about his bias before it can be proven by extrinsic evidence. 3. Conviction Of A Crime The court must admit any conviction involving fraud (i.e., dishonesty or false statement) against any witness, as long as the conviction is less than 10 years old. The court may admit a recent conviction of a criminal defendant for any felony not involving fraud only if the impeaching party first shows that the probative value of the conviction outweighs its prejudicial effect on the accused. The court must admit a recent nonfraud felony conviction of any witness other than a criminal defendant unless the objecting party shows that the prejudicial effect of the impeachment substantially outweighs the probative value of the evidence. A conviction more than 10 years old can be admitted against any witness only if the impeaching party first shows that the probative value of the conviction substantially outweighs its prejudicial effect. Such evidence is presented by asking the witness about the conviction, or by the introduction of a certified copy of the conviction. If the witness was pardoned because of

MicroMash MBE In Brief: Evidence

a finding of innocence or because she has been rehabilitated and has not been convicted since then, the conviction may not be used to impeach. Convictions of a crime while a juvenile are admissible in the discretion of the court against a witness who is not a defendant. 4. Specific Instances Of Conduct The judge in his discretion can permit inquiry on cross-examination into specific conduct that is probative of truthfulness or untruthfulness. Extrinsic evidence of such conduct is not allowed. 5. Character For Truthfulness A witness who demonstrates a knowledge of a witness' reputation in the community, or who shows direct knowledge of a witness' character, may testify either to the reputation of another witness for truth and veracity or give her own opinion about that truth and veracity. Evidence that a witness is truthful, however, is admissible only after the character of the witness for truthfulness has been attacked. On cross-examination, such a character witness may be asked about specific instances of conduct of the witness about whom she is testifying. 6. Inability To Observe, Remember, Or Relate Accurately If a lay witness did not have any opportunity to observe the relevant event, or does not remember anything about the event, then his testimony is inadmissible. A witness can be questioned about any deficiency in testimonial faculties on cross-examination, and extrinsic evidence of a deficiency is admissible if the matter is not collateral. 7. Contradiction Testimony by another witness that contradicts the testimony of an earlier witness impeaches the earlier witness. Such impeachment is always permissible when it concerns a material fact. It is not permissible when the testimony concerns a collateral matter, unless the trial judge in her discretion determines that the witness is a very important one and the contradiction will greatly affect her credibility. 8. Impeachment Of The Hearsay Declarant If hearsay evidence is admissible, the out-of-court declarant can be impeached in the same way he could if he were a witness at the trial. 9. Rehabilitation Once the credibility of a witness has been attacked, it may be rehabilitated by either party. a. Good character If character has been attacked, good character can be shown. b. Prior consistent statements If impeaching evidence has been introduced which implies that the testimony on the witness stand is a recent fabrication, or was given as a result of improper motive or

MicroMash MBE In Brief: Evidence

influence, prior consistent statements are admissible substantively as well as for impeachment purposes.

c. Bias
Evidence of bias may be rebutted, but the bias may not be justified.

d. Impeaching the impeacher


In limited instances, the credibility of the impeachment witness may be impeached.

E. PROCEEDINGS TO WHICH EVIDENCE RULES APPLY


The Federal Rules of Evidence govern proceedings in courts of the United States and district courts in the territories, hearings before bankruptcy judges, and hearings before United States magistrates. However, they do not apply to those proceedings in which the court may act summarily and to proceedings under Title 11 of the United States Code. Except for the rules of privilege, the rules do not apply to preliminary questions of fact determined by the trial judge, grand jury proceedings, extradition proceedings, criminalsentencing proceedings, proceedings to grant or revoke probation, proceedings for the issuance of arrest or seat warrants, and bail hearings.

II. RELEVANCY AND REASONS FOR EXCLUDING RELEVANT EVIDENCE A. CONCEPT OF PROBATIVE VALUE
1. Relevancy Evidence is relevant if it has any tendency to make the existence of any fact that is of consequence to the determination of the action more probable or less probable than it would be without the evidence. The evidence standing alone need not be sufficient to warrant a finding on the issue. Relevancy deals with the ability of evidence to help prove or disprove a fact, whereas materiality adds the condition that the fact proven be of consequence in the lawsuit.

2. Counterweights To Relevancy
Even though evidence passes the extremely liberal test of relevancy, evidence will be inadmissible if its probative value is substantially outweighed by the danger of unfair prejudice, confusion of the issues, misleading the jury, or by considerations of undue delay, waste of time, or needless presentation of cumulative evidence.

B. AUTHENTICATION 1. Method Of Authentication


A writing must be authenticated before it is admissible. It can be authenticated by testimony of a witness who saw the document signed or who is familiar with the signature on the document, or by the introduction of an admittedly genuine specimen of the signer's signature. It may also be authenticated circumstantially because it contains

MicroMash MBE In Brief: Evidence

information known only to the author, because it was over 20 years old and taken from a place of proper custody, or because it was in reply to an earlier document. 2. Voice Identification A voice is authenticated either (a) if the witness recognizes it, or (b) if the witness dials an individual's number, asks for that person, and the person answering the phone identifies herself. 3. Self-Authentication Various kinds of official documents are self-authenticating, provided they bear proper official indicia. In some instances, documents such as wills must be authenticated by testimony of the official witnesses.

C. CHARACTER AND RELATED CONCEPTS


1. Character In Issue When character is directly in issue, it may be proven by specific instances of conduct as well as by reputation evidence and opinion evidence. 2. Character Not Directly In Issue If character is not directly in issue but is offered to show that an individual acted in conformity with a specific trait, the following rules apply. a. Use in criminal cases The criminal defendant may introduce character evidence to show that he is not the type of person who would have committed the crime, and the prosecution may use character evidence only in rebuttal of the defendant's character evidence. b. May not be used in civil cases Character evidence may not be introduced in civil cases unless character or reputation is directly in issue. c. Character of the victim The criminal defendant may introduce evidence of the victim's relevant character traits. In rebuttal, the prosecution can introduce evidence of the victim's character, or the same character trait of the accused, and to show the peaceful character of a homicide victim if self-defense is raised. Federal Rule of Evidence 412(a) prohibits the defendant in a rape case from using opinion or reputation evidence concerning the victim's past sexual behavior. In a civil case such evidence is admissible only if the alleged victim's reputation has been placed in controversy by the alleged victim. d. Character of witness Character evidence may be introduced concerning the truth and veracity of any witness. Evidence of truthful character is admissible only after the character of the witness for truthfulness has been attacked by opinion or reputation evidence or otherwise.

10

MicroMash MBE In Brief: Evidence

3. Methods Of Proving Character


Character may be proven by testimony of a witness who is familiar with the individual's reputation concerning that character trait, or by the character witness' opinion of that individual with respect to that character trait. It may not be proven by specific instances of conduct not material to the issues in the lawsuit, (i.e., when character is not directly in issue, but a character witness may be asked about specific instances of conduct on crossexamination).

4. Other Crimes
Evidence that the defendant has committed other crimes is inadmissible, except when offered to prove some fact other than the character of the defendant, such as motive, opportunity, intent, preparation, plan, knowledge, identity, or absence of mistake. If evidence of other crimes is admissible, it need not be proven by convictions of these crimes.

5. Habit, Custom, And Routine Practice


Habit, a regularized response to a specific type of situation, may be proven to show circumstantially that the response at the time in issue was in accord with that habit. Routine practice in the field of business can be proven for the same purpose. Habits of animals are also admissible to prove their conduct on specific occasions.

6. Similar Happenings And Transactions


Evidence of similar happenings or transactions is not admissible unless it is extremely probative on the transaction in issue, or unless it is relevant on some other issue in the lawsuit.

a. Similar lawsuits
Evidence of similar lawsuits is not admissible unless the other lawsuit is connected in some special way with the lawsuit on trial.

b. Prices
Prices received for the sale of similar property are admissible on the issue of valuation. Similar fraudulent transactions are admissible to prove absence of mistake in the case on trial. Contract dealings between the parties are admissible to aid in interpretation, but dealings between a party and a third person are not.

c. Prior accidents
Prior accidents are not admissible, unless they are offered to show notice of a defective condition or to show causation in the case on trial.

MicroMash MBE In Brief: Evidence

11

D. EXPERT WITNESSES AND SCIENTIFIC EVIDENCE

1. Expert Witnesses a. Qualification


An expert witness must be qualified in her field of testimony before she can give expert testimony.

b. Fields of expert testimony


A judge may permit expert testimony if it will assist the trier of fact to understand the evidence or determine a fact in issue.

c. Ultimate issue
Opinions are permitted on the ultimate issue, except when the question asks for a general conclusion in terms of inadequately explained legal criteria, or when the issue is the criminal defendant's mental state at the time of committing the crime.

d. Sources of knowledge for experts


Many times an expert will have first-hand knowledge of the factual matters upon which his opinion is based. In addition, he may use second-hand information if it is the type reasonably relied upon by experts in the particular field, even if such data is not admissible in evidence. The basis for an expert opinion need not be disclosed in advance of the opinion, but can be elicited on cross-examination.

2. Experimental And Scientific Evidence


Experiments are admissible in the trial judge's discretion if they accurately reproduce the relevant event. Scientific evidence falls into three categories. If the scientific principle is beyond dispute, the court will take judicial notice of the principle, and expert testimony is only necessary on its particular application. Even if the scientific procedure has not become generally accepted, it is possible to introduce evidence of test results, provided that expert evidence is produced at the trial concerning the validity of the test procedure involved and that the trial judge finds sufficient evidence of its reliability. Scientific evidence introduced through a qualified expert is admissible as long as it is both relevant and reliable. If the principle is in dispute and is likely to mislead the jury, such as the principle of the polygraph test, the evidence concerning the test is admissible only if counsel stipulate its admissibility.

E. DEMONSTRATIVE EVIDENCE 1. Real Evidence


Real proof must be relevant and may be excluded even if relevant if it is highly prejudicial. It must also be authenticated. If it is not distinctive or distinctively marked, it must be authenticated by showing a chain of custody from the time of the relevant event until trial.

12

MicroMash MBE In Brief: Evidence

2. Demeanor And Views A trier of fact can base her findings upon observation of the demeanor of witnesses and evidence deduced from observations made in the courtroom and from authorized views. 3. Sound And Picture Recordings Sound and picture recordings of a relevant event are admissible when properly authenticated.

III. PRIVILEGES AND OTHER POLICY EXCLUSIONS


A. INTERSPOUSAL PRIVILEGE 1. Right Of Spouse To Refuse To Testify In Criminal Cases The spouse of a criminal defendant has the right to refuse to testify, provided that the marriage relationship exists at the time of trial. 2. Right To Keep Confidential Communications Out Of Evidence A spouse also has the right to prevent his or her spouse from testifying to confidential communications made during the marriage. The rule does not apply in a criminal case where the victim is the spouse or the children. a. Communications Communications are words and actions designed to transmit information, but do not include observations about physical conditions made by a spouse. b. Confidential The presence of third parties, except young children, destroys the confidential nature of the communication. c. Holder of the privilege The holder of the privilege is the spouse transmitting the information, but if there is a dialogue, each spouse is the holder. d. Timing The privilege only applies to communications made while the parties are married. e. Waiver The privilege is waived if the holder makes the subject matter of the communication public. f. Exceptions The privilege does not apply in suits between spouses.

MicroMash MBE In Brief: Evidence

13

g. Eavesdroppers While the earlier rule was that an eavesdropper is entitled to testify concerning confidential marital communications, there is a growing trend toward excluding such testimony.

B. ATTORNEY CLIENT AND WORK PRODUCT


-

1. Statement Of Rule Confidential communications between a client and lawyer consulted in a professional capacity are privileged at the option of the client. The client need not hire the lawyer or pay a fee. Communications from the lawyer to the client are also privileged. 2. Communications That Are Protected Both written and oral communications to the lawyer are privileged, but the fact of representation, observations of physical conditions made by the lawyer, and preexisting documents are not. 3. Communications Between Lawyer And Subordinate The privilege also extends to communications made to the lawyer by his agent in preparation for a lawsuit. 4. Eavesdroppers Under the modern approach, an eavesdropper cannot testify to privileged lawyer-client communications. 5. When A Communication Is Confidential The presence of third parties necessary for the conduct of the lawyer's business does not destroy the confidential nature of the communication. Communications made in the presence of another client are privileged in a suit against a third person, but they are not privileged when one such client is suing the other. 6. Client Is Holder Of The Privilege The client is the holder of the privilege and can waive it. The attorney must claim the privilege unless the client waives it or authorizes her to waive it. 7. Exceptions To, And Termination Of, The Privilege The privilege does not apply when the purpose of the communication is to plan or perpetrate a crime or fraud. Death does not terminate the attorney-client privilege. It may be claimed by the client's executor, but may not be claimed by anyone in a will contest. Publication of the matter communicated to the lawyer or testimony by the client concerning the conversation with the lawyer will waive the privilege. 8. Work Product A qualified immunity protects from discovery materials prepared for litigation. Federal Rule of Civil Procedure 26(b)(3) states: "A party may obtain discovery of documents and tangible things otherwise discoverable...and prepared in anticipation of litigation

14

MicroMash MBE In Brief: Evidence

or for trial by or for another party or by or for that other party's representative (including his attorney, consultant, surety, indemnitor, insurer, or agent) only upon a showing that the party seeking discovery has substantial need of the materials in the preparation of the party's case and the party is unable without undue hardship to obtain the substantial equivalent of the materials by other means." (Emphasis added). The policy underlying the "work product" immunity is the necessity for the lawyer to investigate all facets of the case and develop his theories without fear of having to disclose his strategies or information that is unfavorable to his client. The work-product immunity now extends to persons other than the attorney who are representing a party, e.g., investigators or insurers. However, the document must have been prepared "in anticipation of litigation"; documents prepared in the regular course of business are not within the work-product immunity. The immunity protects only the documents or things themselves; an adversary can use interrogatories or depositions to discover the existence or nonexistence of the documents. Material prepared in anticipation of one suit should also have qualified immunity in a later suit. Discovery of work-product information may be allowed if it is otherwise unobtainable, e.g., if a witness is dead or his memory is faulty. In determining whether "undue hardship" exists, the court will consider the cost of otherwise obtaining the material, the financial resources of the party seeking it, and the likelihood that a "substantial equivalent" cannot be obtained (e.g., a witness who is an employee of the defendant may be hostile toward a discovering party who seeks a statement, and a "substantial equivalent" may therefore be unobtainable, and thus a transcript of his original statement to his employer's attorney might not be immune). Absolute immunity from discovery protects "the mental impressions, conclusions, opinions or legal theories" of an attorney or other representative of a party concerning the litigation. C. PHYSICIAN/PSYCHOTHERAPIST-PATIENT PRIVILEGE 1. Physician-Patient Privilege The physician-patient privilege is a statutory privilege by which the patient can prevent the disclosure of confidential communications made to a physician, and the disclosure of observations made by her. The confidential nature of the communications or observations is not destroyed by the presence of third persons necessary to the performance of the physician's duties. The privilege is waived if the patient introduces evidence on her physical condition or sues the physician. 2. Psychotherapist-Patient Privilege The United States Supreme Court has recognized the psychotherapist-patient privilege. Conversations between a patient and the therapist and notes taken during the counseling session are protected from compelled disclosure under Fed. R. Evid. 501. The court reasoned that effective psychotherapy depends upon an atmosphere of confidence and trust. The possibility of compelled disclosure would destroy that confidence and trust. In contrast, the evidentiary benefit of compelling disclosure would be modest. The patient-

MicroMash MBE In Brief: Evidence

15

psychotherapist privilege applies not only to conversations with psychiatrists and psychologists, but also with licensed social workers in the course of psychotherapy. Courts will not balance the relative importance of the patient's need for privacy against the evidentiary need for the disclosure of the information. If the conversation is between a patient and a psychotherapist, the privilege applies. However, if there is a serious threat of harm to the patient or others that can only be averted by disclosure by the psychotherapist, the privilege will not apply.

D. SELF - INCRIMINATING TESTIMONY


The privilege against self-incrimination is a constitutional privilege in two parts: the witness privilege and the privilege of a criminal defendant not to testify. 1. Witness Privilege a. Applicability The witness privilege is available in all types of proceedings, including legislative investigations, but only by natural persons who claim that the answer would incriminate them. They cannot claim the privilege on the ground that the answer would incriminate a third party. Loss of social status or humiliation is not a ground for claiming the privilege. The privilege may not be claimed for a crime that is clearly barred by the statute of limitations. It can be claimed for activities that are a crime in another jurisdiction. A witness may refuse to answer, however, if a responsive answer could possibly lead to evidence of a crime. b. Immunity The government can compel a witness to give an answer if it gives him immunity that prevents the prosecution from using the answer or any evidence derived therefrom in any future trial. Immunity granted in one jurisdiction prevents use of the testimony or its derivatives to prosecute in a second jurisdiction. c. Waiver A witness who voluntarily testifies and discloses substantial information about a subject can be required to testify further about the same subject matter. A witness can also claim the privilege and refuse to produce documents that are summoned, except when there is a legal requirement that the records be kept. Except in the Miranda custodial-interrogation situation, a person has no right to be warned of her privilege against self-incrimination. 2. Privilege Of Accused The accused in a criminal proceeding has the right to refuse to be a witness. This right attaches only when formal legal proceedings have been commenced against him. The privilege, however, is testimonial. The accused may be required to stand up, try on clothes, give fingerprints, and give handwriting and voice samples. The prosecution may not comment on the failure of the accused to testify. If the accused voluntarily takes the stand, he waives his privilege against self-incrimination as to his credibility and matters within the scope of direct examination.

16

MicroMash MBE In Brief: Evidence

E. OTHER PRIVILEGES
1. Priest-Penitent The priest-penitent privilege is law in most states. It requires a confidential communication to a member of the clergy for the purpose of obtaining spiritual advice. 2. Required Reports Reports required to be filed with government agencies, except income tax returns, are privileged if the statute confers such privilege. 3. Vote The political vote is privileged, and its tenor need not be disclosed. 4. Accountant-Client A few states recognize a privilege for confidential communications between an accountant and her client, but there is no such federal privilege. 5. Newsperson Sources There is no constitutional privilege for a newsperson to refuse to disclose sources. 6. Government Secrets The government has a right to refuse to disclose secret information concerning national defense or international relations and official information that is protected from disclosure under freedom-of-information acts. The government likewise has the right to refuse to disclose the identity of an informer, provided that he is not known to those who would harm him, and his testimony is not necessary to a fair determination of the case. If the government exercises one of its privileges and is a party to the suit, it must suffer adverse judgment.

F. INSURANCE COVERAGE
Evidence that the defendant is or is not covered by liability insurance is not admissible to prove that the defendant acted negligently or otherwise wrongfully.

G. SUBSEQUENT SAFETY MEASURES


Measures taken after an event which would have made the event less likely to occur if taken previously are not admissible to prove negligence or culpable conduct in connection with the event, but are admissible to show control of premises or to show the feasibility of precautionary measures if they are in issue.

H. COMPROMISE, PAYMENT OF MEDICAL EXPENSES, PLEA NEGOTIATIONS


1. Statement Of Rule Offers to settle or compromise a claim that is in dispute as to either validity or amount are inadmissible. Conversations that are part of the settlement process are likewise

MicroMash MBE In Brief: Evidence

17

inadmissible. Offers to buy off criminal liability by payment of civil damages are admissible in a subsequent criminal case. 2. Exceptions Such offers are admissible when there is no dispute, or when they are offered to show something other than liability. 3. Medical Payments The fact of payment of, or an offer to pay, a party's medical expenses is inadmissible. 4. Settlements With Third Parties Settlements with a third party are inadmissible unless offered to show something other than the defendant's liability. However, statements made by a potential criminal defendant to a victim in an attempt to buy off criminal liability are not protected. 5. Criminal Plea Bargaining In criminal cases, conversations in connection with plea bargaining and withdrawal of guilty pleas are excluded.

I. PAST SEXUAL CONDUCT


Federal Rule of Evidence 412 excludes in civil and criminal proceedings involving alleged sexual misconduct evidence offered to prove that any alleged victim engaged in other sexual behavior, or evidence offered to prove any alleged victim's sexual predisposition, with the following exceptions. In criminal cases, evidence of specific instances of sexual behavior by the alleged victim is admissible if offered to prove that a person other than the accused was the source of semen, injury, or other physical evidence. Evidence of specific instances of sexual behavior by the alleged victim with respect to the person accused of the sexual misconduct is admissible if offered by the accused to prove consent. Evidence, the exclusion of which would violate the constitutional rights of the defendant, is also admissible. In civil cases, the evidence of past sexual behavior must be otherwise admissible and its probative value must substantially outweigh the danger of harm to any victim and of unfair prejudice to any party. Evidence of the alleged victim's reputation is admissible only if it has been placed in controversy by the alleged victim. Any party intending to offer evidence of an alleged victim's other sexual behavior must file a written motion at least 14 days before trial, unless the trial judge for good cause waives this requirement, and the judge must hold an in camera hearing at which the alleged victim and the other parties have a right to be heard. The record of that hearing is sealed unless the judge orders otherwise.

18

MicroMash MBE In Brief: Evidence

IV. WRITINGS, RECORDINGS, AND PHOTOGRAPHS A. REQUIREMENT OF AN ORIGINAL


If the content of a writing is to be proven, the original writing is required unless production of the original is excused. 1. Rule Applies When Contents Must Be Proven The rule only applies when the contents of a document are to be proven, not when an individual is testifying about observations that she memorialized in a document. 2. Original Document The original document is that writing which controls the rights of the parties under substantive law. a. Duplicate A duplicate is a photographic reproduction of the original and is admissible to the same extent as the original, unless there is a genuine question of the authenticity of the original, or if it would be unfair to admit the duplicate. b. Excuse for nonproduction of original The production of the original is excused when it is in the hands of a party opponent, and a notice to produce has been given but not complied with; when the original is in the hands of a third party and is not subject to summons; when the original is lost and there has been a diligent search, or when it has been destroyed by someone other than the party seeking to offer it; when the document is a certified copy of an official document; and when the contents of the document are collateral to the issues in the lawsuit. 3. Degrees Of Secondary Evidence If there is excuse for nonproduction of the original, any degree of secondary evidence is admissible. If there are duplicate originals, there must be excuse for nonproduction of both. a. Admission by opponent Secondary evidence is also admissible if the opponent admits the contents of a document in his testimony, a deposition, or in another writing. b. Function of judge and jury Except when the existence of a document is an issue in a lawsuit, the admissibility of secondary evidence is a preliminary question for the trial judge.

B. SUMMARIES
The contents of voluminous writings, recordings, or photographs that cannot conveniently be examined in court may be presented in the form of a chart, summary, or calculation. The originals, or duplicates, shall be made available for examination or copying, or both, by other

MicroMash MBE In Brief: Evidence

19

parties at a reasonable time and place. The judge may order that they be produced in court. Fed. R. Evid. 1006.

C. COMPLETENESS RULE
Federal Rule of Evidence 106, known as the completeness rule, permits a party to introduce into evidence at the same time that her opponent introduces a document, or part of a document, any other part of the document or any other writing or recorded statement which ought in fairness to be considered contemporaneously with it. This rule does not require that the parts of a document to be introduced be relevant to the issues being tried, and it does not require that such parts of a document meet a hearsay test or any other test. The entire test of its admissibility is whether it is fair to introduce it and consider it contemporaneously with some other document being introduced by the opponent. It can happen that the document introduced by the opponent is inadmissible under these rules of evidence. Nevertheless, Fed. R. Evid. 106 permits the introduction of other evidence relating to the inadmissible document as long as the fairness test is met.

V. HEARSAY AND CIRCUMSTANCES OF ITS ADMISSIBILITY


A. DEFINITION OF HEARSAY
"Hearsay" is a statement other than one made while testifying at the hearing to prove the truth of the matter asserted. The purpose of the hearsay rule is to bring before the trier of fact the most reliable evidence possible. In most instances, testimony by a witness is more reliable than an out-of-court statement because it is under oath, the jury can observe the demeanor of the witness, and, most important of all, the witness is subject to crossexamination. 1. Statement A "statement" is either an oral or a written assertion, or nonverbal conduct if it is intended as an assertion. Words or conduct are assertive when the person making them wants the hearer or observer to believe something is true because he said it. Conduct that was not intended to be assertive when made but which is offered to show the opinion of the actor at the time of the conduct is not a "statement" as that term is defined in the hearsay rule. 2. Out-Of-Court Statements That Are Not Hearsay Out-of-court statements are not hearsay when they are offered for purposes other than the truth contained in them. a. Statements having a legal significance Statements that have legal significance independent of their truth, and statements which are contemporaneous with physical acts and which give the acts a legal effect, are not offered for their truth, and are not hearsay.

20

MicroMash MBE In Brief: Evidence

b. Statements offered circumstantially Statements are not hearsay if offered circumstantially. If knowledge of an individual must be proven, a statement made in her presence containing the requisite information is not hearsay when offered to show that as a result of the statement, the individual possessed the required knowledge. Statements from which an inference about the state of mind of the speaker can be drawn are not hearsay when offered on the issue of her state of mind. An out-of-court statement that is proven false is not hearsay when offered to prove that similar statements made on the stand are also false. c. Out-of-court statements of witnesses Out-of-court statements of witnesses testifying at a trial are admissible to impeach credibility if they are inconsistent with testimony given on the stand, because they are offered only to show that a person who says two different things about a situation should not be believed. Despite the fact that the witness is on the stand and can be cross-examined, such statements are not admissible substantively unless they were made under oath. Outof-court statements that are consistent with in-court testimony are inadmissible, unless they are offered to rebut the inference that the statement on the stand was a recent contrivance or the product of improper influence. Evidence of prior identification made by an eyewitness who testifies at trial is also excluded from the definition of hearsay. 3. Admissions An out-of-court statement of a party is admissible, even though it was in his interest at the time he made it, and even though he had no personal knowledge of the facts contained in it. a. Statement of party In addition to the statement of a party, her actions that are inconsistent with the position she is taking in a case are admissible against her. b. Adoptive admissions Likewise, statements made by others that a party has adopted through his actions are admissions. This most commonly occurs when a party remains silent when a statement is made in his presence which he would deny if it were false. Such statements are not admissible when a defendant is entitled to his Miranda rights. A party may adopt a statement without knowing its precise nature if he indicates that its author is a reliable person. c. Vicarious admissions Statements made by an authorized agent or a partner are admissible.

MicroMash MBE In Brief: Evidence

21

d. Statement of an employee
Even though an employee is not authorized to speak for her employer, her statements made while an employee that involve matters concerning her employment are admissible.

e. Statements of a conspirator
Statements made by a conspirator are admissions against co-conspirators if made in furtherance of the conspiracy and during the course of the conspiracy.

4. Multiple Hearsay
A statement of an out-of-court declarant that is admissible under an exception to the hearsay rule must be one where the out-of-court declarant has personal knowledge of the facts contained in the statement. If he does not, then the statement is totem-pole hearsay and is not admissible unless each layer of hearsay is supported by an exception to the hearsay rule.

B. EXCITED UTTERANCES AND STATEMENTS OF PRESENT SENSE IMPRESSIONS


A statement made by a person while under the stress of an exciting event concerning that event, made of the personal knowledge of the out-of-court declarant, is admissible, even if the out-of-court declarant is available. Even if there is no exciting event, a present sense impression (a statement made by a declarant while viewing an event) is admissible.

C. STATEMENTS OF MENTAL, EMOTIONAL, OR PHYSICAL CONDITION


Statements of present physical or mental or emotional condition are admissible if offered to prove that condition. Statements of present intention are admissible to show future intention and future actions based on that intention.

D. STATEMENTS FOR THE PURPOSE OF DIAGNOSIS OR TREATMENT


Statements of past physical or mental condition are admissible when made for the purpose of diagnosis or treatment. The statement need not be made to a physician, but can be made to any person who might render treatment. A statement made to a doctor for the purpose of having the doctor make a diagnosis and testify about it is admissible.

E. PAST RECOLLECTION RECORDED


If a witness on the stand testifies that she once had a memory of an event, that she recorded her memory at the time of the event, and that she no longer has any memory, then the record of the past memory may be read to the jury but the writing itself is not admissible by its proponent. An adverse party can require that the document be admitted. However, the writing must satisfy the admissibility requirements of a writing, in that it must be authenticated and must not violate the Best-Evidence Rule.

22

MicroMash MBE In Brief: Evidence

F. BUSINESS RECORDS
Records kept in the routine course of business are admissible despite the fact that the declarant is available. Personal knowledge of the person who kept the record is not required, but someone with personal knowledge must have transmitted the information in the usual course of his business. Records that were prepared for litigation, even though kept in the usual course of business, are not admissible. A record is self-authenticating if it is certified to meet the requirement of the business records exception by a qualified person. Business records are also admissible to show the nonoccurrence of an event if it can be shown that the event, if it had occurred, would have appeared in the record.

G. PUBLIC RECORDS
Records of the activity of a public agency, records of observations which a public official has a duty to make, and records of an investigation made pursuant to law are admissible. The original record need not be produced, and a certified copy is admissible. The nonoccurrence of an event can be proven by the absence of an entry in an official record. Evidence of judgments of conviction are admissible to prove any fact necessary to sustain the judgment.

H. LEARNED TREATISES
Learned treatises concerning a subject upon which an expert witness has testified are admissible if established as a reliable authority. Relevant portions may be read to the jury, but the treatise itself is not admissible as an exhibit.

I. THE PRIOR RECORDED TESTIMONY EXCEPTION, DEPOSITIONS


1. Unavailability As A Requirement The prior-recorded-testimony exception, the declaration against interest exception, and the statement made under belief of impending death exception require that the out-ofcourt declarant be unavailable. A declarant is unavailable if she has successfully claimed a privilege, refuses to testify, takes the stand and says she has no memory on the subject, is absent because of death or mental illness or infirmity, or cannot be required to attend by process or other reasonable means. 2. Former Testimony The prior-recorded-testimony exception admits in both criminal and civil cases testimony of a witness in an earlier proceeding who is currently unavailable, if he is testifying against the same party in both cases. In civil cases, prior recorded testimony is admissible if the party against whom it is offered in the second hearing had interests which were similar to the party against whom it was offered in the prior hearing. 3. Depositions A statement in a deposition is generally inadmissible as hearsay. However, any deposition may be used by any party for contradicting or impeaching the deponent's testimony as a witness or for any other purpose permitted by the Federal Rules of Evidence. Likewise, since the statement in a deposition of a deponent who is on the

MicroMash MBE In Brief: Evidence

23

witness stand is made under oath, a prior inconsistent statement in a deposition can be used as substantive evidence. The deposition of an adverse party or of an officer, director, or other agent of an adverse corporate party is an admission and may be admitted for any purpose. The deposition of any witness (party or nonparty) may be used for any purpose if the deponent is (a) dead, (b) more than 100 miles from the trial, (c) infirm or imprisoned and unable to testify, (d) not obtainable by subpoena, or if (e) special circumstances make it desirable in the interests of justice to use the deposition.

J. STATEMENTS AGAINST INTEREST EXCEPTION


When the out-of-court declarant is unavailable, a statement made by her that was contrary to the declarant's pecuniary or proprietary interest and which would likely subject her to criminal or tort liability, or which would likely render invalid a claim which she might possess, is admissible as an exception to the hearsay rule, provided that it was against her interest at the time it was made. If the statement is offered to exonerate a criminal defendant by showing that the out-of-court declarant committed the crime, the evidence must be corroborated.

K. OTHER EXCEPTIONS 1. Dying Declaration


A statement made by a person who believes his death is imminent concerning the cause of or circumstances surrounding that imminent death is admissible as an exception to the hearsay rule in civil cases where the declarant is unavailable and in criminal-homicide prosecutions. 2. Vital Statistics And Family History Records of vital statistics, records of religious organizations, family records, and reputation concerning family history are admissible to prove family relationships. If the declarant is unavailable, her own statement concerning her personal history is admissible. 3. Ancient Documents Documents more than 20 years old in proper custody come within the ancient-documents exception to the hearsay rule. 4. Market Tabulations Tabulations of market-price quotations are admissible. 5. Residual Exception The federal rules contain a residual exception which admits evidence on a material fact if it is more probative on that fact than any other evidence which can reasonably be procured, if its admission serves the interests of justice, and if the adverse party has been given notice of the intention to rely on this exception.

24

MicroMash MBE In Brief: Evidence

MicroMash MBE REVIEW


BAR EXAM ALERTS AT A GLANCE
-

EVIDENCE

I. PRESENTATION OF EVIDENCE
A. INTRODUCTION OF EVIDENCE 1. FIRST HAND KNOWLEDGE
Every witness (except an expert witness and a witness testifying to admissible hearsay) must testify from first-hand knowledge.

2. REFRESHING RECOLLECTION
Present memory refreshed occurs when the witness' memory is revived (e.g., by reference to a document). The witness then testifies to what she remembers. The rules regarding hearsay and admissibility of writings are inapplicable. On the other hand, past recollection recorded occurs when a witness has made a written record on a matter while her memory was fresh, and her memory of that matter is exhausted and cannot be revived. The document itself, which comes within a hearsay exception, is read to the jury and must satisfy the requirements for admissibility of a writing. If a witness brings written documents with her while testifying, opposing counsel can examine them as a matter of right in the course of crossexamination.

3. OBJECTIONS AND OFFERS OF PROOF


An offer of proof is required only when an objection to a question is sustained. The party must state for the record what the answer to the question would be, if known. To preserve an issue for appeal, counsel must object to the admissibility of evidence at the time it is offered.

4. LAY OPINIONS
A layperson can testify in the form of opinion with respect to matters on which laypersons are competent to form opinions, if the opinion is based

MicroMash MBE In Brief: Evidence Bar Exam Alerts

upon personal knowledge and is helpful to an understanding of the testimony. A lay witness cannot testify on matters on which only experts are qualified to give opinions, even if the layperson's opinion is based upon first-hand knowledge. 5. COMPETENCY OF WITNESSES The Federal Rules of Evidence require that a federal court apply the state's rule on matters of competency of witnesses and privilege if state law provides the basis for decision in the federal court (as it would in diversity cases). 6. JUDICIAL NOTICE A jury in a criminal case is not bound to take as true matters that have been judicially noticed. 7. ROLES OF JUDGE AND JURY Hearsay evidence that would be inadmissible at trial is admissible before a judge who is hearing evidence on a preliminary question of fact. B. PRESUMPTIONS If the party seeking the benefit of a presumption introduces evidence from which the jury can find the basic fact giving rise to the presumption, the party against whom the presumption operates must introduce evidence contradicting the presumed fact or face a directed verdict against him on that fact. C. MODE AND ORDER 1. CROSS-EXAMINATION If the opposing party is deprived of his opportunity to cross-examine a witness, the remedy is to strike the direct examination. 2. REDIRECT AND RECROSS-EXAMINATION Generally, testimony on redirect examination must relate to those matters asked on cross-examination. If part of a document is admitted in evidence by one party, the opposing party has the right to introduce any other part of the same document which ought in fairness to be considered with the part already in evidence, even if such evidence would otherwise be inadmissible. 3. FORM OF QUESTIONS Leading questions are not permitted on direct examination except: to cover preliminary matters, where a witness is hostile, where a witness' memory is exhausted, or where the witness because of tender age or weak memory can

MicroMash MBE In Brief: Evidence Bar Exam Alerts

be examined only by leading questions. On cross-examination, leading questions are permissible. An argumentative question (e.g., one that starts "Don't you know that . . .") is inadmissible even on cross-examination.

D. IMPEACHMENT, CONTRADICTION, AND REHABILITATION 1. INCONSISTENT STATEMENTS AND CONDUCT


A prior inconsistent statement is admissible only to impeach unless it comes within an exception to the hearsay rule or it was given under oath, in which case it is admissible to prove the matter asserted. Extrinsic evidence of a prior inconsistent statement is inadmissible to impeach credibility unless the attention of the witness is called to the statement. 2. BIAS AND INTEREST Unless the witness admits the facts relating to bias on cross-examination, extrinsic evidence can be introduced to prove bias. If offered to prove bias, usually inadmissible evidence (such as insurance coverage and other criminal convictions) is admissible. 3. PRIOR BAD ACTS Evidence of bad acts that show fraudulent conduct can be inquired into on cross-examination to impeach credibility, but extrinsic evidence of such conduct cannot be introduced. 4. CHARACTER Character can be attacked either by opinion evidence or by reputation evidence. The character of a witness for truthfulness cannot be introduced until that character trait has been attacked. 5. PRIOR CONVICTIONS Prior convictions of a person can only be introduced to impeach credibility after that person has testified. Evidence of convictions for misdemeanors not involving dishonesty or false statement are always inadmissible to impeach credibility. The party proffering a witness can anticipate impeachment of the witness through the use of prior convictions by introducing the convictions against the witness on direct examination. The court must admit any conviction involving dishonesty or false statement against any witness, as long as the conviction is recent (i.e., less than ten years old).

MicroMash MBE In Brief: Evidence Bar Exam Alerts

The court may admit a recent conviction of the criminal defendant for a crime punishable by death or at least one year imprisonment only if the impeaching party first shows that the probative value of the conviction outweighs its prejudicial effect. The court must admit a recent conviction of a witness (other than the accused) for a nonfraud crime punishable by death or at least one year imprisonment unless the objecting party shows that the prejudicial effect of the impeachment substantially outweighs the probative value of the evidence. A conviction more than ten years old can be admitted against any witness only if the impeaching party first shows that the probative value of the conviction substantially outweighs its prejudicial effect. 6. CONTRADICTION Extrinsic evidence cannot be used to contradict a witness on a collateral matter. 7. IMPEACHING HEARSAY DECLARANTS An out-of-court declarant whose statement is admissible hearsay may be impeached in the same manner as an in-court witness. 8. REHABILITATION A witness' credibility can be rehabilitated only with respect to the manner in which it has been attacked. For example, evidence of good character can only be presented if the witness' character has been attacked. II. RELEVANCY A. AUTHENTICATION Objects that do not have any identifying characteristics must be authenticated by proving a chain of custody from the point at which the object became relevant until the time of trial. A photograph is admissible upon testimony that it fairly and accurately depicts a relevant event. The photographer is not required to so testify; any witness may testify. A telephone voice of an individual is authenticated by testimony either that the witness recognized the voice, or that the witness called the number listed to that individual and the call was answered by a person identifying herself as that individual. A telephone voice of an individual is not authenticated if the witness does not recognize the voice and the person calls the witness and identifies herself.

MicroMash MBE In Brief: Evidence Bar Exam Alerts

A lay witness cannot testify to the genuineness of a signature solely on the basis of comparing the signature in question to an admittedly genuine signature, but a handwriting expert can. Familiarity with a signature by a witness, even at a distant time, is all that is required to authenticate the signature.

B. CHARACTER
Evidence of regularized conduct that can be characterized as a habit is admissible to prove conduct in accordance with that habit. Evidence of other crimes is admissible only to prove identity, motive, notice, opportunity, plan, or similar relevant facts in a criminal case. Conviction of the other crime need not be proven. Evidence of similar events or circumstances is not admissible to prove the relevant event unless the probative value is compelling. Evidence of a character trait to show propensity to act in accordance with that trait is always inadmissible in a civil case. The defense in a criminal case can prove, by opinion or reputation evidence, character traits of the criminal defendant that are inconsistent with the alleged criminal activity. After the defendant introduces such evidence, the prosecution can rebut with similar character evidence. The prosecution cannot initiate proof of character in a criminal case, except to prove the peaceful nature of the victim in a homicide case where the defendant has raised the defense of self-defense.

C. EXPERT TESTIMONY AND SCIENTIFIC EVIDENCE


It is unlikely that you will be given a multiple-choice question in which you must determine if a particular expert is qualified, because qualification involves subjective value judgments made in the discretion of the trial judge. However, beware of the question where a witness is asked a question calling for expert testimony and there is no indication that she has been qualified. Under those circumstances, the question is inadmissible because a proper foundation for the testimony has not been laid. An expert witness need not testify from personal knowledge, but instead may draw inferences from facts presented to him and may rely on the opinions of other experts if to do so is customary in the field of expertise. An expert witness can be cross-examined about specific instances in his background that bear on his qualification as an expert. Except for the mental state of a criminal defendant, an expert witness can give an opinion on the ultimate issue in a case.

MicroMash MBE In Brief: Evidence Bar Exam Alerts

Evidence of a scientific test that fairly represents a relevant event is admissible even if the opposing party had no notice of the test and did not participate in it. D. DEMONSTRATIVE EVIDENCE The judge has wide discretion in deciding whether and in what form to allow demonstrative evidence. The judge may exclude demonstrative evidence as unduly inflammatory, even if it is relevant.

III. PRIVILEGES AND OTHER EXCLUSIONS A. SPOUSAL COMMUNICATION


There are two separate marital privileges, with distinct rules. Under the first, a witness-spouse can refuse to testify in a criminal prosecution of the defendantspouse, but the defendant-spouse cannot keep the witness-spouse off the stand. With respect to this rule, the parties must be married at the time of trial. The second rule applies to confidential communications between individuals who are at the time of the communication, married to each other. The privilege with respect to such communications survives divorce. The presence of third parties capable of understanding the conversation destroys the confidentiality necessary for this spousal privilege.

B. ATTORNEY CLIENT COMMUNICATION


-

The privilege only applies to confidential communications between a client and an attorney for the purpose of obtaining legal advice. The privilege applies even if the attorney is not in fact hired by the client. The privilege applies even if the individual consulted is not an attorney, if the client reasonably believed that he was. The presence of third parties reasonably necessary for either the attorney or the client to perform their duties does not destroy the confidentiality necessary for the privilege. If two clients consult one lawyer, communications in the presence of both clients and the lawyer are privileged in any suit with a third party, but are not privileged in a suit between the clients. The privilege is inapplicable if the purpose of the communication was to commit future fraud or future criminal conduct. The privilege is inapplicable if the client or a disciplinary body calls the attorney's conduct into question and the attorney must reveal the confidential communication to defend himself.

MicroMash MBE In Brief: Evidence Bar Exam Alerts

Turning over preexisting documents to an attorney does not make them privileged, but a letter to an attorney seeking legal advice is privileged. C. PHYSICIAN-PATIENT COMMUNICATION The physician-patient privilege is a statutory privilege by which the patient can prevent the disclosure of confidential communications made to a physician, and the disclosure of observations made by him. The confidential nature of the communications or observations is not destroyed by the presence of third persons necessary to the performance of the physician's duties. The privilege is waived if the patient introduces evidence on his physical condition, or sues the physician.

D. SELF INCRIMINATION
-

The privilege against self-incrimination is a testimonial privilege and does not empower a defendant to refuse to turn over nontestimonial items such as bodily fluids, handwriting, or voice samples. The privilege against self-incrimination (except for cases where the Miranda rule concerning confessions is applicable) operates prospectively, and does not give a defendant the power to suppress a statement already made. A defendant who testifies on a preliminary matter in a criminal case does not waive his right to refuse to testify in the case itself, and cannot be crossexamined in the preliminary hearing on matters beyond the scope of the preliminary hearing. Admissions of ownership for purposes of asserting standing in a hearing on a motion to suppress evidence are not admissible in the criminal trial. The government, by granting use and derivative-use immunity, can compel testimony despite the privilege against self-incrimination.

E. OTHER PRIVILEGES
Other privileges, recognized in some jurisdictions, include a priest-penitent privilege, a social worker-client privilege, and privileges not to disclose one's vote, a newsperson's sources, and government secrets.

F. REMEDIAL MEASURES
Evidence of subsequent remedial measures is not admissible to prove negligence or culpable conduct, but is admissible to prove ownership or control.

G. COMPROMISE, PAYMENT OF MEDICAL EXPENSES, AND PLEA NEGOTIATIONS


An offer to compromise a disputed claim and all statements made in such a context are not admissible to prove liability.

MicroMash MBE In Brief: Evidence Bar Exam Alerts

An offer in compromise is admissible, however, if it is accepted and the party is suing in contract to enforce it. An offer in compromise cannot qualify as such until the other party has made a claim so that a dispute exists. An offer to pay or the payment of medical expenses is likewise not admissible to show liability. However, a statement made in connection with an offer to pay medical expenses is admissible. An offer to plea bargain and statements made in connection therewith are not admissible at a subsequent trial. If the criminal process has not begun, a statement made in an attempt to avoid criminal liability (for example, an offer to pay for goods that were stolen) is admissible at a subsequent trial.

H. PAST SEXUAL CONDUCT


Evidence of a rape victim's sexual conduct is admissible only if: (1) it involves other sexual conduct with the alleged perpetrator or the alleged victim's sexual conduct with a person other than the defendant at the time of the alleged rape and (2) the judge determines that the probative value of the evidence outweighs its prejudicial effect.

IV. BEST EVIDENCE RULE


Secondary evidence used to prove a collateral matter is admissible despite the best evidence rule. The best evidence rule does not require the production of a written record of an event if the witness can testify about that event from first-hand knowledge.

V. HEARSAY AND ITS ADMISSIBILITY

A. DEFINITION OF HEARSAY
Evidence that is hearsay and does not come within any hearsay exception is inadmissible. An out-of-court statement is not hearsay if you do not have to believe the statement is true for it to be relevant in the lawsuit. Nonverbal conduct is hearsay only if the person intended to make an assertive statement by the conduct. Out-of-court statements that are only used circumstantially not to prove the truth of the matter asserted are not hearsay. Examples of this include

MicroMash MBE In Brief: Evidence Bar Exam Alerts

statements admitted to show: (1) the knowledge or state of mind of either the declarant or the recipient of the statement, when such is relevant to a case; (2) the declarant's lack of credibility; or (3) the meaning to the parties of the words involved in a statement.

B. PRIOR STATEMENTS OF WITNESSES 1. PRIOR INCONSISTENT STATEMENTS


If a witness testifies on the stand, a prior inconsistent statement by the witness given under oath subject to the penalty of perjury is admissible substantively to contradict the witness. If the prior inconsistent statement was not made under oath subject to the penalty of perjury, it is only admissible to impeach. If the witness is not on the stand, a prior statement can be admissible under the former-testimony exception to the hearsay rule, if the necessary requirements are met (the declarant is unavailable, etc.).

2. PRIOR CONSISTENT STATEMENTS An out-of-court statement consistent with testimony on the witness stand is
admissible substantively for the purpose of showing that the testimony given on the witness stand is not a recent contrivance when the opposing party has impeached credibility by use of a prior inconsistent statement.

An out-of-court statement consistent with testimony on the witness stand is admissible substantively to rebut an inference of bias if the consistent statement was made prior to the time that the reason for the bias occurred.

3. PRIOR EYEWITNESS IDENTIFICATION


A prior identification by a witness is admissible if the witness is on the stand and testifying subject to cross-examination.

4. STATEMENTS BY PARTY-OPPONENT
An out-of-court statement of a party is admissible, even though it was in his interest at the time he made it, and even though he had no personal knowledge of the facts contained in it. In addition to the statement of a party, his actions that are inconsistent with the position he is taking in a case are admissible against him. Statements made by others that a party has adopted through his actions are admissions. This most commonly occurs when a party remains silent when a statement is made in his presence which he would deny if it were false. Such statements are not admissible when a defendant is entitled to his Miranda rights. A party may adopt a statement without knowing its precise nature if he indicates that its author is a reliable person.

10

MicroMash MBE In Brief: Evidence Bar Exam Alerts

5. STATEMENTS BY AGENT Statements made by an authorized agent, a partner, or a predecessor in title are admissible. The statement of an employee while still employed concerning matters within the scope of her employment is an admission against her employer, even if the employee was not specifically authorized to speak for the employer. 6. ADMISSIONS BY CO-CONSPIRATOR Admissions made by one co-conspirator are only admissible against another co-conspirator if made during the course of the conspiracy. A conspiracy ends with the arrest of the co-conspirators.

C. PRESENT SENSE IMPRESSIONS AND EXCITED UTTERANCES


A present sense impression must be more contemporaneous with the prompting event than an excited utterance, but does not require an exciting event. A witness may testify to a present sense impression stated by the declarant without having been in a position to observe the facts related by the declarant.

D. STATEMENTS OF MENTAL, EMOTIONAL, OR PHYSICAL CONDITION


The present-mental-state exception can be used to prove actions in accordance with that mental state. Statements of present physical condition are admissible if made to anyone. Statements of past physical condition are admissible only if made to a doctor or the like for purposes of medical diagnosis or treatment.

E. PAST RECOLLECTION RECORDED


The declarant must be on the witness stand for either a past recollection recorded or a nonhearsay statement of prior identification to be admissible.

F. DOCUMENTATION
The business record exception is not applicable to a business record made in preparation for a lawsuit. Where the learned treatise exception applies, the passage in the learned treatise is admitted substantively. However, the treatise itself cannot be admitted as an exhibit. The learned treatise exception is available only after the opposing expert testifies and the proponent establishes the authority of the treatise. The opposing expert need not admit to the authority of the treatise, but some expert must establish its qualifications.

MicroMash MBE In Brief: Evidence Bar Exam Alerts

11

G. STATEMENTS AGAINST INTEREST When the out-of-court declarant is unavailable, a statement made by her which was contrary to the declarant's pecuniary or proprietary interest and which would likely subject her to criminal or tort liability, or which would likely render invalid a claim which she might possess, is admissible as an exception to the hearsay rule, provided that it was against her interest at the time that it was made. If the statement is offered to exonerate a criminal defendant by showing that the outof-court declarant committed the crime, the evidence must be corroborated.

H. OTHER HEARSAY EXCEPTIONS


Statements made in contemplation of impending death, declarations against interest, former testimony, and statements of personal and family history require unavailability. Other hearsay exceptions do not. A witness who is available but refuses to answer questions or claims the privilege against self-incrimination is "unavailable" for purposes of the Fed. R. Evid. 804 hearsay exceptions.
If prior testimony is offered in a case where the parties are not identical to the case in which the witness testified, the prior testimony is admissible only if the opposing attorney in the first trial had an opportunity and the same motive for cross-examination as the party against whom the statement is offered in the second trial. A statement of impending death is admissible only in civil cases and criminalhomicide prosecutions. The death of the declarant is not required (only unavailability). Family records and reputation concerning family history are admissible to prove family relationships. If the declarant is unavailable, his own statement concerning his personal history is admissible. Documents more than 20 years old in proper custody come within the ancientdocuments exception to the hearsay rule.

12

MicroMash MBE In Brief: Evidence Bar Exam Alerts

MicroMashe
MBE IN BRIEF REAL PROPERTY

MicroMash BAR REVIEW MBE IN BRIEF


REAL PROPERTY Table of Contents
I. OWNERSHIP
A. PRESENT ESTATES: THE FEE SIMPLE
1. 2. 3. 4. 1. 2. 3. 4. 1. 2. 3. 4. 5. 1. 2. 3. 4. 5. 1. The Fee Simple Defeasible Fees Simple Life Estates Summary Of Basic Estates Tenancy In Common Joint Tenancy Tenancy By The Entirety Rights And Liabilities Of Cotenants Reversions Remainders Vested And Contingent Executory Interests Possibilities Of Reverter, Power Of Termination Summary Of Future Interests Fitness And Suitability Of The Premises Types Of Holdings, Creation, Termination Assignment And Subletting Rent Surrender, Mitigation Of Damages, Anticipatory Breach The Instrument

1
1
1 1 3 4

B. CONTENACY

5
5 5 7 7

C. FUTURE INTEREST

8
9 9 10 10 11

D. THE LAW OF LANDLORD AND TENANT

12
12 13 14 15 17

E. OWNERSHIP INTERESTS IN TRUSTS

18
18

2. 3. 4. 5. 6.

The Trustee Rights Of Beneficiaries Termination Of A Trust Charitable Trusts Implied Trusts The Rule Against Perpetuities Alienability, Descendability, And Devisability Rules of Construction Gifts "To A And His Children"

18 18 19 19 20

F.

SPECIAL PROBLEMS
1. 2. 3. 4.

21
21 24 25 26

II.

RIGHTS IN LAND A. COVENANTS AT LAW AND IN EQUITY


1. 2. 3. 4. 5. Requirements For A Covenant To Run The Common Scheme Denial Of Relief In Equity Comparison Of Land-Use-Control Devices Table Showing Requirements For Running Of Covenants Easements Profits A Prendre Licenses Fixtures Scope And Extent Of Real Property Takings Zoning

26 26
26 28 28 28 29

B.

EASEMENTS, PROFITS, AND LICENSES


1. 2. 3.

29
30 33 34

C.

OTHER INTERESTS IN LAND 1.


2.

34
34 35

D.

TAKINGS AND ASPECTS OF ZONING


1. 2.

39
39 40

III.

REAL PROPERTY CONTRACTS A. B. RELATIONSHIPS INCLUDED


1. 2. Interests In Land Within The Statute Of Frauds Contracts Associated With Land Not Within The Statute Of Frauds
-

42 42
43 43

CREATION AND CONSTRUCTION: THE STATUTE OF FRAUDS43

C.

THE PURCHASE AND SALE AGREEMENT


1. The Enforceability Of A Purchase-And-Sale Agreement
ii

43
43

2. 1. 2. 3. 4. 1. 2. 3. 1. 2.

Implied Conditions Or Terms Fitness And Suitability Of The Premises Marketable Title Required Risk Of Loss Remedies For Breach Of Purchase-And-Sale Agreement

44

D. PERFORMANCE

45
45 45 46 46

E. INTEREST BEFORE CONVEYANCE

47

The Effect Of An Enforceable Purchase And Sale Agreement The 47 Doctrine Of Equitable Conversion 47 Earnest Money Deposits The Closing Effect Of Closing On Purchase-And-Sale Agreement Title Problems 47

F. RELATIONSHIPS AFTER CONVEYANCE

48
48 48

IV. REAL PROPERTY MORTGAGES A. TYPES OF SECURITY DEVICE


1. 2. 3. 4. 1. 2. 3. 4. 1. 2. 3. 4. 5. Mortgages (Including Deeds Of Trust) Land Contracts As A Security Device Absolute Deeds As Security, Equitable Mortgages Sale Leaseback Arrangements The Underlying Obligation Title And Lien Theories Of Mortgages Rights Between Mortgagor And Mortgagee Prior To Default Right To Redeem And Clogging The Equity Of Redemption

48 48
48 48 49 49

B. SOME SECURITY RELATIONSHIPS

49
49 50 50 50

C. TRANSFERS BY THE MORTGAGOR

50

50 Conveyance Free And Clear Of The Mortgage 50 Conveyance Subject To The Mortgage Conveyance Subject To The Mortgage With The Grantee Assuming The 50 Debt 51 Novation 51 Due-On-Sale Clauses

D. TRANSFERS BY MORTGAGEE E. DISCHARGE AND DEFENSES

51 51

iii

F. FORECLOSURE
1. 2. 3. 4. 5. Types Of Foreclosure Rights Of Omitted Parties Deficiency And Surplus Statutory Right Of Redemption Deed In Lieu Of Foreclosure

52
52 52 53 53 54

V. TITLES A. ADVERSE POSSESSION


1. 2. 3. 1. 2. 3. 4. 5. 1. 2. 3. 1. 2. 3. Time When Statute Begins To Run Requirement Of Continuous Possession Title Obtained By Adverse Possession Requirements Of A Valid Conveyance Necessity Of A Grantee Delivery Land Description And Boundaries Covenants Of Title Ademption Exoneration Lapse Types Of Priority Scope Of Coverage Special Problems

54 54
54 55 55

B. CONVEYANCE BY DEED

56
56 56 56 57 57

C. CONVEYANCING BY WILL

59
59 59 60

D. PRIORITIES AND RECORDING

60
60 61 63

iv

REAL PROPERTY

I. OWNERSHIP
A. PRESENT ESTATES: THE FEE SIMPLE
1. The Fee Simple The fee simple absolute is the most complete property interest known to the law. It is an estate of infinite duration. At common law, in order to convey a fee simple inter vivos, the grantor had to deed the property to the grantee "and his heirs." Under the modern law of most states, that phrase is no longer required; a simple conveyance "to B" will suffice. 2. Defeasible Fees Simple a. Types of qualified estates 1) Determinable estates Any estate that automatically terminates and reverts to the grantor on the happening of a specified contingency is a determinable estate. (The grantor's retained future interest is called a possibility of reverter.) A determinable estate is often created by the use of words such as "so long as," "until," or "during." 2) Estates subject to a condition subsequent If the grantor of an estate has the right to enter the property and take back the estate if a contingency comes to pass, the estate is subject to a condition subsequent. (The grantor's retained interest is called a right of entry for condition broken or a power of termination.) (a) Words of creation Any grant retaining a right of entry on a certain condition creates an estate subject to a condition subsequent. However, a court may find, even without such a clear expression of the grantor's interest, that an estate subject to a condition subsequent is created when the condition is preceded by a phrase such as "but if," "on condition that," or "provided that." (b) Action necessary to enter and re-take estate At common law, an actual physical entry on the land was required. Today, most jurisdictions hold that filing an action to recover the land is sufficient to re-vest title in the grantor.

MicroMash MBE In Brief: Real Property

3) Estates subject to an executory limitation An estate is subject to an executory limitation if, on the happening of a contingency, a third party takes (or may take) the grantee's estate. Thus, estates subject to an executory limitation differ from determinable estates and estates subject to a condition subsequent only in that the future interest lies in another grantee, not the grantor. (The future interest held by the third party is called an executory interest.) Executory limitations are distinguished from rights of entry and possibilities of reverter primarily because the Rule Against Perpetuities applies only to future interests in third parties, and thus does not apply to possibilities of reverter or rights of entry. 4) The fee tail The fee tail was an estate that passed automatically from generation to generation of the grantee's heirs. At the death of the grantee, the property passed automatically to the appropriate heir (usually the grantee's oldest child). When that heir died, the property again passed automatically to the heir's heir, and so on, indefinitely. If any of the successive owners of the estate died without heirs, the property reverted to the grantor. (a) Words of creation At common law, a fee tail was created by a grant to the grantee "and the heirs of his body" or to "A, but if A dies without issue, then to B and his heirs." Such a grant was called a "fee tail general." A "fee tail special" was created if the grant restricted the subsequent takers to the heirs of the grantee by a particular spouse. The subsequent takers could also be limited to the male or female heirs of the grantee by use of the "fee tail male" or "fee tail female." (b) Modern developments The fee tail has been eliminated in most states, usually by either converting it into a fee simple absolute, or by allowing the current holder to convey the estate as a fee simple absolute. b. Issues common to all qualified estates 1) Possessory rights of owners of qualified estates The owner of a qualified fee estate is not liable to the holder of the future interest for waste. 2) Grantee's right of alienation Qualified estates are transferable, but the conditions imposed on the estate pass with the estate, even if not listed in the new deed.

MicroMash MBE In Brief: Real Property

3) Impermissible conditions
The contingency which terminates the estate cannot be within the control of the grantor, because such power would reduce the estate to an estate at will. The contingency cannot be a prohibition against ownership or use of the land by members of a particular race. Many states also void conditions that encourage divorce, discourage marriage, or discourage legitimate will contests. Many states also refuse to enforce a condition that would divest a fee owner of her estate on the basis of her alienation of the estate.

3. Life Estates
A life estate is an estate the duration of which is measured by the lifetime of a person.

a. Words of creation and types of life estates


At common law, a life estate was created by the conveyance "to A." This is no longer the rule today; such a conveyance conveys the grantor's entire estate. A life estate pur autre vie (a life estate "for the life of another") can be created in a grantee (A, in this example) by a conveyance "to A for the life of B." A life estate pur autre vie is also created when the holder of a life estate deeds his estate to another.

b. Rights and obligations of the life tenant


Unlike the holders of fees simple and fees tail, life tenants (and nonfreehold tenants) have only limited rights in the property.

1) Right of possession and related rights of enjoyment


The fundamental right of the life tenant is possession. Incident to that right is the right to all rents and profits during the time she is entitled to possession. She is also entitled to evict the grantor, trespassers, adverse possessors, and holders of future interests, and to sue for damages to her present possessory interest in the property.

2) Right of alienation
Absent a specific prohibition, a life tenant may lease, sell, or mortgage his interest in the property.

3) Rights in fixtures
When personal property affixed by a life tenant can be removed without lasting damage to the real property, the personal representative of the deceased life tenant has a reasonable time after the life tenant's death to remove such property.

4) Rights in division of proceeds


If the property is involuntarily converted by an eminent-domain taking, a mortgage foreclosure, or a court order, the usual procedure is to value the life estate by use of actuarial tables in relation to the remainder interest, and divide the proceeds accordingly.

MicroMash MBE In Brief: Real Property

5) Obligation not to commit waste The right of life (and nonfreehold) tenants to the use and enjoyment of property is limited by the doctrine of waste. The modern law of waste makes a life tenant liable to the holder of the future interest for any reduction in the value of the future estate for which the life tenant is responsible. 6) Obligation to pay taxes The life tenant is obligated to pay the ordinary, annual real estate taxes assessed against the property, to the extent that she receives actual or imputed income from the property. The life tenant is also obligated to pay her share of extraordinary real estate taxes assessed to pay for long-term public betterments. 7) Obligation to pay interest on the mortgage If the property was already mortgaged when the life tenant's estate became possessory, the life tenant is obligated to pay the interest due on the mortgage during his estate, to the extent that the property produces or can produce income. The life tenant is not obligated to pay any principal due under the mortgage. 4. Summary Of Basic Estates Name of Interest FreeHold? Example of Words Necessary to Create Interest "to A and his heirs" (common law) "to A" (modem law) "to A and the heirs of his body" "to A" (common law) "to A for life" (modern law) "to A years" for 10 Subsequent Future Interest in Grantor None Subsequent Future Interest Held by Third Party None

Fee Simple

Yes

Fee Tail

Yes

Reversion

Remainder

Life Estate

Yes

Reversion

Remainder

Term Years Periodic Tenancy

for

No

Reversion

Springing Executory Interest Springing Executory Interest

No

"to A from year to year"

Reversion

MicroMash MBE In Brief: Real Property

Tenancy at Will Determinab le Estate Estate Subject to a Condition Subsequent

No

(Implied consensual possession)

by

Reversion

Springing Executory Interest Shifting Executory Interest Shifting Executory Interest

Yes

"to A and his heirs so long as . . . " "to A and his heirs, but if . . . then . . . "

Possibility of Reverter Right of Entry for Condition Broken

Yes

B. CONTENACY
Concurrent estates exist when two or more persons have possessory rights (i.e., estates) in a single parcel of land at the same time. The joint tenancy and tenancy by the entirety are specialized forms of concurrent ownership, with special requirements for their creation. All other concurrent estates are tenancies in common. 1. Tenancy In Common A tenancy in common exists when two or more owners have the right to possess property at the same time, and their tenancy does not meet the criteria for a joint tenancy or tenancy by the entirety. There are no rights of survivorship. If one tenant dies, her interest passes to her heirs or devisees. When the respective shares of tenants in common are not specified, they are presumed to hold equal shares. 2. Joint Tenancy a. The right of survivorship The right of survivorship is an incident to a joint tenancy. A right of survivorship signifies that, at the death of a joint tenant, his interest passes to the surviving joint tenants. It cannot be devised by will. b. Requirements necessary to create a joint tenancy 1) Common law At common law, any conveyance that included "the four unities" was presumed to create a joint tenancy.

MicroMash MBE In Brief: Real Property

(a) Unity of time The interests of all the cotenants had to vest at the same time. This requirement meant that a grantor could not create a joint tenancy between herself and other cotenants at common law, since her interest would always predate the interests of her grantees. (b) Unity of title The interests of all the cotenants had to be acquired from the same instrument. (c) Unity of interest The cotenants had to have equal interests. (d) Unity of possession The cotenants had to have equal and concurrent rights to possess and enjoy the property. 2) Modern law (a) Intent to create must be explicit Generally, under modern law, it must be clear that the grantor intended to create a joint tenancy. Some states require that the grant specifically include a right of survivorship. (b) Unity of interest Most jurisdictions still require that joint tenants have equal interests. c. Destruction 1) Conveyance A joint tenant's conveyance of an interest in the property will destroy the joint tenancy between his interest and the interests of his fellow cotenants. Many jurisdictions also hold that the signing of a contract to sell the joint tenant's interest will destroy the joint tenancy, at least for equitable purposes. A mortgage by a joint tenant will destroy the joint tenancy in a title-theory state (where title is passed to the mortgagee as security). However, a joint tenancy is not destroyed by an attempt to devise the joint tenant's interest. Such a devise is of no effect at all. 2) Partition action A judgment in a partition action will destroy a joint tenancy. 3) Effect of destruction The destruction of a joint tenancy terminates the right of survivorship only of the share conveyed or partitioned. The joint tenancy (and right of survivorship) persists between the other joint tenants.

MicroMash MBE In Brief: Real Property

3. Tenancy By The Entirety Tenancy by the entirety is a form of concurrent ownership that can be held only by a married couple. There is a right of survivorship, but the owners cannot convey or unilaterally partition their individual interests (unlike joint tenants). a. Creation A married couple is presumed to take as tenants by the entirety. If a conveyance to unmarried persons purports to create a tenancy by the entirety, many courts will find that a joint tenancy was created, since the grantor probably intended that the cotenants should have a right of survivorship. b. Destruction A tenancy by the entirety can only be terminated by a divorce, an annulment, or an agreement to partition. Mere separation will not suffice. Once the couple is divorced, the property is held as tenants in common. A tenant by the entirety cannot unilaterally have her interest partitioned, even by a court. The property can be partitioned (and the tenancy by the entirety destroyed) only by agreement of the parties. A tenancy by the entirety cannot be terminated by a conveyance or a devise. 4. Rights And Liabilities Of Cotenants a. Right to possession A cotenant is entitled to possession of the entire property, subject to the same right as his cotenants. Thus, if one of the cotenants actively excludes the others from possession or enjoyment of the whole or any part of the property, his conduct amounts to an ouster, and an action of ejectment will lie. However, permissive possession and use of the entire property by one cotenant is not an ouster. b. Right to convey Both the tenant in common and the joint tenant (but not the tenant by the entirety) may convey their undivided interest in the property. The grantee succeeds to her grantor's concurrent ownership in the property as a tenant in common, even if the grantor held the property as a joint tenant. If one cotenant purports to convey full and exclusive ownership of the property, the conveyance will be effective only to convey the grantee's concurrent interest. Moreover, the conveyance may be treated as an ouster of her cotenants. When one tenant seeks to convey her interest in the property by separating and conveying a particular segment of the property, the conveyance will not be given effect insofar as it interferes with the other cotenant's right of partition. c. Partition A partition terminates concurrent ownership by assigning a physical share of commonly held land corresponding to the interest of a cotenant to the cotenant in exclusive ownership. Property held in tenancy by the entirety can only be voluntarily

MicroMash MBE In Brief: Real Property

partitioned. Property held in joint tenancy or tenancy in common can be partitioned by either of the following methods. 1) Voluntary partition Cotenants may validly agree on a division of the land. This agreement must be in writing to be effective. 2) Involuntary partition Partition of property held in joint tenancy or tenancy in common may also be accomplished by court action. 3) Restraints on right of partition Reasonable restraints on a cotenant's right of partition are generally upheld, since the cotenant always has the right to sell his undivided interest in the property. 4) Obligation to account for rent Since a cotenant has a right to occupy the entire premises, she is not generally liable to account to her cotenants for the fair rental value of her own personal use of the land. 5) Obligation to pay for improvements and repairs In cases of necessary repairs, the majority rule is that contribution from the other tenants will be required. However, in the case of improvements, unnecessary repairs, or repairs which only benefited the paying cotenant, contribution will usually be denied. 6) Obligation to pay taxes and carrying charges Where one tenant pays more than his share of carrying charges, such as interest on a mortgage, mortgage principal, or taxes, he is generally entitled to contribution from the other cotenants. 7) Waste Most jurisdictions have enacted statutes permitting one tenant to sue another for waste. A cotenant commits waste when she exceeds the reasonable use and enjoyment of the land. C. FUTURE INTEREST A future interest is reversionary if it is retained by the grantor at the time he conveys the prior estates. Reversions, possibilities of reverter, and rights of entry are reversionary future interests. A future interest is nonreversionary if it is granted to someone other than the grantor at the time the grantor conveys the prior estates. Remainders and executory interests are nonreversionary future interests. In general, a future interest is vested if the holder has a present (i.e., certain) right to later possess the property. All reversionary future interests are also said to be vested. A future interest is contingent if the grantee's right to later possess the property is uncertain.

MicroMash MBE In Brief: Real Property

That is, a future interest is contingent until the grantee meets all necessary preconditions

to having a right of possession. 1. Reversions

Anytime a grantor conveys a qualified estate or an estate of lesser duration than she owns, without expressly naming a subsequent taker, the grantor automatically retains a future interest. Future interests held by the grantor are never subject to the Rule Against Perpetuities. The grantor retains a reversion when she grants an absolute interest that is less than her own interest.

2. Remainders Vested And Contingent


Any interest in a third party that may become possessory immediately after a prior life estate, fee tail, or freehold estate is a remainder.

a. Vested remainders An individual's remainder is vested if he has a present or certain right to take possession of the property when the prior estates terminate. If the individual is
not yet alive or has not yet met a condition precedent to his taking an interest, then his interest is not yet vested, but is contingent.

1) Legal incidents of a vested remainder


The designation of a remainder as "vested" rather than "contingent" has the following ramifications: A vested remainder does not lapse if the holder dies prior to the time of possession. Vested remainders are not subject to the Rule Against Perpetuities. Contingent remainders were "destructible" at common law, as discussed infra.

2) Classification of vested remainders The Absolutely Vested Remainder. The interest of the person holding
an absolutely vested remainder is subject neither to dilution nor to divestment.

The Vested Remainder Subject to Open. A vested remainder is "subject


to open" (or "subject to partial divestment") if the remainderman's interest is vested but her share of the property is not certain because other persons may be capable of sharing in the grant. The vested remainder subject to open most commonly occurs in class gift situations.

The Vested Remainder Subject to Complete Divestment. A vested remainder subject to complete divestment is not subject to a condition precedent, but a condition subsequent will completely divest the remainder interest.

10

MicroMash MBE In Brief: Real Property

b. Contingent remainders
An individual holds a contingent remainder whenever he may someday have a right to take a remainder interest, but has not yet met a condition precedent to taking such an interest.

1) Words of creation
The condition precedent to vesting can be explicit or implied.

2) Destructibility of contingent remainders


At common law, a contingent remainder was destroyed if it had not vested by the time the preceding estate terminated. The destructibility of contingent

remainders has been abolished today.

3. Executory Interests
Any nonreversionary future interest following a qualified fee or any interest held by the grantor is an executory interest. Executory interests are classified into "shifting" and "springing" interests, depending on the holder of the prior estate.

a. Shifting executory interests


A nonreversionary future interest following a qualified estate held by another grantee is a shifting executory interest. A conveyance "to A, so long as the property is used for church purposes, and if not so used, to B" creates a shifting executory interest in B.

b. Springing executory interests


A grantee holds a springing executory interest when his estate does not begin until a future time and the immediately preceding estate is held by the grantor.

4. Possibilities Of Reverter, Power Of Termination a. Possibilities of reverter


The grantor retains a possibility of reverter when she conveys a determinable estate. A possibility of reverter signifies that title and right to possession will automatically re-vest in the grantor on the happening of the specified condition.

b. Right of entry
The right of entry for condition broken (or power of termination) is created when the grantor conveys an estate subject to a condition subsequent. The right of entry signifies that the grantor has the right to take back the granted estate from the grantee, at his discretion, when and if the stated condition comes to pass. The right of entry for condition broken is considered a personal right and thus not alienable, unlike reversions and possibilities of reverter.

MicroMash MBE In Brief: Real Property

11

5. Summary Of Future Interests Name of Interest Example of Words Necessary to Create Interest Applicability of Rule Against Perpetuities Interest Usually Preceding It Life Estate, Fee Tail, Years, for Term Tenancy, Periodic Contingent Remainder, Executory Interest Fee Determinable Simple

(1) Reversion

(Created by law when No grantor transfers less than she has)

(2) Possibility of Reverter (3) Right of Entry for Condition Broken (4) Absolutely Vested Remainder (5) Vested Remainder Subject to Open (6) Vested Remainder Subject to Complete Divestment (7) Contingent Remainder (8) Executory Interest (Shifting Use)

law No by (Created Simple Fee after Determinable) law No by (Created after a Fee Simple subject to a Condition Subsequent) "(to X for life) No and then to A and her heirs"

Fee Simple subject to a Condition Subsequent

Life Estate, Fee Tail, or Estate for Years

"(to X for life) and Yes (until Class Life Estate, Fee Tail, or Estate for Years then to the children of Closes) A and her heirs" "(to X for life) then No to A and her heirs, but if A does not survive X, then to B and her heirs" "(to X for life), and Yes then to A and her heirs if A survives X" "(to X and her heirs), but if the property is not used for church purposes then to A and her heirs" Yes Life Estate, Fee Tail, or Estate for Years

Life Estate, Fee Tail, or Estate for Years Fee Simple subject to an Interest, Executory Remainder Vested subject to Complete Divestment or Reversion Fee Simple subject to an Executory Interest

(9) Executory Interest (Springing Use)

"(to X for life), and Yes one year after X's death, to A and her heirs"

12

MicroMash MBE In Brief: Real Property

D. THE LAW OF LANDLORD AND TENANT


These estates are typified by the grantee's finite possessory right, which is not measured by a human life. The grantor (landlord) also has an implied right to collect rent from the grantee/tenant during the tenant's possession.

1. Fitness And Suitability Of The Premises a. Landlord's obligations 1) Common law rule
In general, at common law, the landlord made no implied warranties that the premises were or would remain in any particular state of repair. The following situations provided the only exceptions to this general rule. In these cases, the landlord was (and is) liable for any negligence in maintaining the premises. In order to qualify as negligence, the defects should be ones of which the landlord actually was or should have been aware.

(a) Latent defects


The landlord had a duty to disclose (not repair) defects which the tenant would not reasonably be able to discover.

(b) Short-term lease of a furnished dwelling


The landlord would be liable for any damages resulting from defects of which she was or should have been aware.

(c) Short-term public-use lease


If the landlord knew the tenant was going to open the demised premises to the public and the lease was only of a short duration, the landlord was liable for any defects on the premises.

(d) Common areas


The landlord was always liable for defects in the common areas of multi-unit dwellings.

(e) Undertaking to repair


If the landlord undertook to repair a defect on the premises, he was liable for any negligence in the repair, even if he was under no duty to repair the defect in the first place. Thus, if the landlord supposedly repaired a defect, but the defect still injured someone, the landlord would be liable.

2) Leases imposing duty on landlord


In the case of minor defects, the damages would be the cost of repair. In the case of major defects, the damages would be the difference between the fair rental value of the premises in good repair and in the state of repair in which they were actually leased. However, at common law, the landlord was not liable in tort for

MicroMash MBE In Brief: Real Property

13

any injuries sustained as a result of a breach of a covenant to repair. The modern trend is to impose such tort liability. 3) Modern rules of residential leases (a) Implied covenant of habitability The modern law of most states imposes an implied covenant of habitability on any residential lease. These covenants generally not only warrant that the premises are in compliance with local building health and safety codes at the commencement of the lease, but also covenant that the landlord will keep the premises in compliance during the lease term. This covenant cannot be waived by the tenant. The landlord must repair any defects that rise to the level of a violation of the local building codes, unless the defects were caused by the negligent or intentional acts of the tenant. (b) General duty of care Many jurisdictions now impose a general duty of care on landlords in regard to residential tenants. In these states, a landlord is liable for her negligence in failing to discover or repair defects. The defects need not rise to the level of a violation of a building code to create liability for damages or personal injuries. b. Tenant's common-law duty to avoid waste The common-law rule is that the tenant is only under a duty to avoid waste. A tenant is required to repair negligent or intentional damage done to the premises, but he is not obligated to repair the ordinary wear and tear to the property (unless such repair is necessary to avoid more substantial waste). The tenant is also liable for any "ameliorating" waste which changes the property (even if it increases the property's value), unless he holds a very long-term lease. If the tenant covenants to repair, the premises must be returned to the landlord in the condition in which the tenant received them. This duty is absolute and arises regardless of the cause of the damage. The tenant would also be liable to repair everyday wear and tear to the property, unless the lease provided otherwise. 2. Types Of Holdings, Creation, Termination a. The estate for years Any estate with a fixed and certain period of duration is an estate for years. An estate for years terminates at the end of the stated period. No notice is required. Most American jurisdictions now require that any lease for more than one year must be in writing.

14

MicroMash MBE In Brief: Real Property

b. Tenancy at will A tenancy at will is a tenancy which is terminable at the will of either the landlord or the tenant, and has no specified period of duration. c. Tenancy at sufferance A tenancy at sufferance occurs when the tenant wrongfully remains in possession after her tenancy has been validly terminated. No notice is required to terminate a tenancy at sufferance. d. Periodic estates An estate for successive periods of time with no fixed termination date is a periodic estate. 1) Periodic tenancy by implication A periodic tenancy can be implied anytime a tenant pays and the landlord accepts rent for an identifiable period of time. 2) Termination of periodic tenancies Periodic tenancies can be terminated by either party, but the terminating party usually must give notice of termination equal to the length of the tenancy period. 3. Assignment And Subletting Unless there is a lease provision to the contrary, the interests of landlords, tenants for years, and periodic tenants are freely transferable. The transfer of a tenancy at will or at sufferance is valid between the parties, but is not enforceable against the landlord. If the tenant sells her leasehold, the rights of the parties will depend on whether the transfer is an assignment or a sublease. If the tenant conveys the remainder of her entire (temporal and physical) estate, there has been an assignment of her interest. If the tenant conveys less than her entire estate, there has been a sublease of her interest. An assignee of the tenant takes the tenant's estate and thus is said to be in privity of estate with the landlord. A subtenant is not in privity of estate with the landlord. In effect, the sublease creates a second tenancy, between the tenant and her subtenant. a. Running of lease covenants, generally Since the landlord and tenant are in horizontal privity when they make any lease covenants, those covenants may run with the land at law and be enforceable by and against successors. Also, the fact that the landlord owns a reversionary estate in the leased premises will satisfy the requirement that the party seeking to enforce an equitable servitude must hold an estate that will benefit from the covenant.

MicroMash MBE In Brief: Real Property

15

b. The rent obligation, specifically 1) Obligation of the original tenant The tenant cannot discharge his rent obligation by either a sublease or assignment. His contractual obligation to pay rent continues even if he is no longer in possession of the property. However, if the tenant assigns his interest, he is only secondarily liable (as a surety); an assignee assumes the primary obligation to pay rent. 2) Obligation of the tenant's lessee (a) Obligation of an assignee When a lessee accepts an assignment of the tenant's interest in leased land, the tenant's covenant to pay rent runs with the land and is enforceable against the assignee. Since the assignee is liable only by virtue of her possession of the estate and is not held to have assumed the contractual obligation per se, however, she is liable on the rent obligation only for the period of her estate (i.e., her possession). (b) Obligation of a subtenant The rent covenant is not directly enforceable against a subtenant. (c) Assumption of rent obligation by lessee If the lessee expressly assumes the tenant's rent obligation, the landlord can sue the lessee directly as a third-party beneficiary of the contract between the lessee and the tenant. c. Covenants against assignment and subletting Leases often contain a clause prohibiting assignment and/or subletting by the tenant. Since such a clause restricts the tenant's right of alienation, courts construe these provisions narrowly and are quick to find a waiver of the landlord's contractual right to prohibit a sublease or assignment. Under the rule in Dumpor's case, consent to one sublease or assignment extinguishes the landlord's right to object to any future assignments or subleases (unless the right is specifically saved by agreement). 4. Rent The tenant's obligation to pay rent is implied in the landlord-tenant relationship. If the amount is not specified in the lease or by agreement, a reasonable rent will be implied. a. Defenses to the rent obligation 1) Eminent domain If the entire premises have been taken by eminent domain, the lease and the rent obligation are terminated. If less than all of the property is taken, modern law provides for a proportional abatement in the rent for any lost value the tenant cannot recover from the taking authority.

16

MicroMash MBE In Brief: Real Property

2) Destruction of the leasehold


At common law, if the premises consisted solely of space in a building and the building was destroyed, the lease and the rent obligation were terminated. However, if the premises consisted of land as well as buildings, the common law held that the lease and the rent obligation would be fully enforceable. Today, statutes in many jurisdictions or provisions in the lease itself often provide for at least an abatement in rent if the buildings or part of the premises are destroyed.

3) Frustration of purpose/illegality
Common-law courts allowed the tenant to avoid the lease under the doctrine of frustration of purpose only if the only use of the premises allowed by the lease was or later became illegal (e.g., by new zoning regulations). Some states have extended the applicability of the doctrine beyond supervening illegality, to encompass acts of God or third parties that make the property unusable for its intended use.

4) Surrender
A tenant cannot unilaterally abandon the property and avoid his contractual rent obligation. The rent obligation is terminated only if the landlord agrees to "surrender" the lease. The landlord's surrender can be an express agreement to terminate the lease (which must be in writing if more than one year remains on the lease), or it can be found in the landlord's actions. If the landlord takes possession of the premises for his own use, or re-rents the premises on his own account, such action will always be held to be a surrender. If the landlord re-rents the premises on behalf of the tenant, the lease is not discharged.

5) Eviction
An actual, physical eviction of the tenant by the landlord or anyone with superior title violates the covenant of quiet enjoyment and discharges the tenant's rent obligation.

6) Constructive eviction
Constructive eviction occurs when the landlord breaches the covenant of quiet enjoyment by significantly interfering with the use and enjoyment of the demised premises (e.g., by failing to provide heat). The tenant must vacate the premises within a reasonable time after the interference has commenced, in order to take advantage of this defense to the lease.

7) Breach of a covenant of habitability


The modern law of most states imposes an implied covenant of habitability into any lease of residential premises. If the premises fail to meet the standards imposed by this covenant (usually based on local safety and health ordinances) or if the landlord fails to meet the obligations of an actual covenant to repair, the tenant has several options, often including a right to abate rent. Unlike the

MicroMash MBE In Brief: Real Property

17

defense of constructive eviction, the tenant need not quit the premises to utilize the defense of breach of a covenant of habitability. b. Landlord's remedies for failure to pay rent 1) Debt action The landlord can sue the tenant for the rent due, as it becomes due. 2) Eviction The modern law of most states gives landlords the right to evict for failure to pay rent, either by the terms of the lease or by the terms of a statute. The modern trend is to require that the landlord regain possession only through judicial process. 5. Surrender, Mitigation Of Damages, Anticipatory Breach

a. Surrender
A tenant cannot unilaterally abandon the property and avoid her contractual rent obligation. The rent obligation is terminated only if the landlord agrees to "surrender" the lease. If the landlord takes possession of the premises for her own use, such action will always be held to be a surrender. However, the landlord can enter the premises for more limited purposes (e.g., inspection) without automatically surrendering the lease. A re-renting of the premises by the landlord may or may not qualify as a surrender, depending on the circumstances. Most courts presume that the re-renting of the premises operates as a surrender, unless the landlord makes it very clear that she is doing so only on behalf of the tenant. b. Mitigation of damages In most jurisdictions, the landlord will be required to mitigate his damages and will only be able to collect the rent he could not obtain from another source. The common-law rule was that a landlord was not required to mitigate his damages by reletting the premises, unless he evicted the tenant. c. Anticipatory breach The landlord can sue the tenant for the rent due, as it becomes due. The landlord generally cannot sue for future rent due under the contract, because the doctrine of anticipatory breach does not apply to leases. Even if the lease contains an acceleration clause, purportedly making the tenant immediately liable for all the rent due for the life of the contract on any breach of the tenant's rent obligations, the landlord probably will not immediately be able to sue for that amount. Many courts view such clauses as a penalty rather than a reasonable attempt to establish liquidated damages, and refuse to enforce them.

18

MicroMash MBE In Brief: Real Property

E. OWNERSHIP INTERESTS IN TRUSTS Through the use of a trust it is possible to vest the legal title in land in one individual (the trustee) and the equitable or beneficial interest in that land in others (the beneficiaries of the trust). 1. The Instrument In most jurisdictions, the Statute of Frauds applies, requiring equitable interests in land to be created by a written instrument.

2. The Trustee a. Power of sale


Unless the trust instrument gives the trustee the power to sell the trust property, she must obtain court permission to sell.

b. Power of contract
The trustee has the inherent power to enter into contracts to manage the trust property, and is not liable on such contracts personally, as long as the third party knew that the trustee was acting in a fiduciary capacity. c. Right of compensation A trustee is entitled to reimbursement for his reasonable expenses and liabilities incurred in the execution of the trust, and reasonable compensation for his services. d. Duty of loyalty and good faith The trustee is a fiduciary. As such, she owes a duty of loyalty and utmost good faith in all matters pertaining to the trust. She may not put herself in a position where her interests would be contrary to the interests of the trust. Hence, she usually may not enter into any transactions with the trust. If a trustee uses trust funds for her own benefit or in any other improper fashion, she must account to the trust for any profits made or losses incurred as a result of such improper conduct. If the improper conduct leads to both gains and losses, she cannot offset the gains against the losses.

e. Duty of reasonable care


The trustee is required to exercise reasonable care in managing the trust. However, he is not expected to be infallible. Therefore, he is not strictly liable if his decision in a certain matter turns out to be erroneous, provided that he exercised reasonable care. 3. Rights Of Beneficiaries The beneficiary of a trust has only the rights given to her in the trust instrument. If the trust permits the trustee to pay income on a discretionary basis, the beneficiary has no right to the income.

MicroMash MBE In Brief: Real Property

19

The beneficiaries have the right to sue the trustee (and, in some cases, cause the trustee to be removed and another trustee appointed) if the trustee has breached her fiduciary duties.

4. Termination Of A Trust a. By the terms of the trust


A trust will usually establish its own date of termination or a condition that will terminate its existence. A trust can also be terminated prior to its express termination date by the methods described below.

b. By the settlor
A valid trust cannot be unilaterally terminated by the settlor prior to the time set out in the trust instrument, unless the power to revoke was expressly retained.

c. By the beneficiaries
Where the settlor has fixed the period for the termination of a trust, the trust cannot be terminated by agreement of all of the parties in interest. A probate court does, however, possess the power to terminate a trust.

5. Charitable Trusts
Charitable trusts are favored by the law because of their benefit to society. They are liberally construed and are exempt from some of the restrictions that apply to private express trusts.

a. Creation similar to private trusts


A charitable trust may be created by any of the methods for creating a private express trust. The requirements of a settlor with capacity to convey, properly expressed intent, and a specific trust res are the same.

b. Differences from private trusts 1) Indefinite beneficiaries


In order to qualify as a valid charitable trust, the persons to be benefited must be uncertain that is, members of an indefinite class. The reason for this rule is that a charitable trust is basically concerned with a public benefit. As long as the class is indefinite, it does not affect the validity of the trust that only a small number of persons will actually benefit from the trust or that the amount each will receive is small.

2) Rules against perpetuities and accumulations


The Rule Against Perpetuities does not apply to charitable trusts; such trusts may be perpetual. The Rule Against Accumulations also does not restrict charitable trusts, unless the accumulation is found to be unreasonable.

20

MicroMash MBE In Brief: Real Property

3) Trust purpose (a) Must be charitable The essence of a charitable trust is that it is established to accomplish one or more charitable purposes. The purpose must be one recognized in the law as charitable, including the furtherance of health, religion, education, governmental establishments (such as parks or museums), and the relief of poverty or discrimination.

(b) Cy pres doctrine


Since a charitable trust may be perpetual, it is likely that at some point the charitable purpose intended by the settlor will have been accomplished, can no longer be accomplished, or some other change in circumstances renders it impracticable to administer the trust in the precise manner provided by the settlor. In such cases, the court under the cy pres doctrine may apply the gift as nearly as possible to the settlor's particular charitable intent, unless the instrument creating the trust provides to the contrary. 4) Power to enforce The duty of taking action to protect public charitable trusts and to enforce proper application of their funds rests solely upon public officials. Neither the settlor nor any member of the community who might benefit from the trust may bring suit to enforce it. 6. Implied Trusts Implied trusts are an equitable remedy used by courts to avoid the unjust enrichment of a titleholder. Thus, they differ from express trusts in that they arise by operation of law, rather than by any expressed intent of a settlor. They also differ from express trusts in that the trustee's only power and duty is to convey legal title to the property to the "beneficiary." a. Resulting trusts Resulting trusts are found in situations where the court determines that the intent of the parties was that the settlor-beneficiary should have title to the property. 1) Title to property taken in the name of a person other than the one paying the consideration If one party pays (or is responsible for) the full purchase price, but title is taken in the name of another party, a court usually will decree that the titleholder holds the property in trust for the person paying the purchase price. However, if the person paying the purchase price is legally obligated to support the person taking title (e.g., the titleholder is the purchaser's minor child or spouse), no resulting trust will be found.

MicroMash MBE In Brief: Real Property

21

2) Purchase by a fiduciary If a fiduciary uses fiduciary funds to purchase property, but takes title in his own name, he holds title to such property in trust. 3) On termination of express trust Where an express trust is properly created but fails for some reason, the trustee holds any remaining trust funds on a resulting trust for the settlor, her heirs, devisees, or legatees. b. Constructive trusts A constructive trust may be imposed where property has been acquired as the result of fraud or a violation of a fiduciary duty or confidential relationship. F. SPECIAL PROBLEMS 1. The Rule Against Perpetuities The Rule Against Perpetuities states that all nonreversionary future interests must be certain either to fail or vest within 21 years after some life in being at the creation of the interest, or they will be invalid. a. Interests to which the rule applies The rule applies only to contingent remainders, executory interests, and options or rights of first refusal. b. The period of the rule 1) The starting point of the rule period The rule period begins with "the creation of the interest." Generally, an interest is created when the property is no longer freely alienable by any party. If the property is transferred inter vivos, the period begins at the time of the transfer of title to the grantee. If the property is transferred by will, the period does not begin until the death of the testator. If the property is placed in an irrevocable trust, the period begins at the time of the creation of the trust. If the trust is revocable, the period does not begin until the power of revocation is terminated (e.g., at the settlor's death). 2) The lives in being Once the starting point of the rule is determined, we must determine whether there are any lives in being at that time that can serve to extend the period of time allowed by the rule. The lives in being may either be ones artificially or naturally connected with the conveyance. If the lives are only artificially connected with the gift, they must be expressly set forth in the grant. Lives naturally connected with the gift can be implied from the grant. Measuring lives can only be usefully implied from a grant if every member of

22

MicroMash MBE In Brief: Real Property

that class must, by definition, be alive at the time of the creation of the interest. A child conceived but not born at the time of the creation of an interest is a life in being. If no measuring lives are identifiable, the period allowed by the rule for vesting is only 21 years from the creation of the interest. c. Nonreversionary future interests must vest or fail 1) When interests vest A contingent remainder vests when it becomes either a vested remainder or a present possessory interest. An executory interest vests only when it becomes a present possessory interest. 2) Certainty These interests must be certain to vest (or fail) within the prescribed period, from the moment of their creation. If, at the time of the creation of the interest, there is any possibility that the interest will not vest within the rule period, the interest is void. (a) The fertile octogenarian rule For purposes of the Rule Against Perpetuities, every person is presumed to be capable of having children as long as they are alive, even though that may be physically impossible. (b) The unborn widow rule If an instrument gives an interest to a living person's "widow," there is no guarantee that that living person will be married at his death to a person who was alive at the time of the creation of the interest. Thus, the widow might not be a life in being at the creation of the interest. If a subsequent interest does not vest until after the widow's interest, the subsequent interest will vest too remotely and is void. (c) The administrative contingency Anytime that a nonreversionary future interest does not vest until the end of some procedure or event which is not necessarily limited in time to less than 21 years, the interests are probably invalid. 3) Interests must vest or fail The rule does not require that interests actually vest within the period of the rule, only that they be limited such that they either vest or fail within the period. Any nonreversionary future interest that is not explicitly or implicitly limited to vesting within a limited period of time will violate the rule and thus be void.

MicroMash MBE In Brief: Real Property

23

d. The rule's effect on interests and conveyances If the future interest violates the rule, the interest is void and is stricken from the granting instrument. The striking of a future interest from an instrument may also alter the estate of the prior holder. If the condition that terminates the prior estate is part of the description of the prior estate (e.g., a life estate or a fee simple determinable), the prior estate will not be altered. If the condition which terminates the prior estate is part of the description of the future estate only (e.g., a fee simple subject to a condition subsequent), then the prior estate will be converted from a qualified estate into an absolute estate. e. Special applications of the rule 1) Class gifts A class gift exists anytime a gift is to be shared equally among a group of unnamed individuals who all bear the same relationship to the grantor (e.g., "my grandchildren"). The Rule Against Perpetuities has a harsh effect on class gifts. If the rule could be violated by any potential member of the class, then the entire class gift is void. If the class closes, however, at such a time that it is certain that the interests of all the members of the class (as constituted) will vest within the rule, then the class gift is saved (for those members). As a general rule, courts employ the "rule of convenience" a class closes at the time that any member of the class has a right to distribution of her share. If the gift is per capita, the class closes immediately on the testator's death. If the gift is immediate and there are members of the class alive at the testator's death, then the class closes immediately. If the class gift is an immediate lump sum and there are no members of the class alive at the time of distribution, then the class stays open until all potential members of the class are determined. If the class gift is, by its own terms, postponed (e.g., it is to become possessory only after a prior estate ends or some precondition is met), then the class does not close until a class member has a right to receive her share. 2) The charity-to-charity exception The Rule Against Perpetuities does not apply when property is vested in one charity, with a gift over to a different charity on the happening of a condition. f. Modifications of the common-law rule 1) The "wait and see" doctrine Some courts and statutes have adopted the view that unless a limitation does in fact fail to vest within a life in being plus 21 years, it should be held valid.

24

MicroMash MBE In Brief: Real Property

2) The cy pres rule

Some courts and statutes in recent years have used the cy pres doctrine to reform private trusts in such a manner as to effectuate the wishes of the settlor or grantor "as nearly as possible" within the confines of the rule's time limitation.
3) The Uniform Statutory Rule Against Perpetuities

The Uniform Statutory Rule Against Perpetuities, embodies a relatively liberal, modern rule. Under the statute, a nonvested property interest is valid if it is certain to vest within the common-law rule period or actually does vest within 90 years after its creation. The statute embodies a wait-and-see rule allowing the nonvested property interest a grace period of 90 years to vest. Moreover, the statute has a reformation provision allowing a court on the petition of an "interested person" to modify a disposition which is invalid under the rule stated above so that it follows the grantor's intent as nearly as possible, but does in fact vest within 90 years.
The statutory rule (5) also provides that the following rights or interests must be exercised within 30 years of their creation: an option in gross or a preemptive right of first refusal in gross regarding an interest in land or minerals; a lease commencing at a certain time or upon the happening of a future event; and a nonvested easement in gross.

Additionally, a fee simple determinable or a fee simple subject to right of entry for condition broken shall become an absolute fee unless the specified contingency occurs within 30 years. If the contingency does occur within the 30year period, the succeeding interest becomes possessory, or the right of entry exercisable, irrespective of whether the bequest or devise would have violated the Rule Against Perpetuities. These provisions apply whether the succeeding interest is a reversionary type of interest (possibility of reverter or right of entry for condition broken) or an executory interest (shifting use). 2. Alienability, Descendability, And Devisability a. Right of alienation All present estates (except tenancies at will or sufferance) are alienable. All future interests (except rights of entry) are alienable. 1) Restraints on alienation A restraint on the right of alienation is called a disabling restraint if it purports to prohibit alienation of the estate and to render void any conveyance by the grantee. Disabling restraints are void (except, perhaps, when applied to nonfreehold estates).

MicroMash MBE In Brief: Real Property

25

A restraint is called a forfeiture restraint if the grantee loses his estate if he attempts to convey it. A forfeiture restraint would take the form of a qualified estate. Forfeiture restraints are generally effective against all estates, except fees simple. A restraint is called a promissory restraint if the grantee covenants or contracts not to convey the estate. Promissory restraints are generally effective against all estates, except fees simple.

2) Preemptive rights
A preemptive right is a right retained by a grantor to buy back the granted property when the grantee chooses to sell it. Such a right is enforceable as long as it does not function as a restraint on the grantee's right of alienation. A preemptive right restrains a grantee's right of alienation if the grantor has a right to buy the property at a price below market value.

b. Descendability
Descendability is the right of the property owner's heirs to inherit whatever interest she held in the property by intestacy upon the death of the owner. Except for property rights that are limited by the life of the possessor, such as a life estate, all property rights today are descendable and will pass to the heirs of the previous owner if she dies intestate.

c. Devisability
Devisability is the right of the possessor to dispose of his interest in land by will. Except for property rights that are limited by the life of the possessor, such as a life estate, all property rights today are devisable.

3. Rules of Construction a. Remainders in "heirs" or "next of kin": The Rule in Shelley's Case and the Doctrine of Worthier Title
At common law, the Rule in Shelley's Case operated as a rule of law to transform a remainder in the "heirs" or "next of kin" of a life tenant into a remainder in the hands of the life tenant herself. The Doctrine of Worthier Title accomplished much the same function as the Rule in Shelley's Case when the grantor conveyed a limited estate inter vivos with a remainder in her own "heirs." The remainder in the grantor's heirs was treated as a reversion in the grantor. Under modern law, the vast majority of states have rejected the Rule in Shelley's Case and the Doctrine of Worthier Title as rules of law. However, they remain as rules of construction in many states.

26

MicroMash MBE In Brief: Real Property

b. Conditions of survival 1) General rule - survival not required


As a general principle, the holder of a future interest need not survive the holder of a prior possessory estate in order to take the interest.

2) Exceptions - when survival is required


When the condition is expressed When survival to a given age is required When the grant is to "heirs" the individual's family members must survive that person in order to qualify as heirs

4. Gifts "To A And His Children"


A gift "to A and his children" is ambiguous, in that it could create a concurrent estate between A and the children in existence at the time of the conveyance, or it could create a present interest in the parent and a future interest in all of the parent's children. As a rule of construction, if A did not have children at the time of the gift, the conveyance would create a life estate in A and a remainder in all of his children, whenever born. If A had children living at the time of the conveyance, then, as a rule of construction, the conveyance would create a tenancy in common in A and any living children.

II. RIGHTS IN LAND A. COVENANTS AT LAW AND IN EQUITY


A covenant is an enforceable contractual obligation between two parties. A covenant becomes a covenant running with the land if it is enforceable not only between the actual parties to the covenant, but also by or against persons who were not parties to the original agreement, but now hold the land to which the agreement pertains. A covenant can run with the land either at law or as an equitable servitude.

1. Requirements For A Covenant To Run a. A binding covenant


In order for a covenant to be enforceable by or against successors, it must first have been enforceable between the original parties.

b. A writing and signature


A covenant relating to land must be in writing. The only exception is commonscheme restrictions, discussed infra. In general, a covenant must also be signed by the promisor. A covenant in a deed need not be signed by the promisor, if the promisor is the grantee; it will be binding if the promisor accepts the deed.

MicroMash MBE In Brief: Real Property

27

c. Intent
The original parties must have intended that the rights and/or the duties of the covenant run with the land.

d. Touch-and-concern requirement
A covenant must "touch and concern" the land in order to run; i.e., the promise must affect the promisee and promisor as owners of land, and not merely as individuals. If a covenant increases the value of the promisee's land or decreases the value (or uses) of the promisor's land, then most jurisdictions hold that it touches and concerns the land affected. Covenants to pay money touch and concern the land when there is a direct relationship between the payment of the money and services to be rendered to the land. The covenant must touch and concern both the land of the promisor and the land of the promisee to be enforceable by and against the successors of both parties.

e. Special requirements for a covenant to run at law 1) Horizontal privity


In order for a covenant to run at law, the promisor and promisee must have been in privity of estate at the time the covenant was imposed. This means that the

promisor must have granted the promisee's estate to the promisee (or vice versa) by the same instrument that imposed the covenant. 2) Vertical privity

The successor to property can only be held to the covenant if he has succeeded to the original party's estate (that is, his title can be traced back to the original party). If the successor to the property acquired his title by adverse possession or at a foreclosure sale, he is not in privity with the original party to
the covenant, and the covenant is not enforceable by or against him. Likewise, many jurisdictions hold that a successor who owns a lesser estate than the promisor cannot have a covenant enforced against him, since the successor did not acquire the promisor's exact estate.

f. Special requirements for equitable servitudes 1) Privity No horizontal privity is required for an equitable servitude to be enforceable.
Neighboring but unrelated landowners can enter into a covenant that may be enforceable against the successors to the property as an equitable servitude.

Vertical privity is required only for the right to run. Therefore, the party
seeking enforcement of the covenant must be able to trace her estate back to the original promisee.

28

MicroMash MBE In Brief: Real Property

2) Notice A covenant is enforceable in equity against the subsequent possessor of the burdened estate any time that possessor had notice, actual or constructive, of the existence of the servitude. Only a bona fide purchaser can avoid an equitable servitude on a parcel of land. 3) Dominant estate An equitable servitude can be enforced only if the person seeking enforcement is the owner of a parcel benefited by the covenant. 2. The Common Scheme If land is developed under a common scheme (e.g., a subdivision) and the common scheme includes restrictions on some lots, the owners of those lots can force the same restrictions on the owners of other lots which are part of the common scheme but were not expressly so burdened. a. Finding a common scheme In order to find a common scheme, there must at least be similar restrictions imposed by a common grantor upon a significant number of lots in a given area, such that a scheme of development can be inferred. b. Effect of common scheme negative reciprocal restrictions Any owner burdened by the common-scheme restrictions can enforce those restrictions on any land that is part of the common scheme, regardless of who owns it. 3. Denial Of Relief In Equity Equitable enforcement of a covenant running at law may be denied and an equitable servitude may be extinguished entirely in a number of situations. For example, the equitable doctrine of unclean hands will prevent any lot owner who is in violation of restrictions on his land from enforcing those restrictions against others. Also, equitable enforcement of a restriction may be denied if the neighborhood has changed such that the benefit secured by the restriction is significantly reduced. 4. Comparison Of Land-Use-Control Devices A lease is an effective means to control the use of the land, as the lessor has an unqualified right to control the use of the land via the lease. However, a lease is only appropriate where the person who seeks to control the use of the land is and wants to remain the owner of the land. A covenant is often an effective means to control the use of land. However, a covenant is binding on successors to land only in the limited situations described in this chapter. An easement is often a preferable device to a covenant, where it is appropriate. Easements are indefinite in duration and their enforcement is not subject to the requirements of contract law or equitable considerations. However, easements usually

MicroMash MBE In Brief: Real Property

29

cannot be used to limit the uses of land directly or to require the burdened party to perform an affirmative act. A zoning ordinance is rarely an effective form of land-use control. In the first place, it is difficult to obtain and usually cannot be obtained in reference to a single parcel of land. Moreover, its enforcement is uncertain, since it can be enforced only by public authorities and is subject to administrative variances and legislative changes. A qualified estate is also of limited effectiveness. It is only an option if the person who desires to control the land owns it. Its enforceability is certain, but the severity of the penalty of forfeiture usually destroys the marketability (and mortgageability) of the property.

5. Table Showing Requirements For Running Of Covenants Duty to Run at Law Writing
Yes

Right to Run at Law


Yes

Duty to Run in Equity

Right to Run in Equity

Yes, except in Yes commonscheme cases Yes Yes Yes Yes Yes Yes

Intent

Yes

Yes Yes No

and Yes Touch Concern Dominant Estate Notice


No, except where required by recording statutes Yes Yes

Horizontal Privity Vertical Privity

Yes, probably Yes, probably

No No

No Yes

B. EASEMENTS, PROFITS, AND LICENSES


Easements, profits, and licenses are all types of nonpossessory property rights, meaning that the holders of such rights have the right to use or control the use of land which they do not own or possess.

30

MicroMash MBE In Brief: Real Property

1. Easements
The land that is subject to or burdened by the easement is called the servient estate. If there is a parcel of land benefited by the easement, it is called the dominant estate.

a. Classification of easements 1) Appurtenant easements v. easements in gross If the owner of the easement holds it only by virtue of her status as the owner or possessor of land that is benefited by the easement, then the easement is appurtenant. An appurtenant easement is automatically transferred with the land benefited, and cannot be transferred separately from the transfer of the
dominant estate. An easement in gross is one that benefits no particular parcel of land. The benefits of the easement are held personally by the owner.

2) Affirmative v. negative
If one has an affirmative easement, one has the right to physically enter upon and use the land upon which the easement exists. Occurring less frequently is the negative easement, which gives the owner of the easement the right to prevent the owner of the servient estate from using the land in a particular manner.

b. Creation of easements 1) By express grant (a) Writing required (b) Must be recorded to bind successors 2) By express reservation
An easement by express reservation is created when the grantor, in the deed, reserves to himself easement rights in the granted premises.

3) By implication
In order to have an easement by implication, (1) the dominant and servient estates must have been held in common ownership at the time the easement was allegedly created, (2) the servient estate must have been used in an apparent and continuous way such that a quasi easement could be said to exist, and (3) the continued use of the quasi easement must be reasonably necessary to the enjoyment of the dominant estate. It is possible to defeat an easement by implication by showing that the parties did not intend to create an easement.

4) By necessity
An easement by necessity is the same as an easement by implication except that there is no requirement that a quasi easement exist on the land prior to the time of the division of the land, but the easement sought must be "strictly" (not just

MicroMash MBE In Brief: Real Property

31

reasonably) necessary to the enjoyment of the dominant parcel. An easement is strictly necessary if the land is practically incapable of use without the easement. An easement by necessity is unique in that it is not presumed to be infinite; an easement by necessity lasts only as long as the necessity which gave rise to it. 5) Easement by prescription An easement by prescription is based upon the same legal principles as title by adverse possession. The elements required for an easement by prescription follow. (a) Actual use Use of the servient estate must be made. The use need not oust the owner from possession. (b) Open and notorious The use must be such that the owner is put on notice that she has a cause of action. The statutory period runs from the time that the owner has constructive notice or actual knowledge of the use. (c) Continuous The prescriptive use must not be interrupted or even temporarily abandoned during the statutory period. A use is continuous if it is only seasonal, provided that the customary use of the easement would occur only seasonally. Successive owners of the dominant estate are able to "tack" their periods of successive use in computing the statutory period. Continuous use is interrupted and the period must start anew if the owner physically bars the owner of the dominant estate from using the easement, or if the owner initiates court action to prevent the use of his property. (d) Adverse All that is required for the use to be adverse is that it be without the permission of the owner. A use that starts permissively can become hostile when the user communicates the hostile nature of her use to the owner or when the permission is revoked. If the owner grants permission to continue what once was a hostile use, then the prescriptive period ends. (e) Status of the easement at the end of the period Once the requirements to obtain an easement by prescription are met, the easement has the permanence of any other kind of easement. There is no longer a requirement of continuous use. c. Scope of easements 1) Rights of easement holder In the case of an easement by grant or reservation, the easement rights are determined by the language of that grant. If the easement by grant or reservation

32

MicroMash MBE In Brief: Real Property

fails to determine the location and scope of the easement, or the easement is an easement by necessity, the owner of the servient estate has the right to reasonably fix the location of and control the use of the easement. In the case of easements by implication or prescription, the location and scope of the easement will be determined by the prior (quasi easement or prescriptive) use of the servient estate.

(a) Changed circumstances


Courts will generally permit reasonable change in the use of an easement when circumstances have changed since the creation of the easement. If the original use becomes more intense because the dominant estate is more fully developed, there will not be an overburdening of the easement unless the intensity of the use is beyond the reasonable contemplation of the parties.

(b) Use for other than dominant estate


If the holder of the easement attempts to use the easement for the benefit of land other than the dominant estate, he has automatically overburdened it and that use can be enjoined.

2) Repairs The easement holder has the obligation to keep the easement in repair, and
has the right to enter onto the servient estate to meet that obligation.

3) Right of holder of servient estate


The holder of a servient estate may make any use of her land that is not inconsistent with the rights of the easement holder. If the easement is one of passage, she may use the road herself and permit others to do so. If the location and scope of the easement have not been defined by a grant or prior use, she may reasonably control the location and scope of an easement.

d. Termination of easements 1) By the terms of the grant


An easement is of infinite duration unless limited by conditions in the grant.

2) A written release
The holder of an easement, whether in gross or appurtenant, can terminate it by delivering a written release to the owner of the servient estate.

3) By non-use and intent to abandon


Mere non-use of the easement, even for an extended period, does not result in its destruction. Before the easement will be terminated, there must be an affirmative act that is a manifestation of intent to abandon.

MicroMash MBE In Brief: Real Property

33

4) Estoppel If either an oral release or extended non-use is coupled with reliance on the termination by the owner of the servient estate, then the easement will be terminated. 5) By merger If the servient and dominant estates come into identical ownership, then the easement is extinguished. If the servient estate is later separately reconveyed, the easement does not automatically come back into existence. 6) By prescription If the owner of the servient estate bars the easement holder from using the easement for the statutory period, the easement will be extinguished. 7) Destruction of the servient estate If the easement right is one of passage through a building, the easement is terminated if the building is involuntarily destroyed by fire or other cause. 8) Conveyance of servient estate to bona fide purchaser If a granted easement is not recorded and a purchaser takes the servient estate without knowledge of the easement, then the easement is unenforceable against the purchaser. Easements by prescription, implication, and necessity, however, bind a bona fide purchaser, even though not recorded. 9) End of the necessity An easement by necessity ends at the time the necessity ends. 10) Death In a jurisdiction that does not permit the alienation of easements in gross, they are terminated by the death of the holder, or sometimes by an improper attempt to alienate the easement. 2. Profits A Prendre Profits a prendre are a specialized form of easement. They consist of an easement right to go on the land of another coupled with an additional right to sever, remove, and own something from the land, such as gravel, trees, or water. Profits differ from easements in the following ways. a. Creation Profits can be created by express grant, by reservation, by implication (if there was a quasi profit in existence at the time of the division of the common estate), and by prescription. A profit cannot be created by necessity. b. Gross profits vs. appurtenant profits A profit in gross is freely assignable, unlike most easements in gross.

34

MicroMash MBE In Brief: Real Property

c. Exclusive vs. nonexclusive profits If the profit is "exclusive," then the holder of the profit has an unlimited and exclusive right to take the subject matter of the profit from the land. Even the owner is not permitted to do so. If it is "nonexclusive," then either the owner or others may also take the profit, or the profit holder's right to take the profit is limited by quantity, by a time period, or by a use which may be made of the profit taken. 3. Licenses A license is a right given by the owner that permits a person to go onto and use the owner's land. A license differs from an easement in that it is usually revocable at will, whether given gratuitously or in fulfillment of a contractual obligation. It is also valid if oral, because the Statute of Frauds does not apply. A license is irrevocable in two instances. If the licensor permits the licensee to maintain personal property on his land, the licensee has an irrevocable right to enter onto the licensor's land for access to the property. If the licensee justifiably relies on a grant of a long-term license and spends substantial sums of money because of that reliance, then the licensor will be estopped from revoking the license.

C. OTHER INTERESTS IN LAND 1. Fixtures


a. Definition A fixture is an item of personal property that becomes part of the real estate because of its annexation to the real estate. Once an item becomes classified as a fixture, ownership of the item passes to the owner of the real estate. b. Rules for determining when personal property becomes a fixture The intention of the parties at the time the property was attached controls whether or not that personal property becomes a fixture. However, if there are no clear indications of the parties' intent, the following factors will be important in determining their intent. 1) The character of the attachment Any personalty that is attached to realty in such a way that its removal would cause substantial damage to the realty becomes a fixture. 2) The character of the personalty If the personalty was specially designed for the realty (e.g., doors or windows of unusual size) then it follows that it was to become part of the realty and is treated as a fixture. Also, if the personalty is necessary to the use of the realty, it is likely to be considered a fixture. On the other hand, trade fixtures, or personalty of a tenant attached as a necessary part of her business on the premises, are presumed not to be a fixture.

MicroMash MBE In Brief: Real Property

35

3) The relationship between the competing parties


Disputes over fixtures occur between the following groups: (1) vendor and purchaser, (2) mortgagor and mortgagee, (3) landlord and tenant or owner and licensee, and (4) life tenant and remainderman. In the first two cases, the person who annexed the personal property to the realty was the fee owner of the realty at the time it was annexed. Therefore, it is probable that any property attached by

a vendor or mortgagor was intended to be part of the real estate and is therefore to be treated as a fixture. Conversely, any property attached by a

tenant or licensee is presumed to remain personal property.

c. Removal of personalty not considered a fixture


If the property is not a fixture, then it must be removed promptly at the end of the annexor's estate, or it will become part of the realty. Where there is a life estate, periodic tenancy, or tenancy at will, there is a reasonable time after the end of the tenancy when fixtures can be removed.

Removal is only permitted when it can be accomplished without seriously damaging the real estate. Any damage caused by the removal must be repaired by
the owner of the removed personalty.

d. Third-party interest in fixtures


Since most mortgages are written to include later-added fixtures, the holder of a security interest in a chattel which is later annexed to real estate, and a mortgagee who has a security interest in the real estate, will likely have competing security interests in the fixture.

1) No security interest unless property is easily removable


If the personal property is not removable without substantial damage to the realty, the holder of the chattel has no security interest in the property once it has been attached.

2) Security interest must be recorded before mortgage

Generally, a security interest in a chattel prevails over a mortgage only if the security interest in the chattel is recorded prior to the real-estate mortgage. However, a purchase-money security interest in chattels will prevail over even a prior mortgage, provided that the purchase-money security interest is promptly perfected by recording.

2. Scope And Extent Of Real Property a. Rights in airspace


The possessor has a right to possession of the airspace above his land. Most often, such intrusions are made by aircraft.

36

MicroMash MBE In Brief: Real Property

b. Right to support A possessor of land has the right to both lateral and subjacent support for her land. The right of lateral support means, generally, that neighbors cannot use their land in such a manner that the surface of the possessor's land subsides. The right of subjacent support means that if someone else owns areas below the surface of the possessor's land (e.g., mines), they have an obligation to keep the surface of the possessor's land from subsiding. There is no liability for loss of support if the neighbor properly removes water from underneath the possessor's land. 1) Absolute liability A possessor has an absolute right to lateral support of land in its natural condition. The possessor has an absolute right to subjacent support of land in the condition it was in at the time of the conveyance of the right to subjacent areas. These absolute duties of support cannot be avoided by delegation of the job of excavation to an independent contractor. 2) Negligence A possessor always has a cause of action in negligence for subsidence of her land. The possessor may recover, if her neighbor was negligent, even if the land would not have subsided in its natural state. However, a landowner is not ordinarily liable for the negligence of her independent contractor (unless the independent contractor was engaged in an ultrahazardous activity, such as blasting). The owner can be held liable for her own negligence in choosing an incompetent independent contractor, however, if that is the case. c. Rights in party walls Rights to support also exist in party walls (i.e., a wall which is part of two separate buildings and is built along a common boundary line). Each abutting owner owns that part of the wall on his side of the common boundary, and has an easement to use the entire wall for the support of the remaining part of his building. Each party can extend the height or length of a wall. This must be done at his own expense, and no contribution is allowed unless the other party undertakes to use the extended wall. d. Rights in the subsurface The possessor of land has the exclusive right to the area underneath the surface of her land, and can prevent the mining of minerals even though she cannot reach them from the surface. By the same token, the possessor has the right to sell, lease, or grant an easement in subsurface areas. She does not, however, have the exclusive right to underground liquids and gases if they flow from her land to the land of another.

MicroMash MBE In Brief: Real Property

37

e. Rights in common resources of light, air, streams, and bodies of water 1) Rights in light and air To be valid, rights to light and air must be specifically granted in deeds or other conveyances. A negative easement for light and air cannot be created by implication, because it is not apparent (and also because it would create uncertainty in titles and hamper development). Likewise, an easement for light and air cannot be obtained by prescription merely because one landowner enjoyed, without permission, light or air that came across the property of his neighbor for the statutory period. 2) Water rights classification of bodies of water and rights therein (a) Watercourses Any water that follows a well-defined course or channel, whether aboveground or belowground, is a watercourse. A watercourse need not have water in it year-round. If a watercourse is navigable, the riparian owner's rights are subservient to the public rights in the watercourse. If the waterway is nonnavigable, the riparian owner owns the land under the stream (and has full usufructuary rights in the water). The riparian owner's usufructuary rights vary depending on the theory adopted in the jurisdiction. At common law, the riparian owner had the right to use as much water as she needed for domestic purposes (e.g., irrigation of a family garden on riparian land, bathing, etc.) and could use water for nondomestic (i.e., commercial purposes) only insofar as the quality and quantity of the water she returned to the stream were not affected. This standard was called the "natural flow doctrine," meaning that a lower, riparian landowner had a right to receive water in its natural state, unaffected by any nondomestic uses. Many western states have adopted an appropriation system. Under such a system, the priority of all users is determined by the time they began their use, with the prior user always having a superior right to continue her use. Thus, if the water supply becomes inadequate, later users will have to discontinue their use so that the uses of prior users are protected. (b) Aboveground ponds and lakes If a landowner owns all the land around a lake or pond, he usually has a proprietary right in the water, meaning that he owns the water itself and can sell it as he pleases. If the land around a lake is held by several persons, the rights of such littoral landowners are only usufructuary, and are generally identical to the usufructuary rights of riparian landowners. The usufructuary rights of littoral landowners may also be subject to the public rights in a lake that is used for

38

MicroMash MBE In Brief: Real Property

recreational or commercial purposes. Some of the western states that have established appropriation systems have also applied it to lakes and ponds. Thus, even a littoral landowner must acquire a permit to protect his usufructuary rights. (c) Underground ponds and lakes Again, if a landowner owns all the land above an underground pool or lake, she has a proprietary right in the water. If a landowner owns only part of the land above a lake, her rights will depend on the theory applicable in the jurisdiction. At common law, a rule of capture applied in effect, each landowner above a lake had an unqualified, absolute (but not exclusive) right to appropriate the water below. Basically, a landowner was free to use or sell whatever water she could pump out. Most jurisdictions have now gravitated to a reasonable-use standard, whereby landowners have a right to use underground water to a reasonable extent for domestic, riparian uses. Under such a standard, a landowner will have a right to use underground water for nonriparian uses only if no other riparian uses are effected. Some jurisdictions have adopted a correlative-rights standard which gives a landowner a proprietary right to the proportion of water in an underground lake which corresponds to her proportion of the land over the lake. (d) Surface water Surface water consists of bodies of water, which do not follow any regular course or have any identifiable, regular boundaries. The most common forms of surface water are runoff from rain or snow, and temporary tidal pools and marshlands. The owner of land on which surface water is found has a proprietary right in it. However, the more common issue is whether a landowner can freely rid himself of such waters. In most jurisdictions today, a reasonable use standard applies (though it may take a variety of forms or names); a landowner is permitted to repel or remove surface water as long as it does not unduly damage the land of a neighbor. f. Nuisance A possessor of land has the right to quiet enjoyment of the land. In some situations, an action in nuisance will lie to prevent (or compensate for) violations of this right. Generally, actions of nuisance will only lie for nonphysical invasions (noise, odors, etc.). Physical violations (water, trespassers) are more properly the province of trespass actions, as discussed immediately above.

MicroMash MBE In Brief: Real Property

39

1) Interference with use and enjoyment The action for nuisance protects the plaintiff from interference with the use and enjoyment of her property. Traditionally, it involves such interferences as noise, excessive light, odors, fumes, smoke, etc. The plaintiff may sustain the nuisance action by showing that the defendant substantially interfered with the plaintiff's use and enjoyment of her property, and that such interference was unreasonable, even if the defendant's conduct was not negligent or intentional. 2) Reasonableness of conduct Whether defendant's conduct is reasonable or unreasonable involves the balancing of three considerations: (1) the locality and character of the surrounding area; (2) the nature, extent, and frequency of the interference; and (3) the utility and social value of the activity involved. 3) Relief The court in a nuisance action may grant damages and/or injunctive relief Where plaintiff's harm is insignificant when contrasted with the potential harm to defendant if injunctive relief is granted, the court may treat the nuisance as permanent, and award plaintiff damages for the diminution in the value of his property. 4) Public nuisance Recovery for personal injury resulting from a nuisance may be had whether the nuisance is private or public. If the nuisance is public, plaintiff's injury must be of a different kind than that of the general public.

D. TAKINGS AND ASPECTS OF ZONING


I. Takings Both the federal government and state governments and instrumentalities of both have the power to take private property by eminent domain. However, the Fifth Amendment to the Constitution prohibits the United States from taking private property for public use without just compensation. That amendment is applicable to the states through the Fourteenth Amendment. a. Requirement of a public purpose A purported taking is invalid if it is not for a public purpose. However, the court has construed a public purpose broadly, as coterminous with the scope of the sovereign's police power. If the taking is held invalid because it did not serve a public purpose, the landowner is entitled to damages for the temporary taking for the period when she was unable to use her property because of the invalid taking.

40

MicroMash MBE In Brief: Real Property

b. What constitutes a taking Not all actions by the government that regulate the use of land and in many instances substantially diminish its value are takings for which the government must pay compensation. Zoning ordinances, environmental protections laws, and landmark preservation are usually found to be valid regulations under the police power instead of compensible takings. There are a number of situations, however, where the regulations constitute a taking. Any physical intrusion on private property by the government or the establishment of a non-possessory property interest, such as an easement, constitutes a taking. A land-use regulation, even if enacted for valid police-power purposes, will constitute a taking if it deprives the owner of all economically viable use of the land. The only basis upon which the state could justify a regulation depriving the land of all economic value under the police power is to prove that the building on the land would constitute a common-law nuisance. To be a valid regulation instead of a taking, the regulation must also substantially advance the governmental objective being pursued and there must be a tight fit between the regulation and the governmental interest. 2. Zoning Zoning is the governmental control of the land-use by governmental action. Local government (that of a county, city, or town) enacts the ordinance that actually controls the use of land. The local ordinance is authorized when the state acting pursuant to the police power enacts a zoning-enabling act. There are two levels on which the validity of a zoning act can be attacked: a constitutional level, and that it violates the enabling act. a. Constitutional attack on zoning ordinance 1) The police-power justification Over the years, the court has sanctioned the use of zoning control the density of dwelling units to prevent overcrowding, to achieve aestheticism by imposing controls on the style of buildings to prevent the intrusion of buildings that might depress neighboring property values, to preserve the integrity of historical districts by prohibiting building which is out of character in such districts, etc. As courts became less intrusive on legislative judgments on economic matters, all of these types of zoning controls have been authorized under the police power. 2) Takings A zoning ordinance can constitute a taking, however, when it deprives the landowner of all economic use of her land, or when it imposes conditions upon the use of land which are disproportionate to the benefits which are conferred by the government. The ordinance is not unconstitutional because it constitutes a taking; however, the government must pay just compensation for the value of land which the ordinance has taken away from the landowner.

MicroMash MBE In Brief: Real Property

41

3) Due process Since the adoption of a zoning ordinance is a legislative act, owners of land affected are not entitled to notice and hearing. If there is an amendment to the ordinance that will only affect a few specific parcels of land, however, the governmental activity is more administrative in nature, and affected owners will be entitled to procedural due process. A zoning ordinance can violate substantive due process by infringing upon the associational rights of families. 4) Equal protection Zoning ordinances are fundamentally economic regulations, which neither infringe fundamental rights, nor discriminate by suspect classifications, and thus the ordinance must only bear a rational relationship to a permissible state objective to be upheld. If a zoning ordinance operates with respect to a suspect classification the strictscrutiny standard will apply only if the plaintiffs can show that the purpose of the ordinance was to discriminate, not that it has a discriminatory effect. 5) First Amendment issues A zoning ordinance could be unconstitutional if it improperly limited the right of free speech. b. The zoning-enabling act The zoning-enabling act in force in a state sets forth the procedure which a municipality must follow in enacting or amending a local zoning ordinance and contains such substantive limitations on the power of a municipality as the state determines appropriate. Unless the state enabling act runs afoul of one of the constitutional limitations described above, it is valid. Actions of a municipality that violate the act's procedural requirements or substantive limitations are invalid. Because enabling acts vary widely, a question is impossible in this area without specific reference to the text of the enabling act. The proper approach is to test the actions taken under the zoning ordinance against the specific provisions of the enabling act. c. Administration of zoning 1) Passing and amending the zoning ordinance The first step in subjecting property to zoning controls is the passage of the local zoning ordinance. The procedure for passage must be in accordance with the state enabling act. The local ordinance usually divides the municipality into zoning districts, prescribes the uses which are allowed as of right (conditionally allowed and prohibited in each district), and sets forth density regulations, and setback and height requirements for each district. It then sets up an administration and enforcement structure that must be in accordance with the enabling act.

42

MicroMash MBE In Brief: Real Property

2) Protection of nonconforming uses Property that is in existence at the time a zoning ordinance is enacted or amended and which violates the ordinance either in use, density, or dimensional requirements is protected as a nonconforming use. Such uses can usually be modified with permission of the zoning administrative body, usually a zoning board of appeals. Some ordinances provide that non-conforming uses must be amortized and can only continue for a specific period. A majority of courts uphold such limitations as long as they are reasonable in length and the property can be economically used after the nonconforming use is phased out. 3) Conditional uses Many zoning ordinances today permit specific uses only if certain conditions are met. Many times these conditions are general, such as a requirement that the use be "compatible" with the neighborhood. Normally, compliance with such conditions is determined by a zoning board of appeals after a public hearing in which both proponents and neighbors have an opportunity to be heard. Many commercial permits are granted only after some local body reviews the site plan of the development. 4) Variances There are circumstances dealing with unique conditions on a particular lot of land where compliance with the provisions of the zoning ordinance would create a substantial hardship on the landowner. Variances permitting the property to be used for a purpose prohibited by the zoning ordinance are rarely granted whereas dimensional variances are commonly approved. 5) Floating zones and planned unit developments As the zoning process has matured over the past seventy years, ordinances have permitted developers and administrators more flexibility in planning specific uses. Planned unit developments permit developers of large parcels to plan mixed uses of varying densities on the property and gain approval of the entire plan by some planning authority in the municipality. Floating zones set forth the requirements for a particular kind of use and permit developers to petition the proper authorities to place specific land in those zones.

III. REAL PROPERTY CONTRACTS


A. RELATIONSHIPS INCLUDED
The contract for the sale of land is used in two distinct situations. The most common situation is the one in which its function is to bind the parties. Within a short period of time, usually less than 90 days, the contract is performed by the seller conveying title to the buyer in exchange for the agreed-upon consideration. The buyer does not ordinarily take possession until the contract is fully performed.

MicroMash MBE In Brief: Real Property

43

The second situation occurs when the contract is used as a financing device. The buyer takes possession at the time the contract is executed, makes periodic payments on the purchase price, and receives a conveyance only when all of the consideration has been paid.

B. CREATION AND CONSTRUCTION: THE STATUTE OF FRAUDS


The Statute of Frauds requires that any conveyance of an interest in land or promise to convey an interest in land be in writing and signed to be enforceable in a court.

1. Interests In Land Within The Statute Of Frauds


Interests in land include: (1) leaseholds, except those of short duration which are expressly excepted by statute; (2) interests of mortgagor and mortgagee or vendor and purchaser under a specifically enforceable contract; (3) present and future interests, both legal and equitable; (4) easements and profits; (5) interests created by covenants; and (6) option contracts. Contracts by joint tenants or tenants in common to partition land into separate tracts for each tenant are generally also held to be within the Statute of Frauds. 2. Contracts Associated With Land Not Within The Statute Of Frauds Agreements which create a license as opposed to an easement or covenant, boundary agreements and brokers contracts, and a promise not to make a will so that an heir will inherit the real estate, although connected with land are not within the Statute of Frauds.

C. THE PURCHASE AND SALE AGREEMENT


-

1. The Enforceability Of A Purchase-And-Sale Agreement In order to be enforceable, an agreement to convey land must meet the following requirements: the agreement must be a valid contract, and the agreement must satisfy the Statute of Frauds (or the Part Performance Doctrine). In general, it can be said that an agreement to convey land is enforceable only if it is evidenced in a writing or writings that will satisfy the Statute of Frauds. However, a court will order a conveyance of land, without any evidentiary writing at all, if the requirements of the doctrine of part performance (discussed, infra) are met. a. Required contents of the writing(s) Any writing or collection of writings that evidences the agreement will suffice, even if they were not intended to embody the contract. For the contract to be enforceable, the parties and the land must be identified, there must be language indicating a sale was intended, and the purchase price must be included if it has been agreed upon. The document must then be signed by the party to be charged. b. The part-performance doctrine If a court finds sufficient evidence in the parties' actions of an agreement to convey property, it will order a conveyance of the land, despite the lack of a writing. Generally, part performance is not grounds for an action at law for damages; specific performance is the only remedy available.

44

MicroMash MBE In Brief: Real Property

In general, jurisdictions require the purchaser to perform varying combinations of the following acts: (1) payment of the purchase price, (2) possession of the land with the permission of the seller, and (3) improvements to the land. A majority of jurisdictions require all three. Payment of the purchase price alone is not sufficient.

2. Implied Conditions Or Terms a. Time of performance ("closing")


When a purchase-and-sale agreement fixes no date for performance, a court will presume that the parties intended it to be performed within a reasonable time. If a specific date is fixed, strict compliance with that date is required only in an action at law (for damages). Strict adherence to the date for performance is not required in equity, unless the contract specifies that time is of the essence, or that fact can be implied from the circumstances of the contract. Thus, failure to perform on the specified date is usually not grounds for an action for rescission or specific performance.

b. Title required
If the purchase-and-sale agreement is silent on the matter, the seller is required to deliver marketable title at the closing to all of the property specified in the purchaseand-sale agreement. The requirements of marketable title are discussed below. The agreement can provide for a lesser quality of title.

c. Burdens related to title defects 1) Seller's right to notice of defects


When the purchaser finds a defect in the title, he must notify the vendor, who will then have a reasonable time to clear the defect. A purchaser cannot assert a defect that was not brought to the vendor's attention as a justification for the purchaser's failure to perform.

2) Seller's burden to clear liens


If there is a defect in marketable title created by a lien on the property, the seller has the obligation to clear that lien at the time of the closing. The most common lien is a mortgage lien that can be cured by use of a portion of the proceeds at the closing. If the defect is uncurable by the payment of a specified amount of money because another person has some title claim, then the obligations of the parties is governed by the agreement.

3) Seller's right to notice of defects


When the purchaser finds a defect in the title, she must notify the vendor, who will then have a reasonable time to clear the defect. A purchaser cannot assert a defect that was not brought to the vendor's attention as a justification for the purchaser's failure to perform.

MicroMash MBE In Brief: Real Property

45

4) Variance between area owned by seller and area specified in agreement


Minor variances between the land promised in the contract to convey and the land conveyable by the seller will not put the seller in breach of contract, unless there was fraud involved. A variance between the area agreed to and actually conveyable also will not usually result in an adjustment of the purchase price, unless the price was established on a per-acre basis.

5) Buyer's options when defect incurable


The buyer has a right to terminate the agreement and recover his deposit. Some agreements give the seller a period of time to cure a title defect and keep the agreement in force during that time. Some agreements will require the seller to use good-faith efforts to cure a title defect. In any event, the buyer can waive the requirement of marketable title and take any title that the seller possesses. Ordinarily, if the buyer elects to take the seller's unmarketable title, he must pay the full purchase price.

D. PERFORMANCE 1. Fitness And Suitability Of The Premises


Since the Uniform Commercial Code does not apply to sales of realty, there are generally no implied warranties, except for the covenants of title discussed below incident to a deed. The purchaser must rely on express warranties of material fact made by the seller. The one exception is that a warranty of habitability is implied in every deed from a vendor who is also the builder of the improvements on the land. Except for the implied warranty on a new home, obligations of the seller concerning the physical status of the premises conveyed depends upon the language set forth in the purchase-and-sale agreement. If nothing is said about condition and there is no misrepresentation about the condition of the premises, the obligation of the seller is only to convey the premises in their existing condition. Most purchase-and-sale agreements provide that the buyer has a period of time to inspect the condition of the premises and terminate the agreement if it has environmental defects or has physical defects, and terminate the agreement if the defects are not cured. If the buyer does not terminate within the specified period, she is cannot use the defects as a reason to back out of the agreement.

2. Marketable Title Required


Unless the contract provides otherwise, the seller must provide "marketable" title at the closing, even if he is only obligated to give a quitclaim deed. If the seller cannot produce marketable title, he is in breach of the contract to convey; the purchaser need not go through with the conveyance and may sue for damages. A marketable title is one reasonably free from doubt both in fact and in law. It is the type of title that a reasonable, prudent, and knowledgeable businessperson desiring to purchase the property would accept. If there is a factual problem with the title, the facts supporting its marketability must (1) be so conclusive that a judge would not permit a

46

MicroMash MBE In Brief: Real Property

contrary verdict to stand, and (2) be capable of proof whenever challenged. If the challenge is one of law, the law in favor of good title must be clear and not debatable. An adverse possessor must obtain a judicial decree supporting his title before he has marketable title. Defects that render title unmarketable are as follows: a. Gaps or defects in the chain of title b. Inadequate estate c. Encumbrances The purchase-and-sale agreement can specifically provide that the conveyance is to be made subject to certain enumerated encumbrances. Absent an agreement to the contrary, encumbrances have the following effects. Mortgages and liens destroy marketable title. Dower rights destroy marketable title. Restrictions and equitable servitudes destroy marketable title only if they are more burdensome than the zoning ordinances applicable to the land. Leasehold interests destroy marketable title. Easements destroy marketable title only if they actually or potentially interfere with the reasonable use of the land. Zoning ordinances do not destroy marketable title. However, if the zoning is changed between the time of the purchase-and-sale agreement and the time of the closing, some courts would permit the purchaser to rescind the agreement. Also, an existing violation of a zoning ordinance likely renders title unmarketable. 3. Risk Of Loss The parties are always free to determine in their agreement who will bear the risk of loss. However, if they fail to do so, most jurisdictions hold that the doctrine of equitable conversion governs the risk of loss. The minority view is that risk of loss is on the vendor until legal title passes. The Uniform Vendor and Purchaser Act puts the risk of loss on the party in possession. Where the risk of loss is on the purchaser, but only the seller is insured for the loss, then either the seller must deduct the insurance proceeds from the sale price or he must pay those proceeds to the purchaser. 4. Remedies For Breach Of Purchase-And-Sale Agreement a. Damages Either party is entitled to expectancy damages (the difference between the market value of the property at the time of the closing and the purchase price) if the other party breaches. If the seller breaches, the purchaser is also entitled to the return of any deposit she has paid.

MicroMash MBE In Brief: Real Property

47

b. Specific performance
Either side can also elect to obtain specific performance, meaning that the court will force the breaching party to pay the purchase price or convey the property, whichever is appropriate. A court may also award incidental damages as part of a judgment for specific performance.

c. Rescission E. INTEREST BEFORE CONVEYANCE

1. The Effect Of An Enforceable Purchase And Sale Agreement The Doctrine Of Equitable Conversion
The doctrine of equitable conversion provides that a purchaser who has a valid and binding agreement to convey should be treated as the owner of the property. The doctrine does not apply to options to purchase, unenforceable (e.g., oral) agreements to convey, or agreements to convey in which a condition precedent to enforceability (e.g., attainment of a mortgage) has not been met.

a. Right of possession and enjoyment


Equitable conversion does not give the purchaser a right to possession prior to the time of performance. If the contract gives the right to possession to the purchaser before the closing, he has the full use of the property and may commit waste on it as long as the value of the property does not fall below the balance due on the purchase price.

b. Risk of loss
The parties are always free to determine in their agreement who will bear the risk of loss. However, if they fail to do so, most jurisdictions hold that once there is a binding purchase-and-sale agreement, the purchaser bears the risk of loss if the property is destroyed, even if she does not have possession.

c. Third-party damage
The purchaser can bring a tort action against third parties for any damage to the property committed after the execution of the contract to convey.

2. Earnest Money Deposits


A purchase and sale agreement typically requires that the buyer deposit a portion of the purchase price in escrow, and usually provides that the deposit will be liquidated damages if the buyer defaults. Courts will permit the seller to keep the deposit in the event of a buyer default, providing it bears a reasonable relationship to the seller's actual damages.

3. The Closing a. Seller must sign deed


The purchaser is entitled to a deed from and actually signed by the seller if the deed is to contain any warranties of title.

48

MicroMash MBE In Brief: Real Property

b. Purchaser's obligation to produce purchase money The purchaser is expected to produce the full purchase price at the time of the closing, unless other arrangements are made. F. RELATIONSHIPS AFTER CONVEYANCE 1. Effect Of Closing On Purchase-And-Sale Agreement In general, a conveyance discharges the obligations arising from the contract to convey, except those obligations expressly made to survive the closing. After accepting a deed, the purchaser may sue only on the covenants (e.g., the warranties of title) contained in the deed. 2. Title Problems Once the deed is delivered and accepted, the grantee can only sue for defects in the title based on the covenants of title in the deed. Any promises made by the grantor in the purchase-and-sale agreement are superseded and extinguished by the deed, unless those provisions of the purchase-and-sale agreement are made to survive the deed.

IV. REAL PROPERTY MORTGAGES


A. TYPES OF SECURITY DEVICE 1. Mortgages (Including Deeds Of Trust) a. Mortgage A mortgage is a conveyance of an interest in land for the purpose of securing some obligation that the owner of the land ("the mortgagor") owes to a creditor (the "mortgagee"). b. Deeds of trust A trust deed is a conveyance from the owner of property to a third person in trust. The trustee is to hold the property as security for a debt owed by the owner to a lender. The trust instrument will require the trustee to deed the property to the debtor when the obligation is satisfied, or to the creditor in the event of default. The principal difference between the deed of trust and a mortgage is in the method of enforcement; the trustee may be able to sell the property without all of the safeguards required for a foreclosure sale. 2. Land Contracts As A Security Device A purchaser who buys property under an installment contract is in much the same position as a mortgagor. The purchaser will take immediate possession of the property, but will not be the legal owner of the property until she pays the seller the full purchase price in installments and receives a deed. There are two principal problems with this device from the buyer's viewpoint. First, the seller may be unable to deliver clear title when the payments have been completed.

MicroMash MBE In Brief: Real Property

49

Second, if the purchaser under an installment contract breaches by making a late payment or by failing to make a payment at all, she may lose the right to enforce the contract and obtain a deed and also forfeit the payments she has already made. 3. Absolute Deeds As Security, Equitable Mortgages Where a deed, absolute on its face, was delivered solely as security for a debt, the deed is treated as an equitable mortgage. a. Procedure to establish an equitable mortgage To establish that a deed is in fact an equitable mortgage, the grantor (mortgagor) may introduce parol evidence in an equity proceeding to show that the deed was intended only to serve as a mortgage and that he is entitled to a reconveyance when the debt is paid. b. Rights of bona fide purchasers from the equitable mortgagee If the deed that is in fact an equitable mortgage has been recorded in the registry of deeds, however, a bona fide purchaser from the grantee (mortgagee) can cut off the rights of the grantor (mortgagor). 4. Sale Leaseback Arrangements A sale of property accompanied by a lease back to the seller may function as a financing device. In effect, the purchase money given to the seller of the land is a loan, the purchaser merely holds title as security for the repayment of the money loaned, and the seller-lessee's rent payments are mortgage payments. At the end of the lease, the sellerlessee can usually repurchase the property and recover legal title for the amount of the loan still outstanding.

B. SOME SECURITY RELATIONSHIPS


1. The Underlying Obligation A mortgage must serve as security for an underlying obligation. The obligation is
often a debt owed by the mortgagor to the mortgagee. However, a landowner can grant a mortgage to secure any sort of obligation including a duty to perform services. A mortgage is enforceable only as long as there is an underlying, enforceable obligation. If the underlying debt is unenforceable (e.g., for lack of consideration, fraud, duress, etc.) the mortgage is likewise unenforceable and the mortgagor has a right to have the mortgage discharged. There are, however, two exceptions if the debt becomes unenforceable due to a discharge in bankruptcy or due to the running of the statute of limitations, the mortgage will continue to be enforceable. Requirements for a mortgage are as follows: a. Formalities of execution b. Consideration not required

50

MicroMash MBE In Brief: Real Property

2. Title And Lien Theories Of Mortgages In a so-called title-theory state, the mortgagor will deed legal title to the mortgagee, but will retain an "equity of redemption" the right to have legal title revested in him when he discharges his obligation. In a so-called lien-theory state, the mortgagor conveys only a lien on the property to the mortgagee. 3. Rights Between Mortgagor And Mortgagee Prior To Default The mortgagor has the full right of possession (including the corollary right to the rents and profits from the land) until default. The mortgagor also has a possessory right to use the property, as long as her use does not constitute waste that would prejudice the security interest of the mortgagee. 4. Right To Redeem And Clogging The Equity Of Redemption In some states, even after the mortgagee has foreclosed the equity of redemption and in effect vested title in himself or a purchaser from the mortgagee, some states permit the mortgagor to redeem the property for a period of time, usually two years after the foreclosure sale. If the mortgagor pays the lesser of the amount bid at the foreclosure sale or the amount due on the mortgage note and foreclosure costs within the prescribed time, the mortgagee or the person buying from him must deed the property back to the mortgagor.

C. TRANSFERS BY THE MORTGAGOR


1. Conveyance Free And Clear Of The Mortgage The grantor-mortgagor can convey the property free and clear of the mortgage if she discharges her obligation to the mortgagee at the time of or prior to the closing. 2. Conveyance Subject To The Mortgage The grantor can convey the property without making any special arrangements in regard to the mortgage. In this situation, only the grantor is personally liable on the note. The purchaser will lose his land, however, if the mortgage obligation is not discharged, and so will be informally expected to pay the mortgage debt. The grantor-mortgagor can force the mortgagee to foreclose on the land before pursuing the grantor-mortgagor personally. If the grantor-mortgagor does pay on the mortgage obligation, however, he is subrogated to the rights of the mortgagee, and may foreclose on the property if the grantee does not pay him back. 3. Conveyance Subject To The Mortgage With The Grantee Assuming The Debt The grantee may expressly promise the grantor-mortgagor that she will pay the mortgage obligation as it becomes due. By assuming the mortgage obligation, the grantee becomes the principal debtor and the grantor-mortgagor becomes a surety. The grantee's express promise to pay the mortgage obligation gives the grantor-mortgagor a direct cause of action against the grantee if the grantee fails to pay. The mortgagee is a third-party beneficiary of the grantee's promise to pay, and so the mortgagee can sue the grantee directly if the grantee fails to pay.

MicroMash MBE In Brief: Real Property

51

4. Novation
In a few cases, the mortgagee may agree to substitute the grantee as the obligor and release the grantor-mortgagor from any liability. In order to accomplish this, the grantee and mortgagee would have to enter into a new contract (or "novation").

5. Due-On-Sale Clauses
In theory, the mortgagee need not approve of the conveyance of the property to the grantee or the financial arrangements made between the grantor-mortgagor and grantee. However, most mortgages now contain a "due-on-sale" clause that makes the entire mortgage debt due immediately if the grantor-mortgagor conveys the property. If the entire debt is not paid when the mortgagee exercises his right to call the entire debt, the mortgage can foreclose. So, unless the grantor-mortgagor and grantee can arrange to pay off the mortgagee at the time of the sale, they must obtain the mortgagee's approval of the sale. Courts have upheld the validity of due-on-sale clauses.

D. TRANSFERS BY MORTGAGEE
Mortgagees may transfer both the underlying obligation and the mortgage itself. Normally, the two will be transferred together. A conveyance of a mortgage is a conveyance of an interest in property and so must meet the necessary formalities. Words of conveyance, however, are not necessary to transfer the mortgagee's interest in a mortgage; the term "assign" is sufficient. If the obligation is in the form of a negotiable note, it will be endorsed and negotiated to the transferee. Any other obligation must be conveyed by a contract between the mortgagee and the transferee (or a novation between the mortgagor and the new obligee). The first purchaser has the presumed right to both instruments, except that a holder in due course of the note always has a superior right to the mortgage.

E. DISCHARGE AND DEFENSES


When the mortgagor has satisfied his obligations to the mortgagee, he is promptly entitled to receive from the mortgagee a discharge of the mortgage in recordable form so that he can clear his title of the mortgage encumbrance. Most states impose severe penalties on mortgagees who fail to provide a discharge when they are obligated to do so. In addition the mortgagor can sue for any actual damages suffered by failure to provide a discharge. A mortgagee, however, is not required to accept satisfaction of the underlying obligation if the mortgage note does not permit prepayment of the mortgage. If the note only permits the mortgagor to pay the note with interest in installments over a fixed period of time, the mortgagee can refuse to accept any payment except the installment due and not discharge the mortgage. Many states have provisions that in consumer mortgages require the mortgagee to accept prepayment and discharge the mortgage. The mortgage note frequently provides for a prepayment penalty if the mortgagor wants to pay the mortgage off ahead of time. Such clauses are lawful. The mortgagee does not have

52

MicroMash MBE In Brief: Real Property

to discharge the mortgage until he receives both the amount due on the note and the prepayment penalty.

F. FORECLOSURE
If the mortgagor defaults on the underlying obligation, the mortgagee has the power to foreclose on the mortgage. Foreclosure is the process that extinguishes the mortgagor's equity of redemption and allows the mortgagee to use the mortgaged property to satisfy the underlying debt.

1. Types Of Foreclosure a. Bill in equity to foreclose


This is also called "strict foreclosure." In an equitable proceeding, the mortgagee asks the court to set a date for performance of the obligation and to order that, if there is no performance by that date, the mortgagor's equity of redemption be cut off

b. Foreclosure by entry of action


This method is used if the mortgage contains no power-of-sale provision. The mortgagee must enter the land and continue in possession for three years. At the end of the statutory period, the mortgagor's equity of redemption is automatically cut off

c. Foreclosure under a power of sale


Most commonly, a mortgage is foreclosed pursuant to a power of sale contained in the mortgage.

1) Mortgagee must comply with contractual and statutory provisions


In order to foreclose on and sell mortgaged property validly, the mortgagee must adhere strictly to the contractual and statutory requirements for a foreclosure sale. For example, statutes generally require that the foreclosure sale be by public auction after proper notice to the mortgagor and advertisement of the sale.

2) Mortgagee must act in good faith


The mortgagee must act in good faith and use reasonable diligence to protect the interests of the mortgagor. However, the mortgagee is allowed both to bid on the property at the foreclosure sale and to act as the auctioneer.

2. Rights Of Omitted Parties


The mortgagee prior to foreclosure must run the title to determine all persons having an interest in the mortgage premises. These parties include the original mortgagor, any subsequent owner of the property who has received her interest through the mortgagor, and junior mortgage holders; any attaching creditors; and any other persons whose interest in the land is shown in the record title. These parties must be notified of the foreclosure proceeding so that they can object to it if they think the proceeding is improper and so that they can attend the foreclosure sale to protect their interests.

MicroMash MBE In Brief: Real Property

53

3. Deficiency And Surplus The proceeds of a foreclosure sale are distributed as follows: (1) to the foreclosing mortgagee goes the sum of the unpaid balance of the obligation, the interest up to the time of foreclosure (but not thereafter), and the costs of foreclosure; (2) then, junior mortgagees and attaching creditors have access to the proceeds, in the order of their seniority under the recording statute; and (3) any surplus goes to the mortgagor. a. Deficiency judgment In many states, the mortgagee has a right to a deficiency judgment if the property does not bring enough to satisfy his claim. b. Relationships between competing mortgagees at foreclosure The priorities of mortgagees are determined by the recording statute of the jurisdiction. Thus, the first mortgage granted is not necessarily the first in priority. The purchaser at the sale will take the property free of all encumbrances except those senior to the mortgage being foreclosed. The junior mortgagees will have to bid on the property to protect their interests in it. c. Relationship between mortgagees and holders of security interests Mortgages are usually written so that they cover any fixtures, whether attached to the realty at the time of the mortgage or later added. Since fixtures may be the separate subject of security interests, there may be competing security interests in the same fixture. 1) Fixtures that can be subject to security interests Goods irretrievably incorporated into realty (e.g., bricks) cannot properly be the objects of security interests other than mortgages. 2) Priorities of mortgages and security interests Generally, only a security interest in a fixture that is recorded prior to the mortgage takes priority over the mortgage. There is one exception, however, a purchase-money security interest prevails over even a prior mortgage, if it is perfected within a reasonable time after annexation of the fixture to realty. 3) Rights of the superior holder of a security interest If the holder of a security interest prevails over the mortgagees, she may remove the fixtures. She will be liable to the mortgagees for any damage done to the property by removal. 4. Statutory Right Of Redemption Many states allow a mortgagor to recover the property for a reasonable amount of time after the foreclosure sale by paying the amount of the winning bid at the foreclosure sale to the mortgagee or the purchaser at the foreclosure sale. While this procedure makes the property unmarketable for a period of time, it does make it prudent for the mortgagee to

54

MicroMash MBE In Brief: Real Property

bid a fair price at the foreclosure sale. If he bids a nominal price, the property can be redeemed for that nominal price. 5. Deed In Lieu Of Foreclosure When a mortgage is in default, the mortgagor and mortgagee can agree that the mortgagor will deed the property to the mortgagee in satisfaction of the mortgage debt. Such a procedure is valid, and if done properly will transfer the ownership of the property to the mortgagee. The deed in lieu of foreclosure is not a viable option if there are subsequent encumbrances on the property, because the deed will not wipe them out as a mortgage foreclosure will. Therefore the mortgagee taking a deed in lieu of foreclosure must make sure that the deed will give her clear title to the property.

V. TITLES
A. ADVERSE POSSESSION
Adverse possession is a doctrine that vests title to property in a person solely by reason of his long-standing possession of the land. Generally, the possessor must wrongfully possess land, in such a way that the true owner should notice his possession, for the period of the statute of limitations for ejectment. At common law, the period was 20 years. 1. Time When Statute Begins To Run The statute begins to run when the owner becomes or should become aware of the possession. a. Possession must be hostile In general, the possession must be without the owner's permission, but need not be hostile in the sense of ill will. 1) Cotenant cases Possession by a cotenant becomes hostile only when the cotenant either physically ousts the other cotenants or effects a constructive ouster of nonpossessory cotenants by expressly informing them that she is asserting exclusive dominion over the property. 2) Permissive commencement cases If possession commenced under a lease or in some other permissive manner, then it does not become adverse until there is explicit notification that possession is henceforth adverse, or there is activity known to the owner which is inconsistent with the lessee's permissive use of the premises. b. Possession must notify owner; open-and-notorious possession The statute of limitations does not begin to run until the owner actually knows or should know of the possessor's possession. The legal standard has developed that the owner is put on notice of the possession when the possessor takes open and

MicroMash MBE In Brief: Real Property

55

notorious possession of the property. The possession required will be the type of
possession that the average owner of such land would exercise. Physical presence on the land is indispensable.

c. Effect of owner's disabilities


If the owner is disabled at the time his cause of action accrues, the common-law rule is that the statute of limitations does not run against him until the disability is removed. However, the running of the statute is not affected by disabilities later incurred.

2. Requirement Of Continuous Possession


The possessor acquires title by adverse possession only if she has been in continuous possession for the statutory period. Seasonal use qualifies as "continuous," as long as the possessor uses the property during the appropriate season for each and every year of the statutory period.

a. "Tacking" between successive possessors An adverse possessor is entitled to count the period of possession of a prior possessor towards the statutory period, if he is in privity with the prior possessor. Any sort of consensual transfer of possession will put two adverse
possessors into privity. Two possessors are not in privity when the second possessor's possession is adverse to the first possessor.

b. Interruption of possession If the possessor's continuous possession is interrupted by the owner, the statutory period must start all over again. The possession can be interrupted by
any of the following methods.

1) Possession by the owner


The possessor's possession must be exclusive of the owner. Thus, if the owner goes into possession, the statutory period is terminated.

2) Ouster 3) Judicial action


Filing an action for ejectment will terminate the possessor's possession, as long as the owner obtains a decree of ejectment.

3. Title Obtained By Adverse Possession


The possessor takes a new title. Thus, his title is not subject to the defects found in the record owner's title.

The possessor only takes the estate of the owners who no longer have a cause of action to eject him. If the possessor first took possession during a life estate or estate for
years, a bare 20 years of possession will only give him title to those estates.

56

MicroMash MBE In Brief: Real Property

a. Possession no longer required


After obtaining title by adverse possession, the possessor need not remain in possession.

b. Property acquired by possession and constructive adverse possession Generally, the possessor acquires title only to the property she has actually possessed. However, under the doctrine of constructive adverse possession, if a
possessor enters onto land and takes possession of a part of it under "color of title," she will obtain title to the entire tract, even though she didn't possess all of it.

B. CONVEYANCE BY DEED 1. Requirements Of A Valid Conveyance


A deed must meet the requirements of the Statute of Frauds, and often the additional requirements of the deeds statute of the jurisdiction, to be valid.

a. Requirements of a valid deed 1) Signature of the grantor 2) Name of the grantee 3) Words of conveyance 4) Description of the property conveyed b. Elements not generally required 1) Witnesses or acknowledgment 2) Recording 3) Consideration 2. Necessity Of A Grantee
The grantee need only be identifiable from the deed. The grantee need not sign the deed. Also, the grantor need not be the one to place the grantee's name on the deed. The grantor may hand over a deed without a grantee's name and the conveyance will be valid for whomever's name later appears as the grantee (assuming that the grantor had the requisite intent to convey at the time he handed over the deed).

3. Delivery
There must be a "delivery" of the deed for title to be conveyed. The grantor "delivers" the deed at the time that she intends to confer an immediate, irrevocable interest on the grantee. A physical transfer of the deed to the grantee most likely indicates that the grantor intended to create an immediate interest in the grantee. However, it is possible to have delivery without physical transfer, and vice versa.

MicroMash MBE In Brief: Real Property

57

a. Delivery without physical transfer


A grantor can "deliver" the deed without physically giving it to the grantee. All he need do is execute the deed and somehow manifest intent to make it effective immediately.

b. Physical transfer without delivery


By the same token, the grantor can physically transfer the deed to the grantee without affecting delivery. If the grantor hands over the deed without intent to immediately and irrevocably create an interest in the grantee, the deed will not pass title. However, in cases where the grantee has physical possession of the deed, the grantor will have to overcome a presumption that the deed was delivered.

c. Physical transfer to third parties


In line with the general rule above, physical transfer to a third party only qualifies as a delivery if the grantor thereby intends to create an irrevocable and immediate interest in the grantee. Thus, an unconditional transfer of the deed to the grantee's agent will likely qualify as a delivery.

4. Land Description And Boundaries


Aside from the basic requirement that the property be described in a manner sufficient to identify it in order for the deed to be valid, any description of the property in the deed will be important in any subsequent disputes regarding the boundaries of the property. Where different descriptions of land in the same deed are inconsistent, the following rules of construction will be applied. Natural monuments prevail over artificial ones. Natural and artificial monuments both prevail over distances. Specific descriptions prevail over general ones. If a deed refers to a plan, the plan is incorporated into the deed, and the courses and distances on the plan are to be regarded in determining the true construction of the deed. If the grantor conveys property bounded by a road and the grantor owns the fee under the road, the deed conveys property to the middle of the road, unless that does not appear to be the grantor's intent and there is a reason she might want to keep an exclusive interest in the road (e.g., she owns the land on the other side or at the end of the road and needs it for access). The same rules as above apply to waterways the deed is presumed to convey to the middle ("the thread") of a waterway (if the grantor owned the land under the waterway) and any distances are measured from the banks of the waterway.

5. Covenants Of Title
Covenants of title are promises by a grantor in a deed in regard to the title the grantor is conveying. Once the deed is delivered and accepted, the grantee can only sue for defects in the title based on the covenants of title in the deed.

58

MicroMash MBE In Brief: Real Property

a. Types of covenants 1) Present covenants (a) Covenant of seisin Here, the grantor warrants that he has title to and possession of the property or the interest in property conveyed. (b) Covenant of the right to convey Here, the grantor warrants that she has the right to convey the property to the grantee. (c) Covenant against encumbrances Here, the grantor warrants that there are no liens, mortgages, easements, or other interests in third parties which will diminish the ownership rights of the grantee, except those listed in the deed. 2) Future covenants (a) Covenant of quiet enjoyment By the covenant of quiet enjoyment, the grantor warrants that the grantee and his successors will not be disturbed in their possession of the property by the grantor or someone with a claim of title superior to that of the grantor. (b) Covenant of warranty By the covenant of warranty, the grantor guarantees that her title is good, and that she will assist in defending that title against claims by third parties. (c) Covenant for further assurances This covenant is not common, but where it is given, the grantor promises that he will take whatever steps are necessary to perfect any defects in title. b. Types of deeds There are, generally, three types of deeds with respect to covenants. 1) Quitclaim deed The quitclaim (or "release") deed contains no covenants. It conveys to the grantee whatever the grantor had, without making any representations or promises as to the grantor's title. 2) General warranty deed The general warranty deed contains all of the covenants discussed above, except the covenant for further assurances. Such a deed makes the grantor liable for any encumbrances or defects in title that existed at the time of the conveyance, whether attributable to her or her predecessors.

MicroMash MBE In Brief: Real Property

59

3) Special warranty deed The special warranty deed contains the same covenants as the general warranty deed, but the grantor is liable only for defects or encumbrances incurred during his ownership. In effect, the grantor warrants only that the property was not encumbered and the title did not become defective during his ownership. c. Actions for breach of covenants 1) Time of breach Present covenants are breached at the time of conveyance, if at all. Thus, the statute of limitations starts to run at the time of the conveyance, even if the title defect or encumbrance is not discovered until later. Future covenants are not breached until the grantee's title to the property is seriously and validly disputed. The mere existence of an undisclosed mortgage will not breach a future covenant; the grantee must be threatened with eviction. 2) Who can sue A grantee has a cause of action only against her immediate grantor for a breach of a present covenant. Future covenants run with the land. Therefore, once a grantor has made a future covenant, she is liable to any subsequent grantee in her chain of title for any defects or encumbrances that existed at the time she conveyed the property. C. CONVEYANCING BY WILL If an individual dies leaving a valid will, and owns real estate in his individual capacity as opposed to owning it as a joint tenant, the will acts as an instrument of conveyance, and transfers ownership of the property to the persons to whom the property was devised. If the property is not specifically devised by the will, the residuary clause of the will serves as the instrument of conveyance. If the will does not effectively dispose of property, then the laws of intestacy of the state in which the property lies govern the devolution of the property to the decendent's heirs. For the disposition in any of these cases to be effective, there must be a probate of the decedent's estate so that the will is allowed and their heirs determined and there is a record of the distribution of the estate. There are three circumstances where the devise of the real estate made in the will is ineffective: 1. Ademption The will only operates on property that is owned by the decedent at the time of death. If the decedent sold or otherwise disposed of the property during her lifetime, the devise is adeemed by extinction, and the devisee does not receive other property to compensate her. 2. Exoneration The devise of the real estate is effective only if all of the debts and taxes of the estate can be paid without selling the real estate to satisfy them. If the real estate is specifically

60

MicroMash MBE In Brief: Real Property

devised, all of the property not specifically bequeathed or devised that is in the residuary estate will be sold before specifically devised real estate to satisfy debts and taxes. If the real estate is part of the residuary, then it will be sold to satisfy specific and general legacies, and will abate if there are not sufficient unallocated funds to satisfy both the expenses of the estate and the prior legacies. If the executor must sell to pay the debts or prior legacies, the devise of real estate in the will is ineffective.

3. Lapse
A devise is only effective if the devisee survives the testator. It will lapse if the testator survives the devisee and will be disposed of by the residuary clause of the will, or if the devise is to a sole residuary legatee, by intestacy. All jurisdictions have anti-lapse statutes that provide that a devise to a relative survived by issue will go to the issue if the named relative predeceases the testator. Statutes vary with respect to the degree of kindred required between the decedent and the deceased devisee.

D. PRIORITIES AND RECORDING 1. Types Of Priority a. Recording acts


A deed need not be recorded to be valid and convey good title. An unrecorded deed is always valid to give good title to the grantee, at least in regard to the grantor. However, if a grantee does not properly and promptly record his deed, he may lose his title if his grantor later grants the land again to someone else ("the subsequent grantee"). Recording systems give stability to titles by providing a method of verifying a grantor's title and protecting the title of a purchaser who has bought land without knowledge of any prior grantee. A grantee must promptly and properly record the deed he receives from the grantor, or a subsequent purchaser without knowledge of his deed may be able to take good title from the grantor. The public-recording system provides a method for a purchaser of land to give notice to the rest of the world that he is now the record owner of land, so that no subsequent purchaser from the grantor can rob him of title. In essence, although an unrecorded deed is valid as between a grantor and a grantee, only a properly recorded deed is good against the rest of the world.

b. Judgment liens
The priority given to a judgment lien over other instruments depends on the statute in the jurisdiction. Some states require that a judgment be recorded in the registry of deeds before it is a lien on property and it is then governed by the priorities in the recording system. In some states a judgment is a lien as soon as it is rendered by a court, even if it is not recorded in the registry of deeds. The priority of that lien would be governed by the date of the judgment. Some states make a judgment a lien against property acquired subsequent to the judgment where it would take priority over other liens filed at that time except for purchase-money mortgages. If a state

MicroMash MBE In Brief: Real Property

61

gives automatic lien status to a judgment, it is necessary to check for judgment liens in an title search. c. Fraudulent conveyances Almost all states have enacted the Uniform Fraudulent Conveyance Act. When the owner of property makes a fraudulent conveyance, a transfer, or when the conveyance is made to defraud creditors, designated creditors, set forth in the act, have rights in the property superior to the rights of the transferee. They can treat the property as if it were still owned by the transferor and can use it to satisfy their claims against the transferor. However, if the property is transferred for some value to an innocent purchaser who does not know that the transaction is a fraudulent conveyance, that purchaser has superior rights to the extent of the consideration she furnished. An analysis of the specific provisions of the act follows: A deed need not be recorded to be valid and convey good title. An unrecorded deed is always valid to give good title to the grantee, at least in regard to the grantor. However, if a grantee does not properly and promptly record her deed, she may lose her title if her grantor later grants the land again to someone else ("the subsequent grantee"). d. Protection of bona fide purchasers other than under statutes Since most of the remedies for creditors to set aside conveyances and for holders of unrecorded instruments to try to obtain property from bona fide purchasers are equitable in nature, the bona fide purchaser may have the equitable defenses of laches, or unclean hands, in a suit which tries to take the property away from him. 2. Scope Of Coverage a. Recorded documents An individual records an instrument affecting land by filing it in the registry of deeds for the jurisdiction where the land lies. Usually the instrument must be notarized and a fee paid. The recorder's office then photocopies the instrument, binds it in a chronological volume, and indexes it according to the system employed in the jurisdiction (as explained below). b. Mechanics of title searches The only way a purchaser can verify the validity and quality of the grantor's title is by using indexes to find all of the documents making up the grantor's chain of title. How these indexes are used depends on the type of index utilized in the jurisdiction. 1) The tract index The tract index is used in some highly developed urban areas. All of the land in the jurisdiction is subdivided into small parcels known as "tracts." Each instrument is then indexed under the tract designation. The title examiner looks in the tract index under that particular tract, and finds references to the book and page numbers of all documents affecting that tract. She will then go to the record books to examine those documents.

62

MicroMash MBE In Brief: Real Property

2) The grantor-grantee index The most common type of indexing system is the grantor-grantee index, which is actually two separate indexes one arranged by the grantor's name and one arranged by the grantee's name. When an instrument is brought to the registrar's office, it is indexed under the name of the grantor in the grantor index, and under the name of the grantee in the grantee index. c. When a subsequent purchaser gets superior title 1) Subsequent purchaser must receive a valid deed A subsequent purchaser's deed cannot defeat a prior deed if the subsequent deed is forged or obtained from an incapacitated grantor. 2) Subsequent deed must have been purchased In order to qualify as a purchaser, the subsequent grantee must give "value." Value means more than nominal consideration, but does not necessarily mean a fair price. A deed given in exchange for cancellation of a debt is purchased. A deed given as security for a loan (i.e., a mortgage) is purchased, as long as the loan and the deed are simultaneous. 3) Subsequent purchaser usually must have purchased without notice In the overwhelming majority of jurisdictions (i.e., all jurisdictions but those which have a pure "race" system of recording, discussed below), a subsequent purchaser must be a bona fide purchaser to prevail that is, she must have purchased without notice of the prior, unrecorded conveyance. A subsequent purchaser can be notified of a prior purchase by the following three types of notice. (a) Constructive notice A promptly and properly recorded and indexed deed is constructive notice to all. An improperly recorded or indexed deed is usually held not to be constructive notice, unless it is actually discovered by a title examiner. (b) Actual notice (c) Inquiry notice A prospective purchaser is expected to inspect the land before buying. If the land is in the possession of a stranger, then the grantee has the duty to investigate further or he will not be a bona fide purchaser. 4) Subsequent purchaser may have to record first Whether the subsequent purchaser must record her deed to prevail over the prior deed depends on the type of recording system employed in the jurisdiction.

MicroMash MBE In Brief: Real Property

63

(a) Notice system A subsequent bona fide purchaser need not record to prevail over a prior deed, in a notice system of recording. Under a notice system, a subsequent purchaser takes good title by merely purchasing without notice of the prior conveyance. (b) Race-notice system Under a race-notice system, the subsequent bona fide purchaser must record first, in order to prevail over a prior grantee. (c) Race system A race system gives good title to whomever records first, regardless of whether that person had notice of prior conveyances. d. Interests affected by recording system Title obtained by adverse possession; easements obtained by implication, necessity, or prescription; and short-term leases need not be recorded. All other interests must be recorded. e. Doctrine of shelter Once a subsequent bona fide purchaser has achieved superior title as against a prior grantee, he can convey that priority to almost anyone, including someone who had notice of the prior conveyance. However, a purchaser who is not a bona fide purchaser cannot convey to a bona fide purchaser and then buy the property back to obtain the status of a bona fide purchaser. 3. Special Problems a. Instruments recorded out of chain of title 1) The deed recorded early estoppel by deed Estoppel by deed operates in the situation where an individual has conveyed an interest in land by a warranty deed, but does not actually own the land until after that conveyance. In such a situation, the title is transferred automatically to the grantee when the grantor acquires it. The question then becomes whether a subsequent purchaser can get good title from that same grantor. Because a person examining title will not find the conveyance from the grantor to the prior grantee in the chain of title, the majority rule is that a subsequent bona fide purchaser can get good title, despite the doctrine of estoppel by deed. 2) The instrument recorded late Since the registry of deeds records and indexes instruments by the date they are received at the registry, rather than by the date of the conveyance, it is possible that a valid conveyance might not be recorded until after the grantor has conveyed the property a second time. The majority of jurisdictions hold that a

64

MicroMash MBE In Brief: Real Property

purchaser does not have constructive notice of a late-recorded instrument that is out of the chain of title. b. Constructive notice A promptly and properly recorded and indexed deed is constructive notice to all. A properly recorded and indexed deed defeats all subsequent grantees. However, a subsequent purchaser need not actually check the registry of deeds to attain the status of a bona fide purchaser. If the prior deed is not recorded, the subsequent purchaser can be a bona fide purchaser, even if she did not check the record. c. Forged instruments Forged documents, even if properly recorded and notarized and relied upon by bona fide purchasers, are ineffective to affect title. Even if the forged document was back in the chain of title, so that the seller received an unforged deed when he took title, the doctrine of shelter is inapplicable, and the buyer who takes through a forged instrument will lose to the true owner. d. Transfers from corporations and by agents A deed signed by an officer of a corporation on its behalf or by an agent for a principal only conveys the interest of the principal if the agent is authorized to sign the document or if the purchaser is protected under some theory of apparent authority or estoppel. The common practice when a corporation conveys property is to place on record a corporate vote authorizing the officer to execute the deed on behalf of the corporation. Likewise, an agent executing a deed for an individual should place on record a power of attorney authorizing the action. Without these documents on record, there is a flaw in the title. e. Purchase-money mortgages In some jurisdictions, the seller who takes back a purchase-money mortgage may be entitled to additional protection, so that her mortgage is valid against a trustee in bankruptcy even though she did not immediately protect her security interest by recording the purchase-money mortgage. Likewise, if there is a statute in the jurisdiction which gives judgments or government liens an automatic attachment against after-acquired property, those statutes would not apply so as to give priority over purchase-money mortgages.

MicroMash BAR REVIEW BAR EXAM ALERTS AT-A-GLANCE REAL PROPERTY

I. OWNERSHIP
A. LIFE ESTATES Any remainderman (including a contingent remainderman) can enjoin a life tenant from committing waste. The holder of an interest after a qualified fee simple determinable or fee simple subject to a condition subsequent cannot enjoin the holder of the fee simple for waste. If there is a mortgage on property at the time it is conveyed to a life tenant and a remainderman, the life tenant is responsible for interest and current real estate taxes, and the remainderman is responsible for paying the principal. The life tenant is obligated to pay the interest due on the mortgage during his estate, to the extent that the property produces or can produce income.

B. COTENANCY
The granting of a mortgage by one joint tenant does not transform the tenancy into a tenancy in common in a state adopting the lien theory of mortgages, but it does in a title-theory state. If two joint tenants die simultaneously, the estate of each takes one half. A conveyance by all joint tenants of an undivided portion of their interest to a third party does not destroy the joint tenancy between them in the portion they retain.

C. FUTURE INTERESTS
If an interest is created in a third party in the same instrument as the prior possessory interest and can take in possession upon the termination of the prior interest, it is a remainder. A remainder is contingent if there is a condition precedent to its becoming possessory or the holders are unascertained.

MicroMash MBE In Brief: Real Property Exam Alerts

A remainder is vested if it can take whenever and however the previous estate terminates. The persons taking are ascertained and there is no condition precedent to their taking. If the named vested remainderman dies before the life tenant, her devisees take her interest if she leaves a will. Her heirs take if she dies intestate. If there is a class gift (a gift to children or grandchildren), afterborn members of the class can join the class until the class closes. If the grantor does not indicate otherwise, the class closes at the time any member of the class is capable of taking possession of the gift. If there is a remainder to the children of a living person and one or more children are in existence, then during the prior estate the interest in the children is "subject to open," or "subject to partial divestment." A future interest can be alienated prior to its becoming possessory. The interest in the "heirs" of a living person is contingent, because heirs cannot be determined until the person's death. D. THE LAW OF LANDLORD AND TENANT
1. TYPES OF TENANCIES

A periodic tenancy is terminated by notice (from either the landlord or the tenant) before the beginning of a rental period terminating the tenancy at the end of that period. A term for years is terminated at the end of the term without notice by either party. 2. ASSIGNMENT AND SUBLETTING A tenant is liable to pay rent during the term even if she has assigned her interest in the leasehold. An assignee is obligated to pay rent during the time that she possesses the leasehold property, but is not obligated to pay rent if she further assigns her leasehold interest. Only if the landlord, tenant, and assignee enter into a novation is the tenant no longer liable for the rent. When a tenant validly assigns a lease, the assignee and the landlord (or the landlord's successors) are bound by all of the covenants in the lease, such as a covenant to pay taxes or a covenant giving the tenant a right to purchase the property. A covenant against assignment does not prevent a tenant from subletting the property and vice versa.

MicroMash MBE In Brief: Real Propert Bar Exam Alerts

3. RENT A tenant who is denied the beneficial use of the property by the landlord and who moves out is not liable to pay the rent on the theory that she was constructively evicted. E. SPECIAL PROBLEMS 1. THE RULE AGAINST PERPETUITIES A child conceived but not born at the time of the commencement of the rule will be considered a life in being. The Rule Against Perpetuities does not apply to interests in the grantor (reversions, possibilities of reverter, or rights of entry for condition broken) or vested remainders. The time for determining lives in being when the conveyance is by will is at the death of the testator. The time for determining lives in being when the conveyance is by inter vivos deed is at the time of the conveyance. The time for determining lives in being when the conveyance is by irrevocable inter vivos trust is at the time of the conveyance. If the trust is revocable, it is at the time that the power to revoke terminates (either on the death of the testator or earlier if the power to revoke is relinquished). The Rule Against Perpetuities invalidates rights of first refusal which might not be exercised within the period of the rule. Under the common-law Rule Against Perpetuities, any person is irrebuttably presumed capable of having children until death. If the Rule Against Perpetuities invalidates the interest of one member of a class, the disposition to the entire class is invalid. If an interest is invalid because of the Rule Against Perpetuities, the disposition is construed with the invalid gift deleted. If there is no ultimate disposition in a will because of an invalid disposition, then the testator's heirs take. If there is an incomplete disposition by conveyance because of an invalidity, then the grantor or his heirs have a reversion. 2. ALIENABILITY A right of first refusal that only requires the seller to sell at market value is not an invalid restraint on alienation.

MicroMash MBE In Brief: Real Property Exam Alerts

A prohibition of a grantee's right to alienate property or a provision forfeiting an interest if the grantee attempts to alienate is invalid. The owner of property can by contract restrict her own right to alienate property.

II. RIGHTS IN LAND


A. COVENANTS The person who imposes a covenant that runs with the land cannot enforce that covenant against a subsequent purchaser unless he is still the owner of some land which was owned by him at the time he imposed the covenant. For a deed covenant to be enforceable against a subsequent owner of the property restricted, the original parties must have intended that it apply to subsequent owners, the subsequent owners must have actual or record notice of the restriction, and the subject matter of the restriction must touch and concern the land. The recording by a grantee of a deed containing a covenant running with the land is a satisfactory substitute for a memorandum signed by the grantee, and the defense that the covenant is unenforceable because of the Statute of Frauds is invalid. If the grantor consistently imposes similar covenants on a group of lots in a subdivision, he has created a common scheme and the owner of any lots burdened by the restrictions can sue the owner of any other lot to enforce the restrictions. If the grantor imposes similar covenants on a group of lots in a subdivision, she has created a common scheme and can be required to impose similar restrictions on all remaining lots in the subdivision, even if she has not promised in writing that she will do so. B. EASEMENTS AND PROFITS 1. EASEMENTS BY NECESSITY An easement by necessity or implication can only be created at the time of the division of a commonly owned parcel. An easement created by necessity ends when the necessity ends, but the end of the reason for creating an express easement does not terminate an express easement. An easement for light and air does not arise by necessity or implication.

MicroMash MBE In Brief: Real Propert Bar Exam Alerts

2. EASEMENTS BY PRESCRIPTION An easement by prescription need not be recorded to be effective against purchasers. The scope of an easement by prescription depends upon the scope of the use during the prescriptive period. Once an easement by prescription ripens with the passage of the appropriate time, continuous use of the easement is not necessary to maintain it. If use is with the permission of the owner, then no prescriptive rights accrue. If nothing is said, then the use is adverse. The adverse use of the property need not be exclusive to obtain an easement by prescription. 3. SCOPE OF EASEMENTS An easement is overburdened if it is used to benefit land other than the dominant estate. Non-use alone is insufficient to terminate an easement. An easement by grant must be in writing and signed by the grantor to be valid. An easement by grant must be recorded in order to bind bona fide purchasers of the benefited land. An appurtenant easement is automatically transferred with the dominant estate. A person cannot alienate her interest in an appurtenant easement separate from the alienation of the dominant estate, and the attempted alienation destroys the easement. The holder of an easement has the right to make repairs to property such as pipes and roads that are associated with the easement. A person cannot have an easement on land that she owns in fee simple. If the holder of the dominant estate acquires title to the servient estate, the easement is destroyed by merger and is not reinstated by a later conveyance of the servient estate. If the owner of an interest in land induces another person to rely substantially on the fact that the owner will not assert her property right, the owner will be prevented from later asserting that right by reason of estoppel.

4. PROFITS A PRENDRE
A person who holds an exclusive profit a prendre has the right to apportion it.

MicroMash MBE In Brief: Real Property Exam Alerts

A profit a prendre can be unlimited in time and is created in the same manner as an express easement.

C. FIXTURES A tenant has the right to remove personal property that he attached to the real
estate, even though the property might otherwise be characterized as a fixture (real estate). A person having an estate of uncertain duration (e.g., a life estate) who plants crops on that land can enter the land and remove the crops at the end of the growing season.

III. REAL PROPERTY CONTRACTS


A. CREATION AND CONSTRUCTION
A written brokerage-listing agreement is not a memorandum sufficient to satisfy the Statute of Frauds. Payment of the purchase price by the buyer is not sufficient part performance to take an oral agreement out of the Statute of Frauds. A written memorandum is necessary to change co-ownership from one form to another.

B. MARKETABLE TITLE
Restrictions imposed by zoning ordinances do not render title unmarketable. The fact that a buyer would be exposed to nonfrivolous litigation is sufficient to render title unmarketable. An adverse possessor whose title has not been confirmed in a judicial proceeding does not have marketable title. The most useful property device to control the use of land, which does not seriously affect the marketability of title, is usually an easement. However, if the marketability of title is not an issue, a qualified estate is the most certain form of control.

C. INTERESTS BEFORE CONVEYANCE 1. EQUITABLE CONVERSION


In a jurisdiction that recognizes equitable conversion, the risk of loss is on the buyer from the time that a binding purchase-and-sale agreement is executed. If a purchase-and-sale agreement is executed in a jurisdiction which recognizes equitable conversion, the buyer's interest is immediately an

MicroMash MBE In Brief: Real Propert Bar Exam Alerts

interest in realty and the seller's interest is immediately an interest in the proceeds (i.e., personalty). 2. CLOSING If time is of the essence, then the seller and the buyer must each be prepared to close on the date specified in the agreement, or each is in default.

D. RELATIONSHIPS AFTER CONVEYANCE


If a purchase-and-sale agreement is consummated by the delivery of a deed, covenants contained in the purchase-and-sale agreement are no longer enforceable, unless the agreement specifically states that they survive the closing.

IV. REAL PROPERTY MORTGAGES A. TYPES OF DEVICES A deed that is absolute on its face, but was intended only to convey a security
interest, can be reformed by a court into an equitable mortgage, as long as a bona fide purchaser does not now hold title. Other security devices, such as installment sales contracts, will be treated as a mortgage by a court. The usual procedures required for foreclosure and redemption will be applied.

B. TRANSFERS BY MORTGAGE; FORECLOSURE


A person who purchases at a mortgage foreclosure takes free of any encumbrances placed on the land subsequent to the mortgage that is being foreclosed. If the mortgagor sells property without paying off the mortgage and the buyer agrees to assume and pay the mortgage, the buyer is primarily liable and the mortgagor is only secondarily liable on the mortgage note. If the mortgagor sells property without paying off the mortgage and the buyer takes subject to the mortgage (i.e., without agreeing to pay the debt), the buyer is not liable for any deficiency judgment on the mortgage note, but can lose the property through foreclosure if she does not pay the mortgage. If a deed (rather than a mortgage) is given to secure the payment of a debt, the deed is an equitable mortgage. Parol evidence can be used to prove that the deed was intended to be a mortgage.

MicroMash MBE In Brief: Real Property Exam Alerts

If there is an equitable mortgage, the grantor-mortgagor can require a reconveyance of the property upon payment of the debt unless the granteemortgagee has conveyed the property to a bona fide purchaser. A mortgage foreclosure is not effective against a junior encumbrance unless notice is given to the holder of the encumbrance. A purchase-money mortgage (a mortgage from the grantee to the grantor to secure part of the purchase price) which is recorded immediately after the deed takes precedence over any other liens on the property.

V. TITLES A. ADVERSE POSSESSION


Open, notorious, and exclusive possession by one cotenant for the statutory period will not establish adverse possession unless the other cotenant was ousted at the beginning of that period. Joint possession with the rightful owner interrupts the adverse possessor's exclusive possession. Adverse possession must start all over again after the rightful owner leaves. If adverse possession commences against a competent adult, the subsequent ownership by a minor or a person with a disability does not interrupt the statutory period. It is possible to obtain title by adverse possession to airspace by projections from a structure that overhangs another's property. Transfer of ownership by the true owner does not interrupt the running of the period of adverse possession. Transfer of rights from one adverse possessor to a subsequent adverse possessor does not interrupt the running of the period of adverse possession.

B. CONVEYANCING BY DEED 1. VALID CONVEYANCE A forged deed is a nullity conveying no title.


The time of the transfer of title dates back to the time when the deed was delivered into a commercial escrow if the transaction is consummated. If the owner of property delivers a valid deed to a grantee, title is transferred to the grantee even though the deed is not recorded. The subsequent redelivery of the original deed from the grantee to the grantor does not retransfer title to the grantor. A new deed signed by the grantee is required for that retransfer.

MicroMash MBE In Brief: Real Propert Bar Exam Alerts

A grantee who receives a warranty deed is not required to be a bona fide purchaser to sue his grantor for a breach of a warranty. If the grantee of a validly delivered deed objects to owning the property, title has not been transferred because the grantee has not accepted the deed. 2. LAND DESCRIPTION AND BOUNDARIES Any description of property describing the property deeded with reasonable certainty is sufficient to make the deed effective. A reference to a survey or plan is sufficient, even if the survey or plan is not recorded. In cases of ambiguity, parol evidence is admissible to clarify the parties' intent. A deed that does not sufficiently describe the property, even after consideration of parol evidence, is invalid. Where there is a conflict, a description of the property by monuments prevails over a description of the property by distances. 3. COVENANTS OF TITLE A quitclaim deed contains no covenants. A warranty deed usually contains both present and future covenants. Future covenants run with the land, while present covenants do not. Thus, the grantee may sue only the immediate grantor for breach of a present covenant (such as the covenant against encumbrances). Present covenants are also breached, if at all, at the time of conveyance. The covenant of quiet enjoyment (a future covenant) is breached only when the grantee is ousted from possession of (even part of) the land. If a person grants an interest in land that he does not own to a grantee by a warranty deed, the grantee automatically becomes the owner of that interest as soon as the grantor acquires it, because of estoppel by deed.

C. PRIORITIES AND RECORDING


Recording is not required for an effective transfer of interests between the parties to the transaction. If the owner of property delivers a deed to a grantee and she records immediately, and the owner then delivers a deed of the same property to a subsequent grantee, the subsequent grantee loses because she has (constructive) notice of the prior deed.

If the owner of property deeds first to one grantee and then to a second grantee, the issue of which grantee prevails does not turn on whether the first grantee is a bona fide purchaser. That inquiry is relevant only with respect to the second grantee. A deed that is recorded out of order in the chain of title is not constructive notice to a subsequent bona fide purchaser.

10

MicroMash MBE In Brief: Real Property Exam Alerts

In a notice jurisdiction, the subsequent grantee cuts off the interest of the prior grantee who fails to record if the subsequent grantee is a bona fide purchaser. In a race-notice jurisdiction, the subsequent grantee cuts off the interest of the prior grantee who fails to record if the subsequent grantee is a bona fide purchaser and records prior to the first grantee.

MicroMash BAR REVIEW


MBE IN BRIEF TORTS
Table of Contents
1 1
1 1

I.

INTENTIONAL TORTS A. HARMS TO THE PERSON


I. 2. 3. 4. Assault Battery False Imprisonment Intentional Infliction Of Mental Distress Trespass To Land Trespass To Chattels Conversion Consent Privilege Immunity Necessity

B.

HARMS TO PROPERTY 1.
2. 3.

2 2 2 2 3 3 3 3 3
4 5

C.

DEFENSES TO CLAIMS FOR PHYSICAL HARMS


1. 2. 3. 4.

D. II.

DAMAGES RECOVERABLE FOR INTENTIONAL HARMS

5 5 5
6 6 6 7 7 8

NEGLIGENCE A. DUTY
1. 2. 3. Duty To Act Unforeseeable Plaintiffs Obligations To Control The Conduct Of Third Parties The Reasonable Prudent Person Rules Of Conduct Derived From Statutes And Custom

B.

STANDARD OF CARE
1. 2.

C. PROBLEMS RELATING TO PROOF OF FAULT INCLUDING RES 8 IPSA LOQUITUR


1. 2. 1. 2. 3. 4. Res Ipsa Loquitur Direct Evidence Of Negligence "But For" Test Substantial Factor Test Harms Traceable To Multiple Causes Apportionment Between Multiple Defendants 9 9

D. PROBLEMS RELATING TO CAUSATION

10
10 10 10 11

E. LIMITATIONS ON LIABILITY AND SPECIAL RULES OF 11 LIABILITY


1. Proximate Cause 11

2.
3. 4. 1. 2. 3. 1. 2. 3. 4. 1. 2.

Claims Against Owners And Occupiers Of Land

12

Claims For Mental Distress Not Arising From Physical Harm; Other 13 Intangible Injuries 14 Claims For Pure Economic Loss

F. LIABILITY FOR THE ACTS OF OTHERS


Joint Liability Vicarious Liability Master-Servant Relationship Contributory Fault Assumption Of The Risk Imputed Negligence Immunity Damages Recoverable Collateral Source Rule

14
14 15 15

G. DEFENSES TO NEGLIGENCE ACTIONS

15
15 16 16 16

H. DAMAGES IN NEGLIGENCE ACTIONS

17
17 17

III. STRICT LIABILITY


A. B. ABNORMALLY DANGEROUS THE RULE OF RYLANDS V. FLETCHER
1. 2. Trespassing Animals Personal Injury Caused By Wild Animals Or Domestic Animals

17 17 17 17
17 17

C. KEEPING ANIMALS

IV.

PRODUCTS LIABILITY
A. B. NEGLIGENCE STRICT LIABILITY
1. 2. 3. Individuals Protected When Strict Liability Is Applicable Defenses Misrepresentation Under Restatement 402B Warranties Under The Uniform Commercial Code

18 18 18
19 19 19

C.

MISREPRESENTATION AND WARRANTY THEORIES


1. 2.

20
20 20

V.

OTHER TORTS
A. NUISANCE
1. 2. 3. 4. Interference With Use And Enjoyment Reasonableness Of Conduct Relief Public Nuisance

20 20
20 21 21 21

B. CLAIMS BASED UPON DEFAMATION AND INVASION OF 21 PRIVACY


1. 2. 1. 2. Defamation Invasion Of Privacy Deceit Negligent Misrepresentation 21 22

C. CLAIMS BASED UPON MISREPRESENTATIONS

22
22 23

D. CLAIMS BASED UPON INTENTIONAL INTERFERENCE WITH 23 BUSINESS RELATIONS


1. 2. 1. 2. Interference With Contractual Relations Interference With Advantageous Relations Elements Application To Civil Proceedings 23 24

E. CLAIMS BASED UPON MALICIOUS PROSECUTION

24
24 24

VI. IMPACT OF INSURANCE, WORKERS' COMPENSATION, AND 24 "NO-FAULT" SYSTEMS A. B. AREAS AFFECTED BY INSURANCE TYPES OF INSURANCE
Hi

24 24

C. D.

WORKERS' COMPENSATION SYSTEMS "NO-FAULT" SYSTEMS

25 25

iv

TORTS
I. INTENTIONAL TORTS
A. HARMS TO THE PERSON 1. Assault
In an action for assault, plaintiff must prove: (1) that defendant placed plaintiff in actual apprehension of an immediate harmful or offensive touching; (2) that defendant intended either to bring about the offensive or harmful touching, or the apprehension thereof (as, for example, by threatening plaintiff with an unloaded gun); and (3) that plaintiff did not consent. The defendant must have had the apparent present ability to bring about such a contact. A conditional threat of immediate harm constitutes an assault if the condition is one which defendant is not privileged to impose under the circumstances.

2. Battery a. Elements
In an action for battery, plaintiff must prove that defendant: (1) intended to cause plaintiff to suffer a harmful or offensive touching, or to create apprehension in plaintiff of an imminent harmful or offensive touching; (2) actually caused such a harmful or offensive touching; and (3) did so without plaintiffs consent. Substantial certainty that the contact will result satisfies the intent requirement. Intent to bring about the touching does not necessarily mean intent to harm the plaintiff. The doctrine of transferred intent applies to battery as well as to assault.

b. Lack of consent
Whether the action is for assault or battery or both, plaintiff must prove lack of consent. If the plaintiff has not consented, expressly or impliedly (by conduct or custom), the touching is unpermitted, and defendant is liable even if she acted with the best of motives (e.g., unlawful extension of surgery). Consent is not purely a subjective matter; the defendant is entitled to act upon reasonable appearances in implying plaintiff's consent. Consent induced by fraud is not effective if it goes to the essence of the touching; it does, however, bar the action if defendant's fraud related solely to a collateral matter. Likewise, consent induced by defendant's misrepresentation, even though not intentional, does not bar the action if it goes to the essence of the touching. Normally, consent bars the action even though the consent was to an unlawful act. There are, however, two exceptions: (1) a minority of jurisdictions do not recognize consent to a breach of the peace (e.g., fist fighting), and allow the participants to sue one another for assault and battery; and (2) where criminal law invalidates consent in order to protect a class of persons against their own lack of judgment (e.g., statutory rape), such invalidation will likewise apply in a civil action.

MicroMash MBE In Brief: Torts

The touching may not go beyond the scope of consent given unless, in a medical situation, the doctor extends an operation to protect the life or health of the patient. 3. False Imprisonment In an action for false imprisonment, plaintiff must prove that defendant: (1) confined the plaintiff within a limited area from which there was no reasonable and apparent means of escape merely impeding plaintiffs movement in a certain direction is insufficient; (2) acted intentionally; and (3) did not have plaintiff's consent. There is a split of authority on the issue of whether plaintiff must be aware of his confinement at the time it takes place; the Restatement (Second) of Torts requires awareness. Awareness is not required if plaintiff suffered harm from the confinement. 4. Intentional Infliction Of Mental Distress In an action for intentional infliction of mental distress, plaintiff must show that defendant's conduct caused plaintiff severe emotional distress. Slight distress, embarrassment, or humiliation is insufficient. The more recent cases do not require physical harm resulting from the distress, although if it occurs, such physical harm is also compensable. Plaintiff must also prove that defendant acted intentionally. Reckless disregard for the plaintiffs rights will satisfy the requirement of intent. Common carriers, innkeepers, and public utilities are strictly liable to their patrons for mental distress caused by highly offensive insults.

B. HARMS TO PROPERTY
The torts involving intentional harm to property are: (1) trespass to land; (2) trespass to chattels; and (3) conversion. 1. Trespass To Land a. Elements of action In an action for trespass to realty, plaintiff must establish: (1) that she had possession or the right to possession of the land in question; (2) that defendant either herself made an entry onto the land, or projected an object onto the land, and (3) that defendant did so intentionally. In trespass to property, consent is an affirmative defense. Except where defendant is the real owner or the person having the right to possession, the plaintiff in actual possession (even though wrongfully) is entitled to prevail. While defendant's entry must be intentional, she need not know she is committing a trespass; a good-faith mistake does not excuse the entry, even if it is reasonable. She is also liable for any harm caused by her trespass to the person or property of the possessor or members of the possessor's family; a showing of negligence is not required. b. Nuisance distinguished While trespass to land protects the plaintiff's possessory interest in the land, the action for nuisance protects the plaintiffs use and enjoyment of his property. Recent cases in some jurisdictions have rejected the distinction between nuisance and

MicroMash MBE In Brief: Torts

trespass that is based solely upon whether the invasion is by tangible items (e.g., stones) or intangible items (e.g., chemicals). See section V(A) on nuisance.

c. Damages
Nominal damages may be recovered for trespass, even though no actual harm is done to the property.

d. Privilege
A person may be privileged to enter the land of another to protect her own person or property; the privilege is, however, incomplete, in that she must pay damages for any actual harm that she causes. The state may take a person's property for the public good under its power of eminent domain; it must, however, pay the owner for its fair value. Under the police power, the state may destroy or limit the use of property that represents a danger to the public safety, health, or welfare. In such case, no damages are awarded to the owner.

2. Trespass To Chattels
Trespass to chattels and conversion both involve an interference with the plaintiff's possessory interest in personal property. If the dominion over the property is not complete, the interference constitutes a trespass to the chattel, in which case damages are for any actual damage to the property or actual loss of its use.

3. Conversion
Where defendant's interference is sufficient to constitute an act of dominion over the property, the plaintiff in an action for conversion may recover its fair market value. In the alternative the plaintiff may sue for return of the goods taken in an action of replevin.

C. DEFENSES TO CLAIMS FOR PHYSICAL HARMS 1. Consent


Since lack of consent is an element in many intentional torts, the issue of consent is discussed with the discussion of the tort itself.

2. Privilege
There are two basic affirmative defenses to the torts of intentional harm to the person: privilege and immunity. Privilege justifies defendant's conduct so that no tort occurs.

a. Self-defense Self-defense is a privilege. A person is entitled to use reasonable force to prevent


injury to himself. Deadly force cannot be used to repel force of a nondeadly nature. A person may not use deadly force even to repel deadly force, if there is a reasonable means of escape. Most states, however, hold that this retreat doctrine does not apply to a person while in his home or place of business.

MicroMash MBE In Brief: Torts

b. Defense of a third person

A person may use reasonable force to protect a third person. A majority of jurisdictions, however, take the view that the actor takes the risk whether the person she is defending would be privileged to defend herself in like manner, and is thus liable in case of mistake.
c. Defense of property Reasonable force may be used to recapture property, provided the original taking was wrongful (as opposed to merely a wrongful refusal to return), and there is fresh pursuit. The force used solely to recover or protect property may not be deadly; traps such as spring guns, likely to cause death or serious injury, may not be used. Most jurisdictions grant a privilege to a shopkeeper to use reasonable force to detain a person for a reasonable period upon reasonable grounds that such person has stolen property. The privilege ends when the goods are recovered; the shopkeeper cannot continue to hold such person in order to obtain a signed confession. d. Arrest without a warrant In the absence of a statute restricting or expanding the common law, a peace officer may arrest without a warrant: (1) a person who has committed or is committing a felony; (2) a person he reasonably suspects of having committed a felony; or (3) a person who has committed or is committing in the officer's presence a misdemeanor which is a breach of the peace. Reasonable grounds as to the commission of a misdemeanor are insufficient. A private person may arrest without a warrant for: (1) a felony, only if a felony has in fact been committed, and he has reasonable grounds to believe that the person arrested committed the felony or (2) a misdemeanor constituting a breach of the peace committed in the presence of the arresting person. e. Discipline of children Parents are privileged to use reasonable force to discipline their children. In the absence of a statute to the contrary, a teacher may use reasonable force to discipline a child. 3. Immunity Immunity does not make defendant's conduct nontortious, but does bar an action for such conduct on policy grounds. a. Interspousal and parent-child The majority of jurisdictions have partially or totally abolished intrafamily immunity, principally on the basis that the presence of liability insurance precludes any argument that such suits destroy family harmony. Immunity does not exist between siblings, nor does it generally apply against a spouse's employer being sued under respondeat superior.

MicroMash MBE In Brief: Torts

b. Charitable immunity The doctrine of charitable immunity has been abolished in practically all states, although some states by statute limit the amount recoverable against a charity. c. Governmental immunity The federal government cannot be sued in tort except as allowed under the Federal Tort Claims Act. Under the act, the federal government is liable only if a private person would be liable under the law of the place where the tort occurred. No recovery is allowed for intentional torts, for the performance or nonperformance of discretionary functions, or on a strict liability theory. In most jurisdictions, a state may be sued for torts only to the extent permitted by statute. This doctrine is, however, eroding. Further, some states have enacted legislation similar to the Federal Tort Claims Act, allowing suits against the state. Municipalities are generally held liable for torts performed while carrying out proprietary functions, but not while carrying out governmental functions. However, as with states, the doctrine of municipal immunity is eroding. d. Immunity of public officials Judges and high governmental officials enjoy an absolute immunity from tort liability. Lower officials are protected when performing quasi-judicial, legislative, or discretionary responsibilities, but not when performing purely ministerial acts. 4. Necessity The defense of necessity is available when the defendant's tortious conduct was necessary to avoid greater harm.

D. DAMAGES RECOVERABLE FOR INTENTIONAL HARMS


Compensatory damages for intentional harm to the person, unlike negligence, do not require proof of actual injury. Compensatory damages may be recovered for the humiliation, indignity, and injury to feelings resulting from defendant's conduct. Where defendant's conduct is outrageous, punitive damages are allowed in many states. Jurisdictions differ on whether punitive damages are covered by a liability insurance policy. A majority of jurisdictions allow assessment of punitive damages against an employer under respondeat superior for the outrageous conduct of its servant, even though the employer did not order or condone the conduct; a minority of states, and the Restatement, disagree.

II. NEGLIGENCE
A. DUTY
The initial step in a negligence action is to establish the duty owed plaintiff by defendant. Duty may involve either: (1) a duty to act or (2) a duty not to act in a negligent or reckless manner.

MicroMash MBE In Brief: Torts

1. Duty To Act a. No duty to act generally


Absent some special relationship between the parties, the law does not impose a general duty to render aid to another.

b. Creation of duty
Some special relationships upon which courts have imposed a duty to aid include: (a) employer-employee; (b) innkeeper-guest; (c) common carrier-passenger; (d) business invitor-invitee; (e) social invitor-invitee. A duty to act has also been imposed upon the person in control of the instrumentality which has caused the harm or is causing the harm, even though there was no initial negligence on the part of the person in control. Also, a person who has put into public distribution a product that she subsequently discovers is defective has a duty to warn.

c. Good Samaritan rule


If a person, having no duty to act, does undertake to act, he must exercise reasonable care. This is sometimes called the "Good Samaritan Rule." Most states have enacted statutes making the "Good Samaritan Rule" inapplicable to doctors and nurses who render emergency treatment at the scene of an accident.

2. Unforeseeable Plaintiffs
An individual has a duty to avoid negligent conduct which runs not only to those persons whom the plaintiff can foresee will be harmed if she does not fulfill her duty but also to those unforeseen plaintiffs who are within the scope of the risk of being harmed by her conduct. A further discussion of this issue is contained in the section dealing with proximate cause. One who has acted negligently is liable not only to direct victims of her negligence, but also to anyone who undertakes to rescue persons in peril from the defendant's negligence. A defendant who endangers only herself is also liable for her negligence to anyone who tries to rescue her from her own misconduct.

3. Obligations To Control The Conduct Of Third Parties a. Owners of automobiles


The owner of an automobile is not generally liable for the negligence of another person driving the automobile. Some states have, however, enacted statutes holding the owner liable up to a certain amount (consent statutes), on the theory that the owner is able to obtain liability insurance. Such statutes have been held to replace the family purpose doctrine. The owner of an automobile is not generally liable for the negligence of the operator simply on the basis of the presence of the owner in the automobile. Where, however, the owner is present in his automobile which is being driven in a negligent manner, he may be held for his own negligent failure to take steps to prevent the automobile from being so operated.

MicroMash MBE In Brief: Torts

b. Torts of infants Infants are liable for their tortious conduct provided that they have the capacity to commit the particular tort. Under the common law, parents are not vicariously liable for the torts of their children. However, a parent may be liable for his or her own negligence in entrusting a child with a dangerous instrumentality which results in injury to another, or where the parent is aware of the child's propensity for violent conduct and fails to take reasonable steps to discipline the child or warn potential victims. B. STANDARD OF CARE 1. The Reasonable Prudent Person a. The general objective standard Absent some relationship between the parties, which may create a greater or lesser duty, a person owes others the duty of exercising reasonable care. The defendant's conduct is measured by the objective standard of a reasonable person under the circumstances. What is reasonable depends upon a great variety of factors. Basically, a comparison is made between the burden of avoiding the occurrence of harm with the probability that harm will occur and the gravity of that type of harm if it does occur. The defendant is not excused simply because she did the best that she could, considering her knowledge, experience, and intelligence; she is held to have the knowledge, experience, and intelligence of a reasonable person. b. Emergencies An actor confronted with a sudden emergency will not be required to act as if he had had adequate time to weigh alternatives and decide on the most reasonable course of action. While an objective standard of reasonableness should still be applied in such cases, the standard is one of a reasonable person under all the circumstances and the emergency will be considered as one of the circumstances. c. Minors This rule does not apply to children. A child's conduct will be measured by that of a person of the child's age, knowledge, and experience. Where the child's activity involves the operation of an automobile, airplane, or powerboat, she will be held to an adult standard. d. Physically and mentally impaired individuals While the mental deficiencies of the adult defendant are not considered in determining the reasonableness of his conduct, the physical infirmities of the defendant are considered. Thus, the defendant is not responsible for a sudden, unforeseeable heart seizure that causes him to lose control of his automobile. On the other hand, if the defendant knows that he is subject to "spells" which render him unconscious, he is liable in negligence if, while driving, he becomes unconscious and loses control of the automobile.

MicroMash MBE In Brief: Torts

e. Greater knowledge than ordinary reasonable person A person who has greater intelligence, knowledge, or experience than the reasonable person will be held to that higher standard. 1) Physicians For years, the standard of care for a physician was whether her conduct measured up to the standard of care and skill ordinarily possessed by others in her profession practicing in the same community as the defendant. This has changed in recent years. Some courts use a "same or similar community" standard, while others have completely abandoned a community standard by holding the physician to the standard of the average qualified practitioner nationwide. 2) Other professionals Other professionals such as lawyers, architects, and engineers are measured by the care and skill possessed by other members of their profession. f. Automobile operator In some states, the duty of the operator of a motor vehicle to his social guest in the car is something less than ordinary care; this is either by statute (guest statutes) or common law. Many courts have recently held that guest statutes are unconstitutional as a denial of equal protection. 2. Rules Of Conduct Derived From Statutes And Custom a. Violation of criminal statute Jurisdictions differ on the effect to be given to defendant's violation of a criminal statute. A majority hold that the unexcused violation of a criminal statute is negligence per se; a minority hold that it is evidence of negligence. Even in negligence per se jurisdictions, the courts generally consider whether defendant's conduct, although technically in violation of the criminal law, was nevertheless excused. In any case, no effect will be given to the violation unless the harm to plaintiff was of the type that the statute was intended to prevent. b. Rules of conduct derived from custom Proof of a company safety rule and its violation by an employee is admissible as evidence of negligence. Proof of an industry-wide standard or custom is admissible on the issue of the appropriate standard of care, but is not conclusive on that issue. Even though the defendant acted consistently with the standard of care currently applied in the community, the defendant may be negligent because the community standard was set too low.

C. PROBLEMS RELATING TO PROOF OF FAULT INCLUDING RES IPSA LOQUITUR


Defendant's breach of duty may be proven by circumstantial evidence (res ipso loquitur), or direct evidence, which includes violation of a criminal statute.

MicroMash MBE In Brief: Torts

1. Res Ipsa Loquitur The doctrine of res ipsa loquitur permits the fact finder to find the defendant negligent even though the plaintiff has produced no direct evidence of negligence. The plaintiff, however, must establish by circumstantial evidence that the greater likelihood is that the harm to the plaintiff resulted from defendant's negligence, rather than from some other cause. Plaintiffs evidence should warrant a finding that defendant at some time had control of the instrumentality which caused the harm, that the harm was such as does not ordinarily occur in the absence of negligence, and that the harm was not the result of the conduct of plaintiff herself or of some person other than defendant. A res ipsa case merely justifies an inference of negligence. Most jurisdictions permit res ipsa loquitur to be used in medical malpractice cases, but in most instances it must be accompanied by expert testimony. 2. Direct Evidence Of Negligence Obviously, the plaintiff may establish the defendant's breach of duty by the introduction of direct evidence of negligence even apart from violations of criminal statutes. The major problems in this area are: (a) the need in some cases for expert testimony; (b) the sufficiency of the evidence to avoid a directed verdict; (c) the relevancy or materiality of the evidence on the issue of negligence. a. Expert evidence on professional standards or custom in the trade Expert testimony is necessary where plaintiffs claim of negligence involves a matter of such a nature that a jury cannot, on the basis of its own common knowledge and experience, determine whether reasonable care has been exercised. While most medical malpractice cases require expert testimony, there are some instances where lay persons are capable of determining negligence based upon common knowledge and experience. Evidence that defendant's conduct was consistent with the custom in the trade or calling is merely evidence of due care; it does not establish due care. b. Sufficiency of the evidence The majority of negligence cases involve either a claimed defect or unsafe condition in the premises owned or controlled by the defendant, or the claimed negligent operation of an automobile by the defendant. 1) Defect in premises With respect to premises, the mere existence of a slight defect (e.g., a slight ridge in a carpet) does not establish negligence, since the appropriate standard is generally reasonable care and not perfection. The issue is whether the defendant's conduct has created an unreasonable risk of harm. Many factors are involved in that determination, including expense. With respect to even ordinary defects, the defendant is not an insurer. He is given a reasonable opportunity to discover defects and make repairs. With respect to foreign substances on a floor or step, it is incumbent upon the plaintiff to prove either that the defendant or his

10

MicroMash MBE In Brief: Torts

employees had actual knowledge of the presence of the foreign substance and failed to remove it, or that they had a reasonable time to discover the presence of the foreign substance and remove it, and failed to do so. As mentioned previously, whether a defect or foreign substance is involved, there is no duty on the defendant to warn the plaintiff of obvious conditions. 2) Operation of motor vehicle With respect to the operation of a motor vehicle, the alleged negligent act may be one of many types: excessive speed, inattention, mechanical defects, etc. The mere fact that the defendant's automobile struck the plaintiff or the plaintiff's automobile does not of itself establish negligence. While intoxication may be evidence of negligence, it does not generally ipso facto establish negligence, in the absence of a showing of some unreasonable conduct in the driving of the motor vehicle. An owner is not an insurer that her vehicle is in safe mechanical condition. She is required to exercise ordinary care to keep her vehicle in a reasonably safe condition.

D. PROBLEMS RELATING TO CAUSATION


An essential element of plaintiff's negligence action is that defendant's negligent act caused plaintiff's injury. This is referred to as actual or factual cause. Sometimes, particularly in medical malpractice cases, expert testimony is needed to show actual cause. Such expert testimony must indicate a probability (and not merely a possibility) that defendant's negligence resulted in plaintiff's harm. 1. "But For" Test Except in some cases involving joint tortfeasors, defendant's conduct cannot be considered the cause of plaintiff's harm if such harm would have occurred even had the defendant not so acted. The question is not, however, whether a like harm or similar harm would have occurred without defendant's negligent act, but whether the same harm would have occurred. 2. Substantial Factor Test In most cases involving joint tortfeasors, plaintiff's injury is the result of the combined negligent acts of two or more tortfeasors, and the harm would not have resulted from either act standing alone. Occasionally, however, there are situations where either act alone would have caused the harm. While the "but for" rule of actual causation does not literally apply in these latter situations, the courts treat each defendant's act as the cause of plaintiff's harm, despite the fact that the harm would have occurred anyway (i.e., from the other defendant's act). 3. Harms Traceable To Multiple Causes a. Concurrent causes Where the plaintiff is injured from the negligent act of co-defendants who shared a common duty toward the plaintiff or acted in concert with each other, the "but for" test can usually be readily applied to make the defendants jointly and severally liable

MicroMash MBE In Brief: Torts

11

for the full amount of the plaintiffs damages. Occasionally, however, the plaintiffs harm may result from two independent acts or events, either alone being insufficient to cause the plaintiffs harm. The courts have taken the position that where the defendant's negligent act unites with another event, the defendant's negligence will be considered the cause of at least part of the harm. If the other human agent is another negligent defendant, the defendants will be held jointly and severally liable. Also, where two defendants have acted negligently, although independently, but due to the circumstances it is impossible to determine which defendant caused the plaintiffs injury (as where two defendants fire shotguns in the general direction of the plaintiff but only one hit the plaintiff), the courts have treated the defendants as joint tortfeasors, even though in actuality only one of the defendants has caused the plaintiffs injury.

b. Indivisible injury caused by multiple defendants


Occasionally, two or more defendants commit negligent acts, each probably causing injury to the plaintiff, but it is impossible under the circumstances for the plaintiff to show which of his injuries were caused by each defendant. Where the triers of fact cannot ascertain the amount of damage each wrongdoer has inflicted, they are authorized to hold all of the wrongdoers liable for all of the plaintiffs injuries (i.e., joint and several liability), on the ground that each has participated in the infliction of a single indivisible injury.

4. Apportionment Between Multiple Defendants


Sometimes the negligent acts of two or more defendants cause separate injuries to plaintiff, but it is impossible under the circumstances to show which injuries were caused by each defendant. Most courts, in this situation, will hold the defendants jointly and severally liable for all the injuries. A few states hold that the court should instruct the jury to make a rough apportionment, or, if such apportionment is not possible, the jury should apportion the damages equally among the defendants.

E. LIMITATIONS ON LIABILITY AND SPECIAL RULES OF LIABILITY 1. Proximate Cause


Courts are less likely to apply the rules of proximate cause strictly in cases of intentional torts, rather than negligence. The area of proximate cause may be divided into two categories: (1) harm within the risk, and (2) persons within the risk.

a. Harm within the risk


Plaintiff must prove that she suffered personal injury or tangible property damage as a legal result of the defendant's action.

1) Foreseeability
A majority of courts hold that a defendant is liable only for the foreseeable harm that results from his conduct. However, the defendant can be held liable for even unusual harm if some harm was foreseeable.

12

MicroMash MBE In Brief: Torts

2) Sequential harm and intervening acts Some courts adopt the view that a defendant is liable for all of the harm which follows in unbroken sequence from her negligent act. Thus, the most common proximate cause problem is whether the intervening negligence of a third party, which combines with the negligence of defendant to cause plaintiffs injury, will legally break the chain of proximate cause. In a foreseeability jurisdiction, the defendant will be held liable for the entire harm (usually jointly and severally with the second actor) if the conduct of the second actor was reasonably foreseeable. The conduct of the second actor may be reasonably foreseeable in some circumstances even though it is intentional; in fact, even criminal conduct may, under some circumstances, be foreseeable. In a jurisdiction holding defendant liable for the direct unbroken consequences of her act, defendant is liable even though the second actor's act was not foreseeable, as long as it was not of such a nature as to take over as the efficient cause of the harm. b. Persons within the risk 1) Foreseeable plaintiff the Palsgraf case The two views of proximate cause from the concept of persons within the risk are set out in the Palsgraf case. In Palsgraf the majority held that defendant is liable to plaintiff only if plaintiff was a person to whom harm was reasonably foreseeable when defendant acted; the act must have created an unreasonable risk of harm to plaintiff. The dissenting opinion held that a person should be entitled to recover if in fact injured by defendant's negligent act, even though defendant's act initially created no reasonable risk of harm to plaintiff The only restriction on liability is that the harm may not be too remote in time or space, or limited by public policy considerations. 2) Rescue doctrine A person who negligently places another in a position of danger is liable to a third person who comes to the aid of the victim and is injured in so doing, unless the attempted rescue was foolhardy. Assumption of risk is no defense. This is sometimes referred to as the rescue doctrine. It also applies where defendant negligently places himself in a position of danger, and plaintiff is injured while attempting a rescue. 2. Claims Against Owners And Occupiers Of Land a. By business invitees The duty of a business invitor to an invitee is ordinary care to keep the premises in a reasonably safe condition; this includes the duty to make reasonable inspections. The duty extends also to one accompanying a business invitee. There is a difference of judicial opinion whether one entering a business establishment for a purpose other than one financially benefiting the owner (e.g., to make a telephone call or to get out of the rain) is a business invitee or a mere licensee, but the Restatement does not require a business purpose for an invitee.

MicroMash MBE In Brief: Torts

13

The duty of ordinary care owed to a business invitee does not apply when the plaintiff goes into areas of the premises to which the invitation did not extend. The duty of ordinary care to a business invitee extends to using reasonable care to prevent injury to the invitee by the acts of third persons. While a few recent cases hold to the contrary, most jurisdictions treat police and firefighters as licensees rather than invitees. Some states hold that they are invitees when they come on the property under the same circumstances as other members of the public and to a part of the premises that is normally open to the public.

b. By social guests and licensees


Most states hold that the duty owed to a social guest or licensee does not require the landowner to make reasonable inspections of the premises; she is only required to make safe those unsafe conditions known to her, or to warn the social guest or licensee of such conditions. A few states have recently abolished all distinctions among business invitees, social guests, and licensees, holding the owner to a duty of reasonable care to all. Most states hold that with respect to active conduct (as opposed to the condition of the premises), the duty of the landowners to social guests and licensees is ordinary care.

c. By trespassers
The duty of a landowner to a trespasser is, in most jurisdictions, to avoid either gross negligence or willful and wanton conduct. Exceptions to this rule include infant trespassers (sometimes referred to as the "attractive nuisance doctrine") and technical trespassers. Also, the landowner owes a duty of ordinary care with regard to active conduct toward discovered trespassers.

3. Claims For Mental Distress Not Arising From Physical Harm; Other Intangible Injuries a. Negligent infliction of emotional distress
Most jurisdictions allow recovery for severe mental distress negligently inflicted, and for any physical harm resulting from such mental distress despite the absence of any physical impact, provided plaintiff was within the zone of physical danger from defendant's negligence, and was thus fearful for his or her own safety. Many jurisdictions also allow recovery for mental distress resulting from witnessing injury to a close relative (e.g., a child).

b. Prenatal injuries
Most jurisdictions allow recovery for prenatal injuries whether or not resulting in death, provided that the fetus is born alive. There is a split of authority whether a wrongful death action is allowed when the fetus is stillborn. A majority of states allow recovery provided the injury took place when the fetus was viable (capable of living apart from its mother approximately 24 weeks).

14

MicroMash MBE In Brief: Torts

c. Wrongful death 1) Damages standard for liability In almost all states, recovery for wrongful death is statutory, recovery at common law having been denied. While a few states base recovery on the loss to the decedent's estate, most base and measure damages on the loss to survivors for such elements as loss of support, loss of services and companionship, and, in the case of a spouse, loss of consortium. The standard for liability in wrongful death is generally derivative; that is, recovery may be had if the decedent, had he survived, could have recovered damages for his injury. Therefore, substantive defenses which would have been available against the decedent are available in the death action. Also, the contributory negligence of the survivor-plaintiff(s) will preclude recovery. 2) Survival statutes Unlike the death statute, under which recovery is had for the decedent's death, survival statutes preserve certain claims which decedent had at the time of her death. Thus, where decedent's death resulting from defendant's negligence is not instantaneous, recovery for such elements as pain and suffering and medical expenses are recoverable under a survival statute for the benefit of the estate, whereas generally, recovery for decedent's death under the death statute goes directly to the survivors. 4. Claims For Pure Economic Loss While a person may be liable in negligence for injury to the plaintiffs person or physical property, it is generally held that, except for wrongful death actions, proximate cause does not extend to economic harm suffered by the plaintiff as a result of injury to another individual's person or property. Thus, an employer may not normally recover in negligence for injury to an employee. Nor may a person recover in negligence for lost wages against a defendant who negligently burned down the plaintiffs place of employment. Nor may an insurance company recover in negligence the amount which it had paid out on a life insurance policy against the defendant who had negligently caused the death of the insured. In all these situations, however, if the plaintiff could establish that the defendant acted with the intent to injure the plaintiff, recovery may be had on a theory of intentional interference with contractual or advantageous relations.

F. LIABILITY FOR THE ACTS OF OTHERS


1. Joint Liability a. Concurrent negligent acts If two or more defendants, by their concurrent negligent acts, bring about harm to the plaintiff, and it is not possible to separate the portions of the harm resulting from each act, the defendants may be held jointly liable. The effect of joint liability is that each defendant is liable for the entire harm, although obviously plaintiff can collect only up to the amount of the judgment. If a joint tortfeasor pays more than his pro rata

MicroMash MBE In Brief: Torts

15

share of the judgment, he has the right of contribution against the other joint tortfeasor. In most jurisdictions, proration is determined by the number of joint tortfeasors involved, and not upon relative degrees of fault. b. Contribution and indemnification Contribution should be distinguished from indemnification. Contribution involves the sharing of the financial burden among joint tortfeasors. Indemnification involves recovery by a nonwrongdoer who has incurred a judgment against the actual wrongdoer or the one responsible for the harm. It seeks the full amount of the judgment rather than a part of it. For example, under respondeat superior a master may obtain indemnification from the negligent servant. 2. Vicarious Liability Vicarious liability is based upon a relationship between the defendant and the one committing the tort, as, for example, a master's liability for the tort of her servant. There is no vicarious liability on the part of a parent for the tort of a child, nor is there vicarious liability on one spouse for the tort of the other. 3. Master-Servant Relationship A master is liable for the tort of his servant if the tort occurred while the servant was acting within the scope of his authority. Except where the activity is inherently dangerous, an employer is not liable for the tort of an independent contractor. The test of a master-servant relationship is whether the employee is subject to the employer's right to control the details of the work performed. It is not necessary to prove that defendant paid the employee to perform the services in order to establish a master-servant relationship. Under the borrowed servant rule, the defendant may be responsible for the servant's tort where defendant controlled the servant in carrying out the work, even though the servant was paid by a third person. The liability of a master for the servant's torts under respondeat superior includes intentional torts, even where expressly prohibited by the master, where the intentional tort was committed by the servant while advancing the master's business. A master is not liable for a servant's tort where the tort occurred while the servant had substantially deviated from his authorized route. Where the detour is slight, that is, reasonably foreseen by the master, the master is liable. General partners and persons engaged in a joint enterprise for profit are vicariously liable for the torts committed by each other in conducting the business. The rule does not generally apply to joint enterprises of a noncommercial nature (i.e., social).

G. DEFENSES TO NEGLIGENCE ACTIONS


1. Contributory Fault a. Contributory negligence The defense of contributory negligence means that plaintiffs own negligence contributed proximately to her injury. While most jurisdictions have adopted rules of comparative negligence, contributory negligence still operates as a complete defense in some jurisdictions.

16

MicroMash MBE In Brief: Torts

b. Last clear chance doctrine The Last Clear Chance Doctrine was designed to mitigate the harshness of the defense of contributory negligence as a complete bar to recovery. Thus, most states which adopted comparative negligence have abolished the Last Clear Chance Doctrine. Under the Last Clear Chance Doctrine, plaintiff, even though guilty of contributory negligence, could recover provided: (1) plaintiff was in a position of peril; (2) defendant was aware of plaintiffs position of peril and of plaintiffs inability to extricate himself; and (3) defendant could then have prevented the harm by the exercise of ordinary care. c. Comparative negligence Most states which have adopted comparative negligence have done so by statute. Several states have recently adopted comparative negligence as a matter of common law. Under comparative negligence, plaintiff may recover damages even though guilty of contributory negligence, but damages are reduced by the percentage of plaintiffs negligence. In states having pure comparative negligence, plaintiff may still recover a percentage of her damages even though her negligence exceeds defendant's. In a majority of states which have adopted a modified comparative negligence, plaintiff will not recover if her negligence exceeds defendant's. 2. Assumption Of The Risk The defense of assumption of risk requires proof that plaintiff knowingly entered into, or stayed in, a position of danger. The defense applies irrespective of the reasonableness of plaintiffs conduct. This defense is being abolished in some states, particularly in the jurisdictions which have adopted comparative negligence. 3. Imputed Negligence The general rule is that the negligence of one party cannot be attributed to another. Thus, the contributory negligence of a driver cannot be raised by a negligent third party to bar an action by a passenger in the driver's car. Likewise, the negligent supervision of parents will not act to bar a suit by their child to recover for harm resulting from the negligence of a third party, and the negligence of the plaintiffs spouse will not defeat the plaintiffs cause of action against a third party. The exceptions to this rule are that the negligence of the deceased will bar (or reduce) a wrongful death recovery by the decedent's estate and the negligence of the spouse will bar or reduce a claim for loss of consortium. 4. Immunity Immunity is also a defense that may be raised in a negligence action as well as an action for intentional tort.

MicroMash MBE In Brief: Torts

17

H. DAMAGES IN NEGLIGENCE ACTIONS


1. Damages Recoverable In a personal injury action, plaintiff may recover for: (1) diminution of earning capacity, past and future; (2) medical expenses, past and future; and (3) pain and suffering, past and future. Most states also allow recovery for loss of consortium for injury to either the husband or wife.

2. Collateral Source Rule


In most states, amounts received by injured plaintiffs under medical insurance plans or wage compensation plans are not deducted from damages recovered for medical expenses or impairment of earning capacity; this is referred to as the collateral source rule.

III. STRICT LIABILITY A. ABNORMALLY DANGEROUS


Most jurisdictions allow recovery on a strict liability theory for harm resulting from abnormally dangerous activities. Conduct is abnormally dangerous if it: (1) necessarily involves a risk of serious harm to the person, land, or chattels of others which cannot be eliminated by the exercise of utmost care and (2) is not a matter of common usage.

B. THE RULE OF RYLANDS V. FLETCHER


In Rylands v. Fletcher, Justice Blackburn stated that "the person who for his own purposes brings on his land and collects and keeps there anything likely to do mischief if it escapes, must keep it in at his peril, and if he does not do so, is prima facie answerable for all the damage which is the natural consequence of its escape." The rule was modified by the House of Lords, which confined the doctrine to non-natural uses of the land, i.e., activity that is not ordinary or appropriate for its locality.

C. KEEPING ANIMALS
1. Trespassing Animals Apart from statute, owners are strictly liable for harm done by animals trespassing off the owner's land only if the animals constitute livestock or wild animals. However, some states by statute also make the owners of domestic pets liable for harm done by trespassing animals. 2. Personal Injury Caused By Wild Animals Or Domestic Animals Apart from the trespassing situation, under the common law, strict liability applies only to harm done by wild animals kept by the defendant. Negligence is required to impose liability on the owner of livestock or a domestic pet unless the owner knew or had reason to know of the animal's dangerous propensities; in that event, strict liability is imposed. By statute, some states impose strict liability on the owner of a dog for harm done by the dog, unless the victim was committing a trespass or other tort.

18

MicroMash MBE In Brief: Torts

IV. PRODUCTS LIABILITY


A. NEGLIGENCE
Sellers of products, like anyone else, can be held liable for injuries caused by their failure to exercise due care. A breach of this duty can be found in any one of the following circumstances. A defendant can be liable under a negligence products liability theory when his failure to exercise due care causes a product to differ from its intended design and this difference causes the product to be more dangerous than others of its type. This type of negligence may result from carelessness in manufacturing (as when a product is improperly assembled) or from a mishandling of the product (as when it is dropped or improperly exposed to the elements). Negligence may be found, even if a product meets the specifications of its designer, if the design unreasonably fails to protect potential plaintiffs from harm. This so-called "negligence in design" requires a careful consideration of whether the defendant could have reasonably foreseen the danger, the extent to which the technology in existence at the time of its manufacture could minimize the risks, and the degree to which the consumer could be reasonably expected to appreciate and protect himself from harm. A product might be suitable for its intended purpose when used properly, but unreasonably dangerous if it is handled differently. In such a case, a defendant can be liable in negligence if he fails to provide adequate instructions or warnings with regard to foreseeable uses to which the product may be put. Any seller who is physically responsible for the product's dangerous condition can be liable under a negligence theory. In addition, sellers further down the chain of distribution can be liable if a reasonable inspection would have revealed a product's dangers and they unreasonably failed to protect possible plaintiffs from such dangers. The question whether a wholesaler or retailer should inspect goods at all, and the kind of inspection that is appropriate, is determined by ordinary concepts of reasonableness and foreseeability. Factors which would weigh in the balance include: (a) the potential dangerousness of the product, (b) any safety history with regard to the product or predecessors in possession, (c) the presence of any physical evidence of danger (as when the product arrived in a damaged container), and (d) the practical ability of the defendant to inspect the product (including costs).

B. STRICT LIABILITY
All jurisdictions today recognize the liability of a manufacturer for negligence, despite the absence of privity between the manufacturer and the consumer. Most jurisdictions also recognize the liability of a manufacturer, seller, or supplier for a defective product without any showing of fault and despite the absence of privity. Liability is based upon either a warranty theory or a theory of strict liability in tort. A supplier may be strictly liable, even though selling such product is not its principal business (e.g., sale of popcorn at a movie

MicroMash MBE In Brief: Torts

19

theatre); however, a person who is an occasional seller (e.g., sale of automobile by private person to used car dealer) is not strictly liable. Manufacturers are strictly liable for defective component parts which are supplied by others but assembled into the final product. Further, they are liable for a final defective product, even when they rely upon a third person (e.g., a dealer) to complete assembly, and the third person causes the product to be defective. Whether the defect was discoverable by a reasonable inspection is irrelevant under strict liability. Strict liability does not apply to services; negligence must be proved. In hybrid (serviceproduct combination) situations, strict liability may apply to harm caused by the product.

1. Individuals Protected
Consumers as well as users (e.g., passengers in an automobile) are protected by the strict liability rule. Further, the majority rule allows recovery even by a bystander injured by the product. Misuse of a product does not bar strict liability where the misuse was foreseeable. Obviousness of the defect is not a defense in defective design cases. The rescue doctrine applies in strict liability cases.

2. When Strict Liability Is Applicable a. Defective manufacture or design


Strict liability has been applied to so-called "defective design" cases as well as to manufacturing defects. Most cases involve automobiles where plaintiff claims that severe injury from a collision could have been avoided by a safer design. These are often referred to as "second collision" cases.

b. Unavoidably unsafe products


Unavoidably unsafe products, products which in the present state of human knowledge are incapable of being made safe for their intended and ordinary use, are justified by the need for the product. Such a product is not defective, nor unreasonably dangerous, if properly prepared and accompanied by proper directions and warnings.

c. Failure to warn
Strict liability has also been applied where, even though the product was not defective, defendant failed to warn consumers adequately of its dangerous characteristics (e.g., child killed by drinking furniture polish).

3. Defenses
Usually, neither contributory negligence nor assumption of the risk alone constitutes a defense to the manufacturer's or seller's liability based upon either warranty or strict liability. Where, however, plaintiff, with knowledge of the defective product, unreasonably continues to use the product, the defense of unreasonable assumption of risk is available to defendant. A few jurisdictions have used plaintiff's contributory negligence to diminish her damage recovery in a strict liability suit even where there was no assumption of the risk.

20

MicroMash MBE In Brief: Torts

C. MISREPRESENTATION AND WARRANTY THEORIES


Several potential products liability theories are based upon express or implied assertions concerning the nature and quality of a seller's goods.

1. Misrepresentation Under Restatement 402B


Under 402B of the Restatement (Second) of Torts, a commercial seller's public

misrepresentation of material fact concerning the character or quality of a product


is actionable if the plaintiff is injured thereby.

2. Warranties Under The Uniform Commercial Code a. Express warranty under U.C.C. 2-313
Section 2-313 of the Uniform Commercial Code imposes liability for injuries resulting from a breach of express warranty. Insofar as its applicability to torts questions is concerned, an express warranty claim is essentially the same as an action under 402B except that: (1) liability can be imposed on any seller (not just on commercial suppliers), (2) the warranty need not be made to the public at large, (3) specific reliance on the warranty is not required so long as it was "part of the basis of the bargain," and (4) the plaintiff must fall within the class of people protected by the relevant state's version of 2-313.

b. Implied warranty of merchantability under U.C.C. 2-314


Under U.C.C. 2-314, every commercial seller warrants that the goods he sells are of "fair average quality within the description" and "fit for the ordinary purposes for which such goods are used" unless this implied warranty of merchantability is clearly disclaimed or modified. The privity notice, and defense issues with regard to this action, are the same as those relating to express warranty as discussed above.

c. Implied warranty of fitness for a particular purpose under U.C.C. 2-315


When a seller has reason to know of the buyer's particular purpose for certain goods and that the buyer is relying on the seller's skill or judgment to select a product appropriate for such needs, the seller impliedly warrants that the goods are fit for that use. This implied warranty of fitness for a particular purpose applies whether or not made by a commercial distributor.

V. OTHER TORTS A. NUISANCE


1. Interference With Use And Enjoyment
The action for nuisance protects the plaintiff from interference with the use and enjoyment of her property. Traditionally, it involves such interferences as noise, excessive light, odors, fumes, smoke, etc. The plaintiff may sustain the nuisance action by showing that the defendant substantially interfered with the plaintiff's use and enjoyment of her property, and that such interference was unreasonable, even if the defendant's conduct was not negligent or intentional.

MicroMash MBE In Brief: Torts

21

2. Reasonableness Of Conduct
Whether defendant's conduct is reasonable or unreasonable involves the balancing of three considerations: (1) the locality and character of the surrounding area; (2) the nature, extent, and frequency of the interference; and (3) the utility and social value of the activity involved.

3. Relief
The court in a nuisance action may grant damages and/or injunctive relief. Where plaintiffs harm is insignificant when contrasted with the potential harm to defendant if injunctive relief is granted, the court may treat the nuisance as permanent, and award plaintiff damages for the diminution in the value of his property.

4. Public Nuisance
Recovery for personal injury resulting from a nuisance may be had whether the nuisance is private or public. If the nuisance is public, plaintiffs injury must be of a different kind than that of the general public.

B. CLAIMS BASED UPON DEFAMATION AND INVASION OF PRIVACY 1. Defamation a. Elements


The elements of an action for defamation are: (1) defendant published a false communication of fact concerning the plaintiff which held plaintiff up to scorn and ridicule in the eyes of a respectable minority in the community; (2) the defamatory matter was communicated to at least one person other than plaintiff herself, and its defamatory meaning was understood by the third person; and (3) if the defamation is slander (oral), and does not fall into one of the four categories of slander per se, special damages must be proven. Proof of special damages requires a showing that, as a result of the slander, someone denied plaintiff an existing or prospective benefit, as for example, the loss of a job.

b. Fault requirements; damages


Under the First and Fourteenth Amendments to the United States Constitution, statements concerning public officials and public figures are not actionable unless there is clear and convincing evidence that defendant knew the statement was false, or made the statement with reckless disregard for its truth or falsity. A person may be a public figure generally or a public figure solely with respect to the matter involved in the publication. A private person may recover upon a state-imposed standard, but not on a strict liability standard. Usually the state-imposed standard will be negligence. If plaintiff establishes liability on the basis of a standard less than knowledge of falsity or recklessness, plaintiff may recover actual damages and presumed damages, but no punitive damages will be allowed. Pretrial discovery into the thoughts and editorial processes of the alleged defamer is allowed to enable the plaintiff to prove malice.

22

MicroMash MBE In Brief: Torts

c. Defense Falsity is an element of the plaintiff's case, but truth is still an absolute defense. The other primary defense in an action for defamation is privilege, which, depending upon the importance of the policy advanced by the privilege, is either absolute or conditional. An absolute privilege protects the person making the defamatory statement, even where that person acts maliciously. A conditional privilege protects only the nonmalicious publisher of defamatory matter. 2. Invasion Of Privacy a. Categories There are four categories of the tort of invasion of privacy: (1) intrusion upon the plaintiff's physical solitude for example, placing a listening device in his home this category does not require a publication of the matter to sustain recovery; (2) publication of matters concerning plaintiff which violate common decencies recovery under this category is extremely difficult unless the matter published is not newsworthy; (3) publication of matters which, while not necessarily defamatory, place plaintiff in a false light in a way which would be highly offensive to a reasonable person; and (4) appropriation of some element of plaintiff's personality for commercial purposes, as for example the unauthorized use of the picture of a famous athlete on defendant's product. b. Constitutional considerations The constitutional considerations with regard to fault requirements for media defendants set out by the United States Supreme Court in the defamation area apply also in the area of invasion of privacy.

C. CLAIMS BASED UPON MISREPRESENTATIONS


1. Deceit The elements of an action for deceit are: the defendant made a misrepresentation of an existing fact; with intent to induce reliance from plaintiff or from a class of which plaintiff is a member; with knowledge of falsity or with reckless disregard for truth or falsity (referred to as scienter), or in some cases, the legal equivalent of scienter; the misrepresentation was of a material nature; and plaintiff relied upon the misrepresentation to his or her detriment. a. Misrepresentation While the misrepresentation is usually of an objective fact, it may involve a misrepresented intention or opinion since there the existing fact is the speaker's state of mind. A broken promise, standing alone, is not a misrepresented intention; fraud at the time of the statement of intent must be shown. A misrepresentation may be

MicroMash MBE In Brief: Torts

23

established by showing defendant's silence when there was a duty to speak, as for example where the seller of property fails to disclose a dangerous, concealed defect which the buyer could not have discovered by a reasonable inspection. b. Knowledge of falsity Scienter (knowledge of falsity or recklessness) was an essential element of the deceit action at common law. Today, however, some jurisdictions allow plaintiff to recover at least out-of-pocket damages where the defendant made an unqualified assertion of fact as to her own knowledge concerning a matter which was susceptible of exact knowledge. c. Reasonableness of belief Most jurisdictions today do not deny recovery for deceit on the basis that plaintiff was unreasonable in believing defendant's misrepresentations. A distinction, however, still exists between misrepresentations of fact and so-called "seller's talk." The reliance must be at least justifiable. d. Damages Damages in deceit may be either "benefit-of-the-bargain" damages or "out-of-pocket" damages. Most jurisdictions, where defendant's conduct was intentional or reckless, allow plaintiff to recover "benefit-of-the-bargain" damages; that is, the difference between what plaintiff received and what he would have received if the representation had been true, provided such damages can be proved with reasonable certainty. Otherwise, plaintiff may only recover "out-of-pocket" damages; that is, the difference between the value of what the plaintiff gave up and what he received. 2. Negligent Misrepresentation An action for negligent misrepresentation will lie against a defendant who, in the performance of her trade or profession, negligently provides erroneous information which is used by the plaintiff to his or her detriment despite the absence of privity between plaintiff and defendant, provided that such liability would not expose the defendant to an unlimited liability to an indeterminate number of persons. Generally, a defendant will be held liable despite lack of privity only if the defendant knew that the plaintiff would be likely to rely on the representation.

D. CLAIMS BASED UPON INTENTIONAL INTERFERENCE WITH BUSINESS RELATIONS 1. Interference With Contractual Relations a. Elements
The elements of the action for interference with contractual relations are: (1) plaintiff had an existing contract with a third person; (2) defendant knew of the contract; and (3) defendant intentionally interfered with the contract. While occasionally a defendant's conduct may be privileged, business competition does not justify

24

MicroMash MBE In Brief: Torts

intentional interference with contractual relations, although some courts hold otherwise where the contract was terminable at will.

b. Distinguish procuring refusal to contract


The action for intentional interference with existing contractual relations should be distinguished from the tort of procuring a person's refusal to contract with plaintiff which may, unless the means used are unlawful, be justified on the basis of business competition.

2. Interference With Advantageous Relations


A defendant may be liable in tort for intentionally interfering with a prospective benefit to plaintiff; for example, interfering with the making of a will in which plaintiff was to be named as a beneficiary.

E. CLAIMS BASED UPON MALICIOUS PROSECUTION 1. Elements


The elements of an action for malicious prosecution are: (1) defendant procured the initiation of criminal proceedings against plaintiff; (2) defendant did not have probable cause to procure such proceedings; (3) defendant acted with malice; and (4) plaintiff received a favorable termination in the proceedings. Arrest of the plaintiff is not a necessary element of the tort. While malice may be inferred from lack of probable cause, lack of probable cause cannot be inferred from malice.

2. Application To Civil Proceedings


Some jurisdictions recognize the action for malicious prosecution in the civil sense, as, for example, the unwarranted initiation of bankruptcy proceedings. The elements of lack of probable cause, malice, and favorable termination are still present.

VI. IMPACT OF INSURANCE, WORKERS' COMPENSATION, AND "NO-FAULT" SYSTEMS A. AREAS AFFECTED BY INSURANCE
Insurance, particularly liability insurance, has played a significant role in recent decades in the development of tort law. Particularly affected are: (1) the extension of vicarious liability; (2) extension of strict liability for defective products; (3) abolition of various immunity doctrines; (4) development of Workers' Compensation plans; and (5) development of nofault systems.

B. TYPES OF INSURANCE
There are basically two types of insurance:

(1) First-party insurance, which pays the insured or designated persons (usually the insured's family) for their loss. It is not based upon liability. Medical insurance is a
typical form of first-party insurance.

MicroMash MBE In Brief: Torts

25

(2) Third-party insurance, in which the insurer pays a third person to whom the insured has become liable; it is liability insurance.

C. WORKERS' COMPENSATION SYSTEMS


Under Workers' Compensation, an injured employee recovers medical expenses and a percentage of lost wages without having to prove fault and even if the employee himself was guilty of contributory negligence. However, the employer is immune from any liability in tort for having caused the employee's injury.

D. "NO FAULT" SYSTEMS


-

Under no-fault a person injured due to an accident arising from the use of an automobile (i.e., operator, passenger, pedestrian) may recover medical expenses and a percentage of lost wages (up to some prescribed maximum amount) without proving negligence, even though the victim was guilty of contributory negligence. The owner or operator of the vehicle is immune from liability up to the prescribed amount. Beyond that, the owner or operator may be liable in tort.

26

MicroMash MBE In Brief: Torts

MicroMash BAR REVIEW BAR EXAM ALERTS AT-A-GLANCE TORTS

I. INTENTIONAL TORTS A. HARMS TO THE PERSON 1. ASSAULT


Intent to cause apprehension of contact is all that is required for the tort of assault. Contact or intent to actually contact is not required. Words alone are not enough unless accompanied by an ability and intent to act. There is a privilege to use any assault or reasonable battery in self-defense, defense of others, and to eject trespassers. These privileges exist as long as the actor reasonably believed the circumstances called for the conduct, even if there was a mistake. 2. BATTERY To recover from battery, the plaintiff must show (1) an intentional and (2) unconsented (3) harmful or offensive (4) touching. No actual harm is required, though. There is a privilege to use any assault or reasonable battery in self-defense, defense of others, and to eject trespassers. These privileges exist as long as the actor reasonably believed the circumstances called for the conduct, even if there was a mistake. 3. FALSE IMPRISONMENT The plaintiff need not resist confinement to have an action for false imprisonment. The imprisonment must be (1) intentional and (2) without consent to be actionable. A shopkeeper has a privilege to reasonably detain someone reasonably suspected of shoplifting. The plaintiff must have been aware of the confinement.

MicroMash MBE In Brief: Torts Bar Exam Alerts

Any reasonable exit or alternate route from confinement defeats the cause of action. Only a wrongful arrest constitutes false imprisonment. The fact that the plaintiff did not actually commit the crime does not automatically give rise to a cause of action. 4. INTENTIONAL INFLICTION OF MENTAL DISTRESS The defendant's conduct must be extreme and outrageous such that it would be substantially certain to cause severe emotional distress in a person of normal sensitivities. However, the defendant can also be held liable if she knows of the victim's peculiar sensitivities. Bystanders can recover for emotional distress resulting from an intentional tort to a family member only if the defendant knew the bystander was a witness.

B. HARMS TO PROPERTY
1. TRESPASS TO LAND Force need not be used to gain entry, but consent (even implied consent) will defeat the trespass to land cause of action. There is a privilege to trespass in emergency situations and to protect one's own property, but the trespasser must pay for any damage done by the trespass. 2. TRESPASS TO CHATTELS; CONVERSION Both trespass to chattels and conversion require an intentional interference with the personal property of another. However, like trespass to land, the defendant need not know that the trespass was wrongful for it to be actionable. The defendant need only intend to commit the act that constitutes the trespass. Only a substantial interference with personal property can be the basis for an action of conversion. If the defendant has substantially damaged or lost the personal property or if the defendant refuses to return it after demand has been made, the plaintiff can bring an action for conversion to recover the value of the personal property at the time that the defendant first asserted dominion over it. Any lesser interference with personal property is only the basis for a trespass to chattels action to recover damages for the harm done to the chattel and the plaintiff's lost use of the chattel.

MicroMash MBE In Brief: Tort Bar Exam Alerts

II. NEGLIGENCE
A. DUTY Generally, one has a duty to act reasonably, but one usually does not have a duty to rescue someone from a danger that he or she did not create. Only the creator of the peril or a close family member (usually a parent) has any duty to rescue someone in danger. Once one undertakes to rescue, though, one has a duty to act with reasonable care. This is called the Good Samaritan doctrine.

B. RES IPSA LOQUITUR


The plaintiff can prevail without direct proof of the defendant's negligence if she can prove (or the circumstances alone indicate) that she would not have been harmed if the defendant had not been negligent. The plaintiff need only show that it is more likely than not that the defendant was negligent (and that there is a causal connection to the plaintiff's harm). Res ipsa loquitur is irrelevant if there is direct evidence that the defendant was negligent. The defendant can defeat a res ipsa loquiturcase against her by showing that it is just as likely that someone else's negligence caused the plaintiff's harm. C. CAUSATION The test of factual causation is "but for," i.e., the plaintiff's harm would not have occurred if not for the defendant's negligence. The chain of causation is broken by a superseding intervening cause. However, the defendant is liable for any harm which would have occurred if not for the superseding cause. A foreseeable intervening cause is not superseding. An unlawful or negligent act may be foreseeable. Once a plaintiff has recuperated from an injury by the defendant, the defendant is not liable for subsequent injuries just because they would not have happened if not for the plaintiff's weakened condition from the first injury. Even if there is factual causation, there must be proximate cause (i.e., the plaintiff's harm must not be too remote). A cause cannot be the "proximate" cause unless it is also the factual cause.

D. CONTRIBUTION
Where two or more independent defendants are responsible for the plaintiff's harm, each is generally ultimately liable only for the part of plaintiff's harm for which he is responsible. However, the plaintiff can recover all of his damages

MicroMash MBE In Brief: Torts Bar Exam Alerts

from the defendants if their combined negligence causes more harm than their actions alone would have caused (or even if, alone, their actions would not have caused any harm). Contribution is available between joint tortfeasors no matter what the relative degrees of fault. Contribution allows a defendant who was held liable to recover a pro rata share of her liability from her joint tortfeasors. A negligent defendant is only entitled to contribution (not complete indemnification) from a joint tortfeasor.

E. CLAIMS AGAINST OWNERS OF LAND


At common law, a business invitee was owed a duty of reasonable care. A licensee (social guest) was only owed a duty to warn of dangers known to the owner or occupant but not obvious to the licensee. A trespasser was only owed a duty to avoid gross negligence or wanton, willful misconduct. The doctrine of "attractive nuisance" only applies if the landowner or occupant has reason to know both that children might come onto the land, and that the nuisance might be dangerous to them. Then, the landowner owes a duty of reasonable care to the infant trespasser(s). It is not necessary that the children be attracted onto the land by the nuisance.

F. NEGLIGENT INFLICTION OF MENTAL DISTRESS


A plaintiff can recover for negligent infliction of mental distress only if (a) she experiences some actual physical harm from the defendant's negligence, (b) she is within the zone of danger, or (c) she witnesses harm to a family member. The plaintiff can only recover if the defendant's conduct was sufficient to cause emotional distress in a person of normal sensitivities. Once this objective test is met, the plaintiff can recover for any emotional harm, even if it is unusual.

G. LIABILITY FOR THE ACTS OF OTHERS 1. EMPLOYEES AND OTHER AGENTS


Employers are liable for the torts of their servants committed within the scope of their employment. An intentional tort is within the scope of employment if it was committed to further the master's business. A plaintiff who is injured by a servant may sue the servant and/or the employer. 2. INDEPENDENT CONTRACTORS Generally, the employer of an independent contractor is not liable for the torts of the independent contractor. An employee is an independent contractor as opposed to a servant if the employer does not control the employee's performance.

MicroMash MBE In Brief: Tort Bar Exam Alerts

However, an employer is liable for ultrahazardous activity (e.g., blasting) undertaken by an independent contractor. There is said to be a nondelegable duty to see that such activity is performed properly. An employer of an independent contractor can also be held liable for her negligence in hiring an unfit contractor. 3. JOINT ENTERPRISE When two parties enter into a joint enterprise, they are liable for each other's torts within the scope of the joint enterprise activity. 4. INDEMNIFICATION A right of indemnification exists when a nonnegligent defendant has been held vicariously liable (i.e., for the negligence of another, e.g., a servant). The nonnegligent defendant has the right to recover all of the amount for which she was held liable from the negligent party. H. DEFENSES 1. CONTRIBUTORY FAULT Under a contributory (as opposed to comparative) negligence rule, any negligence on the part of the plaintiff would bar his recovery against the defendant. However, in some jurisdictions, a negligent plaintiff could still recover if the defendant had the last clear chance to avoid the accident. The contributory negligence of another party will not be attributed to the plaintiff, even if the parties are related. However, under most wrongful death statutes, the contributory negligence of either the decedent or the beneficiaries will bar recovery. 2. COMPARATIVE NEGLIGENCE Under a comparative negligence statute, the negligence of the plaintiff will not defeat the plaintiff's cause of action, but her recovery will be reduced by her share of the negligence. Under a "pure" comparative negligence statute, the plaintiff will recover no matter how negligent she was. Under a "hybrid" comparative negligence statute, she will recover only if her negligence was equal to or less than the combined negligence of the other parties. Comparative negligence does not change the rule of joint and several liability between joint tortfeasors. A plaintiff can recover all of her damages (minus her share of the negligence) from one defendant; that defendant will then have to seek contribution from the joint tortfeasor (based on the joint tortfeasor's share of the negligence).

MicroMash MBE In Brief: Torts Bar Exam Alerts

3. DAMAGES IN NEGLIGENCE ACTIONS Expert testimony is usually required to prove future medical expenses. A defendant is liable for all of the damages proximately caused by her negligence, even if the defendant's particular damages were unforeseeable.

III. STRICT LIABILITY A. ABNORMALLY DANGEROUS ACTIVITIES There is strict liability for any harm which results from a use of land which (1) is
not common to the area and (2) presents a serious risk of harm even if undertaken with due care. In such a case, the plaintiff need not show that the defendant's conduct of the abnormally dangerous activity was negligent to recover.

B. ANIMALS
Owners of wild animals are strictly liable for the harm caused by them; owners of domesticated animals are only liable for their own negligence regarding the animals.

IV. PRODUCTS LIABILITY A. NEGLIGENCE


Contributory negligence is a defense in a products liability action based on negligence. Exercise of due care by the defendant in manufacturing or handling the product will defeat plaintiff's products liability claim in negligence.

B. STRICT PRODUCTS LIABILITY


The manufacturer of a defective component is liable for the defective parts included in the finished product. A supplier (i.e., a seller or manufacturer) of a product is strictly liable if: (1) the product was defective when it left the party's hands and (2) that defect causes the plaintiff harm. Foreseeable users and even bystanders may recover under a strict liability theory. Assumption of the risk is a defense in strict products liability actions, but contributory negligence is not.

MicroMash MBE In Brief: Tort Bar Exam Alerts

Exercise of utmost care by the defendant in manufacturing or handling the product will not defeat plaintiff's strict liability claim. Misuse by the plaintiff is not a defense, unless that misuse was unforeseeable. Alteration of the product after it left the defendant's hands can defeat the plaintiff's strict liability action. An assembler is liable for the defective parts included in its finished product. Unavoidably unsafe drugs (including blood) are the basis of strict liability only if the supplier does not notify the physician of the potential dangers. C. DEFENSES A plaintiff's cause of action is defeated by assumption of the risk only if he had actual, subjective knowledge of the risk and voluntarily assumed it.

V. OTHER TORTS
A. NUISANCE
A nuisance exists when the defendant's use of neighboring land unreasonably interferes with the use and enjoyment of the plaintiff's land. A defendant can be held liable for nuisance even if the offending use is not negligent.

1. PUBLIC NUISANCE
A nuisance is a private nuisance if it interferes with only one neighbor's use and enjoyment of her land. That neighbor has a cause of action for private nuisance. A nuisance is a public nuisance if it interferes with the use and enjoyment of several neighboring parcels. In general, only the relevant political subdivision (city, county, etc.) has the right to sue for a public nuisance. However, a private plaintiff can sue for public nuisance if the harm to that plaintiff from the nuisance is different in kind from the harm to the public (or if a statute gives the neighbor a private cause of action). 2. DEFENSES TO NUISANCE ACTION If there is an actual physical invasion of the plaintiff's land, the cause of action is for trespass, rather than nuisance. Which use commenced first is a factor to be considered in deciding whether a particular use is a nuisance, but it is not dispositive. (That is, one can "move to the nuisance" and still enjoin it or recover damages in some cases.)

MicroMash MBE In Brief: Torts Bar Exam Alerts

The fact that a particular use is permitted by the applicable zoning regulations does not establish that it is not a nuisance, although it is some proof that the use is reasonable. Some courts will deny any judgment for plaintiff if the defendant's use of the land is socially useful. Other courts will award damages, but will refuse to enjoin such a use.

B. DEFAMATION 1. LIBEL
Some jurisdictions treat libel per se and libel per quoddifferently. Libel per se is that which is libelous on its face. Libel per quod is that which is libelous only when taken in conjunction with facts known by those to whom the libel is published. These jurisdictions allow recovery for libel per quod only if there are "special" (i.e., actual monetary) damages.

2. PUBLICATION
There must be a publication meaning that the defendant must intend or allow that at least one person (other than the defamer and defamed) receive and understand the statement (even if they don't believe it). 3. DEFENSES Truth is an absolute defense in a defamation action (and a "false light" privacy action).

4. "DEFAMATORY" DEFINED
Material is defamatory if it would lower the person's esteem in the eyes of a reputable segment of the community. 5. FAULT Where a showing of malice is not required, any other plaintiff must at least prove negligence to recover for defamation. 6. MALICE REQUIRED A public official or figure must prove "malice" that the defendant acted with knowledge of falsity or reckless disregard for the truth in order to recover for defamation (or invasion of privacy). 7. DAMAGES The common law rule is that a plaintiff suing for slander must show special damages unless the slander constitutes slander per se (charging the plaintiff with a crime, a loathsome disease, sexual misconduct, or business incompetence).

MicroMash MBE In Brief: Tort Bar Exam Alerts

8. DEFENSES There is no defamation if the statement was privileged. Relevant statements made in court hearings are absolutely privileged. (A perjury action, not a defamation action, is the proper action for lies stated in court) All other privileged statements (those made by an employer to another employer about an employee, statements in administrative hearings) are only protected by a qualified privilege. Such qualified privileges can be overcome if the statement was made with knowledge that it was false or with reckless disregard for its truth. A public official or figure must prove "actual malice" to recover for defamation. "Actual malice" requires that the plaintiff prove that the defendant either knew that the information was false, or acted with reckless disregard for its truth or falsity. C. INVASION OF PRIVACY Publication is not required for the tort of intentional intrusion upon seclusion. Truth is not a defense to the torts of invasion of privacy (except "false light" publicity). Newsworthiness is a defense to the tort of invasion of privacy. Publishing information in the public record cannot generally be the basis for a privacy tort. A defendant can be held liable for giving 'unreasonable publicity to the plaintiff's private life if the material published would be highly offensive to a reasonable person and is not of legitimate public concern (i.e., newsworthy). A defendant can be held liable for false-light publicity even if the material published is complimentary (as long as it is untrue).

D. MISREPRESENTATION
In order to be deceit, there must usually be an affirmative misrepresentation. Silence is a misrepresentation only when the defendant had some legal (e.g., fiduciary) duty to disclose. A statement of opinion is not an actionable misrepresentation.

10

MicroMash MBE In Brief: Torts Bar Exam Alerts

MicroMash
Bar Review
6402 South Troy Circle Englewood, CO 80111-6424 303-799-0099 Fax: 303-799-1425 email: info@MicroMash.com

MicroMash publishes a comprehensive line of Bar Exam preparation materials and computerbased continuing legal education courses.

...a

MicroMash MBE Review


for the Multistate Bar Exam

MicroMash State Bar lie4tiews


for the state portion of the Bar Exam
,

MicroMash MPRE review


for the Multistate Profeisional Responsibility Exam

MicroMash MPT Review


for the Multistate Performance Test

MicroMash MEE Review


for the Multistate Essay Exam

MicroMash Continuing Legal Education (CLE)


Over 100 continuing legal education courses are on-line, downloadable, and disktbased.

n O
m

Você também pode gostar